Вы находитесь на странице: 1из 304

Continuum: Lifelong Learning in Neurology—Movement Disorders, Volume 22, Issue 4,

August 2016

Issue Overview

Movement Disorders August 2016;22(4)

Continuum: Lifelong Learning in Neurology® is designed to help practicing neurologists stay

abreast of advances in the field while simultaneously developing lifelong self-directed learning

skills.

Learning Objectives
Upon completion of this Continuum: Lifelong Learning in Neurology Movement Disorders issue,
participants will be able to:
 Differentiate the clinical presentation of Parkinson disease from its mimics and other

forms of parkinsonism and describe the appropriate ancillary tests and the indications for

their use

 Discuss the various pharmacologic and nonpharmacologic strategies for treating early

and advanced motor symptoms of Parkinson disease

 Recognize and manage neuropsychiatric problems in patients with Parkinson disease

 Manage and counsel patients (and their families) affected by advanced Parkinson disease

 Diagnose and manage atypical parkinsonian syndromes

 Diagnose and manage tremor disorders

 Discuss the broad spectrum of movement disorders that may present in childhood and

their management strategies

 Diagnose and manage Huntington disease and other causes of chorea

 Diagnose and manage the spectrum of disorders in which ataxia is a prominent sign

 Recognize key clinical features of dystonia, categorize dystonia using the new

classification schema, and describe the approach to treatment

Copyright © American Academy of Neurology. Unauthorized reproduction of this article is prohibited.


 Recognize the many clinical presentations of Wilson disease, perform the appropriate

investigations to expeditiously diagnose the disease, and discuss the appropriate

treatments of this condition

 Discuss the ethics of ordering dopamine transporter imaging in asymptomatic patients

concerned about developing Parkinson disease

Core Competencies
This Continuum: Lifelong Learning in Neurology Movement Disorders issue covers the
following core competencies:
 Patient Care and Procedural Skills
 Medical Knowledge
 Practice-Based Learning and Improvement
 Interpersonal and Communication Skills
 Professionalism
 Systems-Based Practice

Disclosures
CONTRIBUTORS

Susan H. Fox, MBChB, MRCP(UK), PhD, Guest Editor


Associate Professor of Neurology, University of Toronto; Associate Director, Movement
Disorder Clinic, Toronto Western Hospital, University Health Network, Toronto, Ontario,
Canada
a
Dr Fox has received salary support for serving as the co-editor of the International Parkinson and Movement
Disorder Society website, as a speaker for Ipsen Pharmaceuticals, Inc; Teva Pharmaceutical Industries Ltd; and
Zambon Company SpA; and for serving on the advisory boards of Lundbeck, Orion Pharma Ltd, and Novartis
International AG. Dr Fox has received research/grant support from Avanir Pharmaceuticals, the Michael J. Fox
Foundation, the National Institutes of Health, and Parkinson Society of Canada and has received research/grant
support as site principal investigator for clinical studies from Adamas Pharmaceuticals Inc, Cynapsus
Pharmaceuticals Inc, and Kyowa Hakko USA Inc. Dr Fox receives royalties from Oxford University Press.
b
Dr Fox discusses the unlabeled/investigational use of amantadine and methylphenidate for freezing gait disorders,
the use of clozapine and mirtazapine for tremor in Parkinson disease, and the use of anti–α-synuclein antibodies,
caffeine, coenzyme Q10 , creatine, inosine, isradipine, nicotine, nilotinib, pioglitazone, and rasagiline as failed or
potential disease-modifying therapies.

Tetsuo Ashizawa, MD, FAAN


Director, Neurosciences Research Program, Houston Methodist Hospital, Houston, Texas
a
Dr Ashizawa receives research/grant support as principal investigator of studies for Ionis Pharmaceuticals, Inc
(1515598769-CS2); the Myotonic Dystrophy Foundation; and the National Institutes of Health (NS083564).
b
Dr Ashizawa reports no disclosure.

Copyright © American Academy of Neurology. Unauthorized reproduction of this article is prohibited.


Susan B. Bressman, MD, FAAN
Mirken Chair, Departments of Neurology, Mount Sinai Beth Israel, Mount Sinai St. Luke’s, and
Mount Sinai West; Professor of Neurology, Icahn School of Medicine at Mount Sinai, New
York, New York
a
Dr Bressman receives personal compensation for serving on the scientific advisory boards of the Bachmann-
Strauss Dystonia & Parkinson Foundation, Inc and the Michael J. Fox Foundation and for serving on the editorial
board of Movement Disorders. Dr Bressman receives research/grant support from the Michael J. Fox Foundation.
b
Dr Bressman discusses the unlabeled/investigational use of oral medications for the treatment of dystonia, none of
which are approved by the US Food and Drug Administration.

Thomas I. Cochrane, MD, MBA


Associate Neurologist, Senior Ethics Consultant, Brigham and Women’s Hospital; Assistant
Professor of Neurology, Director of Neuroethics, Center for Bioethics, Harvard Medical School,
Boston, Massachusetts
a
Dr Cochrane serves as a consultant for Merck & Co, Inc.
b
Dr Cochrane reports no disclosure.

Russell C. Dale, MBChB, MRCP, PhD


Professor of Paediatric Neurology, University of Sydney, Sydney, Australia
a
Dr Dale has received personal compensation for serving on the editorial advisory board of Multiple Sclerosis and
Related Disorders and the editorial boards of the European Journal of Paediatric Neurology and Neurology:
Neuroimmunology & Neuroinflammation, and on the scientific advisory board for the Queensland Children’s
Medical Research Institute. Dr Dale has received honoraria for speaking engagements as well as publishing
royalties from Biogen Inc and Bristol-Myers-Squibb and has received research support from Multiple Sclerosis
Research Australia and the National Health and Medical Research Council.
b
Dr Dale reports no disclosure.

Dawn Eliashiv, MD, FAAN


Professor of Neurology, Department of Neurology, David Geffen School of Medicine at
University of California, Los Angeles; Codirector, University of California, Los Angeles Seizure
Disorder Center, Los Angeles, California
a
Dr Eliashiv receives personal compensation for serving on the speaker’s bureau of Cyberonics, Inc; Sunovion
Pharmaceuticals, Inc; and UCB, Inc and receives research/grant support as co-investigator of a study by the
Defense Advanced Research Projects Agency.
b
Dr Eliashiv discusses the unlabeled/investigational use of botulinum toxins as a generic biologic agent that can be
used with appropriate medication by physicians across all of these indications based on labeling approved by the
US Food and Drug Administration.

Jennifer G. Goldman, MD, MS, FAAN


Associate Professor, Rush University Medical Center, Department of Neurological Services,
Section of Parkinson Disease and Movement Disorders, Chicago, Illinois
a
Dr Goldman has received personal compensation for serving as a consultant for Acadia Pharmaceuticals Inc,
Pfizer Inc, and Teva Pharmaceutical Industries Ltd. Dr Goldman receives research support from Acadia
Pharmaceuticals Inc, Biotie Therapies, the Michael J. Fox Foundation, the National Institute of Neurological
Disorders and Stroke/National Institutes of Health, and Teva Pharmaceutical Industries Ltd.
b
Dr Goldman discusses the unlabeled/investigational use of antidepressants, antipsychotics, anxiolytics, and
cognitive impairment medications for the treatment of Parkinson disease, none of which are approved by the US
Food and Drug Administration except rivastigmine.

Copyright © American Academy of Neurology. Unauthorized reproduction of this article is prohibited.


Christopher W. Hess, MD
Assistant Professor of Neurology, Center for Movement Disorders and Neurorestoration,
University of Florida; Staff Neurologist, Malcom Randall VA Medical Center, Gainesville,
Florida
a
Dr Hess has received personal compensation as a speaker for the Davis Phinney Foundation and the National
Parkinson Foundation, has received grant support from the University of Florida, and has participated in CME and
educational activities on movement disorders sponsored by Allergan, Ipsen, Mertz Pharma, PeerView Press, and
QuantiaMD.
b
Dr Hess reports no disclosure.

Manju A. Kurian, MBBChir, MRCPCH, PhD


Wellcome Trust Intermediate Fellow, Honorary Consultant in Paediatric Neurology,
Developmental Neurosciences, University College London Institute of Child Health; Department
of Neurology, Great Ormond Street Hospital, London, United Kingdom
a
Dr Kurian receives research/grant support from Gracious Heart Charity and Rosetrees (IPAAK/515006), Great
Ormond Street Hospital Children’s Charity (507837), Neurodegeneration With Brain Iron Accumulation Disorders
Association (5150081), and a Wellcome Trust Intermediate Clinical Fellowship (WT098524MA).
b
Dr Kurian reports no disclosure.

Elan D. Louis, MD, MS, FAAN


Professor of Neurology and Epidemiology; Chief, Division of Movement Disorders, Yale
University School of Medicine, New Haven, Connecticut
a
Dr Louis reports no disclosure.
b
Dr Louis discusses the unlabeled/investigational use of acetazolamide, alprazolam, baclofen, carbamazepine,
clonazepam, cyclophosphamide, ethosuximide, gabapentin, phenobarbital, phenytoin, pregabalin, primidone, and
topiramate for the treatment of tremor as well as carbidopa/levodopa for the treatment of orthostatic tremor and
trihexyphenidyl for the treatment of dystonic tremor.

Nikolaus R. McFarland, MD, PhD


Assistant Professor of Neurology; Chief, Movement Disorders Division; Wright/Falls/Simmons
Director of Progressive Supranuclear Palsy and Atypical Parkinsonism Clinic, University of
Florida College of Medicine, Gainesville, Florida
a
Dr McFarland is supported by a career development grant from the National Institutes of Health/National Institute
of Neurological Disorders and Stroke (K09 NS067024) and the Michael J. Fox Foundation.
b
Dr McFarland reports no disclosure.

Tiago A. Mestre, MSc, MD


Assistant Professor, University of Ottawa; Associate Scientist, the Ottawa Hospital Research
Institute, Ottawa, Ontario, Canada
a
Dr Mestre receives personal compensation for serving on the scientific advisory board of AbbVie, for speaking
engagements for Teva Pharmaceutical Industries Ltd, and for educational events for WebMD. Dr Mestre receives
grant support from the Parkinson Study Group/Parkinson Disease Foundation.
b
Dr Mestre reports no disclosure.

Janis M. Miyasaki, MD, MEd, FRCPC, FAAN


Professor, Division of Neurology, Department of Medicine, University of Alberta, Edmonton,
Alberta, Canada
a
Dr Miyasaki received personal compensation as a consultant for Merz Pharma Group and as a lecturer for Teva
Pharmaceuticals Industries Ltd, has received research/grant support as principal investigator of studies for

Copyright © American Academy of Neurology. Unauthorized reproduction of this article is prohibited.


Parkinson Alberta and Patient-Centered Outcomes Research Institute, and receives royalties from UpToDate Inc.
b
Dr Miyasaki reports no disclosure.

John C. Morgan, MD, PhD


Associate Professor; Director National Parkinson Foundation Center of Excellence, Movement
Disorders Program, Department of Neurology, Medical College of Georgia, Augusta, Georgia
a
Dr Morgan has received personal compensation as a speaker and consultant for Impax Laboratories Inc and Teva
Pharmaceutical Industries Ltd, as a speaker for the National Parkinson Foundation, and as a consultant for
AbbVie, Acadia Pharmaceuticals, Acorda Therapeutics, Cynapsus Pharmaceuticals Inc, Lundbeck, UCB Inc, and
Veloxis Pharmaceuticals. Dr Morgan has received research/grant support as principal investigator or
subinvestigator from AbbVie, ACADIA Pharmaceuticals Inc, Biotie Therapies, Civitas Therapeutics, Kyowa Hakko
USA Inc, Lundbeck, the National Institutes of Health, National Parkinson Foundation, and the Parkinson Study
Group, and has served as an expert witness in various court cases.
b
Dr Morgan discusses the unlabeled/investigational use of amantadine and methylphenidate for freezing gait
disorders, the use of clozapine and mirtazapine for tremor in Parkinson disease, and the use of anti–α-synuclein
antibodies, caffeine, coenzyme Q10 , creatine, inosine, isradipine, nicotine, nilotinib, pioglitazone, and rasagiline as
failed or potential disease-modifying therapies.

Marc Nuwer, MD, PhD, FAAN


Professor and Vice Chair, Department of Neurology, David Geffen School of Medicine at
University of California, Los Angeles, Department Head, Department of Clinical
Neurophysiology, Ronald Reagan University of California, Los Angeles Medical Center, Los
Angeles, California
a
Dr Nuwer receives research/grant support as investigator of studies for the Congressionally Directed Medical
Research Programs (SC130209), the National Institutes of Health/National Institute of Neurological Disorders and
Stroke (1R01NS078494-01A1), and the United States Army Medical Research Acquisition Activity (11501944). Dr
Nuwer has received personal compensation for book royalties from Cambridge University Press and has provided
expert medical testimony for trials and depositions as a medical expert and treating physician.
bDr Nuwer discusses the unlabeled/investigational use of botulinum toxins as a generic biologic agent that can be
used with appropriate medication by physicians across all of these indications based on labeling approved by the
US Food and Drug Administration.

Michael S. Okun, MD, FAAN


Adelaide Lackner Professor and Chairman of Neurology, University of Florida, Gainesville,
Florida; National Medical Director, National Parkinson Foundation, Miami, Florida
a
Dr Okun serves as a consultant for the National Parkinson Foundation and has received research grants from the
Bachmann-Strauss Dystonia & Parkinson Foundation Inc, the Michael J. Fox Foundation, the National Institutes of
Health, the National Parkinson Foundation, the Parkinson Alliance, Smallwood Foundation, the Tourette
Association of America, Tyler’s Hope, and the University of Florida Foundation. Dr Okun has participated as a site
principal investigator and coinvestigator for several trials sponsored by the National Institutes of Health and other
foundations and industries, but has not received compensation. Dr Okun has received royalties from Amazon.com
Inc, Books4Patients LLC, Cambridge University Press, Demos, Manson Publishing Ltd, Smashwords, and Taylor &
Francis Group. Dr Okun serves as associate editor for the New England Journal of Medicine Journal Watch
Neurology, and has lectured in CME activities for Henry Stewart Conferences & Events, PeerView Press, Prime
Education Inc, QuantiaMD, and Vanderbilt University.
b
Dr Okun reports no disclosure.

Ronald F. Pfeiffer, MD, FAAN


Professor, Department of Neurology, Oregon Health & Science University, Portland, Oregon
a
Dr Pfeiffer received personal compensation for serving on the advisory boards of Acadia Pharmaceuticals Inc,
Lundbeck, and Pfizer Inc, as a consultant for Clintara LLC, for serving as co-editor-in-chief of Parkinsonism &
Related Disorders, and for serving on the speakers bureau of Teva Pharmaceutical Industries Ltd. Dr Pfeiffer
served on the editorial boards of the International Journal of Brain Science and the Journal of Parkinson’s Disease

Copyright © American Academy of Neurology. Unauthorized reproduction of this article is prohibited.


and receives publishing royalties from CRC Press, Taylor & Francis Group, and Humana Press/Springer. Dr
Pfeiffer served as principal investigator of a study for the Michael J. Fox Foundation, the Parkinson Study Group,
and Rhythm Pharmaceuticals, Inc.
b
Dr Pfeiffer discusses the unlabeled/investigational use of zinc for the treatment of Wilson disease.

Vicki Shanker, MD
Assistant Professor, Neurology Residency Program Director, Icahn School of Medicine at Mount
Sinai, New York, New York
a
Dr Shanker reports no disclosure.
b
Dr Shanker discusses the unlabeled/investigational use of oral medications for the treatment of dystonia, none of
which are approved by the US Food and Drug Administration.

Allan D. Wu, MD
Associate Clinical Professor, Department of Neurology, University of California, Los Angeles,
Los Angeles, California
a
Dr Wu has received personal compensation for serving on the advisory board of Merz Pharma Group, and
received research/grant support as co-investigator of a study for the Dr Miriam & Sheldon G. Adelson Medical
Research Foundation.
b
Dr Wu discusses the unlabeled/investigational use of botulinum toxins as a generic biologic agent that can be used
with appropriate medication by physicians across all of these indications based on labeling approved by the US
Food and Drug Administration.

Guangbin Xia, MD, PhD


Assistant Professor, Department of Neurology, University of Florida, Gainesville, Florida
a
Dr Xia receives research/grant support as principal investigator of studies for Acorda Therapeutics, Grand Aerie
Fraternal Order of Eagles, the National Institutes of Health (AR065836-01), ReproCell Inc, and ThermoFisher
Scientific.
b
Dr Xia reports no disclosure.

James W. M. Owens Jr, MD, PhD


Associate Professor of Neurology and Adjunct Associate Professor of Pediatrics, University of
Washington, Seattle, Washington
a
Dr Owens receives personal compensation for medicolegal record review and royalties from Up-to-date, Inc.
b
Dr Owens reports no disclosure.

Joseph Safdieh, MD, FAAN


Vice Chairman for Education; Associate Professor, Department of Neurology, Weill Cornell
Medical Center/New York–Presbyterian Hospital, New York, New York
a
Dr Safdieh receives personal compensation for providing expert legal testimony and for the development of
educational presentations for Elsevier.
b
Dr Safdieh reports no disclosure.
a
Relationship Disclosure
b
Unlabeled Use of Products/Investigational Use Disclosure

Copyright © American Academy of Neurology. Unauthorized reproduction of this article is prohibited.


Methods of Participation and Instructions for Use

Continuum: Lifelong Learning in Neurology® is designed to help practicing neurologists stay

abreast of advances in the field while simultaneously developing lifelong self-directed learning

skills. In Continuum, the process of absorbing, integrating, and applying the material presented is

as important as, if not more important than, the material itself.

The goals of Continuum include disseminating up-to-date information to the practicing

neurologist in a lively, interactive format; fostering self-assessment and lifelong study skills;

encouraging critical thinking; and, in the final analysis, strengthening and improving patient

care.

Each Continuum issue is prepared by distinguished faculty who are acknowledged leaders in

their respective fields. Six issues are published annually and are composed of review articles,

case-based discussions on ethical and practice issues related to the issue topic, coding

information, , and comprehensive CME and self-assessment offerings, including a self-

assessment pretest, multiple-choice questions with preferred responses, and a patient

management problem. For detailed instructions regarding Continuum CME and self-assessment

activities, visit aan.com/continuum/cme.

The review articles emphasize clinical issues emerging in the field in recent years. Case reports

and vignettes are used liberally, as are tables and illustrations. Video material relating to the

issue topic accompanies issues when applicable.

The text can be reviewed and digested most effectively by establishing a regular schedule of

study in the office or at home, either alone or in an interactive group. If subscribers use such

regular and perhaps new study habits, Continuum’s goal of establishing lifelong learning patterns

can be met.

Copyright © American Academy of Neurology. Unauthorized reproduction of this article is prohibited.


LIFELONG LEARNING IN NEUROLOGY ®

Movement Disorders
Volume 22 Number 4 August 2016

CONTRIBUTORS
Susan H. Fox, MBChB, MRCP(UK), PhD, Guest Editor
Associate Professor of Neurology, University of Toronto; Associate Director,
Movement Disorder Clinic, Toronto Western Hospital, University Health
Network, Toronto, Ontario, Canada
aDr Fox has received salary support for serving as the co-editor of the International
Parkinson and Movement Disorder Society website, as a speaker for Ipsen Pharmaceuticals,
Inc; Teva Pharmaceutical Industries Ltd; and Zambon Company SpA; and for serving on the
advisory boards of Lundbeck, Orion Pharma Ltd, and Novartis International AG. Dr Fox has
received research/grant support from Avanir Pharmaceuticals, the Michael J. Fox Foundation,
the National Institutes of Health, and Parkinson Society of Canada and has received
research/grant support as site principal investigator for clinical studies from Adamas
Pharmaceuticals Inc, Cynapsus Pharmaceuticals Inc, and Kyowa Hakko USA Inc. Dr Fox
receives royalties from Oxford University Press.
bDr Fox discusses the unlabeled/investigational use of amantadine and methylphenidate for
freezing gait disorders, the use of clozapine and mirtazapine for tremor in Parkinson disease,
and the use of anti–α-synuclein antibodies, caffeine, coenzyme Q10 , creatine, inosine,
isradipine, nicotine, nilotinib, pioglitazone, and rasagiline as failed or potential
disease-modifying therapies.

Tetsuo Ashizawa, MD, FAAN


Director, Neurosciences Research Program, Houston Methodist Hospital, Houston, Texas
aDr Ashizawa receives research/grant support as principal investigator of studies for Ionis
Pharmaceuticals, Inc (1515598769-CS2); the Myotonic Dystrophy Foundation; and the
National Institutes of Health (NS083564).
bDr Ashizawa reports no disclosure.

Susan B. Bressman, MD, FAAN


Mirken Chair, Departments of Neurology, Mount Sinai Beth Israel, Mount Sinai
St. Luke’s, and Mount Sinai West; Professor of Neurology, Icahn School of
Medicine at Mount Sinai, New York, New York
aDr Bressman receives personal compensation for serving on the scientific advisory boards
of the Bachmann-Strauss Dystonia & Parkinson Foundation, Inc and the Michael J. Fox
Foundation and for serving on the editorial board of Movement Disorders. Dr Bressman
receives research/grant support from the Michael J. Fox Foundation.
bDr Bressman discusses the unlabeled/investigational use of oral medications for the
treatment of dystonia, none of which are approved by the US Food and Drug Administration.

aRelationship Disclosure
bUnlabeled Use of Products/Investigational Use Disclosure

Continuum (Minneap Minn) 2016;22(4) www.ContinuumJournal.com

Copyright © American Academy of Neurology. Unauthorized reproduction of this article is prohibited.


LIFELONG LEARNING IN NEUROLOGY ®

CONTRIBUTORS continued
Thomas I. Cochrane, MD, MBA
Associate Neurologist, Senior Ethics Consultant, Brigham and Women’s
Hospital; Assistant Professor of Neurology, Director of Neuroethics, Center for
Bioethics, Harvard Medical School, Boston, Massachusetts
aDr Cochrane serves as a consultant for Merck & Co, Inc.
bDr Cochrane reports no disclosure.

Russell C. Dale, MBChB, MRCP, PhD


Professor of Paediatric Neurology, University of Sydney, Sydney, Australia
aDr Dale has received personal compensation for serving on the editorial advisory board of
Multiple Sclerosis and Related Disorders and the editorial boards of the European Journal
of Paediatric Neurology and Neurology: Neuroimmunology & Neuroinflammation, and
on the scientific advisory board for the Queensland Children’s Medical Research Institute.
Dr Dale has received honoraria for speaking engagements as well as publishing royalties
from Biogen Inc and Bristol-Myers-Squibb and has received research support from Multiple
Sclerosis Research Australia and the National Health and Medical Research Council.
bDr Dale reports no disclosure.

Dawn Eliashiv, MD, FAAN


Professor of Neurology, Department of Neurology, David Geffen School of
Medicine at University of California, Los Angeles; Codirector, University of
California, Los Angeles Seizure Disorder Center, Los Angeles, California
aDr Eliashiv receives personal compensation for serving on the speaker’s bureau of
Cyberonics, Inc; Sunovion Pharmaceuticals, Inc; and UCB, Inc and receives research/grant
support as co-investigator of a study by the Defense Advanced Research Projects Agency.
bDr Eliashiv discusses the unlabeled/investigational use of botulinum toxins as a generic
biologic agent that can be used with appropriate medication by physicians across all of these
indications based on labeling approved by the US Food and Drug Administration.

Jennifer G. Goldman, MD, MS, FAAN


Associate Professor, Rush University Medical Center, Department of
Neurological Services, Section of Parkinson Disease and Movement Disorders,
Chicago, Illinois
aDr Goldman has received personal compensation for serving as a consultant for Acadia
Pharmaceuticals Inc, Pfizer Inc, and Teva Pharmaceutical Industries Ltd. Dr Goldman receives
research support from Acadia Pharmaceuticals Inc, Biotie Therapies, the Michael J. Fox
Foundation, the National Institute of Neurological Disorders and Stroke/National Institutes
of Health, and Teva Pharmaceutical Industries Ltd.
bDr Goldman discusses the unlabeled/investigational use of antidepressants, antipsychotics,
anxiolytics, and cognitive impairment medications for the treatment of Parkinson disease,
none of which are approved by the US Food and Drug Administration except rivastigmine.

aRelationship Disclosure
bUnlabeled Use of Products/Investigational Use Disclosure

www.ContinuumJournal.com August 2016

Copyright © American Academy of Neurology. Unauthorized reproduction of this article is prohibited.


LIFELONG LEARNING IN NEUROLOGY ®

CONTRIBUTORS continued
Christopher W. Hess, MD
Assistant Professor of Neurology, Center for Movement Disorders and
Neurorestoration, University of Florida; Staff Neurologist, Malcom Randall VA
Medical Center, Gainesville, Florida
aDr Hess has received personal compensation as a speaker for the Davis Phinney
Foundation and the National Parkinson Foundation, has received grant support from the
University of Florida, and has participated in CME and educational activities on movement
disorders sponsored by Allergan, Ipsen, Mertz Pharma, PeerView Press, and QuantiaMD.
bDr Hess reports no disclosure.

Manju A. Kurian, MBBChir, MRCPCH, PhD


Wellcome Trust Intermediate Fellow, Honorary Consultant in Paediatric Neurology,
Developmental Neurosciences, University College London Institute of Child Health;
Department of Neurology, Great Ormond Street Hospital, London, United Kingdom
aDr Kurian receives research/grant support from Gracious Heart Charity and Rosetrees
(IPAAK/515006), Great Ormond Street Hospital Children’s Charity (507837),
Neurodegeneration With Brain Iron Accumulation Disorders Association (5150081), and a
Wellcome Trust Intermediate Clinical Fellowship (WT098524MA).
bDr Kurian reports no disclosure.

Elan D. Louis, MD, MS, FAAN


Professor of Neurology and Epidemiology; Chief, Division of Movement
Disorders, Yale University School of Medicine, New Haven, Connecticut
aDr Louis reports no disclosure.
bDr Louis discusses the unlabeled/investigational use of acetazolamide, alprazolam,
baclofen, carbamazepine, clonazepam, cyclophosphamide, ethosuximide, gabapentin,
phenobarbital, phenytoin, pregabalin, primidone, and topiramate for the treatment of
tremor as well as carbidopa/levodopa for the treatment of orthostatic tremor and
trihexyphenidyl for the treatment of dystonic tremor.

Nikolaus R. McFarland, MD, PhD


Assistant Professor of Neurology; Chief, Movement Disorders Division;
Wright/Falls/Simmons Director of Progressive Supranuclear Palsy and Atypical
Parkinsonism Clinic, University of Florida College of Medicine, Gainesville, Florida
aDr McFarland is supported by a career development grant from the National Institutes of
Health/National Institute of Neurological Disorders and Stroke (K09 NS067024) and the
Michael J. Fox Foundation.
bDr McFarland reports no disclosure.

aRelationship Disclosure
bUnlabeled Use of Products/Investigational Use Disclosure

Continuum (Minneap Minn) 2016;22(4) www.ContinuumJournal.com

Copyright © American Academy of Neurology. Unauthorized reproduction of this article is prohibited.


LIFELONG LEARNING IN NEUROLOGY ®

CONTRIBUTORS continued
Tiago A. Mestre, MSc, MD
Assistant Professor, University of Ottawa; Associate Scientist, the Ottawa
Hospital Research Institute, Ottawa, Ontario, Canada
aDr Mestre receives personal compensation for serving on the scientific advisory board of
AbbVie, for speaking engagements for Teva Pharmaceutical Industries Ltd, and for
educational events for WebMD. Dr Mestre receives grant support from the Parkinson Study
Group/Parkinson Disease Foundation.
bDr Mestre reports no disclosure.

Janis M. Miyasaki, MD, MEd, FRCPC, FAAN


Professor, Division of Neurology, Department of Medicine, University of
Alberta, Edmonton, Alberta, Canada
aDr Miyasaki received personal compensation as a consultant for Merz Pharma Group and
as a lecturer for Teva Pharmaceuticals Industries Ltd, has received research/grant support as
principal investigator of studies for Parkinson Alberta and Patient-Centered Outcomes
Research Institute, and receives royalties from UpToDate Inc.
bDr Miyasaki reports no disclosure.

John C. Morgan, MD, PhD


Associate Professor; Director National Parkinson Foundation Center of Excellence,
Movement Disorders Program, Department of Neurology, Medical College of
Georgia, Augusta, Georgia
aDr Morgan has received personal compensation as a speaker and consultant for Impax
Laboratories Inc and Teva Pharmaceutical Industries Ltd, as a speaker for the National
Parkinson Foundation, and as a consultant for AbbVie, Acadia Pharmaceuticals, Acorda
Therapeutics, Cynapsus Pharmaceuticals Inc, Lundbeck, UCB Inc, and Veloxis
Pharmaceuticals. Dr Morgan has received research/grant support as principal investigator or
subinvestigator from AbbVie, ACADIA Pharmaceuticals Inc, Biotie Therapies, Civitas
Therapeutics, Kyowa Hakko USA Inc, Lundbeck, the National Institutes of Health, National
Parkinson Foundation, and the Parkinson Study Group, and has served as an expert witness
in various court cases.
bDr Morgan discusses the unlabeled/investigational use of amantadine and methylphenidate
for freezing gait disorders, the use of clozapine and mirtazapine for tremor in Parkinson
disease, and the use of anti–α-synuclein antibodies, caffeine, coenzyme Q10 , creatine,
inosine, isradipine, nicotine, nilotinib, pioglitazone, and rasagiline as failed or potential
disease-modifying therapies.

Marc Nuwer, MD, PhD, FAAN


Professor and Vice Chair, Department of Neurology, David Geffen School of
Medicine at University of California, Los Angeles, Department Head, Department
of Clinical Neurophysiology, Ronald Reagan University of California, Los Angeles
Medical Center, Los Angeles, California
aDr Nuwer receives research/grant support as investigator of studies for the Congressionally
Directed Medical Research Programs (SC130209), the National Institutes of Health/National
Institute of Neurological Disorders and Stroke (1R01NS078494-01A1), and the United States
Army Medical Research Acquisition Activity (11501944). Dr Nuwer has received personal
compensation for book royalties from Cambridge University Press and has provided expert
medical testimony for trials and depositions as a medical expert and treating physician.
bDr Nuwer discusses the unlabeled/investigational use of botulinum toxins as a generic
biologic agent that can be used with appropriate medication by physicians across all of these
indications based on labeling approved by the US Food and Drug Administration.
aRelationship Disclosure
bUnlabeled Use of Products/Investigational Use Disclosure

www.ContinuumJournal.com August 2016

Copyright © American Academy of Neurology. Unauthorized reproduction of this article is prohibited.


LIFELONG LEARNING IN NEUROLOGY ®

CONTRIBUTORS continued
Michael S. Okun, MD, FAAN
Adelaide Lackner Professor and Chairman of Neurology, University of Florida,
Gainesville, Florida; National Medical Director, National Parkinson Foundation,
Miami, Florida
aDr Okun serves as a consultant for the National Parkinson Foundation and has received
research grants from the Bachmann-Strauss Dystonia & Parkinson Foundation Inc, the
Michael J. Fox Foundation, the National Institutes of Health, the National Parkinson
Foundation, the Parkinson Alliance, Smallwood Foundation, the Tourette Association of
America, Tyler’s Hope, and the University of Florida Foundation. Dr Okun has participated as
a site principal investigator and coinvestigator for several trials sponsored by the National
Institutes of Health and other foundations and industries, but has not received compensation.
Dr Okun has received royalties from Amazon.com Inc, Books4Patients LLC, Cambridge
University Press, Demos, Manson Publishing Ltd, Smashwords, and Taylor & Francis Group.
Dr Okun serves as associate editor for the New England Journal of Medicine Journal Watch
Neurology, and has lectured in CME activities for Henry Stewart Conferences & Events,
PeerView Press, Prime Education Inc, QuantiaMD, and Vanderbilt University.
bDr Okun reports no disclosure.

Ronald F. Pfeiffer, MD, FAAN


Professor, Department of Neurology, Oregon Health & Science University,
Portland, Oregon
aDr Pfeiffer received personal compensation for serving on the advisory boards of Acadia
Pharmaceuticals Inc, Lundbeck, and Pfizer Inc, as a consultant for Clintara LLC, for serving as
co-editor-in-chief of Parkinsonism & Related Disorders, and for serving on the speakers
bureau of Teva Pharmaceutical Industries Ltd. Dr Pfeiffer served on the editorial boards of
the International Journal of Brain Science and the Journal of Parkinson’s Disease and
receives publishing royalties from CRC Press, Taylor & Francis Group, and Humana
Press/Springer. Dr Pfeiffer served as principal investigator of a study for the Michael J. Fox
Foundation, the Parkinson Study Group, and Rhythm Pharmaceuticals, Inc.
bDr Pfeiffer discusses the unlabeled/investigational use of zinc for the treatment of
Wilson disease.

Vicki Shanker, MD
Assistant Professor, Neurology Residency Program Director, Icahn School of
Medicine at Mount Sinai, New York, New York
aDr Shanker reports no disclosure.
bDr Shanker discusses the unlabeled/investigational use of oral medications for the
treatment of dystonia, none of which are approved by the US Food and Drug Administration.

Allan D. Wu, MD
Associate Clinical Professor, Department of Neurology, University of California,
Los Angeles, Los Angeles, California
aDr Wu has received personal compensation for serving on the advisory board of Merz
Pharma Group, and received research/grant support as co-investigator of a study for the
Dr Miriam & Sheldon G. Adelson Medical Research Foundation.
bDr Wu discusses the unlabeled/investigational use of botulinum toxins as a generic biologic
agent that can be used with appropriate medication by physicians across all of these
indications based on labeling approved by the US Food and Drug Administration.

aRelationship Disclosure
bUnlabeled Use of Products/Investigational Use Disclosure

Continuum (Minneap Minn) 2016;22(4) www.ContinuumJournal.com

Copyright © American Academy of Neurology. Unauthorized reproduction of this article is prohibited.


LIFELONG LEARNING IN NEUROLOGY ®

CONTRIBUTORS continued
Guangbin Xia, MD, PhD
Assistant Professor, Department of Neurology, University of Florida,
Gainesville, Florida
aDr Xia receives research/grant support as principal investigator of studies for Acorda
Therapeutics, Grand Aerie Fraternal Order of Eagles, the National Institutes of Health
(AR065836-01), ReproCell Inc, and ThermoFisher Scientific.
bDr Xia reports no disclosure.

SELF-ASSESSMENT AND CME TEST WRITERS


James W. M. Owens Jr, MD, PhD
Associate Professor of Neurology and Adjunct Associate Professor of Pediatrics,
University of Washington, Seattle, Washington
aDr Owens receives personal compensation for medicolegal record review and royalties from
Up-to-date, Inc.
bDr Owens reports no disclosure.

Joseph Safdieh, MD, FAAN


Vice Chairman for Education; Associate Professor, Department of Neurology, Weill
Cornell Medical Center/New York–Presbyterian Hospital, New York, New York
aDr Safdieh receives personal compensation for providing expert legal testimony and for the
development of educational presentations for Elsevier.
bDr Safdieh reports no disclosure.

aRelationship Disclosure
bUnlabeled Use of Products/Investigational Use Disclosure

www.ContinuumJournal.com August 2016

Copyright © American Academy of Neurology. Unauthorized reproduction of this article is prohibited.


Volume 22 n Number 4 n August 2016

LIFELONG LEARNING IN NEUROLOGY ®


www.ContinuumJournal.com

Movement Disorders Denotes Video Content

Guest Editor: Susan H. Fox, MBChB, MRCP(UK), PhD Denotes Supplemental Digital Content

Editor’s Preface . . . . . . . . . . . . . . . . . . . . . . . . . . . . . . . . . . . . . . . . . . . . . . . . . . . . . . . 1045

REVIEW ARTICLES
Diagnosing Parkinson Disease . . . . . . . . . . . . . . . . . . . . . . . . . . . . . . . . . . . . . . . . . . 1047
Christopher W. Hess, MD; Michael S. Okun, MD, FAAN
Treating the Motor Symptoms of Parkinson Disease . . . . . . . . . . . . . . . . . . . . . 1064
John C. Morgan, MD, PhD; Susan H. Fox, MBChB, MRCP(UK), PhD
Neuropsychiatric Issues in Parkinson Disease . . . . . . . . . . . . . . . . . . . . . . . . . . 1086
Jennifer G. Goldman, MD, MS, FAAN
Treatment of Advanced Parkinson Disease and Related Disorders . . . . . . . 1104
Janis M. Miyasaki, MD, MEd, FRCPC, FAAN
Diagnostic Approach to Atypical Parkinsonian Syndromes . . . . . . . . . . . . . . 1117
Nikolaus R. McFarland, MD, PhD
Diagnosis and Management of Tremor . . . . . . . . . . . . . . . . . . . . . . . . . . . . . . . . . 1143
Elan D. Louis, MD, MS, FAAN
Movement Disorders Presenting in Childhood . . . . . . . . . . . . . . . . . . . . . . . . . 1159
Manju A. Kurian, MBBChir, MRCPCH, PhD; Russell C. Dale, MBChB, MRCP, PhD
Chorea . . . . . . . . . . . . . . . . . . . . . . . . . . . . . . . . . . . . . . . . . . . . . . . . . . . . . . . . . . . . . . . 1186
Tiago A. Mestre, MSc, MD
Ataxia . . . . . . . . . . . . . . . . . . . . . . . . . . . . . . . . . . . . . . . . . . . . . . . . . . . . . . . . . . . . . . . 1208
Tetsuo Ashizawa, MD, FAAN; Guangbin Xia, MD, PhD
Diagnosis and Management of Dystonia . . . . . . . . . . . . . . . . . . . . . . . . . . . . . . . 1227
Vicki Shanker, MD; Susan B. Bressman, MD, FAAN
Wilson Disease. . . . . . . . . . . . . . . . . . . . . . . . . . . . . . . . . . . . . . . . . . . . . . . . . . . . . . . . 1246
Ronald F. Pfeiffer, MD, FAAN

Volume 22 n Number 4 www.ContinuumJournal.com 1041

Copyright © American Academy of Neurology. Unauthorized reproduction of this article is prohibited.


LIFELONG LEARNING IN NEUROLOGY ®

ETHICAL ISSUES
Ethics of Preclinical Dopamine Transporter Imaging . . . . . . . . . . . . . . . . . . . . 1262
Thomas I. Cochrane, MD, MBA

PRACTICE ISSUES
Chemodenervation Coding for Neurologists . . . . . . . . . . . . . . . . . . . . . . . . . . . 1266
Allan D. Wu, MD; Dawn Eliashiv, MD, FAAN; Marc Nuwer, MD, PhD, FAAN

SELF-ASSESSMENT AND CME


Learning Objectives . . . . . . . . . . . . . . . . . . . . . . . . . . . . . . . . . . . . . . . . . . . . . . . . . . 1043
Instructions for Completing Postreading Self-Assessment and CME Test
and Tally Sheet . . . . . . . . . . . . . . . . . . . . . . . . . . . . . . . . . . . . . . . . . . . . . . . . . . . . . . . 1279
Postreading Self-Assessment and CME Test . . . . . . . . . . . . . . . . . . . . . . . . . . . . 1281
Postreading Self-Assessment and CME Test—Preferred Responses . . . . . . 1295
James W. M. Owens Jr, MD, PhD; Joseph E. Safdieh, MD, FAAN
Patient Management Problem . . . . . . . . . . . . . . . . . . . . . . . . . . . . . . . . . . . . . . . . . 1321
Patient Management Problem—Preferred Responses . . . . . . . . . . . . . . . . . . . 1327
Susan H. Fox, MBChB, MRCP(UK), PhD

Index . . . . . . . . . . . . . . . . . . . . . . . . . . . . . . . . . . . . . . . . . . . . . . . . . . . . . . . . . . . . . . . . 1337
List of Abbreviations . . . . . . . . . . . . . . . . . . . . . . . . . . . . . . . . . . . . . . . . . . . .Back Cover

1042 www.ContinuumJournal.com August 2016

Copyright © American Academy of Neurology. Unauthorized reproduction of this article is prohibited.


Learning Objectives
Upon completion of this Continuum: Lifelong Learning in Neurology
Movement Disorders issue, participants will be able to:

s Differentiate the clinical presentation of Parkinson disease from its mimics and
other forms of parkinsonism and describe the appropriate ancillary tests and
the indications for their use
Discuss the various pharmacologic and nonpharmacologic strategies for treating
s

early and advanced motor symptoms of Parkinson disease


Recognize and manage neuropsychiatric problems in patients with
s

Parkinson disease
Manage and counsel patients (and their families) affected by advanced
s

Parkinson disease
Diagnose and manage atypical parkinsonian syndromes
s

Diagnose and manage tremor disorders


s

Discuss the broad spectrum of movement disorders that may present in


s

childhood and their management strategies


Diagnose and manage Huntington disease and other causes of chorea
s

Diagnose and manage the spectrum of disorders in which ataxia is a


s

prominent sign
Recognize key clinical features of dystonia, categorize dystonia using the new
s

classification schema, and describe the approach to treatment


Recognize the many clinical presentations of Wilson disease, perform the
s

appropriate investigations to expeditiously diagnose the disease, and discuss


the appropriate treatments of this condition
Discuss the ethics of ordering dopamine transporter imaging in asymptomatic
s

patients concerned about developing Parkinson disease

Core Competencies
This Continuum: Lifelong Learning in Neurology Movement Disorders issue
covers the following core competencies:

Patient Care and Procedural Skills


s

Medical Knowledge
s

Practice-Based Learning and Improvement


s

Interpersonal and Communication Skills


s

Professionalism
s

Systems-Based Practice
s

Continuum (Minneap Minn) 2016;22(4) www.ContinuumJournal.com

Copyright © American Academy of Neurology. Unauthorized reproduction of this article is prohibited.


Copyright © American Academy of Neurology. Unauthorized reproduction of this article is prohibited.
Editor’s Preface

* 2016 American Academy

A Kinetic and of Neurology.

Nonrigid Approach to
Movement Disorders

This issue of Continuum tailor treatment strategies


is devoted to the diagnosis to the individual needs
and management of our of our patients. Next,
patients with movement Dr Jennifer G. Goldman
disorders. Dr Susan H. Fox, summarizes the diag-
associate professor of neu- nosis and current ap-
rology at the University of proaches to management
Toronto and associate di- of the neuropsychiatric
rector of the Movement issues that frequently
Disorder Clinic at Toronto occur in patients with
Western Hospital, has PD, including disorders
brought together an out- of mood, cognition, and
standing group of experts behavior. Dr Janis M.
in the field of movement Dr Susan H. Fox has Miyasaki then reviews
disorders who provide brought together an the recognition and man-
their authoritative but outstanding group of agement of the many
flexible approaches to the experts in the field of nonmotor symptoms
diagnosis and manage- movement disorders that may occur in PD
ment of each of the many who provide their and related movement
disorders of movement authoritative but disorders, highlighting
we may encounter in the importance of these
flexible approaches
clinical practice. symptoms as they affect
to the diagnosis and
The issue begins with the the quality of life of our
management of each
article by Drs Christopher patients, especially as
of the many disorders their diseases advance.
W. Hess and Michael S.
Okun who review the ap-
of movement we Dr Miyasaki also stresses
proach to the clinical diag- may encounter in the importance of the
nosis of Parkinson disease clinical practice. neurologist being in-
(PD), emphasizing how the volved and engaged in
neurologic history and examination, symptom management and palliative
informed by knowledge of clinical diag- approaches to our patients with ad-
nostic criteria, can help establish PD or vanced disease.
suggest alternative diagnoses. Drs John C. Moving on from the articles primar-
Morgan and Susan H. Fox then discuss ily about PD, Dr Nikolaus R. McFarland
the various current pharmacologic and reviews the diagnosis and manage-
nonpharmacologic approaches to treat- ment of the atypical parkinsonian
ing the motor symptoms of PD, using syndromes, including progressive
prototypical case examples to help in- supranuclear palsy, multiple system
form us about how to optimize and atrophy, and corticobasal degeneration,
Continuum (Minneap Minn) 2016;22(4):1045–1046 www.ContinuumJournal.com 1045

Copyright © American Academy of Neurology. Unauthorized reproduction of this article is prohibited.


Editor’s Preface

and addresses how their distinctive with a family history of PD who is re-
(although overlapping) clinical features questing such testing. Drs Allan D. Wu,
can help distinguish them from PD. Dawn Eliashiv, and Marc Nuwer pres-
Dr Elan D. Louis next discusses the clin- ent two cases to illustrate their discus-
ical approach to the categorization, di- sion of the various considerations
agnosis, and management of patients neurologists need to be aware of with
with any of the various tremor disorders regard to diagnostic coding for
who may present for our care. chemodenervation procedures.
Drs Manju A. Kurian and Russell C. As with every issue of Continuum,
Dale provide an overview of the a number of opportunities exist for
diagnosis and management of the CME. By taking the Postreading Self-
multitude of acquired and genetic Assessment and CME Test, written by
movement disorders that may present Drs James W. M. Owens Jr and Joseph
in childhood. Turning back toward E. Safdieh, after reading the issue, you
adults, Dr Tiago A. Mestre reviews the may earn up to 12 AMA PRA Category 1
approach to the diagnosis and manage- CreditsTM toward self-assessment and
ment of patients with disorders CME. The Patient Management Prob-
manifesting with chorea as a clinical lem, written by Dr Fox, describes the
sign, including Huntington disease case of a 70-year-old woman who pre-
and the many other genetic and ac- sents with a 6-month history of abnor-
quired choreic syndromes. Drs Tetsuo mal mouth and truncal movements
Ashizawa and Guangbin Xia next dis- and vocalizations. By following her
cuss their approach to the clinical as- case and answering multiple-choice
sessment, classification, diagnosis, and questions corresponding to diagnostic
laboratory investigation of patients with and management decision points along
ataxia. In the subsequent article, Drs the course of her disorder, you will
Vicki Shanker and Susan B. Bressman have the opportunity to earn up to 2
review the clinical phenomenology, AMA PRA Category 1 CME Credits.
classification, and the diagnosis and I want to sincerely thank Dr Fox for
treatment of patients with dystonia. her thoughtfulness, dedication, re-
In the final review article of the sponsiveness, collegiality, and hard
issue, Dr Ronald F. Pfeiffer discusses work beginning from the develop-
the most current approaches to the ment and organization of this issue
diagnosis and treatment of Wilson and culminating in its publication.
disease, a disorder that is so important Thanks also to each of the expert
to keep in mind when seeing any authors in this issue for providing the
patient with unexplained neurologic readers of Continuum with the bene-
or psychiatric (or hepatic) dysfunction fit of their clinical experience and
since it is treatable and reversible if expertise so that we can use their
caught early but progressive and irre- insights to inform our own individual-
versible if not. ized approaches to the clinical diag-
In the Ethical Issues article, Dr nosis and treatment of our patients
Thomas I. Cochrane dissects the eth- with movement disorders.
ical considerations involved in the
decision whether to order preclinical
dopamine transporter imaging in the VSteven L. Lewis, MD, FAAN
hypothetical case of a 52-year-old man Editor-in-Chief

1046 www.ContinuumJournal.com August 2016

Copyright © American Academy of Neurology. Unauthorized reproduction of this article is prohibited.


Review Article

Diagnosing Parkinson
Address correspondence to
Dr Christopher W. Hess,
University of Florida Health
Center for Movement

Disease Disorders and


Neurorestoration, 3450 Hull
Rd, Gainesville, FL 32607,
Christopher.Hess@neurology.
Christopher W. Hess, MD; Michael S. Okun, MD, FAAN ufl.edu.
Relationship Disclosure:
Dr Hess has received personal
ABSTRACT compensation as a speaker
for the Davis Phinney
Purpose of Review: While establishing the diagnosis of Parkinson disease (PD) can Foundation and the National
be straightforward, it can be challenging in some patients, even for the experienced Parkinson Foundation, has
neurologist. The misdiagnosis rate ranges from 10% to 20% or greater depending received grant support from
the University of Florida,
on clinician experience. and has participated in CME
Recent Findings: Despite promise in the search for a biomarker that can establish and educational activities
the presence of PD and act as a marker of its progression, the diagnosis of PD continues on movement disorders
sponsored by Allergan, Ipsen,
to be based on clinical examination. Core criteria, exclusion criteria, and suppor- Mertz Pharma, PeerView
tive criteria have been developed to aid the clinician in establishing the diagnosis. Press, and QuantiaMD.
Nonmotor symptoms of PD are usually present at the time of diagnosis, may precede Dr Okun serves as a consultant
for the National Parkinson
motor symptoms, and should be specifically sought during evaluation. Ancillary testing Foundation and has received
can be appropriate, but its indications and utility must be clearly understood. research grants from the
Summary: The diagnosis of PD requires the recognition of the core features of PD Bachmann-Strauss Dystonia &
Parkinson Foundation Inc,
and the differentiation of its clinical presentation from other entities with similar and the Michael J. Fox Foundation,
potentially overlapping symptoms. A careful history and examination guided by the National Institutes of
clinical diagnostic criteria will usually establish the diagnosis of PD or uncover red Health, the National Parkinson
Foundation, the Parkinson
flags for the possibilities of other diagnoses. Appropriate selection and interpretation Alliance, Smallwood Foundation,
of ancillary testing is critical to avoid misdiagnosis and unnecessary tests. the Tourette Association of
America, Tyler’s Hope, and the
University of Florida Foundation.
Continuum (Minneap Minn) 2016;22(4):1047–1063. Dr Okun has participated as a
site principal investigator and
coinvestigator for several trials
sponsored by the National
INTRODUCTION diagnostic pitfalls. This article focuses Institutes of Health and other
on establishing the clinical diagnosis of foundations and industries, but
Parkinson disease (PD) is the second has not received compensation.
most common neurodegenerative dis- PD and discusses the evolving defini- Dr Okun has received royalties
order after Alzheimer disease. Typically, tion of PD. The article also addresses from Amazon.com Inc,
Books4Patients LLC, Cambridge
PD develops in the fifth and sixth de- when PD occurs (premotor and motor), University Press, Demos,
cades. The disease affects 1% of persons the potential for biomarkers, genetic Manson Publishing Ltd,
Smashwords, and Taylor &
over the age of 60, and up to 60,000 testing, and the role of neuroimaging in Francis Group. Dr Okun serves
new cases are diagnosed each year in the diagnosis. as associate editor for the New
England Journal of Medicine
the United States.1,2 While establishing Journal Watch Neurology, and
the diagnosis of PD in many patients is MOTOR SYMPTOMS has lectured in CME activities for
Henry Stewart Conferences &
relatively straightforward, in some pa- Parkinsonism refers to the constellation Events, PeerView Press, Prime
tients it can be quite challenging, even of bradykinesia, rigidity, resting tremor, Education Inc, QuantiaMD,
for the experienced neurologist. The and postural and gait impairments. and Vanderbilt University.
Unlabeled Use of
rate of misdiagnosis of PD likely ranges While parkinsonism can have a variety Products/Investigational
from 10% to 20% or more, depending of etiologies (Table 1-1), the most com- Use Disclosure:
Drs Hess and Okun report
on the degree of specialization of the mon cause of parkinsonism is idio- no disclosures.
diagnosing physician.3,4 Being able to pathic PD. The motor symptoms are * 2016 American Academy
identify the core features of the disease secondary to the progressive loss of do- of Neurology.
as well as to evaluate the more nu- paminergic neurons in the pars compacta
anced characteristics that typify PD and of the substantia nigra.5 These neurons
its mimics can be helpful in avoiding project to the striatum (caudate and

Continuum (Minneap Minn) 2016;22(4):1047–1063 www.ContinuumJournal.com 1047

Copyright © American Academy of Neurology. Unauthorized reproduction of this article is prohibited.


Diagnosing Parkinson Disease

KEY POINTS
h Parkinsonism is a clinical TABLE 1-1 Classification of Parkinsonism
syndrome that can have a
variety of possible causes, b Idiopathic parkinsonism (Parkinson disease, including sporadic and
only one of which is genetic cases)
Parkinson disease.
b Atypical parkinsonian syndromes (eg, multiple system atrophy, progressive
h There are no clinically supranuclear palsy, corticobasal syndrome)
available biomarkers to
b Heredodegenerative parkinsonism (usually other additional neurologic
indicate the presence of
symptoms are present) (eg, PLA2G6-associated neurodegeneration,
Parkinson disease or track
aceruloplasminemia, X-linked dystonia-parkinsonism, spinocerebellar ataxias)
disease progression.
b Secondary parkinsonism (drug-induced, vascular, structural, infectious,
h The diagnosis of immunologic, toxic, traumatic, metabolic)
Parkinson disease
during the patient’s
lifetime is based on the
clinical examination. putamen) and modulate corticostriatal American Academy of Neurology (AAN),
h Core criteria, exclusion transmission. PD must be distin- the most common clinical criteria used
criteria, and supportive guished from many other forms of to establish the diagnosis of idiopathic
criteria have been parkinsonism.6 Additional or atypical PD is the UK Parkinson’s Disease Society
established that can neurologic symptoms as well as key Brain Bank (UKPDSBB) clinical diag-
assist in the diagnosis aspects of the reported history can nostic criteria (Table 1-2).8 These cri-
of Parkinson disease. help point to the correct diagnosis. teria provide essential clinical findings
Despite a concerted and continued as well as supportive and exclusion-
effort to develop a biomarker that can ary criteria for diagnosing PD.
accurately confirm or refute the pres-
ence of PD and monitor disease pro- Bradykinesia
gression, none has been found. Thus, The first and most important step in
the diagnosis of PD during the patient’s diagnosing PD is to establish the pres-
lifetime is based on a careful clinical ence of bradykinesia, or slowness of
examination. Practically speaking, this movement. While akinesia (the failure
is the most important criterion as few or delay in execution of a purposeful
patients diagnosed with PD undergo movement) and hypokinesia (a de-
postmortem neuropathologic evalua- crease in movement size) are distinct
tion.7 Recently, the use of clinical criteria entities from the motor control stand-
as the practical gold standard for the point, for the purposes of the clinical
diagnosis of PD has been questioned, examination they are often included
and the manner in which PD is best under the umbrella term of bradykinesia,
defined continues to be a current topic especially when describing the de-
of interest, as the clinical diagnosis of crease and slowing of movements as
PD does not always match the patho- well as the loss or decrease of automatic
logic diagnosis (especially in genetic movements that occur in PD.9 With
forms of PD, in which the presence of continued movement, true bradykinesia
pathologic gene mutations is compli- typically gets worse (movements be-
cated by incomplete penetrance).7 come progressively smaller or decre-
These detailed discussions and debates mental), a feature that is required for
are outside the scope of this review, the UKPDSBB criteria and more recent
and this article focuses on diagnosing European guidelines.8,10 Common early
PD for the practicing clinician. manifestations of bradykinesia in PD
Although no clinical diagnostic include decreased facial expression
criteria are formally endorsed by the (hypomimia), soft speech (hypophonia),

1048 www.ContinuumJournal.com August 2016

Copyright © American Academy of Neurology. Unauthorized reproduction of this article is prohibited.


TABLE 1-2 UK Parkinson’s Disease Society Brain Bank Clinical
Diagnostic Criteriaa

b Step 1: Establish the presence of bradykinesia (slowness of initiation of


voluntary movement with reduced speed and amplitude of movements and/or
the loss or decrease of automatic movements) plus at least one of the following:
Rigidity
4Y6 Hz resting tremor
Postural instability (not otherwise explained by primary visual, vestibular,
cerebellar, or proprioceptive dysfunction)
b Step 2: Evaluate for exclusionary criteria for the diagnosis of Parkinson disease
The absence of a history of:
Repeated strokes with a stepwise progression of parkinsonism
Repeated head injury
Definite encephalitis
Neuroleptic treatment at onset of symptoms
More than one affected relative (although this exclusion criterion is
no longer commonly used)
Any period of sustained remission
Oculogyric crises
Early severe autonomic involvement
Early severe dementia with disturbances of memory, language, and praxis
1-Methyl-4-phenyl-1,2,3,6-tetrahydropyridine (MPTP) exposure
OR a finding of:
Strictly unilateral features after 3 years
Negative response to large doses of levodopa (if malabsorption is excluded)
Supranuclear gaze palsy
Cerebellar signs
Babinski sign
Cerebral tumor or communicating hydrocephalus on neuroimaging
b Step 3: Establish the presence of three or more of the supportive criteria
for the diagnosis of Parkinson disease
Unilateral onset
Resting tremor present
Progressive disorder
Persistent asymmetry with symptoms worse on the side of onset
Clear and definite response (70Y100% improvement on Unified
Parkinson’s Disease Rating Scale) to levodopa
Severe levodopa-induced chorea
Levodopa response for 5 years or more
Clinical course of 10 years or more
a
Modified with permission from Hughes AJ, et al, J Neurol Neurosurg Psychiatry.8 B 1992
British Medical Journal Publishing Group. jnnp.bmj.com/content/55/3/181.short.

Continuum (Minneap Minn) 2016;22(4):1047–1063 www.ContinuumJournal.com 1049

Copyright © American Academy of Neurology. Unauthorized reproduction of this article is prohibited.


Diagnosing Parkinson Disease

KEY POINTS
h Rigidity giving rise and small handwriting (micrographia). presenting symptoms of PD. Prominent
to decreased range of The latter can be tested by asking pa- axial rigidity early in the course of symp-
motion and shoulder tients to write the same sentence at toms is not typical and is commonly
pain (often misdiagnosed least three times sequentially, observing observed in progressive supranuclear
as orthopedic or the possibility of a progressive decrease palsy (PSP).
arthritic) and difficulty in sentence length and the size of the
turning over in bed at characters. Although bradykinesia is Resting Tremor
night are common required to make the diagnosis of PD, The classic parkinsonian resting tremor
presenting symptoms of it can sometimes be difficult to elicit in is present in approximately 70% of
Parkinson disease. strongly tremor-predominant patients patients with PD.13 It is typically 4 Hz
h Parkinson tremor can with PD, especially early in the PD course. to 6 Hz and starts unilaterally and distally
fluctuate significantly in In addition to bradykinesia, at least one and, over time, progresses more proxi-
amplitude with mental additional cardinal symptom of rigidity, mally and often to the contralateral side.
activity or voluntary resting tremor, or postural instability The resting tremor can variably involve
movements of other oscillations around the wrist and finger
must be present as a second step to
limbs, and, thus, it is
make the clinical diagnosis of PD.8 joints, although a pronation/supination
not unusual to be
axis at the wrist and a pill-rolling qual-
initially described as
Rigidity ity to finger tremor are classic in PD.14
intermittent. It often has
a reemergent quality Rigidity refers to the resistance of mus- Parkinson tremor can fluctuate signifi-
such that it can reappear cles to passive movement around a cantly in amplitude with mental activity
with postural sustention joint. While rigidity is a clinical sign or voluntary movements of other limbs,
after a variable delay, and detected by the examiner, it is experi- and, thus, it is not unusual to be initially
patients may report enced and often described by patients described as intermittent.14 It often has
noticing it for the first as stiffness.9 When absent or mild on a reemergent quality such that it can
time with sustained examination, rigidity can be augmented reappear with postural sustention after
postures, such as holding by asking the patient to perform volun- a variable delay, and patients may
a telephone or newspaper. tary movements of the contralateral report noticing it for the first time with
limb. In evaluating rigidity, it is impor- sustained postures, such as holding a
tant to distinguish true parkinsonian telephone or newspaper (Case 1-1).15
rigidity from both paratonia (involuntary This manifestation can sometimes lead
variable and proportional resistance in to confusion as it can be reported as an
response to passive movement [also action tremor. Some patients also re-
known as gegenhalten]) and the cog- port a sensation of ‘‘inner tremor’’ that
wheel phenomenon that can be ob- does not involve oscillations around a
served with essential and other types of joint.16 A jerky quality to tremor can
tremor without any underlying in- sometimes be observed in younger
creased tone (Froment sign, which is patients, but in older patients this can
the increase in resistance to passive signal the presence of multiple system
movement about a joint with voluntary atrophy (MSA). Although more charac-
action in another part of the body).11 teristic of essential tremor, kinetic
Similarly, reduced arm swing on one tremor (occurring during active move-
side can be misleading without the ment) is not unusual in PD, and
appropriate clinical context, as some amelioration of tremor with alcohol
degree of arm swing asymmetry can be (due to its relaxing effects) can also be
observed in healthy adults.12 Rigidity reported in PD, emphasizing the im-
giving rise to decreased range of motion portance of the clinical examination.17
and shoulder pain (often misdiagnosed Parkinsonian tremor does not uniformly
as orthopedic or arthritic) and difficulty respond to levodopa, so occasionally
turning over in bed at night are common patients, in whom the other cardinal

1050 www.ContinuumJournal.com August 2016

Copyright © American Academy of Neurology. Unauthorized reproduction of this article is prohibited.


KEY POINTS

Case 1-1 h The lack of resting


tremor does not
A 68-year-old right-handed man was referred for a neurologic evaluation of
exclude the diagnosis
tremor. The patient had first noted tremor of the right hand when driving
of Parkinson disease.
and holding a newspaper approximately 2 years previously. Initially, tremor
seemed to be intermittent, but it gradually progressed to be present h The cardinal symptoms
continuously throughout the day. Two glasses of wine in the evening could of Parkinson disease are
reduce but not stop his tremor, and he had not noticed any tremor in his bradykinesia, rigidity,
nondominant hand. He also had chronic right shoulder pain of 6-months resting tremor, and
duration for which his primary care doctor ordered a shoulder x-ray, which was postural instability.
read as normal. Despite these symptoms, he did not feel that he was h Actual falls occurring
functionally impaired in any way. Upon further questioning, he reported mild early in the disease
difficulty getting up from the low sofa in his basement, and he had both course and especially
chronic constipation and erectile dysfunction for at least 5 years. He denied in the first year of
noticing any change in his sense of smell, but his wife stated that he often presenting symptoms
commented that her cooking was less flavorful. His wife had begun to sleep should elicit suspicion
in the guest room due to the patient hitting her in his sleep. of the possibility of
His neurologic examination was remarkable for a ‘‘poker face’’ and soft an atypical form
voice, a delay in right shoulder shrug, and mild cogwheel rigidity in the right of parkinsonism.
arm that was only noticeable with voluntary movement of the opposite
hand. He had a low-amplitude, low-frequency right hand resting tremor
that reappeared with sustained posture after a short pause. His finger
movements were slightly smaller and slower on the right side, and rapid
sequential movements became progressively smaller with repetition. His
posture was slightly stooped, and he walked with decreased right arm swing
and a more pronounced pronation/supination tremor in the right wrist. He
turned easily and had normal postural stability.
Comment. This patient’s presentation is typical for tremor-predominant
idiopathic Parkinson disease. It illustrates the occurrence of the cardinal
motor symptoms of parkinsonism with multiple supportive criteria that were
preceded by nonmotor symptoms commonly reported in Parkinson disease.

symptoms of the disease are mild first year of presenting symptoms


and not noticeable, can report a lack should elicit suspicion of the possi-
of benefit from levodopa despite an bility of an atypical form of parkin-
observable improvement with levo- sonism. Postural instability is also not
dopa administration. specific to parkinsonism and is com-
monly observed in the elderly, often
Postural Instability making it the least helpful criterion
Impairment in the ability to recover in the initial diagnosis of PD, but it is
one’s balance is often very subtle in helpful in suggesting to the clinician
patients first presenting with PD and the possibility of other diagnoses or
sometimes does not develop until well comorbid diagnoses.18
into the disease course. When present
in early PD, patients will often report Other Motor Abnormalities
tripping or requiring increased con- Outside of the diagnostic criteria, other
centration to avoid tripping on uneven motor features can be seen both at the
surfaces, yet will have a normal re- time of diagnosis and as the disease
sponse to pull testing in the clinic. progresses. Impaired finger dexterity,
Actual falls occurring early in the difficulty multitasking, and increased
disease course and especially in the concentration required for both motor

Continuum (Minneap Minn) 2016;22(4):1047–1063 www.ContinuumJournal.com 1051

Copyright © American Academy of Neurology. Unauthorized reproduction of this article is prohibited.


Diagnosing Parkinson Disease

KEY POINT
h Nonmotor symptoms and cognitive task completion are Since the initial publication of the
such as hyposmia, common early reported symptoms, while UKPDSBB criteria, the inclusion of a
erectile dysfunction, a progressively flexed posture, freezing strong family history (more than one
constipation, and rapid phenomenon, gait initiation difficul- affected relative) has been challenged
eye movement sleep ties, postural deformities, and swal- as an exclusionary criterion and is
behavior disorder can lowing difficulties tend to appear later often overlooked in the appropriate
precede the onset of in the disease. clinical context. Other features (such
motor symptoms of as hyposmia, rapid eye movement [REM]
Parkinson disease Exclusionary and Supportive sleep behavior disorder, and constipa-
by years. Criteria for Diagnosing tion) that are known to be strongly
Parkinson Disease associated with the diagnosis of PD
The exclusionary criteria for PD listed have been proposed as additional
in Table 1-2 need to be practically formal supportive criteria, especially as
considered on an individual basis, as they often precede the motor symp-
some criteria reflect confounders in toms of the disease.8,21Y23 Although
attributing symptoms to PD while early severe dementia is an exclusion
others suggest the presence of other criterion in the UKPDSBB criteria, more
diseases. For example, a history of than 15% of patients with PD can have
multiple concussions that preceded at least mild cognitive impairment at
the onset of parkinsonism is a poten- the time of diagnosis.24 However the
tial confounder in diagnosing PD, but presence of visual hallucinations at the
it does not exclude the diagnosis of time of presentation and especially in
PD or specifically suggest an alterna- untreated patients is suggestive of
tive diagnosis. However, cerebellar dementia with Lewy bodies, which
signs or a supranuclear gaze palsy de- some argue should be considered a
veloping simultaneously with progres- subtype of PD.7
sive parkinsonism would strongly
suggest an alternative diagnosis of an NONMOTOR SYMPTOMS
atypical parkinsonism such as MSA or The function of the basal ganglia was
PSP, respectively. traditionally thought to be limited to
Supportive criteria (of which three the modulation and processing of in-
or more are required) are all charac- formation related to motor control,
teristics that commonly are observed with projections solely to motor corti-
in PD. Patients with PD usually pres- ces.25 However, it is now accepted
ent with a clear asymmetry of symp- that the basal gangliaYthalamocortical
tom onset and respond well to trials of circuits consist of multiple subcircuit
levodopa when appropriate, but this loops projecting cortically to diverse
is not always the case. Resting tremor targets.26,27 These circuits are largely
is absent in 30% of patients with PD, parallel, somatotopically arranged, and
and patients without resting tremor functionally specific, and are organized
can experience a longer delay in into motor, oculomotor, associative,
diagnosis due to physician familiarity and limbic circuits. They are involved
with the association between resting in a variety of activities including emo-
tremor and PD.19 Patients with path- tion, reward behavior and habit forma-
ologically confirmed PD can go many tion, time estimation, attention, working
years from the onset of resting tremor memory, and learning.28 Thus, it would
before developing other symptoms and be expected that the basal ganglia
might never develop unequivocal dysfunction that occurs in PD would
bradykinesia.20 produce a more complicated clinical
1052 www.ContinuumJournal.com August 2016

Copyright © American Academy of Neurology. Unauthorized reproduction of this article is prohibited.


picture than isolated motor dysfunc-
tion. Similarly, basal ganglia pathology TABLE 1-3 Continued
is now recognized to be only one aspect
of the neuropathologic changes that b Gastrointestinal Dysfunction
occur in PD, which include changes Constipation (39%)
to multiple neurotransmitter systems
Early satiety (23%)
throughout the brain, further suggesting
that the clinical course of PD should Loss of appetite
include nonmotor as well as motor b Sexual Dysfunction
symptoms.29,30 The most common non- Erectile dysfunction (28%)
motor symptoms that occur in PD are
listed in Table 1-3.31 Reduced libido (42%)
Approximately 90% of patients with b Sleep Disturbances
PD will develop at least one nonmotor Excessive daytime sleepiness
symptom, and nonmotor symptoms can (28%)
be the presenting feature in PD.23,32
Insomnia (36%)
Later in the disease, the disability from
Rapid eye movement (REM)
nonmotor symptoms often overshadows
sleep behavior disorder (28%)
that from motor symptoms, and uncon-
trolled nonmotor symptoms can be a Sleep fragmentation

deciding factor in the timing of place- Vivid dreams (24%)


ment in a long-term care facility.33,34 For b Somatosensory Dysfunction
Restless legs syndrome (17%)

TABLE 1-3 Common Nonmotor Unexplained pain (20%)


Symptoms of
b Special Sensory
Parkinson Disease at the
Time of Diagnosisa,b Impaired olfaction (46%)
Impaired taste (14%)
b Affective Disorders
b Urinary Dysfunction
Anhedonia (32%)
Frequency (28%)
Anxiety (30%)
Incontinence (10%)
Apathy (31%)
Urgency (34%)
Depression (43%)
Weak urinary stream (25%)
b Autonomic Dysfunction
b Visual Disturbances
Orthostatic hypotension (21%)
Impaired color discrimination
b Cognitive Dysfunction
Visuospatial abnormalities
Bradyphrenia
a
Modified with permission from Pont-
Inattention (28%) Sunyer C, et al, Mov Disord.31 B 2014
International Parkinson and Move-
Memory impairment (32%) ment Disorder Society. onlinelibrary.
wiley.com/doi/10.1002/mds.26077/
Word-finding difficulties abstract.
b
Reported prevalence of symptom
b Fatigue (36%) (%) within a year of diagnosis when
information is available.

Continuum (Minneap Minn) 2016;22(4):1047–1063 www.ContinuumJournal.com 1053

Copyright © American Academy of Neurology. Unauthorized reproduction of this article is prohibited.


Diagnosing Parkinson Disease

KEY POINTS
h Parkinson disease more information, refer to the article commonly referred to as early PD in
results in a variety of ‘‘Treatment of Advanced Parkinson Dis- clinical trials (in which motor symptoms
nonmotor symptoms ease and Related Disorders’’ by Janis M. are just beginning to manifest) is a point
that can sometimes be Miyasaki, MD, MEd, FRCPC, FAAN,35 in in time that is actually relatively later
more disabling than the this issue of Continuum. In the patient in the disease course. Therefore, it has
motor symptoms, and
presenting with PD, nonmotor symp- been recognized by many experts that a
studies suggest that the potential window for disease-modifying
toms often precede or present with
nonmotor symptoms and neuroprotective treatments exists,
have a greater effect motor symptoms but are not commonly
but that earlier diagnosis and a bio-
on quality of life. reported unless uncovered by the clini-
marker are necessary. Many current
h At least two subtypes cian during the encounter.36
neuroprotective and disease-modifying
(tremor-predominant strategies are being tested in patients
PARKINSON DISEASE SUBTYPES
Parkinson disease and where presumably a large proportion
postural instability and One of the most remarkable aspects of
PD is the degree of clinical heterogeneity of neurodegeneration has already
gait difficulty Parkinson
that exists across patients, to such a de- occurred.47 A variety of terms have
disease) have been
gree that some have questioned whether been proposed, including prephys-
identified, and both of
these entities manifest PD can be considered a unified disease.7 iologic, preclinical, prodromal, premotor,
different rates of It has long been recognized that and prediagnostic, to describe the
progression, disability, considerable variability exists between various stages that can be identified
nonmotor symptoms, patients with PD in the expression of prior to the diagnosis of PD.7,48 From
and complications clinical symptoms, the rate of disease the practical and clinical standpoint, it
of therapy. is most important to remember two
progression, and the response to
main points. First, the vast majority of
therapies.37 The approaches that have
patients with PD already manifest one
been used to try to organize this het-
or more nonmotor symptoms of the
erogeneity into distinct and useful clin-
disease at the time of diagnosis and
ical subtypes have largely been based
often years prior. 23 Evaluation of
on specific clinical or demographic
nonmotor symptoms in PD is a critical
features (empirical approach) or on
part of the history at the initial presen-
methods of statistical clustering (data-
tation and should be uncovered, if
driven approach) with a variety of
present, in all patients. Second, some
classifications proposed.38 The obser- patients may have nonmotor symp-
vation that a predominance of tremor toms and mild soft signs of parkinson-
in PD portends a slower disease pro- ism, but not enough clinical findings to
gression was one of the earliest indi- confer the diagnosis of PD. In these
cators of the existence of PD subtypes patients, monitoring symptom stability
and dates back more than 45 years.39 A and progression over time is impor-
tremor-predominant phenotype has tant, especially prior to labeling the
been associated with a slower disease patient as having PD.
progression and less cognitive impair-
ment, depression, and apathy when DIFFERENTIAL DIAGNOSIS
defined empirically in data-driven stud- A myriad of diseases and syndromes can
ies and retrospective clinicopathologic be mistaken for PD, including other
studies.40Y46 forms of parkinsonism (Table 1-1)
as well as nonparkinsonian mimics.
PREDIAGNOSTIC PARKINSON Table 1-4 lists the commonly consid-
DISEASE ered differential diagnoses in the ini-
In the field of PD research, it is be- tial presentation of PD. Atypical
coming increasingly clear that what is parkinsonism can sometimes initially
1054 www.ContinuumJournal.com August 2016

Copyright © American Academy of Neurology. Unauthorized reproduction of this article is prohibited.


KEY POINTS
difficult to distinguish from PD; the h Atypical parkinsonism
TABLE 1-4 Common Differential most important factor in its consider-
Diagnoses of can sometimes initially
Parkinson Disease ation is a careful and complete medica- be mistaken for
tion history (Case 1-2). In cases of idiopathic Parkinson
b Atypical parkinsonian parkinsonism with a predominance of disease. Falls and
syndromes (multiple system lower body symptoms and urinary and significant autonomic
atrophy, progressive cognitive difficulties, structural neuro- dysfunction early within
supranuclear palsy, imaging can uncover the diagnosis of the disease course, a
corticobasal syndrome) poor response to
normal pressure hydrocephalus. For
b Dementia with Lewy bodies more information on normal pressure levodopa, symmetry
of symptoms at onset,
b Drug-induced parkinsonism hydrocephalus, refer to the article ‘‘Di-
and rapid symptom
agnosis and Treatment of Idiopathic
b Dystonic tremor progression are useful
Normal Pressure Hydrocephalus’’ by red flags indicating the
b Essential tremor Michael A. Williams, MD, FAAN, and possibility of atypical
b Frontotemporal dementias Jan Malm, MD, PhD,49 in the 2016 parkinsonism.
b Functional or psychogenic
Dementia issue of Continuum. Tradi-
h Drug-induced
parkinsonism tionally, vascular parkinsonism was parkinsonism can be
thought to produce a nontremulous very difficult to
b Normal pressure
hydrocephalus
lower body predominance of symptoms distinguish from Parkinson
with a poor levodopa response and a disease; the most
b Vascular parkinsonism stepwise progression; however, signifi- important factor in its
cant overlap exists in the clinical find- consideration is a
ings of vascular and other forms of careful and complete
be mistaken for idiopathic PD. Falls parkinsonism, and the entity remains medication history.
and significant autonomic dysfunction poorly defined. MRI can be helpful in
early within the disease course, a poor the diagnosis of vascular parkinsonism
response to levodopa, symmetry of depending on the location of white
symptoms at onset, and rapid symp- matter changes.50 Functional or psy-
tom progression are useful red flags chogenic parkinsonism can usually be
indicating the possibility of atypical diagnosed readily by the recognition of
parkinsonism. The presence of other unusual or inconsistent examination
signs and symptoms can be suggestive findings.
of other diagnoses. Visual hallucina-
tions within the first year of motor LEVODOPA CHALLENGE AND
symptoms are suggestive of dementia SMELL TESTING
with Lewy bodies. Essential tremor is In some cases, clinical response to an
usually 5 Hz or faster and is most adequate dose of levodopa can be help-
prominent with action, although es- ful in establishing the PD diagnosis
sential tremor that is long-standing (Case 1-3). However, a lack of re-
can sometimes occur at rest (usually sponse to a levodopa challenge (even
bilaterally), and its frequency can fall at doses greater than 200 mg per dose)
into the parkinsonian range.11 Pa- does not strictly rule out PD, and
tients with dystonia can have mild patients with atypical parkinsonism
parkinsonism, and patients with PD can have a partial levodopa response
can occasionally present with dystonia as discussed in the article ‘‘Diagnostic
(typically lower limb dystonia in young- Approach to Atypical Parkinsonian
onset patients), although dystonia is Syndromes’’ by Nikolaus R. McFarland,
not commonly observed early in PD. MD, PhD,51 in this issue of Continuum.
Drug-induced parkinsonism can be very A levodopa challenge can be useful
Continuum (Minneap Minn) 2016;22(4):1047–1063 www.ContinuumJournal.com 1055

Copyright © American Academy of Neurology. Unauthorized reproduction of this article is prohibited.


Diagnosing Parkinson Disease

KEY POINT
h Laboratory testing can Case 1-2
sometimes be useful in
A 56-year-old woman with a past medical history of hypertension,
ruling out metabolic
hyperlipidemia, and refractory major depressive disorder presented for
abnormalities that may
a neurologic evaluation because of a 4-month history of difficulty walking.
present as parkinsonism.
She reported problems with handwriting due to tremor and micrographia
and difficulty getting in and out of her car. She denied any sleep
disturbances, constipation, changes in her sense of smell, or urinary dysfunction.
A CT scan of her brain ordered by her primary care doctor 1 year previously
after an episode of acute dizziness was remarkable only for mild chronic
microvascular ischemic changes. She had been tried on multiple antidepressants
but denied ever taking a neuroleptic, and none was uncovered following
inspection of her current medication bottles. Her neurologic examination
was remarkable for decreased facial expression and blink rate, bilateral
rigidity, and low-frequency resting and action tremors. Perioral tremor was
noted as well. Since she had used the same pharmacy for the last 5 years, the
pharmacy was called and a list of all recently prescribed medications was
reviewed. The list included an atypical neuroleptic that had been prescribed
for depression 2 weeks prior to the onset of her neurologic symptoms. This
medication had been discontinued the month prior to her presentation, and
her symptoms gradually improved over the next few weeks and resolved
completely within 2 months.
Comment. Although features such as a subacute onset and relative
symmetry of symptoms can suggest a secondary cause of parkinsonism
such as drug-induced parkinsonism, it may not be possible to distinguish
these entities on a clinical basis alone. An accurate history of current and
recently prescribed medications is critical in establishing the accurate
diagnosis when a patient presents with parkinsonism.

in the diagnosis of PD; however, the from PSP or corticobasal degeneration


absence of a response to levodopa does as less olfactory impairment occurs in
not rule out PD and can result from these diseases compared to MSA.52
gastric emptying issues. Thus, in these
cases, clinicians will sometimes order a LABORATORY TESTING
gastric emptying study to uncover this There are currently no available labo-
potential issue. ratory tests that can establish the
Olfactory impairment can be a diagnosis of PD, despite a number of
useful screening test for PD, and recent potential serum-based biomarkers un-
European guidelines recommend its der evaluation.53 In patients with less
use in the differentiation of PD from than a classical presentation or an ap-
atypical and secondary parkinsonism.10 propriate history, exclusion of thyroid
A number of smell-identification tests and parathyroid dysfunction (which can
are commercially available, including result in parkinsonism) and basic labo-
the University of Pennsylvania Smell ratory testing (including liver function
Identification Test, Brief Smell Iden- tests) can be useful in ruling out sec-
tification Test, and Sniffin’ Sticks. A ondary causes of parkinsonism. In ad-
caveat of such tests is their low speci- dition, any patient who is less than
ficity, as decreased sense of smell can 50 years old and presenting with
be seen in normal aging and other parkinsonism should be screened for
neurodegenerative diseases. It may be Wilson disease with appropriate copper
more helpful in differentiating PD studies including serum ceruloplasmin

1056 www.ContinuumJournal.com August 2016

Copyright © American Academy of Neurology. Unauthorized reproduction of this article is prohibited.


KEY POINT
h Genetic testing in
Case 1-3 Parkinson disease
A 60-year-old right-handed woman presented for a second neurologic opinion is generally not
for the diagnosis of parkinsonism. She had developed masked face, stiffness, recommended except
and slowing in her activities of daily living 3 years earlier, and she had been for very specific
diagnosed with parkinsonism the following year. She had been started on patient populations.
carbidopa/levodopa 25 mg/100 mg 1 tablet 3 times a day, which she had
continued, but she had not noticed any benefit. Over the past year she had
developed freezing when going through doorways and when turning to get
into a chair. Over the past 6 months she had fallen backward multiple times,
and she reported a subjective difficulty with swallowing and bothersome
urinary urgency and frequency. She had been frustrated by her lack of
improvement and heard about ‘‘the brain scan that could diagnose Parkinson
disease’’ in her support group and requested it to be ordered. Neurologic
examination was remarkable for masked facies, hypophonia, moderately
asymmetric rigidity and bradykinesia, and diffusely increased reflexes without
tremor. She had a lack of postural response on pull testing, and mental
status examination was within normal limits.
Comment. This patient presented with a parkinsonian syndrome with the
most important differential diagnosis being Parkinson disease versus an
atypical parkinsonism such as multiple system atrophyYparkinsonism type.
Neuroimaging of the dopamine transporter will be abnormal in both diseases
and would not add any additional useful diagnostic or treatment information.
A further titration of her levodopa to at least 900 mg/d (up to 300 mg per dose
if she can tolerate it) to determine her responsiveness to an adequate dose of
carbidopa/levodopa would be far more helpful in clarifying her diagnosis.

levels, and 24-hour urine copper excre- the diagnosis of PD recommends con-
tion should be considered. For more sideration of specific gene testing in
information, refer to the article ‘‘Wilson patients without a family history of PD
Disease’’ by Ronald F. Pfeiffer, MD, but onset before the age of 40, patients
FAAN,54 in this issue of Continuum. with a strong family history of either a
recessive pattern of inheritance (espe-
GENETIC TESTING cially with onset before the age of 50)
Most patients (approximately 90%) or an autosomal dominant pattern of
diagnosed with PD do not have a family inheritance, and patients who belong
history of the disease and are, there- to ethnic groups associated with foun-
fore, considered to be sporadic cases.55 der mutations.10 These recommenda-
While monogenic forms of PD have tions are very similar to those put forth
been identified and may provide insight by the Consortium on Risk for Early-
into the pathophysiology and genetic Onset Parkinson Disease (CORE PD) in
heterogeneity of PD, they constitute a 2010.58 In such patients, genetic test-
very small portion of patients with PD.56 ing can be helpful in family and finan-
Recently, alleles that are more com- cial planning, aiding in prognosis, or in
mon in the general population and patients with a strong personal desire
that may increase the risk for PD have for testing.
been identified.57 No formal guidelines Clinicians also need to be aware that
exist regarding which patients with PD the first direct-to-consumer genetic
should be considered for genetic test- tests have been approved by the US
ing. A recent European guideline for Food and Drug Administration (FDA)

Continuum (Minneap Minn) 2016;22(4):1047–1063 www.ContinuumJournal.com 1057

Copyright © American Academy of Neurology. Unauthorized reproduction of this article is prohibited.


Diagnosing Parkinson Disease

KEY POINT
h While the research and may become available for PD.59 diseases, FDG-PET is currently lim-
utility of dopamine These tests can potentially mislead ited to the differentiation of Alzheimer
transporter single-photon asymptomatic patients into thinking disease and frontotemporal dementia.
emission computed they are sure to develop the disease if FDG-PET may be especially useful in
tomography is universally they are positive for an allele that the differentiation of atypical parkin-
agreed upon, its use increases the risk of PD, or that they sonism in the future as the research in
in the clinical setting are no longer at increased risk when this area evolves.64
for differential diagnosis a single gene or a limited number of In 2011, ioflupane I-123 single-
in the individual genes are tested. Any decision to pur- photon emission computed tomogra-
remains controversial. sue clinical genetic testing should be phy (SPECT) was approved in the
preceded by genetic counseling and United States as an adjunctive evalua-
family discussions to ensure patients tion tool for use in the diagnosis of
are aware of the implications for them- suspected parkinsonian syndromes.
selves and their families. This compound binds to the presyn-
aptic dopamine transporter (DAT) and
NEUROIMAGING acts as a marker for dopamine levels in
The clinical utility of structural neuro- the striatum. While the research utility
imaging in the evaluation of PD has of DAT-SPECT is universally agreed
traditionally been limited to ruling upon, its use in the clinical setting for
out secondary causes of parkinsonism, differential diagnosis in the individual
such as vascular parkinsonism or nor- remains controversial.65
mal pressure hydrocephalus, as MRI It is generally agreed on by experts
and CT are largely unremarkable in PD that DAT-SPECT does not currently
beyond generalized cortical atrophy. have a role in the differentiation of PD
Most experts make a bedside decision from other neurodegenerative forms
on the need for structural neuroimag- of parkinsonism as DAT binding will
ing on a case-by-case basis, and the most be reduced in all of these entities
recent AAN practice parameter on diag- (Case 1-3). Further, progression of
nosing PD did not render a recommen- PD cannot yet be followed by DAT-
dation regarding the need for structural SPECT as it has not been shown to
imaging prior to making the diagnosis define a clear and reliable relationship
of PD. However, structural imaging is between DAT binding and disease
often recommended to rule out sec- severity.66 Therefore, DAT-SPECT is
ondary causes of parkinsonism.60 Novel currently limited to the differentiation
techniques of diffusion, perfusion, and of a neurodegenerative parkinsonian
functional MRI (fMRI) continue to be syndrome from entities such as essential
developed and show promise as po- tremor, dystonia, or secondary causes
tential biomarkers of PD.61,62 of parkinsonism. Yet even in these
Metabolic imaging with fludeoxyglu- populations, findings are not always
cose positron emission tomography straightforward. DAT binding would be
(FDGYPET) measures regional differ- expected to be normal in essential
ences in glucose metabolism, and these tremor and thus useful for ruling out a
studies have uncovered patterns of dopamine deficiency in the long-
activity that are characteristic for the standing patient with essential tremor
motor and cognitive symptoms of PD, who has developed a superimposed
as well as patterns specific to atypical slowing of movement speed beyond
forms of parkinsonism such as MSA or that which can be seen in essential
PSP.17,63 Although approved in the tremor.11 It should also be normal in
United States for neurodegenerative dystonia, which is believed to account
1058 www.ContinuumJournal.com August 2016

Copyright © American Academy of Neurology. Unauthorized reproduction of this article is prohibited.


for a significant proportion of patients
clinically diagnosed with PD who had TABLE 1-5 Relevant Drug Interactions With Dopamine
Transporter Single-Photon Emission Computed
DAT imaging scans without evidence Tomography (DAT-SPECT)a
of dopaminergic deficit (SWEDD).67
Yet even in such patients, false positives Effect on DAT-SPECT Time to Be Stopped Prior to
have been reported and some patients and Drug DAT-SPECT
with essential tremor may have mild Minor effectb
abnormalities with DAT imaging.68,69
Amantadine 6 days
Similarly, patients with parkinsonism
who have normal DAT-SPECT imaging Budipine 6 days
have been reported to both transition Citalopram 8 days
to abnormal scans on follow-up and Clomipramine 21 days
respond well to dopaminergic therapy.70
Duloxetine 3 days
Rarely, patients with vascular parkinson-
ism and drug-induced parkinsonism Ephedrine, epinephrine, 6Y10 hours
phenylephrine,
can also have abnormal DAT-SPECT
pseudoephedrine
imaging, and in these patient groups
its utility may be primarily focused on Escitalopram 8 days
those patients observed to have nor- Fluoxetine 45 days
mal DAT binding and a clinical presen- Fluvoxamine 5 days
tation of parkinsonism.66 However, DAT Imipramine 5 days
is usually negative in cases of drug-
Memantine 5 days
induced parkinsonism.
DAT-SPECT also has a number of Paroxetine 5 days
limitations that need to be considered, Sertraline 6 days
even when used judiciously. With the Venlafaxine 3 days
exception of a few studies that have Ziprasidone 2 days
included clinicopathologic correlation, c
Significant effect
most studies that have evaluated the ac-
curacy of DAT-SPECT in the diagnosis Amphetamine 7 days
of PD have used the clinical diagnosis Benztropine 5 days
of PD as the gold standard, which is Bupropion or 8 days
obviously problematic when one is amfebutamone
looking to the DAT-SPECT for support- Cocaine 2 days
ive data when the clinical diagnosis is
Dextroamphetamine 7 days
not clear.71 Additionally, interpretation
of the DAT-SPECT is currently qualita- Mazindol 3 days
tive, with a binary normal or abnormal Methylamphetamine 3 days
reading assigned based on visual in- Methylphenidate 2 days
spection. While objective quantitative
Modafinil 3 days
measures of DAT binding have been
developed, they are not currently clin- Phentermine 14 days
ically available.72 a
Modified with permission from Kägi G, et al. J Neurol Neurosurg Psychiatry.73
If DAT-SPECT imaging is pursued, it B 2010 BMJl Publishing Group Ltd. jnnp.bmj.com/content/81/1/5.abstract.
b
is important to note that a number of May have small effect on uptake (at most 15%). This is acceptable for routine
DAT-SPECT but not for research.
commonly prescribed medications can c
All of these drugs are likely to alter (usually decrease) radioligand uptake by at
interfere with DAT binding and pro- least 20% and often substantially more and, therefore, have to be stopped
prior to routine DAT-SPECT.
duce erroneous results (Table 1-5).73
Most of the drugs listed in this resource
Continuum (Minneap Minn) 2016;22(4):1047–1063 www.ContinuumJournal.com 1059

Copyright © American Academy of Neurology. Unauthorized reproduction of this article is prohibited.


Diagnosing Parkinson Disease

KEY POINT
h If dopamine transporter reduce DAT binding (potentially can emerge over time, it is important for
single-photon emission resulting in a false-positive interpreta- the clinician to continue to follow the
computed tomography tion). The duration of discontinuation patient long-term, monitor the exami-
imaging is pursued, it is recommended for some of these agents nation, and document the response to
important to note that a is significant (such as for some of the dopaminergic therapy. In many cases,
number of commonly selective serotonin reuptake inhibitors the diagnosis may be revised based on
prescribed medications [SSRIs]), and the decision to stop medi- the emergence of a pattern of symp-
can interfere with cations needs to be considered on an toms suggesting an atypical parkinso-
dopamine transporter individual basis. In contrast, dopami- nian syndrome or other diagnosis.
binding and produce nergic medications such as levodopa and
erroneous results.
dopamine agonists are generally not held REFERENCES
prior to scanning, as their influence on 1. de Lau LM, Breteler MM. Epidemiology of
DAT binding is not thought to be a Parkinson’s disease. Lancet Neurol 2006;5(6):
significant confounder.73 Recommenda- 525Y535. doi:10.1016/S1474-4422(06)70471-9.

tions on which medications to hold 2. Tanner CM, Goldman SM. Epidemiology of


Parkinson’s disease. Neurol Clin 1996;14(2):
prior to DAT scanning and for how long 317Y335. doi:10.1016/S0733-8619(05)70259-0.
have yet to be agreed upon by experts.
3. Jellinger KA. How valid is the clinical diagnosis
of Parkinson’s disease in the community?
CONCLUSION J Neurol Neurosurg Psychiatry 2003;74(7):
Making an accurate diagnosis of PD in 1005Y1006. doi:10.1136/jnnp.74.7.1005.

the early stages of the disease can be 4. Wermuth L, Lassen CF, Himmerslev L, et al.
Validation of hospital register-based
challenging. A careful history and
diagnosis of Parkinson’s disease.
examination guided by clinical diag- Dan Med J 2012;59(3):A4391.
nostic criteria will usually establish PD
5. Ehringer H, Hornykiewicz O. Verteilung von
or alternatively uncover red flags sug- Noradrenalin und Dopamin (3-hydroxytyramin)
gesting other diagnoses. The symptoms im Gehirn des Menschen und ihr Verhalten
bei Erkrankungen des extrapyramidalen
of PD can be variable, and subtypes
systems. Klin Wochenschr 1960;38:
exist that have differing clinical symp- 1236Y1239.
toms and rates of progression. Some 6. Fahn S, Jankovic J, Hallett M. Principles and
patients may not fulfill criteria for the practice of movement disorders. 2nd ed.
clinical diagnosis in the initial stages New York, NY: Elsevier/Saunders, 2011.
of their disease. The key is to closely 7. Berg D, Postuma RB, Bloem B, et al.
follow patients, as disease progression Time to redefine PD? Introductory
statement of the MDS Task Force on
usually gives the answer. Over time, the definition of Parkinson’s disease.
patients initially suspected of having PD Mov Disord 2014;29(4):454Y462.
will either have their diagnosis solidi- doi:10.1002/mds.25844.
fied by their clinical course and re- 8. Hughes AJ, Daniel SE, Kilford L, Lees AJ.
Accuracy of clinical diagnosis of idiopathic
sponse to medication or an alternative Parkinson’s disease: a clinico-pathological
diagnosis will emerge. Structural neu- study of 100 cases. J Neurol Neurosurg
roimaging and laboratory testing can be Psychiatry 1992;55(3):181Y184.
doi:10.1136/jnnp.55.3.181.
helpful in ruling out other diseases.
While imaging of the striatal DAT can 9. Mazzoni P, Shabbott B, Cortés JC.
Motor control abnormalities in
be useful in specific situations, clinical Parkinson’s disease. Cold Spring Harb
evaluation by a movement disorders Perspect Med 2012;2(6):a009282.
specialist or neurologist experienced in doi:10.1101/cshperspect.a009282.
movement disorders is usually ade- 10. Berardelli A, Wenning GK, Antonini A, et al.
EFNS/MDS-ES/ENS [corrected] recommendations
quate to the establish the diagnosis of for the diagnosis of Parkinson’s disease.
PD for most patients presenting with Eur J Neurol 2013;20(1):16Y34.
parkinsonism. Since other PD syndromes doi:10.1111/ene.12022.

1060 www.ContinuumJournal.com August 2016

Copyright © American Academy of Neurology. Unauthorized reproduction of this article is prohibited.


11. Thenganatt MA, Louis ED. Distinguishing 24. Kang GA, Bronstein JM, Masterman DL,
essential tremor from Parkinson’s disease: et al. Clinical characteristics in early
bedside tests and laboratory evaluations. Parkinson’s disease in a central
Expert Rev Neurother 2012;12(6):687Y696. California population-based study. Mov
doi:10.1586/ern.12.49. Disord 2005;20(9):1133Y1142. doi:10.1002/
12. Plate A, Sedunko D, Pelykh O, et al. mds.20513.
Normative data for arm swing asymmetry: 25. Middleton FA, Strick PL. Basal ganglia and
how (a)symmetrical are we? Gait Posture cerebellar loops: motor and cognitive
2015;41(1):13Y18. doi:10.1016/ circuits. Brain Res Brain Res Rev 2000;
j.gaitpost.2014.07.011. 31(2Y3):236Y250. doi:10.1016/S0165-
13. Hughes AJ, Ben-Shlomo Y, Daniel SE, Lees AJ. 0173(99)00040-5.
What features improve the accuracy 26. Obeso JA, Rodrı́guez-Oroz MC,
of clinical diagnosis in Parkinson’s Benitez-Temino B, et al. Functional
disease: a clinicopathologic study. organization of the basal ganglia:
Neurology 1992;42(6):1142Y1146. therapeutic implications for Parkinson’s
doi:10.1212/WNL.42.6.1142. disease. Mov Disord 2008;23(suppl 3):
14. Hess CW, Pullman SL. Tremor: clinical S548YS559. doi:10.1002/mds.22062.
phenomenology and assessment techniques. 27. Alexander GE, DeLong MR, Strick PL. Parallel
Tremor Other Hyperkinet Mov (N Y) 2012;2. organization of functionally segregated
pii: tre-02-65-365-1. circuits linking basal ganglia and cortex.
Annu Rev Neurosci 1986;9:357Y381.
15. Jankovic J, Schwartz KS, Ondo W. Re-emergent
doi:10.1146/annurev.ne.09.030186.002041.
tremor of Parkinson’s disease. J Neurol
Neurosurg Psychiatry 1999;67(5):646Y650. 28. Obeso JA, Rodriguez-Oroz MC, Stamelou M,
doi:10.1136/jnnp.67.5.646. et al. The expanding universe of disorders
of the basal ganglia. Lancet 2014;384
16. Shulman LM, Singer C, Bean JA, Weiner WJ.
(9942):523Y531. doi:10.1016/S0140-6736
Internal tremor in patients with Parkinson’s
(13)62418-6.
disease. Mov Disord 1996;11(1):3Y7.
doi:10.1002/mds.870110103. 29. Braak H, Del Tredici K, Rüb U, et al.
17. Kraus PH, Lemke MR, Reichmann H. Staging of brain pathology related to
Kinetic tremor in Parkinson’s diseaseVan sporadic Parkinson’s disease. Neurobiol
underrated symptom. J Neural Transm Aging 2003;24(2):197Y211. doi:10.1016/S0197-
2006;113(7):845Y853. doi:10.1007/ 4580(02)00065-9.
s00702-005-0354-9. 30. Sung VW, Nicholas AP. Nonmotor symptoms
18. Weiner WJ, Nora LM, Glantz RH. Elderly in Parkinson’s disease: expanding the view
inpatients: postural reflex impairment. of Parkinson’s disease beyond a pure motor,
Neurology 1984;34(7):945Y947. pure dopaminergic problem. Neurol Clin
doi:10.1212/WNL.34.7.945. 2013;31(3 suppl):S1YS16. doi:10.1016/
j.ncl.2013.04.013.
19. Breen DP, Evans JR, Farrell K, et al.
Determinants of delayed diagnosis in 31. Pont-Sunyer C, Hotter A, Gaig C, et al.
Parkinson’s disease. J Neurol 2013; The onset of nonmotor symptoms in
260(8):1978Y1981. doi:10.1007/s00415- Parkinson’s disease (the ONSET PD study).
013-6905-3. Mov Disord 2015;30(2):229Y237. doi:10.1002/
mds.26077.
20. Selikhova M, Kempster PA, Revesz T,
et al. Neuropathological findings in 32. Park A, Stacy M. Non-motor symptoms
benign tremulous parkinsonism. in Parkinson’s disease. J Neurol 2009;
Mov Disord 2013;28(2):145Y152. 256(suppl 3):293Y298. doi:10.1007/
doi:10.1002/mds.25220. s00415-009-5240-1.

21. Lees AJ, Hardy J, Revesz T. Parkinson’s 33. Seppi K, Weintraub D, Coelho M, et al.
disease. Lancet 2009;373(9680):2055Y2066. The Movement Disorder Society
doi:10.1016/S0140-6736(09)60492-X. Evidence-Based Medicine Review Update:
treatments for the non-motor symptoms
22. Reichmann H. Clinical criteria for the of Parkinson’s disease. Mov Disord
diagnosis of Parkinson’s disease. 2011;26(suppl 3):S42YS80. doi:10.1002/
Neurodegener Dis 2010;7(5):284Y290. mds.23884.
doi:10.1159/000314478.
34. Hely MA, Morris JG, Reid WG, Trafficante R.
23. Shulman LM, Taback RL, Bean J, Weiner WJ. Sydney Multicenter Study of Parkinson’s
Comorbidity of the nonmotor symptoms disease: non-L-dopa-responsive problems
of Parkinson’s disease. Mov Disord 2001; dominate at 15 years. Mov Disord 2005;
16(3):507Y510. doi:10.1002/mds.1099. 20(2):190Y199. doi:10.1002/mds.20324.

Continuum (Minneap Minn) 2016;22(4):1047–1063 www.ContinuumJournal.com 1061

Copyright © American Academy of Neurology. Unauthorized reproduction of this article is prohibited.


Diagnosing Parkinson Disease

35. Miyasaki JM. Treatment of advanced therapy in Parkinson’s disease. Mov Disord
Parkinson disease and related disorders. 2013;28(1):86Y95. doi:10.1002/mds.24997.
Continuum (Minneap Minn) 2016; 48. Siderowf A, Lang AE. Premotor Parkinson’s
22(4 Movement Disorders):1104Y1116. disease: concepts and definitions. Mov
36. O’Sullivan SS, Williams DR, Gallagher DA, Disord 2012;27(5):608Y616. doi:10.1002/
et al. Nonmotor symptoms as presenting mds.24954.
complaints in Parkinson’s disease: a 49. Williams MA, Malm J. Diagnosis and
clinicopathological study. Mov Disord 2008;23(1): treatment of idiopathic normal pressure
101Y106. doi:10.1002/mds.21813. hydrocephalus. Continuum (Minneap Minn)
2016;22(2 Dementia):579Y599.
37. Zetusky WJ, Jankovic J, Pirozzolo FJ. The
heterogeneity of Parkinson’s disease: 50. Vizcarra JA, Lang AE, Sethi KD, Espay AJ.
clinical and prognostic implications. Vascular Parkinsonism: deconstructing a
Neurology 1985;35(4):522Y526. doi:/10.1212/ syndrome. Mov Disord 2015;30(7):886Y894.
doi:10.1002/mds.26263.
WNL.35.4.522.
38. Marras C, Lang A. Parkinson’s disease 51. McFarland NR. Diagnostic approach to
subtypes: lost in translation? J Neurol atypical parkinsonian syndromes. Continuum
Neurosurg Psychiatry 2013;84(4):409Y415. (Minneap Minn) 2016;22(4 Movement
doi:10.1136/jnnp-2012-303455. Disorders):1117Y1142.
52. Wenning GK, Shephard B, Hawkes C, et al.
39. Hoehn MM, Yahr MD. Parkinsonism: onset,
Olfactory function in atypical parkinsonian
progression and mortality. Neurology
syndromes. Acta Neurol Scand 1995;91(4):
1967;17(5):427Y442.
247Y250.
40. Rajput AH, Voll A, Rajput ML, et al. Course 53. Chahine LM, Stern MB, Chen-Plotkin A.
in Parkinson disease subtypes: a 39-year Blood-based biomarkers for Parkinson’s
clinicopathologic study. Neurology disease. Parkinsonism Relat Disord
2009;73(3):206Y212. doi:10.1212/ 2014;20(suppl 1):S99YS103. doi:10.1016/
WNL.0b013e3181ae7af1. S1353-8020(13)70025-7.
41. Oh JY, Kim YS, Choi BH, et al. Relationship 54. Pfeiffer RF. Wilson disease. Continuum
between clinical phenotypes and cognitive (Minneap Minn) 2016;22(4 Movement
impairment in Parkinson’s disease (PD). Arch Disorders):1246Y1261.
Gerontol Geriatr 2009;49(3):351Y354. 55. Thomas B, Beal MF. Parkinson’s disease.
doi:10.1016/j.archger.2008.11.013. Hum Mol Genet 2007;16(spec no. 2):
42. Burn DJ, Landau S, Hindle JV, et al. R183YR194. doi:10.1093/hmg/ddm159.
Parkinson’s disease motor subtypes and 56. Klein C, Westenberger A. Genetics of
mood. Mov Disord 2012;27(3):379Y386. Parkinson’s disease. Cold Spring Harb
doi:10.1002/mds.24041. Perspect Med 2012;2(1):a008888.
43. Reijnders JS, Ehrt U, Lousberg R, et al. The doi:10.1101/cshperspect.a008888.
association between motor subtypes and 57. Gasser T. Usefulness of genetic testing
psychopathology in Parkinson’s disease. in PD and PD trials: a balanced review.
Parkinsonism Relat Disord 2009;15(5):379Y382.
J Parkinsons Dis 2015;5(2):209Y215.
doi:10.1016/j.parkreldis.2008.09.003. doi:10.3233/JPD-140507.
44. Jankovic J, McDermott M, Carter J, et al. 58. Alcalay RN, Caccappolo E, Mejia-Santana H,
Variable expression of Parkinson’s disease:
et al. Frequency of known mutations in
a base-line analysis of the DATATOP cohort. The early-onset Parkinson disease: implication
Parkinson Study Group. Neurology 1990;40(10): for genetic counseling: the consortium on
1529Y1534. doi:10.1212/WNL.40.10.1529.
risk for early onset Parkinson disease study.
45. Lewis SJ, Foltynie T, Blackwell AD, et al. Arch Neurol 2010;67(9):1116Y1122.
Heterogeneity of Parkinson’s disease in the doi:10.1001/archneurol.2010.194.
early clinical stages using a data driven 59. US Food and Drug Administration.
approach. J Neurol Neurosurg Psychiatry
FDA permits marketing of first
2005;76(3):343Y348. doi:10.1136/ direct-to-consumer genetic carrier test for
jnnp.2003.033530. Bloom syndrome. www.fda.gov/NewsEvents/
46. Selikhova M, Williams DR, Kempster PA, Newsroom/PressAnnouncements/
et al. A clinico-pathological study of UCM435003. Updated February 23, 2015.
subtypes in Parkinson’s disease. Brain Accessed June 7, 2016.
2009;132(pt 11):2947Y2957. doi:10. 60. Pahwa R, Lyons KE. Early diagnosis of
1093/brain/awp234.
Parkinson’s disease: recommendations from
47. Lang AE, Melamed E, Poewe W, Rascol O. diagnostic clinical guidelines. Am J Manag
Trial designs used to study neuroprotective Care 2010;16(suppl implications):S94YS99.

1062 www.ContinuumJournal.com August 2016

Copyright © American Academy of Neurology. Unauthorized reproduction of this article is prohibited.


61. Baglieri A, Marino MA, Morabito R, et al. 2016;87(3):319Y323. doi:10.1136/
Differences between conventional and jnnp-2004-310256.
nonconventional MRI techniques in
68. A multicenter assessment of dopamine
Parkinson’s disease. Funct Neurol
transporter imaging with DOPASCAN/SPECT
2013;28(2):73Y82. doi:10.11138/FNeur/
in parkinsonism. Parkinson Study Group.
2013.28.2.073.
Neurology 2000;55(10):1540Y1547.
62. Ofori E, Pasternak O, Planetta PJ, et al. doi:10.1212/WNL.55.10.1540.
Longitudinal changes in free-water
within the substantia nigra of Parkinson’s 69. Waln O, Wu Y, Perlman R, et al.
disease. Brain 2015;138(pt 8):2322Y2331. Dopamine transporter imaging in essential
doi:10.1093/brain/awv136. tremor with and without parkinsonian
features. J Neural Transm (Vienna)
63. Poston KL, Eidelberg D. FDG PET in the 2015;122(11):1515Y1521. doi:10.
evaluation of Parkinson’s disease. PET Clin 1007/s00702-015-1419-z.
2010;5(1):55Y64. doi:10.1016/j.cpet.
2009.12.004. 70. Menéndez-González M, Tavares F, Zeidan N,
et al. Diagnoses behind patients with
64. Stoessl AJ, Brooks DJ, Eidelberg D.
hard-to-classify tremor and normal
Milestones in neuroimaging. Mov Disord
DaT-SPECT: a clinical follow up study.
2011;26(6):868Y978. doi:10.1002/mds.23679.
Front Aging Neurosci 2014;6:56.
65. Perlmutter JS, Eidelberg D. To scan or not doi:10.3389/fnagi.2014.00056.
to scan: DaT is the question. Neurology
71. de la Fuente-Fernández R, Lövblad KO.
2012;78(10):688Y689. doi:10.1212/
DaTscan and Parkinson’s disease: DAT
WNL.0b013e3182494c72.
binding should not lead to binding decisions
66. Ba F, Martin WR. Dopamine transporter in clinical practice. Eur J Neurol 2014;21(11):
imaging as a diagnostic tool for parkinsonism 1351Y1352. doi:10.1111/ene.12458.
and related disorders in clinical practice. 72. Scherfler C, Nocker M. Dopamine transporter
Parkinsonism Relat Disord 2015;21(2):87Y94. SPECT: how to remove subjectivity?
doi:10.1016/j.parkreldis.2014.11.007. Mov Disord 2009;24(suppl 2):S721YS724.
67. Erro R, Schneider SA, Stamelou M, et al. doi:10.1002/mds.22590.
What do patients with scans without 73. Kägi G, Bhatia KP, Tolosa E. The role of
evidence of dopaminergic deficit (SWEDD) DAT-SPECT in movement disorders. J Neurol
have? New evidence and continuing Neurosurg Psychiatry 2010;81(1):5Y12.
controversies. J Neurol Neurosurg Psychiatry doi:10.1136/jnnp.2008.157370.

Continuum (Minneap Minn) 2016;22(4):1047–1063 www.ContinuumJournal.com 1063

Copyright © American Academy of Neurology. Unauthorized reproduction of this article is prohibited.


Review Article

Treating the Motor


Address correspondence to
Dr John C. Morgan, Movement
Disorders Program,
Department of Neurology,
Medical College of Georgia,
Augusta University, 1429 Harper
Street, HF-1154, Augusta, GA
Symptoms of Parkinson
30912, jmorgan@augusta.edu.
Relationship Disclosure:
Dr Morgan has received
Disease
personal compensation as a
speaker and consultant for John C. Morgan, MD, PhD; Susan H. Fox, MBChB, MRCP(UK), PhD
Impax Laboratories Inc and
Teva Pharmaceutical
Industries Ltd, as a speaker
for the National Parkinson
ABSTRACT
Foundation, and as a consultant Purpose of Review: After a patient is diagnosed with Parkinson disease (PD), there
for AbbVie, Acadia are many therapeutic options available. This article provides examples of prototypical
Pharmaceuticals, Acorda
Therapeutics, Cynapsus patients encountered in clinical practice and illustrates the various pharmacologic and
Pharmaceuticals Inc, nonpharmacologic treatment options for the motor symptoms of PD.
Lundbeck, UCB Inc, Recent Findings: Levodopa became available in the late 1960s and remains the gold
and Veloxis Pharmaceuticals.
Dr Morgan has received standard for the treatment of PD even today. Since that time, amantadine, monoamine
research/grant support as oxidase type B inhibitors, dopamine agonists, and catechol-O-methyltransferase inhib-
principal investigator or itors have emerged as monotherapy, add-on therapies, or both, in the armamentarium
subinvestigator from AbbVie,
ACADIA Pharmaceuticals Inc, against PD. The most appropriate time to start such drugs remains a clinical decision
Biotie Therapies, Civitas according to patient symptoms. However, earlier use of levodopa is the more common
Therapeutics, Kyowa Hakko practice due to its superior benefit and the side effects of dopamine agonists. Deep
USA Inc, Lundbeck, the
National Institutes of Health, brain stimulation continues to be the most effective treatment for motor symptoms in
National Parkinson appropriate patients, and advances in technology may improve efficacy. New ways to
Foundation, and the Parkinson deliver levodopa have emerged (effective extended-release oral preparations and
Study Group, and has served
as an expert witness in various levodopa/carbidopa intestinal gel), and these medications provide additional options
court cases. Dr Fox has for certain patients. Exercise and neurorehabilitation are increasingly recognized as
received salary support for important tools to combat the motor symptoms of PD. Nondopaminergic drugs may
serving as the co-editor of the
International Parkinson and help nonYlevodopa-responsive motor symptoms.
Movement Disorder Society Summary: Treatment of PD is multifaceted and requires a tailored pharma-
website, as a speaker for Ipsen cotherapeutic and nonpharmacologic approach for a given patient. Patients should
Pharmaceuticals, Inc; Teva
Pharmaceutical Industries Ltd; be at the center of care, and clinicians should try to provide optimum benefit through
and Zambon Company SpA; the many treatment options available.
and for serving on the advisory
boards of Lundbeck, Orion
Pharma Ltd, and Novartis Continuum (Minneap Minn) 2016;22(4):1064–1085.
International AG. Dr Fox has
received research/grant support
from Avanir Pharmaceuticals,
the Michael J. Fox Foundation, INTRODUCTION were the main treatment for PD for
the National Institutes of
Health, and Parkinson Society This article reviews treatment options perhaps a century, until the late
of Canada and has received of the motor symptoms of early through 1960s, when Cotzias and others defini-
research/grant support as site
principal investigator for advanced Parkinson disease (PD). The tively proved that levodopa was effec-
clinical studies from Adamas management of early PD affords the tive for the motor symptoms of the
Pharmaceuticals Inc, Cynapsus
Pharmaceuticals Inc, and widest array of pharmacotherapies and disease.2 Thus, for the past 50 years,
Kyowa Hakko USA Inc. other modalities for the clinician to treatment has largely focused on ame-
Dr Fox receives royalties from
Oxford University Press. choose from, depending on the pre- liorating the dopaminergic deficit in PD.
Unlabeled Use of senting patient. Early PD has an evolv- While levodopa remains the most effec-
Products/Investigational ing definition, but the diagnosis still tive pharmacotherapy for PD motor
Use Disclosure:
Drs Morgan and Fox discuss depends on the core motor features of symptoms, there are many other op-
the unlabeled/investigational asymmetry, bradykinesia, resting tremor, tions that are quite useful in the ap-
use of amantadine and
methylphenidate for freezing and cogwheel rigidity.1 Anticholinergics propriate patient.
Continued on page 1065

1064 www.ContinuumJournal.com August 2016

Copyright © American Academy of Neurology. Unauthorized reproduction of this article is prohibited.


Continued from page 1064
As PD advances, the key consider- in clinical trials with drugs and supple-
gait disorders, the use of
ations in treatment of motor symptoms ments such as coenzyme Q10, creatine, clozapine and mirtazapine for
are motor fluctuations and dyskinesia pioglitazone, rasagiline, and others, tremor in Parkinson disease,
and the use of antiY!-synuclein
related to levodopa therapy. There are currently no known definitive disease- antibodies, caffeine,
numerous approaches to help patients slowing therapy exists (Table 2-1).4 At coenzyme Q10, creatine,
inosine, isradipine, nicotine,
minimize ‘‘off’’ time and maximize ‘‘on’’ the time of writing, general media in- nilotinib, pioglitazone, and
time without worsening dyskinesia. terest exists in a clinically available rasagiline as failed or potential
Levodopa-resistant motor symptoms anticancer drug, nilotinib (a small mol- disease-modifying therapies.
* 2016 American Academy
(falls, changes in posture and speech, ecule that inhibits the tyrosine kinase of Neurology.
and refractory tremor) can also be pro- c-Abl and turns on autophagy) at the
minent in certain patients and typically preclinical and phase 1 level5; however,
require rehabilitative therapy, medica- further clinical studies are needed to
tion changes, or deep brain stimula- evaluate this agent in PD. Many other
tion (DBS) in appropriate cases. Better drugs are at more advanced stages of Supplemental digital content:
ways to deliver levodopa have emerged study in the search to slow the progres- Videos accompanying this ar-
ticle are cited in the text as
(carbidopa/levodopa extended release sion of PD (eg, isradipine, nicotine, caf- Supplemental Digital Content.
and levodopa/carbidopa intestinal gel), feine, inosine, and antiY!-synuclein Videos may be accessed by
and, hopefully, these therapies will antibodies). Hopefully, one of the many clicking on links provided in
the HTML, PDF, and app ver-
provide a smoother day of function for avenues being pursued to attack PD sions of this article; the URLs are
patients with moderate and advanced mechanistically will be successful (eg, provided in the print version. Video
legends begin on page 1083.
PD. DBS coupled with dopaminergic oxidative stress, neuroinflammation,
therapy has also emerged as an effective autophagy, antiY!-synuclein therapy).
long-term therapy for motor symptoms,
including refractory tremors, in patients Exercise and Rehabilitative
with moderate to advanced PD. Therapies
Exercise and neurorehabilitation (occu-
EARLY PARKINSON DISEASE pational therapy, physical therapy, and
One of the most important principles in speech therapy) have emerged as pro-
the treatment of early PD is individu- mising therapies in the past decade, with
alizing therapy for a given patient. more objective data on the benefit of
While slowing, stopping, and eventu- so-called brain neuroplasticity.6 These
ally reversing disease progression is the exercise modalities are now empha-
ultimate goal in the treatment of PD, sized in clinical practice by many clini-
selecting the right combination of ther- cians, but perhaps less used in primary
apies for each patient to achieve the care until later in the disease course.
best quality of life is most important. While exercise, in general, is considered
important from a common sense point
Slowing Disease Progression of view for overall patient health, the
Improving symptoms is key in the treat- actual evidence supporting the best
ment of PD, but sadly, patients with PD type and long-term benefit of exercise
continue to experience significant is limited based upon clinical trial data
motor and nonmotor morbidity and (mainly due to limitations inherent in
increased mortality compared to age- such studies, such as blinding).7 Thus,
matched peers despite levodopa ther- yoga, dance, tai chi, boxing, music ther-
apy.3 The holy grail of PD treatment is apy, and multiple other nonpharmaco-
slowing neurodegeneration and reduc- logic interventions have all shown
ing the increased morbidity and mor- benefit and might be useful in a given
tality associated with the disease. patient.6 Cognitive exercises (eg, cross-
Despite significant resources invested word puzzles, word finding, Sudoku,
Continuum (Minneap Minn) 2016;22(4):1064–1085 www.ContinuumJournal.com 1065

Copyright © American Academy of Neurology. Unauthorized reproduction of this article is prohibited.


Parkinson Disease Motor Symptoms

TABLE 2-1 Potential Disease-Modifying Agents for Parkinson aDisease:


Recent Failures and Agents Currently Under Study

Disease-Modifying Agent Target Mechanism


Recent failures
AAV2-neurturin Neurotrophic factor
Coenzyme Q10 Antioxidant, bioenergetics
Creatine Bioenergetics
Pioglitazone PPAR-, agonist, others
Pramipexole D2/D3 dopamine agonist
Rasagiline MAO-B inhibitor
Currently under study
AAV2-GDNF Neurotrophic factor
Caffeine Adenosine antagonist
Dimethyl fumarate Anti-inflammatory
Exenatide Neuroprotection
Glutathione Antioxidant
Inosine Increased uric acid (antioxidant)
Isradipine Calcium channel antagonist
N-Acetylcysteine Glutathione precursor
Nicotine Nicotinic acetylcholine receptor agonist
Nilotinib c-Abl tyrosine kinase inhibitor
PD03A plus adjuvant Active antiY!-synuclein immunization
PRX002 AntiY!-synuclein passive immunization
Sargramostim GM-CSF
Ursodeoxycholic acid Mitochondrial rescue effect

AAV2 = adeno-associated type-2 vector; GDNF = glial cell line-derived neurotrophic factor;
GM-CSF = granulocyte-macrophage colony-stimulating factor; MAO-B = monoamine oxidase type B;
PPAR = peroxisome proliferator-activated receptor.
a
See Kalia et al, 20154 for a full review on this subject.

card games, current events) are also Treatment of Early Parkinson


likely to be beneficial for patients and Disease Through Three Cases
should be encouraged in addition to
The following section discusses several
physical exercise and rehabilitation
patients with prototypical early PD and
strategies. Exercise and neurorehabi-
litation should be part of essentially all discusses various options in the treat-
patients’ treatment regimens, at all ment of their motor symptoms.
stages of disease, and should, of course, In a young patient with few co-
be tailored to the level of function morbidities, there are many options
and disability. to choose from within the context of
1066 www.ContinuumJournal.com August 2016

Copyright © American Academy of Neurology. Unauthorized reproduction of this article is prohibited.


KEY POINTS
a typically less-complex medical his- women are the prototypical patients h A younger patient with
tory (Case 2-1). who are most likely to develop motor early Parkinson disease is
fluctuations and dyskinesia with levo- very different from an
Therapeutic Options in Young, dopa therapy.8,9 Some data, however, older, retired patient.
Cognitively Intact Patients indicate that starting levodopa is asso- Young, thin patients
With Parkinson Disease ciated with better long-term motor appear to have higher
While there are many pharmacologic benefit, less physical disability, and risk of developing
options to choose from in patients with better physical functioning after many dyskinesias with
early PD (Table 2-2), some clinicians years of disease, with admittedly more levodopa therapy.
may try to avoid initial levodopa therapy motor complication risk in the short h While more recently
in a 45-year-old woman, as in Case 2-1, term.10 Clinicians should not hesitate neurologists have used
given that her symptoms and disabil- to start levodopa if motor disability levodopa-sparing
ity were mild and that thin, younger warrants it. The watch and wait strategy therapies or
levodopa-delaying
therapies in younger
Case 2-1 patients with Parkinson
A 45-year-old left-handed woman presented, accompanied by her husband, disease, physicians
with a 6-month history of tremor in her left hand. The tremor occurred at rest should not hesitate to
and improved with action. She worked as a seamstress and had noticed start levodopa if motor
that using her left hand to manipulate fabric had become more challenging disability warrants it.
and time consuming. Her productivity had minimally declined. She had some
mild depression and anxiety that had increased in anticipation of this visit.
Her past medical history was unremarkable except for mild depression
and some intermittent anxiety that had been controlled with sertraline
50 mg/d. She lived with her husband and had three children at home, ages
16, 12, and 10. She enjoyed playing tennis and practicing yoga. There
was no family history of Parkinson disease (PD).
Examination revealed that the patient was thin (49 kg [108 lbs]) and had
very mild facial masking and good vocal volume. She had an intermittent
resting tremor of the left hand with slowed rapid alternating movements
on the left, but intact on the right. She had some micrographia. She had a
steady gait with reduced arm swing on the left and had normal turns, stride
length, and a normal pull test. Prior workup had demonstrated normal
ceruloplasmin and 24-hour urine copper testing with a normal slit-lamp
examination on ophthalmologic examination.
She was diagnosed with PD, and many possible therapies were discussed,
including the watch and wait strategy, levodopa, dopamine agonists,
monoamine oxidase type B inhibitors, amantadine, and anticholinergics.
She opted to take the monoamine oxidase type B inhibitor rasagiline 1 mg/d
and to increase her exercise regimen through an intensive whole-body,
amplitude-based therapy and a new membership at her local gym.
Comment. A young patient with early PD who has mild motor symptoms
has the most options for therapy. It is always important to consider Wilson
disease and genetic contributions in patients with young-onset PD. Younger
patients with PD have many life considerations that a retired, elderly patient
may not have (eg, children at home and an ongoing occupation). In this case,
the patient had a mild disturbance at work with a minimal decrease in
productivity as a seamstress. Her neurologic examination demonstrated mild
parkinsonism, and she had good function overall in her activities of daily
living. There were numerable therapeutic options to choose from in her case,
of which this patient chose rasagiline and exercise as initial therapy.

Continuum (Minneap Minn) 2016;22(4):1064–1085 www.ContinuumJournal.com 1067

Copyright © American Academy of Neurology. Unauthorized reproduction of this article is prohibited.


Parkinson Disease Motor Symptoms

TABLE 2-2 Medications Used in the Initial Therapy of Early Parkinson Disease

Maintenance
Generic Name Mechanism Initial Dosage Dosage Major Side Effects
Amantadine Multiple 100 mg 100 mg 2 to Nausea, confusion,
(N-methyl-D-aspartate 2 times a day 3 times a day hallucinations, leg edema,
[NMDA] receptor livedo reticularis
antagonist, potentiates
dopamine release)
Benztropine Anticholinergic 0.5 mg every 1 mg 2 to Dry mouth, reduced
night at 3 times a day cognition, urinary
bedtime retention, hallucinations,
blurry vision, constipation
Bromocriptine Dopamine agonist 1.25 mg 5Y10 mg 2 to Nausea, impulse control
(ergot derived) 2 times a day 3 times a day disorders, hallucinations,
orthostatic hypotension,
fibrosis, sleep attacks
Carbidopa/levodopa Aromatic amino 25 mg/100 mg Variable Nausea, orthostatic
acid decarboxylase 2 times a day hypotension, confusion,
inhibitor/dopamine hallucinations, dizziness
precursor
Carbidopa/levodopa Converted to 23.75 mg/95 mg Variable Nausea, orthostatic
extended release dopamine 3 times a day hypotension, confusion,
capsules hallucinations, dizziness
Pramipexole Dopamine agonist 0.125 mg 2 to 0.5Y1.5 mg 2 to Nausea, hallucinations,
(nonYergot derived) 3 times a day 3 times a day impulse control disorders,
orthostatic hypotension,
sleep attacks, leg edema
Pramipexole Dopamine agonist 0.375 mg/d 1.5Y4.5 mg/d Nausea, hallucinations,
extended release (nonYergot derived) impulse control disorders,
orthostatic hypotension,
sleep attacks, leg edema
Rasagiline Monoamine oxidase 0.5Y1 mg/d 1 mg/d Dizziness, flu syndrome,
type B inhibitor arthralgia
Ropinirole Dopamine agonist 0.25 mg 2 to 2Y8 mg 2 to Nausea, hallucinations,
(nonYergot derived) 3 times a day 3 times a day impulse control disorders,
orthostatic hypotension,
sleep attacks, leg edema
Ropinirole Dopamine agonist 2 mg/d 8Y24 mg/d Nausea, hallucinations,
extended release (nonYergot derived) impulse control disorders,
orthostatic hypotension,
sleep attacks, leg edema
Rotigotine Dopamine agonist 2 mg/d 6 mg per 24 hours Nausea, hallucinations,
(nonYergot derived) (maximum dose) impulse control disorders,
orthostatic hypotension,
sleep attacks, leg edema,
and may have patch
application site reactions
Continued on page 1069

1068 www.ContinuumJournal.com August 2016

Copyright © American Academy of Neurology. Unauthorized reproduction of this article is prohibited.


TABLE 2-2 Medications Used in the Initial Therapy of Early Parkinson Disease Continued from page 1068

Maintenance
Generic Name Mechanism Initial Dosage Dosage Major Side Effects
Selegiline Monoamine oxidase 5 mg 1 to 5 mg 2 times Confusion, hallucinations,
type B inhibitor 2 times a day a day orthostatic hypotension,
insomnia, nausea, benign
cardiac arrhythmias
Trihexyphenidyl Anticholinergic 2 mg/d 2 mg 2 to Dry mouth, reduced
3 times a day cognition, urinary
retention, hallucinations,
blurry vision, constipation

is also falling out of favor by many agonists are also available as once-daily
clinicians given that untreated patients extended-release (pramipexole ex-
have a worse self-rated quality of life over tended release, ropinirole extended
time compared to those who receive release) and patch (rotigotine) formu-
treatment around the time of diagnosis.11 lations. Currently, for the patient with
Of the many options, the patient in early PD, no evidence exists of differ-
Case 2-1 chose monoamine oxidase ences in efficacy or in the propensity
type B (MAO-B) inhibitor therapy with to induce motor complications or side
rasagiline 1 mg/d. In practice, this med- effects with longer-acting dopamine
ication is well tolerated, is taken once agonists versus regular release.
daily, and provides mild symptomatic Anticholinergics (benztropine,
improvement. Many patients taking ra- trihexyphenidyl) have largely fallen out
sagiline may not notice obvious benefit of favor due to potential adverse cog-
after several weeks, and it may take nitive effects, but they may be useful in
8 to 12 weeks to notice full benefit.12 younger, cognitively intact patients with
Selegiline 5 mg 2 times per day would tremor.15 Side effects can also include
also be expected to provide similar be- dry mouth, blurry vision, worsened con-
nefit. Patients on rasagiline or selegiline stipation, and urinary retention.
will eventually need additional therapy Amantadine would also be useful as a
after 6 months to 2 years, depending potential therapy for the young patient
on the patient. with early PD, as in Case 2-1. Dosing is
Other options (outside of levodopa) typically 100 mg titrated to 2 or 3 times
in the early, young patient include do- daily. Side effects with this medication
pamine agonists, anticholinergics, and include leg edema, livedo reticularis,
amantadine. All of these medications and, rarely, corneal edema. It also has
could provide some benefit. Dopamine mild anticholinergic properties and can
agonists (bromocriptine, pramipexole, lead to hallucinations and negative
ropinirole, rotigotine) are more apt to cognitive effects as well.
cause neuropsychiatric issues such as A younger patient with early PD is
hallucinations, impulse control disor- very different from an older, retired
ders, and somnolence as potential side patient. Physicians should try to un-
effects.13 In addition to motor benefit, derstand where the patient is in his or
there is some benefit of additional anti- her life with regard to children, career,
depressant effects.14 Some dopamine and concerns about long-term financial

Continuum (Minneap Minn) 2016;22(4):1064–1085 www.ContinuumJournal.com 1069

Copyright © American Academy of Neurology. Unauthorized reproduction of this article is prohibited.


Parkinson Disease Motor Symptoms

KEY POINTS
h Exercise, if safe for the issues, and should consider the fears Therapeutic Options in Older,
patient, should be a part a patient might have for a future living Cognitively Intact Patients
of every patient’s with PD. One of the most important With Parkinson Disease
regimen to battle against tasks for the clinician and the care In most patients with a typical age of
Parkinson disease. This team is to help the patient accept his onset (late fifties to early sixties), there
not only includes physical or her diagnosis and decide to battle are numerous pharmacologic and
exercise, but cognitive against the disease through all means nonpharmacologic treatments that can
exercise such as reading, at their disposal. improve motor symptoms and quality
crossword puzzles, and of life (Case 2-3).
current events. Complementary and Data indicate that dopamine ago-
h Many younger patients Alternative Therapies nists of all types are effective in the
with Parkinson disease Be aware that many younger patients treatment of motor symptoms of early
embrace alternative with PD may want to pursue alternative PD. Ropinirole, pramipexole, and
therapies, and it is therapies and modalities for treat- rotigotine are nonYergot-derived dopa-
important, as a clinician, ment of their PD motor and non- mine agonists with multiple clinical trials
to work within the
motor symptoms. In many series of supporting their use in untreated PD.20
patient’s framework as
patients with PD throughout the world, Ergot-derived dopamine agonists were
much as possible. At the
same time, discouraging
use of alternative therapies has been previously used in practice but have
use of unproven, as high as 75% in some cohorts.16 The fallen out of favor given potential fi-
expensive therapies that data supporting the use of most alter- brosis of pleuropulmonary tissues and
are highly unlikely to native therapies such as vitamins and heart valves.21 Pergolide is now off the
provide any subjective or nutritional supplements, acupuncture, market in the United States, but bro-
objective benefit is key. or manual therapies are scant, how- mocriptine remains available, although
ever.17 Some more promising therapies it is used extremely rarely for treating
that have emerged with better study PD today.
design and data are speech therapy for Some data offer evidence of an
hypophonia in PD and physical and antidepressant effect with dopamine
occupational therapy focused on inten- agonists (especially pramipexole), and
sity and amplitude of movement. Tai they clearly improve quality of life.14,22
chi for gait and balance, tandem cycling, While sudden onset of sleep is a black
and other therapies have also proven box warning for dopamine agonists,
to be useful in recent work.18,19 sudden onset of sleep does not occur
Many patients with PD embrace commonly in practice.23 However, pa-
exercise and alternative therapies, and tients with PD who are driving need
as a clinician, it is important to work to be advised about sleep risks of all
within the patient’s framework as much drugs. Lower extremity edema is not
as possible to improve quality of life. uncommon in longer-term therapy with
Discouraging use of unproven, expen- dopamine agonists and is often unrec-
sive therapies that are highly unlikely ognized as a side effect by neurologists
to provide any subjective or objective and primary care physicians alike.24 The
benefit, however, is also very important. major issue with these medications is
impulse control disorders that can
Therapeutic Options in Older emerge at any stage, although generally
Patients With Parkinson Disease occur early on in treatment.25 Often,
and Cognitive Impairment changes in behavior are subtle and do
In an older patient with significant co- not necessarily fulfill the impulse con-
morbidities and significant motor dis- trol disorder criteria of the Diagnostic
ability, therapeutic options are usually and Statistical Manual of Mental
more restricted (Case 2-2). Disorders, Fifth Edition (DSM-5).26

1070 www.ContinuumJournal.com August 2016

Copyright © American Academy of Neurology. Unauthorized reproduction of this article is prohibited.


KEY POINTS

Case 2-2 h Levodopa is the drug


of choice for patients
A 77-year-old right-handed man with hypertension, diabetes mellitus,
with significant
hyperlipidemia, osteoarthritis, and coronary artery disease presented
comorbidities, more
with a 1-year history of resting tremor of the right hand, slowness of the
advanced age, postural
right side, and dragging of the right leg. Symptom onset had been
instability, or advancing
gradual, but his wife had wondered if he had experienced a stroke that
disability in activities
caused these symptoms. He had had an MRI of the brain demonstrating
of daily living
cerebral atrophy with moderate microischemic white matter disease and
or occupation.
an old left thalamic lacune. He had been a little more forgetful, according
to his wife, but had been doing all of his activities of daily living fairly well. h Physicians should exercise
He had had intermittent rapid eye movement (REM) sleep behavior caution when prescribing
disorder symptoms for at least 10 years. dopamine agonists,
His medications included propranolol, lisinopril, rosuvastatin, aspirin, amantadine, monoamine
and metformin. He had not been exposed to neuroleptics. He lived in a oxidase type B inhibitors,
single-level home with his wife and played golf frequently as a retiree. or anticholinergic agents
He routinely participated in his church services as a lay minister. in patients presenting
Examination demonstrated a Montreal Cognitive Assessment (MoCA) with significant
score of 23 out of 30, with evidence of some visuospatial and executive cognitive impairment.
dysfunction, with 2 points missed for delayed recall. He had facial
masking and some hypophonia. He had a resting tremor that was mild
in his right hand and had bradykinesia with sequence effect/decrement on
finger taps on the right and somewhat on the left as well. Cogwheel
rigidity was evident at the right elbow and wrist. He had no signs of
ataxia and had mild distal sensory loss. The patient’s gait was slightly
hunched with some shuffling and mild dragging of the right leg. He
turned fairly well and had mild retropulsion, taking five steps on the
pull test, but recovering unaided. He was diagnosed with early
Parkinson disease (PD).
He was initiated on carbidopa/levodopa 25 mg/100 mg titrated
to 1 tablet 3 times per day before meals. Thyroid-stimulating hormone
(TSH), rapid plasma reagin (RPR), and vitamin B12 testing were all
unremarkable. Neuropsychological testing revealed mild cognitive
impairment with executive and visuospatial dysfunction and mild memory
impairment typical of PD. He and his wife returned for a follow-up
visit 1 month later, and he noted significant improvement in his tremor,
slowness, and rigidity. He had begun reading books a lot and staying
aware of current events.
Comment. In an older patient with multiple comorbidities and some
cognitive impairment, it is perhaps best to go straight to levodopa as a first
therapeutic agent in order to avoid potential adverse neuropsychiatric
effects of other medications. Patients with PD and mild cognitive
impairment are more prone to hallucinations and other neuropsychiatric
effects with dopamine agonists, amantadine, monoamine oxidase type B
inhibitors, and anticholinergics.
Another consideration is the impairment and disability of the patient. If
patients experience significant decline in their activities of daily living,
have a change in gait that makes them more prone to falls, or if they have
an expected shorter time frame of therapy (eg, cancer, advanced age,
comorbid disease that is likely to shorten lifespan) then it would be most
prudent to start levodopa.
Continued on page 1072

Continuum (Minneap Minn) 2016;22(4):1064–1085 www.ContinuumJournal.com 1071

Copyright © American Academy of Neurology. Unauthorized reproduction of this article is prohibited.


Parkinson Disease Motor Symptoms

KEY POINT
h When possible, the dose
Continued from page 1071
Remember that levodopa provides the most effective motor benefit for
of levodopa should be
PD, and while there are risks of motor fluctuations and dyskinesia with this
kept lower to reduce
medication, it should be used in any patient with PD regardless of age
the risk of wearing off
or other factors when a patient experiences significant disability (in activities
and dyskinesia.
of daily living or occupation) due to motor symptoms. When possible,
keeping the dose of levodopa lower, however, reduces risk of developing
dyskinesias and motor fluctuations. In many cases, when motor complications
do initially arise, they are not severe, and there are numerous adjunctive
therapies that can ameliorate wearing off and reduce dyskinesia.

Case 2-3
A 62-year-old right-handed man with hypertension and hyperlipidemia
presented for slowness and stiffness of his left side. He had not noticed much,
if any, tremor on either side of his body, but his wife had noticed an occasional
lower lip tremor. He was somewhat bothered in his daily life (work and
running) by the left-sided slowness and stiffness. He worked as a teacher and
did research as a biologist at the local college; he had been doing well in his
daily function and had received great reviews by students and his superiors.
He was taking two medications, hydrochlorothiazide and atorvastatin.
He was active and ran 15 to 20 miles per week. He had noticed that it took
him longer to run 4 miles than it did before. His handwriting did not appear
to be affected yet, and he wanted to keep teaching until he reached 68,
when his mortgage on his house would be paid off.
Examination demonstrated normal cognition with a score of 29 out of
30 on the Montreal Cognitive Assessment (MoCA) and exquisite recall of
current news events. He had mild bradykinesia and cogwheel rigidity of the
left upper extremity and left lower extremity that worsened with contralateral
distraction. He did have a mild occasional lower lip tremor. His gait was
steady with great turns, reduced arm swing on the left, and a normal pull
test. There were no signs of ataxia, and his orthostatic blood pressures were
normal. The patient was diagnosed with Parkinson disease (PD). After
discussing the options with the patient and his wife, the patient decided to
start a dopamine agonist, pramipexole, titrating from 0.125 mg 2 times a day
to 0.75 mg 2 times a day over 3 weeks. He was counseled about potential
side effects, including nausea, hypersomnolence (including sudden onset of
sleep), edema, hallucinations, and impulse control disorders. Screening for
impulse control disorders (eg, hypersexuality, gambling, overshopping) at
baseline was negative.
The patient returned 6 weeks later and noted significant improvement
in his left-sided slowness and rigidity. He had noticed some nausea and
somnolence within an hour after each dose of pramipexole, but it was
mild and did not affect his ability to drive, perform activities of daily living,
or work. These side effects improved over time. His wife noted that his
lip tremor had improved somewhat as well. He had not had symptoms of
an impulse control disorder or hallucinations.
Two years later, his symptoms had gradually progressed, and he was
interested in further therapy for slowness and tremor that had become
apparent and more bothersome in his left hand.
Continued on page 1073

1072 www.ContinuumJournal.com August 2016

Copyright © American Academy of Neurology. Unauthorized reproduction of this article is prohibited.


KEY POINTS
Continued from page 1072 h It is important to consider
Pramipexole was continued, and he was initiated on carbidopa/levodopa all pharmacotherapies in
25 mg/100 mg, titrating to 1 tablet 3 times per day, 45 minutes before patients with early
meals (to avoid competition for absorption from protein in his meals), with Parkinson disease and
significant benefit in his parkinsonian symptoms. to consider the patient’s
Comment. There are many choices in treating older patients with PD goals as primary
and, in this case, the patient chose to start a dopamine agonist after in helping them select
discussion with his neurologist. For this patient, starting levodopa would what medication
also be quite acceptable, and clinicians should not withhold levodopa from they begin.
patients due to fear of motor complications. There were some dopaminergic
side effects that occurred (somnolence and nausea) with the dopamine h Clinicians should not
agonist, but these had improved (somnolence) or resolved (nausea) over withhold levodopa from
time, and the patient experienced motor benefit from the medication. patients due to fear of
motor complications.
h Always screen for
The patients most at risk of impulse start to experience a range of fluctua- impulse control disorders
control disorders seem to be younger tions in their response to levodopa.8 in patients with
men and those with a family history of At this stage, patients report that the Parkinson disease and
addiction. Other side effects of dopa- counsel patients and
benefit does not last up to the next
mine agonists include triggering hallu- families about the
dose, and they notice a return of PD potential for these issues
cinations and delusions, although this motor symptoms (tremor, slowness, to arise during therapy
is more common in advanced PD rigidity), termed wearing off. Patients with any dopaminergic
when used as adjunct therapy. become aware of a return of symptoms drug, but especially with
ADVANCED PARKINSON DISEASE if they are late taking a dose or miss a dopamine agonists.
This section reviews the methods of dose and may experience nighttime
treating the patient with advancing PD. or early morning symptoms prior to
Patients with advancing PD have often the first dose of the day (Case 2-4). If
had their symptoms well controlled on motor symptoms are generally reli-
oral medications for a few years but will ably worsened (ie, predictable wear-
begin to develop fluctuations in re- ing off) toward the end of a levodopa
sponse to medications, as well as symp- dose, recognizing this problem is
toms that emerge or become resistant easy. However, many subjects experi-
to dopaminergic therapies. ence a range of nonmotor symptoms
Three common scenarios will be either alone or associated with motor
reviewed: symptoms during off periods, and it
1. PD with predictable on-off may not always be apparent to the
fluctuations patient that these are related to the
2. Advanced PD with motor timing of their medications (Table 2-3).
complications despite optimal The most common of such symptoms
medical therapies include mood issues such as anxiety
or nonspecific fatigue.27 Autonomic
3. PD with motor symptoms
symptoms can also occur such as
that are resistant to urinary frequency and pain, and
dopaminergic therapies nonspecific sensory symptoms (eg,
numbness, paresthesia) are frequent
Parkinson Disease With off-period symptoms. The challenge
Predictable Fluctuations comes in recognizing the timing in
After a few years of levodopa therapy relation to the levodopa dosing.
(the average is 2 to 3 years or, in some Preventing the development of
cases, longer), patients with PD may bothersome fluctuations in response to

Continuum (Minneap Minn) 2016;22(4):1064–1085 www.ContinuumJournal.com 1073

Copyright © American Academy of Neurology. Unauthorized reproduction of this article is prohibited.


Parkinson Disease Motor Symptoms

Case 2-4
A 66-year-old man with a 3-year history of Parkinson disease (PD) presented
with uncomfortable curling of the toes in his right foot on waking up
several mornings per week. This was often associated with feeling anxious.
During the day he managed reasonably well, but reported that he
experienced more right hand tremor and slower dexterity midafternoon
and at bedtime. He was on carbidopa/levodopa 25 mg/100 mg 1 tablet
every 4 to 5 hours (total of 4 tablets per day), plus selegiline 10 mg/d. He had
anxiety and was also on citalopram 10 mg/d. Examination 2 hours
postYcarbidopa/levodopa revealed a good ‘‘on’’ response with some mild,
nonbothersome neck chorea.
He was advised to take carbidopa/levodopa 30 minutes before meals, and
the fourth dose was switched to a controlled-release carbidopa/levodopa
25 mg/100 mg taken at bedtime. He had no known risk factors for impulse
control disorders, apart from gender, and pramipexole was cautiously
started and titrated up to 0.5 mg 3 times a day with better overall
symptom control.
He was reassessed 4 months later and had less off symptoms but was noted
to be spending more time at night on the computer on an online gambling
site. His family expressed concern, although he was not aware of the change
in behavior. Pramipexole was tapered and stopped, with resolution of his
gambling. Carbidopa/levodopa dosing was increased to 5 tablets per day,
with 1 tablet every 3.5 to 4 hours, and the fifth dose at bedtime of
controlled-release carbidopa/levodopa. Although he had more neck chorea
in the evening, this was not bothersome, and he was happy with his PD control.
Comment. The patient had developed early-morning off-period dystonia
of the foot with nonmotor off symptoms as well as wearing off midafternoon,
which may have been due to effect of food on lunchtime dosing. Evaluating
issues related to poor absorption of the levodopa (eg, timing of medications
in relation to food and constipation) can help. He had head chorea
already (a common issue in managing these patients) and so adding in a
catechol-O-methyltransferase inhibitor, entacapone, will increase dyskinesia.
Dopamine agonists are helpful for reducing wearing off and can also help
depression but must be used with caution due to impulse control disorders, as
occurred in this patient.

levodopa is key to maintaining the suggest that current treatment strate-


quality of life of patients with PD. Motor gies are working and that marked,
fluctuations are due to progressive disabling dyskinesia is less common
disease, combined with chronic levo- than in prior years.30 In general, sub-
dopa use. The frequency increases jects with akinetic-rigid PD appear
with time but is generally estimated at less likely to develop fluctuations, but
10% per year after starting levodopa. also commonly have less effective anti-
Epidemiologic studies (over the past parkinsonian benefit of levodopa com-
2 decades) have consistently reported pared to tremor-dominant PD.31 Such
factors associated with a higher risk subjects, however, may tolerate higher
of developing fluctuations, including levodopa doses if required.
younger age at disease onset, longer Managing patients with PD with
use of levodopa, and higher individual fluctuations is a stepwise process
doses of levodopa.28,29 Recent reviews (Table 2-4).32 Initially altering the

1074 www.ContinuumJournal.com August 2016

Copyright © American Academy of Neurology. Unauthorized reproduction of this article is prohibited.


TABLE 2-3 Phenomenology of Motor and Nonmotor Complications of Advanced Parkinson
Disease in Relation to Levodopa Dosing Cycle

b Motor Symptoms
Off State
Predictable wearing off: Reemergence of Parkinson disease (PD) symptoms before next dose
of medication
Unpredictable off: Random, sudden return of PD symptoms, not related to timing of levodopa
Dose failure/partial response: A dose of levodopa that has a delayed effect (delayed on) or no effect
or a reduced effect (dose failure)
Off freezing: A transient difficulty in initiating or continuing a movement (eg, when starting to walk
[start hesitation], while turning [turning hesitation], when going through a narrow doorway, or due
to sudden stress or anxiety [startle hesitation])
Off-period dystonia: Usually painful affecting distal leg, foot, toes with abnormal postures
On State
Peak-dose dyskinesia: Mixed chorea and dystonia of neck and limbs; increases with mental and
physical activity
On freezing: Rare but may occur
Transitional state
Diphasic dyskinesia: Legs, dystonic and high amplitude; stereotypic kicking, ‘‘funny’’ gait
Beginning-of-dose worsening and end-of-dose rebound: A transient worsening of symptoms at the
beginning of dose or end of a dose, often presenting as an increase in tremor
b Nonmotor Symptoms
Off State
Neuropsychiatric: Anxiety, depression, irritability, panic attacks, apathy, fatigue, verbal fluency,
attention, executive functions
Autonomic: Sweating, facial flushing, pallor, hyperthermia, cold limb extremities (hands and feet),
urinary disturbances, bloating, abdominal discomfort, drooling, dyspnea (air hunger)
Pain/sensory: neuropathic, dysesthesia, numbness, paresthesia, restlessness/akathisia
On State
Neuropsychiatric: Euphoria, agitation, illusions, hallucinations, delusion, paranoia, distracted, impulsive

timing of levodopa doses is essential, ment to improve efficacy.34 In prac- KEY POINT
such as reducing the time between tice, a lower dose of levodopa more h Wearing off is generally
each levodopa dose to 4 hours or less frequently becomes the best option more bothersome to
patients than on-period
so that subjects take 4 or 5 doses per for managing motor complications.
dyskinesia, so clinicians
day (as in Case 2-4). The longer-acting Gastrointestinal factors can be very
should focus on
carbidopa/levodopa, recently approved important in some individuals. Sub- strategies to improve
by the US Food and Drug Administra- jects may notice an effect after eating, off symptoms.
tion (FDA), may also be useful to im- especially after consuming meat pro-
prove duration of action.33 Other novel tein, which reduces the efficacy of
levodopa preparations are in develop- levodopa action (Case 2-4). Such

Continuum (Minneap Minn) 2016;22(4):1064–1085 www.ContinuumJournal.com 1075

Copyright © American Academy of Neurology. Unauthorized reproduction of this article is prohibited.


Parkinson Disease Motor Symptoms

a
TABLE 2-4 Drug Therapies for Levodopa-Induced Complications

Complication Class Drug Side Effects


Wearing off Dopamine agonist Ropinirole immediate Impulse control disorders, sleep
release and extended release, disorders, peripheral edema,
pramipexole immediate hypotension, hallucinations
release and controlled release
Rotigotine patch Impulse control disorders, sleep
disorders, peripheral edema,
hypotension, hallucinations,
skin reactions
Apomorphine subcutaneous Impulse control disorders, sleep
disorders, peripheral edema,
hypotension, hallucinations,
skin reactions, nausea
Catechol-O- Entacapone Abdominal pain and
methyltransferase diarrhea, usually delayed; urinary
inhibitor discoloration; increased
peak-dose dyskinesia
Monoamine oxidase Selegiline, rasagiline Increased peak-dose dyskinesia;
type B inhibitor counsel patients about potential
drug interaction with co-use of
certain serotonergic drugs and
sympathomimetic drugs; there are
no food interaction concerns
Levodopa preparations Carbidopa/levodopa Variable absorption with food
controlled release causes unpredictable efficacy if
used during the day
Carbidopa/levodopa
extended release
Off-period Any of the Any of the above drugs Any of the above side effects,
dystonia above classes depending of the drug chosen
Anticholinergic Many (eg, trihexyphenidyl) Confusion, drowsiness, dry mouth,
constipation
Focal off-period Botulinum toxin Botulinum toxin A Transient weakness
painful dystonia
Diphasic Dopamine agonist Same as above Same as above
dyskinesia
Peak-dose N-Methyl-D-aspartate Amantadine Peripheral leg edema, livedo
dyskinesia (NMDA) antagonist reticularis, confusion, hallucinations,
corneal edema, myoclonus
a
Reviewed in Fox et al, 2011,20 Fox et al, 2015,7 Lang and Connolly, 2014,13 and Giugni and Okun, 2014.32

individuals may also notice fluctuations Thus, delayed effects or even no effect
in benefit that are unrelated to the of a dose can be due to food in-
timing of the medications (termed teractions. Levodopa competes with
unpredictable offs and sudden offs). protein amino acids for transport across

1076 www.ContinuumJournal.com August 2016

Copyright © American Academy of Neurology. Unauthorized reproduction of this article is prohibited.


KEY POINTS
the gastrointestinal tract as well as across Inhibition of enzymatic degrada- h If a patient has
the blood-brain barrier. Recent studies tion of levodopa with the catechol-O- levodopa-induced
suggest Helicobacter pylori in the methyltransferase (COMT) inhibitor, peak-dose dyskinesia
stomach may also affect levodopa ab- entacapone, has a mild benefit on already, then adding
sorption and exacerbate fluctuations.35 wearing off; however, there are min- in a monoamine
Delayed gastric emptying reduces ab- imal side effects, so tolerance is good. oxidase type B inhibitor
sorption of levodopa as well and is a When entacapone is prescribed, dos- (selegiline or rasagiline),
part of the disease process in PD, but ing is required with each dose of or a catechol-O-
is worsened by the presence of food levodopa and compliance can be methyltransferase inhibitor
in the stomach, constipation, or use of improved with the combined tablet (entacapone) will likely
increase dyskinesia.
anticholinergics. Thus, taking levodopa (carbidopa/levodopa and entacapone).
on an empty stomach (30 minutes Tolcapone (administered 3 times h When prescribing
prior to food) and actively managing daily) is both a peripheral and centrally dopamine agonists for
constipation are important lifestyle is- acting COMT inhibitor that may have patients with Parkinson
disease, impulse control
sues to review in patients with PD. Be greater efficacy, but its use is limited
disorders and milder
especially careful with patients who due to risk of elevated liver trans-
variations are far more
have marked dietary interactions with aminases causing fatal hepatotoxicity, common than clinical
medications as they may lose weight which occurred prior to now manda- studies suggest.
due to extreme protein avoidance and tory monitoring of liver function tests Physicians should always
even develop anorexia. Patients with and recognition of the need for mon- have a very high index
PD often lose weight as part of the itoring. The other add-on drug class is of suspicion and should
disease process, and additional calorie the MAO-B inhibitor, either selegiline withdraw the offending
reduction can be challenging. Other or rasagiline. (Safinamide is a new drug slowly to reduce the
nonpharmacologic factors that may MAO-B inhibitor recently approved in risk of dopamine agonist
impact levodopa include effects of Europe.) Compliance is easy due to withdrawal syndrome.
anxiety and stress that may lead to once or twice daily dosing and good h Coprescribing certain
reduced benefit or even sudden loss tolerance. No comparative trials have antidepressants with
of benefit. occured between rasagiline and rasagiline/selegiline as
Specific medication options for selegiline. Side effects of selegiline add-on therapy has
patients with Parkinson disease with can include insomnia due to amphet- been shown to be safe
from use in phase 3
predictable wearing-off. Several op- amine metabolites; thus, selegiline
randomized controlled
tions exist to add on to levodopa to may be helpful in patients with day-
trials. These include
reduce off times (Table 2-4). In gen- time fatigue. Coprescribing with sero- sertraline 100 mg/d,
eral, clinical trials with all these agents tonergic drugs can theoretically cause citalopram 20 mg/d,
have shown an overall decrease in serotonin syndrome. However, co-use paroxetine 30 mg/d,
daily off time, between 1 and 2 hours of an MAO-B inhibitor with several fluvoxamine 150 mg/d,
per day.7,20,36 All such add-on thera- commonly used antidepressants has trazodone 100 mg/d,
pies can increase peak-dose dyskine- been shown to be safe from allowed and amitriptyline
sia (refer to the following section use in randomized control trials. These 50 mg/d.
on the clinical features of dyskinesia). include sertraline 100 mg/d, citalopram
All the clinically available dopamine 20 mg/d, paroxetine 30 mg/d, fluvoxamine
receptor agonists, ropinirole, prami- 150 mg/d, trazodone 100 mg/d, and
pexole, or rotigotine added into levo- amitriptyline 50 mg/d. Patients who
dopa, will improve wearing off, and have both PD and depression should
there is currently no difference in effi- not be deprived of antidepressants, if
cacy between these drugs. The main clinically required.
limiting factor in use of all dopamine Clinical features of dyskinesia.
agonists is the risk of impulse con- Involuntary movements (levodopa-
trol disorders. induced dyskinesia), usually begin
Continuum (Minneap Minn) 2016;22(4):1064–1085 www.ContinuumJournal.com 1077

Copyright © American Academy of Neurology. Unauthorized reproduction of this article is prohibited.


Parkinson Disease Motor Symptoms

KEY POINTS
h Clinicians should assess after a few years of levodopa therapy. side or upgaze eye movements) or res-
the timing of movements Most commonly, levodopa-induced piratory muscles (breathlessness). Many
in relation to the dose of dyskinesia will start on the side of the patients with PD are unaware of such
levodopa as high dose body initially affected by PD. Managing peak-dose dyskinesia, unless severe,
(peak-dose of levodopa dyskinesia involves identifying the time and it is often family members who will
action) or low dose when the movements occur in relation comment on the presence of dyskine-
(onset and end of to a dose of levodopa (Table 2-3). sia. Thus, treating peak-dose levodopa-
levodopa dose, or in the Thus, one simple classification relates induced dyskinesia should be targeted
off periods) to help to the level of levodopa: high-dose to patient disability, and clinicians
classify and then treat the dyskinesia occurs when levels of dopa- should only treat dyskinesia that is inter-
type of dyskinesia.
mine stimulation are at their highest fering with daily life. Many patients,
h Often, patients with and is the classic peak-dose dyskinesia. even when aware of involuntary move-
Parkinson disease are The opposite is low-dose dyskinesia, ments, prefer to be on with some dys-
unaware of the which occurs when the levels of dopa- kinesia than off without any dyskinesia.
movements, especially
mine are low. The most common type Dyskinesia occurring toward the
chorea, and as such do
of high-dose dyskinesia occurs at the end of a dose of levodopa or between
not report any disability.
Painful dystonia is more
peak level of levodopa action (ie, 1 to doses (off periods) tends to be pre-
likely to be reported. 2 hours post dose), called peak-dose dominantly dystonic and affects the legs
dyskinesia (Case 2-5), and is a mixture and feet. Patients often experience a
h Not all dyskinesia needs
of chorea and dystonia affecting the more fixed posture (eg, ankle intorsion
to be treated. Rather,
focus on reducing only
neck, limbs, and trunk, with rarer high- with toe flexion or extension), especially
disabling, bothersome amplitude limb ballism. Rarely, myoclo- in the early morning (early-morning
dyskinesia with medical/ nus or even dyskinesia can occur dystonia). As off-period dystonia can be
surgical strategies. affecting the eyes (including side-to- painful and disabling, patients are more

Case 2-5
A 62-year-old woman with Parkinson disease (PD) presented for a follow-up
visit with neck pain. She had a history of PD for 6 years, which started with
left hand tremor, and was on carbidopa/levodopa 25 mg/100 mg 1.5 tablets
every 4 hours, 4 doses a day, with 1 tablet of entacapone with each dose,
and ropinirole 3 mg 3 times per day. She worked as a teacher, and her
students reported that she moved her head when talking to them. She
otherwise managed well throughout the day, but had some nighttime
insomnia and felt sleepy in the afternoons. On examination, 2 hours after
her last dose of levodopa, she had neck chorea and mild finger chorea
and dystonia in the left hand but with no tremor, and her gait was normal.
The entacapone was tapered and stopped with reduced neck chorea
and less pain. She did not want to reduce ropinirole, as she had tried this
previously but felt anxious and had more tremor, including an inner sense
of tremulousness. She was aware of a slightly increased appetite, especially
for sweets.
Comment. The patient’s pain was likely due to the involuntary movements
in her neck on a background of some mild degenerative spine disease.
Rarely, some patients can develop spinal cord lesions from severe involuntary
neck movements. Patients often prefer to put up with some mild dyskinesia
rather than the PD symptoms. Ropinirole is likely exacerbating sleep
disturbances and causing increased eating, but she had previously had a
mild dopamine agonist withdrawal syndrome on trying to stop taking the
drug. Inner tremor is common in PD and often associated with anxiety.

1078 www.ContinuumJournal.com August 2016

Copyright © American Academy of Neurology. Unauthorized reproduction of this article is prohibited.


KEY POINT
aware and will report this symptom. fective add-on drug for peak-dose dys- h Referral to a specialist
Treatment thus involves preventing and kinesia (Table 2-4). Dosing should be center and careful
treating off periods. A rarer form of dys- away from the bedtime period due to a selection criteria are
kinesia, occurring at the beginning or risk of insomnia. A longer-acting prep- key to successful surgery
end of a dose of levodopa, when the aration is under development that may for patients with
levels of levodopa are rising and falling, is Parkinson disease.
reduce nocturnal side effects.38
called diphasic dyskinesia or dystonia-
improvement-dystonia. Diphasic dyski- Severe Parkinson Disease With
nesia tends to be dystonic, affecting the Advanced Motor Fluctuations
legs with typical stereotypic alternating Refractory to Oral Therapies
kicking leg movements (Supplemental
On-off fluctuations is the term used to
Digital Content 2-1, links.lww.com/
describe predictable, or often unpre-
CONT/A175). Walking can be affected
dictable, switching from being ‘‘on’’ and
with high-stepping and so-called ‘‘funny’’
mobile with dyskinesia to being ‘‘off’’
gaits. In many cases, a mixture of
and immobile.
parkinsonian signs, and often tremor,
Surgical options. DBS of the STN,
is associated with dystonic posture of
or more recently, DBS of the globus
the contralateral foot and arm (Supple-
pallidus internus (GPi) are the most
mental Digital Content 2-2, links.lww.
common targets for the patient with PD
com/CONT/A176). Increasing dopa-
with advanced motor fluctuations.
mine levels, with dopamine agonists, Comparison studies of STN versus GPi
for example, can be helpful. Rarer as targets currently suggest both are
causes of diphasic dyskinesia have been clinically effective, but GPi DBS may
seen in subjects with PD who under- have fewer neuropsychiatric side ef-
went fetal transplants as well as rarely in fects; however, this is not clear.39 In
subjects with bilateral subthalamic general, surgery has been shown to be
nucleus (STN) DBS as a form of superior to best medical therapy for the
stimulation-induced dyskinesia. management of fluctuations and dyski-
Managing dyskinesia. The main nesia in advanced PD.40
strategy for managing dyskinesia is pre- Patient selection is of paramount
vention. Using lower individual levo- importance and realistic patient ex-
dopa doses over time seems to reduce pectation is key to success.32,41 A multi-
the risk. Early use of dopamine agonists disciplinary approach to patient selection
may help prevent levodopa-induced including input from neurosurgery,
dyskinesia for 1 to 2 years but not long neurology, neuropsychology, and psy-
term. Once a patient starts to report chiatry before a final decision on suit-
bothersome peak or on-period dyski- ability is vital. Referral to a specialist
nesia, then keeping dopamine levels as center is important for ensuring only
low as possible without compromising suitable subjects are offered surgery
and also to maximize outcomes from
PD motor symptom control is the best
surgery for patients with PD. STN DBS
strategy. Reducing concomitant COMT
will only improve symptoms to the
and MAO-B inhibitors can also reduce same extent as the ‘‘best on’’ effect of
bothersome peak-dose dyskinesia. levodopa, but without dyskinesia and
Add-on drugs targeting dyskinesia wearing off. In the long term, subjects
have been a focus of research for will develop nonYlevodopa-responsive
many years.37 Currently the N-methyl- (and thus nonYDBS-responsive) symp-
D-aspartate (NMDA) glutamate receptor toms, including falls, speech distur-
antagonist, amantadine, is the most ef- bances, and nonmotor issues due to

Continuum (Minneap Minn) 2016;22(4):1064–1085 www.ContinuumJournal.com 1079

Copyright © American Academy of Neurology. Unauthorized reproduction of this article is prohibited.


Parkinson Disease Motor Symptoms

KEY POINTS
h Early referral for deep disease progression.42 With STN DBS, taking levodopa-carbidopa gel possibly
brain stimulation in patients can reduce medication by linked to low vitamin B levels.46 Sup-
subjects not yet about 50%, with less reduction in plementation with vitamin B12 may be
optimized on medication following GPi DBS. The useful, although not yet proven. Infusion
medical therapies main concerns of surgery are cognitive of apomorphine has been available in
remains controversial. impairment and, as such, younger many countries as another means of
h Infusional therapies patients are more suitable. The clinical continuous dopaminergic stimula-
require nursing and care advantage of early referral for DBS tion. This method is not currently
support for patients with (average of 7.5 years disease duration) available in the United States and, to
advanced Parkinson remains unclear; in general, subjects date, there appears to be no advantage
disease and their need to have optimized medical thera- over levodopa infusion.
families, and pies prior to considering surgery.43
device-complications Advanced Parkinson Disease
Significant depression is also a contra-
are common. With Levodopa-Resistant
indication due to higher risk of suicide.
h Counseling patients that Motor Symptoms
Too-rapid reduction of medications
some symptoms postsurgery can also induce apathy Disease progression in PD involves
(eg, falling, dysphagia) and mood issues. A prior history of structures beyond the dopamine sys-
will not respond to a visual hallucinations can be a red flag for tem, including brainstem and cortical
higher dose of levodopa pending cognitive impairment, and pa- regions. As such, some symptoms are,
is important, and tients with reversible drug-induced vi- or can become, resistant to dopamine
nondrug management replacement therapy. Motor symp-
sual hallucinations should be carefully
strategies are far
evaluated. Rarely, lesioning, including toms that are often harder to treat due
more effective.
newer techniques under development, to poor response to levodopa include
such as focused ultrasound of the tremor, gait and balance with falling,
thalamus for tremor-dominant PD, is and speech issues. Thus, part of
used in certain clinical scenarios, such managing such symptoms includes
as in elderly patients in whom DBS is education of patients and families of
considered too high risk.44 limited benefit of current drugs and
Infusional levodopa/carbidopa in- the use of nonpharmacologic strate-
testinal gel. Infusional levodopa may
gies (eg, physical therapy and occu-
be useful in patients with advanced
pational and speech therapy).
PD who are considered poor surgical
Falls. Falls are common and fre-
candidates because of age constraints
quently multifactorial. A careful history,
or cognitive or behavioral concerns.45
Carbidopa/ levodopa (5 mg/mL/ with good collateral history, is therefore
20 mg/mL) in a stable methylcellulose always important.47 The major cause
gel is pumped through a gastrostomy of falls is postural instability, a disor-
tube with a jejunal extension. The pa- der of balance that becomes increas-
tient carries the pump. The infusion is ingly common with advancing disease
commenced during waking hours but duration. This can be tested in clinic
can be extended overnight if required with the pull test. A normal pull test is
for nocturnal symptoms. Additional one to two steps of retropulsion after
bolus doses can also be used. Technical a brief tug backward; always warn the
hardware complications associated with patient what you are going to do and
gastrostomy tube placement and its be prepared to catch them if they fall.48
chronic use are common (eg, infection, Slow, shuffling gait with short strides,
occlusion, or displacement). More re- reduced or absent arm swing, start
cently, there have been reports of hesitation, and freezing of gait often
Guillain-Barré syndromeYtype sub- accompany postural instability. Some
acute peripheral neuropathy in subjects patients fall because of dystonia/chorea

1080 www.ContinuumJournal.com August 2016

Copyright © American Academy of Neurology. Unauthorized reproduction of this article is prohibited.


KEY POINT

Case 2-6 h Less is more in the


treatment of patients
A 68-year-old man with a 12-year history of Parkinson disease (PD)
with Parkinson disease,
developed painful cramping in his right leg that occurred daily and appeared
and simplifying drug
less responsive to levodopa. These symptoms were often associated with a
regimens allows for less
feeling of breathlessness. He had tried crushing his carbidopa/levodopa in
adverse effects and
apple sauce as well as using a carbonated soda for a more rapid onset effect.
better patient
He had several periods during the day when he had difficulty getting up
compliance in
out of a chair and had used a walker to assist his balance during these times. He
advanced disease.
took rasagiline 1 mg every morning and carbidopa/levodopa (25 mg/100 mg)
tablets, 1.5 tablets at 3-hour intervals (a total of 7 times per day), but had
found that the effect lasted less than 2 hours, with involuntary head
movements and sometimes trunk movements that made him slide out of his
chair. He had previously been on entacapone 200 mg 4 times a day, and this
was stopped 2 years previously, with some improvement in dyskinesia. He
had nocturia and found it more difficult to get out of bed. He had tried
amantadine but developed a leg rash. He experienced a tremor in his arm at
night that kept him awake. He had been on pramipexole 8 years prior for a
period of 2 years, but had developed hypersexuality and so this was stopped.
Comment. This is a typical patient with advanced PD with fluctuations
including painful off-period dystonia and nonmotor symptoms such as
breathlessness, pain, and nocturia. At this stage in PD, the overall goal is for
simplification, and withdrawal of nonessential drugs is usually the best
strategy to limit side effects. Stopping rasagiline may reduce some of the
peak-dose dyskinesia but will not help other symptoms. Alternative rescue
for off periods may include subcutaneous apomorphine injections that would
help painful dystonia. He may be suitable for bilateral subthalamic nucleus
DBS, or if he has contraindications (ie, cognitive or major mood issues), then
infusion therapy of carbidopa/levodopa would be an alternative.

due to dyskinesia (Case 2-6). Nonmotor Postural deformities. Postural defor-


factors include orthostatic hypotension, mities involving the axial skeleton can
which may contribute to some falls and complicate advanced PD.50 Forward
may not manifest until a few minutes flexion of the thoracolumbar spine,
of walking. Cognitive issues play a sig- termed camptocormia, can occur when
nificant role as patients will fall when walking. The posture is correctable
trying to perform two tasks at once when sitting or lying supine; this can
(dual tasking) and are associated with help to differentiate from fixed bony
reduced attention and impulsivity. Treat- deformities. Forward flexion of the cer-
ment options are limited (Table 2-5). vical spine, or head drop, can also be
Physical therapy may be helpful to seen in PD (Supplemental Digital
teach adaptive mechanisms to prevent Content 2-3, links.lww.com/CONT/
falls. In clinic, simple advice to patients A177) but, like camptocormia, is not
and families can include turning in a specific to PD and is also seen in
semicircle rather than pivoting on the multiple system atrophy as well as motor
spot, focusing on the single task of walk- neuron diseases and myopathies. Lat-
ing only, highlighting risks and adaptive eral flexion of the spine, or Pisa syn-
strategies at home such as reducing drome, is of similar etiology, although
clutter, increasing turning space, and the term was originally used to describe
using floor cues for freezing.49 tardive dystonia of the spine secondary

Continuum (Minneap Minn) 2016;22(4):1064–1085 www.ContinuumJournal.com 1081

Copyright © American Academy of Neurology. Unauthorized reproduction of this article is prohibited.


Parkinson Disease Motor Symptoms

TABLE 2-5 Treatment for Levodopa-Resistant Parkinson Disease Motor Symptoms

Symptom Drug/Therapy Comments


Gait Methylphenidate One small randomized controlled trial using 1 mg/kg/d reported
positive as add-on to subthalamic nucleus deep brain stimulation;
second study in advanced Parkinson disease (PD) was negative;
evidence-based medicine recommendations conclusion is
inconclusive7; off-label use
Amantadine Open-label data only; off-label use
Pedunculopontine Undergoing study
nucleus deep
brain stimulation
Postural deformity
Camptocormia None available Surgical correction rarely beneficial
Pisa syndrome, None available May respond to stopping dopamine agonists
head drop
Tremor Anticholinergics Effective as per evidence-based medicine recommendations7;
tolerance is poor
Clozapine Effective at low doses of 25Y50 mg/d (post hoc data); off-label use;
mandatory white blood cell and granulocyte monitoring required
Mirtazapine Anecdotal; no evidence-based medicine recommendations;
off-label use; can also help depression in PD
Propranolol No evidence-based medicine recommendations; for intention
and postural component; watch for lowering of blood pressure and
heart rate

to antipsychotics. All these postural ab- monly used, such as Lee Silverman
normalities have a poor response to Voice Treatment.51 Simple measures
levodopa. The cause is unclear and may such as reading out loud and singing
be due to postural lean implicated in ab- can help patients with PD.
normal postural reflexes from PD, rigidity Refractory tremor. Drug-resistant
combined with dystonia, or a secondary tremor can be an issue throughout
myopathic process. Use of dopamine the course of PD. Levodopa may help
agonists has also been suggested as a but often high doses are required. Im-
cause, as improvements in posture have portantly, unlike all other parkinsonian
been reported when dopamine ago- features, the absence of a levodopa
nists have been stopped (Table 2-5). response does not predict a poor op-
Speech. The characteristic speech erative outcome if DBS is planned. A
disturbance in PD is low volume, mo- number of medical strategies using
notonous pitch, loss of prosody, impre- nondopaminergic therapies have
cise consonants, variability of rate, and been suggested (Table 2-5).
occasional bursts of rapid, unintelligible
speech (tachyphemia). Early in the CONCLUSION
disease, some features may improve Treatment of the motor symptoms of
with levodopa but over time become early PD is relatively straightforward given
less responsive. Speech therapy is com- the choices of medications available in

1082 www.ContinuumJournal.com August 2016

Copyright © American Academy of Neurology. Unauthorized reproduction of this article is prohibited.


our arsenal today. It is important to REFERENCES
customize treatment for the given pa- 1. Postuma R, Berg D, Stern M, et al. MDS
tient, and it is not one drug or therapy clinical diagnostic criteria for Parkinson’s
disease. Mov Disord 2015;30(12):1591Y1601.
fits all. Levodopa remains the gold doi:10.1002/mds.26424.
standard in treatment, and clinicians 2. Fahn S. The medical treatment of Parkinson
should not hesitate to use it when pa- disease from James Parkinson to George
tients’ symptoms warrant its use. As Cotzias. Mov Disord 2015;30(10):4Y18.
patients advance, then comorbidities doi:10.1002/mds.26102.
and risks of side effects tend to limit 3. Hely MA, Reid WG, Adena MA, et al. The
the options. Exercise and neurore- Sydney multicenter study of Parkinson’s
disease: the inevitability of dementia at
habilitation should be used liberally 20 years. Mov Disord 2008;23(6):837Y844.
in the therapy of PD at all stages. doi:10.1002/mds.21956.

VIDEO LEGENDS 4. Kalia LV, Kalia SK, Lang AE. Disease-modifying


strategies for Parkinson’s disease. Mov Disord
Supplemental Digital Content 2-1 2015;30(11):1442Y1450. doi:10.1002/mds.26354.
Diphasic dyskinesia in the legs. This
5. Hebron ML, Lonskaya I, Moussa CE. Nilotinib
video shows a 50-year-old woman with reverses loss of dopamine neurons and
idiopathic Parkinson disease who has improves motor behavior via autophagic
levodopa-induced repetitive stereo- degradation of !-synuclein in Parkinson’s
disease models. Hum Mol Genet 2013;22(16):
typed kicking movements of her feet 3315Y3328. doi:10.1093/hmg/ddt192.
that are predominantly dystonic in
6. Bloem BR, de Vries NM, Ebersbach G.
nature. The movements appear about Nonpharmacological treatments for
20 minutes after taking levodopa. patients with Parkinson’s disease. Mov
Disord 2015;30(11):1504Y1520. doi:10.1002/
links.lww.com/CONT/A175 mds.26363.
B 2016 American Academy of Neurology.
7. Fox SH, Katzenschlager R, Lim SY, et al.
Supplemental Digital Content 2-2 Evidence Based Medicine Publications.
International Parkinson and Movement
Diphasic dyskinesia when walking. This Disorder Society. www.movementdisorders.
video shows a 68-year-old man (who is org/MDS/Resources/Publications-Reviews/
PARK2 gene positive) who has abnor- EBM-Reviews1.htm. Accessed June 17, 2016.
mal intermittent ballistic movements in 8. Aquino CC, Fox SH. Clinical spectrum of
the arm and an unusual high-stepping levodopa-induced complications. Mov Disord
2015;30(1):80Y89. doi:10.1002/mds.26125.
gait, which is another pattern of dipha-
sic dyskinesia. 9. Warren Olanow C, Kieburtz K, Rascol O, et al.
Factors predictive of the development of
links.lww.com/CONT/A176 Levodopa-induced dyskinesia and wearing-off
B 2016 American Academy of Neurology. in Parkinson’s disease. Mov Disord 2013;28(8):
1064Y1071. doi:10.1002/mds.25364.
Supplemental Digital Content 2-3 10. PD Med Collaborative Group, Gray R, Ives N,
Antecollis in idiopathic Parkinson Rick C, et al. Long-term effectiveness
disease. A 65-year-old woman with a of dopamine agonists and monoamine
oxidase B inhibitors compared with
6-year history of idiopathic Parkinson levodopa as initial treatment for
disease initially presented with right Parkinson’s disease (PD MED): a large,
arm tremor and slow gait that re- open-label, pragmatic randomised
trial. Lancet 2014;384(9949):1196Y1205.
sponded to levodopa. Over the next 2
doi:10.1016/S0140-6736(14)60683-8.
to 3 years she developed a progressive
11. Grosset D, Taurah L, Burn DJ, et al. A
neck flexion (antecollis), as seen on
multicentre longitudinal observational
this video. There was no improvement study of changes in self-reported health
with levodopa. status in people with Parkinson’s disease left
untreated at diagnosis. J Neurol Neurosurg
links.lww.com/CONT/A177 Psychiatry 2007;78(5):465Y469. doi:10.1136/
B 2016 American Academy of Neurology. jnnp.2006.098327.

Continuum (Minneap Minn) 2016;22(4):1064–1085 www.ContinuumJournal.com 1083

Copyright © American Academy of Neurology. Unauthorized reproduction of this article is prohibited.


Parkinson Disease Motor Symptoms

12. Parkinson Study Group. A controlled 22. Holloway RG, Shoulson I, Fahn S, et al.
trial of rasagiline in early Parkinson disease: Pramipexole vs levodopa as initial
the TEMPO Study. Arch Neurol 2002 treatment for Parkinson disease: a 4-year
Dec;59(12):1937Y1943. doi:10.1001/ randomized controlled trial. Arch Neurol
archneur.59.12.1937. 2004;61(7):1044Y1053. doi:10.1001/
13. Connolly BS, Lang AE. Pharmacological archneur.61.7.1044.
treatment of Parkinson disease: a review. 23. Hobson DE, Lang AE, Martin WR, et al.
JAMA 2014;311(16):1670Y1683. doi:10.1001/ Excessive daytime sleepiness and
jama.2014.3654. sudden-onset sleep in Parkinson disease: a
survey by the Canadian Movement Disorders
14. Barone P, Poewe W, Albrecht S, et al.
Pramipexole for the treatment of Group. JAMA 2002;287(4):455Y463.
depressive symptoms in patients with doi:10.1001/jama.287.4.455.
Parkinson’s disease: a randomised, 24. Kleiner-Fisman G, Fisman DN. Risk factors for
double-blind, placebo-controlled trial. the development of pedal edema in patients
Lancet Neurol 2010;9(6):573Y580. using pramipexole. Arch Neurol 2007;64(6):
doi:10.1016/S1474-4422(10)70106-X. 820Y824. doi:10.1001/archneur.64.6.noc60158.
15. Katzenschlager R, Sampaio C, Costa J, Lees A. 25. Weintraub D, David AS, Evans AH, et al.
Anticholinergics for symptomatic Clinical spectrum of impulse control
management of Parkinson’s disease. Cochrane disorders in Parkinson’s disease. Mov
Database Syst Rev 2003;(2):CD003735. Disord 2015;30(2):121Y127. doi:10.1002/
doi:10.1002/14651858.CD003735. mds.26016.
16. Wang Y, Xie CL, Wang WW, et al. Epidemiology 26. American Psychiatric Association. Diagnostic
of complementary and alternative medicine and statistical manual of mental disorders,
use in patients with Parkinson’s disease. J Clin fifth edition. Washington, DC: American
Neurosci 2013;20(8):1062Y1067. doi:10.1016/ Psychiatric Publishing, 2013.
j.jocn.2012.10.022. 27. Storch A, Schneider CB, Wolz M, et al.
17. Suchowersky O, Gronseth G, Perlmutter J, Nonmotor fluctuations in Parkinson disease:
et al. Practice parameter: neuroprotective severity and correlation with motor
strategies and alternative therapies complications. Neurology 2013;80(9):800Y809.
for Parkinson disease (an evidence-based doi:10.1212/WNL.0b013e318285c0ed.
review): report of the Quality Standards 28. Ahlskog JE, Muenter MD. Frequency of
Subcommittee of the American Academy levodopa-related dyskinesias and motor
of Neurology. Neurology 2006; fluctuations as estimated from the cumulative
66(7):976Y982. doi:10.1212/01.wnl. literature. Mov Disord 2001;16(3):448Y458.
0000206363.57955.1b. doi:10.1002/mds.1090.
18. Ni X, Liu S, Lu F, et al. Efficacy and safety 29. Cilia R, Akpalu A, Sarfo FS, et al. The modern
of Tai Chi for Parkinson’s disease: a pre-levodopa era of Parkinson’s disease:
systematic review and meta-analysis of insights into motor complications from
randomized controlled trials. PLoS One sub-Saharan Africa. Brain 2014;137(pt 10):
2014;9(6):e99377. doi:10.1371/journal. 2731Y2734. doi:10.1093/brain/awu195.
pone.0099377. 30. Scott NW, Macleod AD, Counsell CE. Motor
19. Ridgel AL, Peacock CA, Fickes EJ, Kim CH. complications in an incident Parkinson’s
Active-assisted cycling improves tremor and disease cohort. Eur J Neurol 2016;
bradykinesia in Parkinson’s disease. Arch 23(2):304Y312. doi:10.1111/ene.12751.
Phys Med Rehabil 2012;93(11):2049Y2054. 31. Thenganatt MA, Jankovic J. Parkinson disease
doi:10.1016/j.apmr.2012.05.015. subtypes. JAMA Neurol 2014;71(4):499Y504.
20. Fox SH, Katzenschlager R, Lim SY, et al. doi:10.1001/jamaneurol.2013.6233.
The Movement Disorder Society 32. Giugni JC, Okun MS. Treatment of advanced
Evidence-Based Medicine Review Update: Parkinson’s disease. Curr Opin Neurol
treatments for the motor symptoms of 2014;27(4):450Y460. doi:10.1097/
Parkinson’s disease. Mov Disord 2011; WCO.0000000000000118.
26(suppl 3):S2YS41. doi:10.1002/
33. Hauser RA, Hsu A, Kell S, et al; IPX066
mds.23829.
ADVANCE-PD Investigators. Extended-release
21. Junghanns S, Fuhrmann JT, Simonis G, carbidopa-levodopa (IPX066) compared with
et al. Valvular heart disease in Parkinson’s immediate-release carbidopa-levodopa in
disease patients treated with dopamine patients with Parkinson’s disease and motor
agonists: a reader-blinded monocenter fluctuations: a phase 3 randomised, double-blind
echocardiography study. Mov Disord 2007; trial. Lancet Neurol 2013;12(4):346Y356.
22(2):234Y238. doi:10.1002/mds.21225. doi:10.1016/S1474-4422(13)70025-5.

1084 www.ContinuumJournal.com August 2016

Copyright © American Academy of Neurology. Unauthorized reproduction of this article is prohibited.


34. Pilleri M, Antonini A. Novel levodopa Arch Neurol 2011;68(12):1550Y1556.
formulations in the treatment of Parkinson’s doi:10.1001/archneurol.2011.182.
disease. Expert Rev Neurother 2014;14(2):
43. Schuepbach WM, Rau J, Knudsen K,
143Y149. doi:10.1586/14737175.2014.877840.
et al. Neurostimulation for Parkinson’s
35. Fasano A, Bove F, Gabrielli M, et al. The role disease with early motor complications.
of small intestinal bacterial overgrowth in N Engl J Med 2013;368(7):610Y622.
Parkinson’s disease. Mov Disord 2013; doi:10.1056/NEJMoa1205158.
28(9):1241Y1249. doi:10.1002/mds.25522.
44. Strauss I, Kalia SK, Lozano AM. Where
36. Stowe R, Ives N, Clarke CE, et al. Evaluation are we with surgical therapies for Parkinson’s
of the efficacy and safety of adjuvant treatment disease? Parkinsonism Relat Disord 2014;
to levodopa therapy in Parkinson’s disease 20(suppl 1):S187YS191. doi:10.1016/
patients with motor complications. Cochrane S1353-8020(13)70044-0.
Database Syst Rev 2010;(7):CD007166. 45. Senek M, Nyholm D. Continuous drug
doi:10.1002/14651858.CD007166.pub2. delivery in Parkinson’s disease. CNS Drugs
37. Calabresi P, Di Filippo M, Ghiglieri V, et al. 2014;28(1):19Y27. doi:10.1007/s40263-013-
Levodopa-induced dyskinesias in patients 0127-1.
with Parkinson’s disease: filling the 46. Uncini A, Eleopra R, Onofrj M.
bench-to-bedside gap. Lancet Neurol Polyneuropathy associated with duodenal
2010;9(11):1106Y1117. doi:10.1016/ infusion of levodopa in Parkinson’s disease:
S1474-4422(10)70218-0. features, pathogenesis and management.
38. Pahwa R, Tanner CM, Hauser RA, et al. J Neurol Neurosurg Psychiatry 2015;86(5):
Amantadine extended release for 490Y495. doi:10.1136/jnnp-2014-308586.
levodopa-induced dyskinesia in Parkinson’s 47. van der Marck MA, Klok MP, Okun MS,
disease (EASED Study). Mov Disord et al.; NPF Falls Task Force. Consensus-based
2015;30(6):788Y795. doi:10.1002/mds.26159. clinical practice recommendations for
39. Odekerken VJ, van Laar T, Staal MJ, et al. the examination and management of falls
Subthalamic nucleus versus globus pallidus in patients with Parkinson’s disease.
bilateral deep brain stimulation for advanced Parkinsonism Relat Disord 2014;20(4):
Parkinson’s disease (NSTAPS study): a 360Y369. doi:10.1016/j.parkreldis.2013.10.030.
randomised controlled trial. Lancet Neurol 48. Goetz C, Vittal P, Maldonado J. How do I
2013;12(1):37Y44. doi:10.1016/S1474-4422(12) test for postural stability? Mov Disord
70264-8. Clin Pract 2014;1(2):144. doi:10.1002/
40. Williams A, Gill S, Varma T, et al; PD SURG mdc3.12029.
Collaborative Group. Deep brain stimulation 49. Okuma Y. Practical approach to freezing
plus best medical therapy versus best medical of gait in Parkinson’s disease. Pract Neurol
therapy alone for advanced Parkinson’s 2014;14(4):222Y230. doi:10.1136/practneurol-
disease (PD SURG trial): a randomised, 2013-000743.
open-label trial. Lancet Neurol 2010;9(6):
581Y591. doi:10.1016/S1474-4422(10)70093-4. 50. Doherty KM, van de Warrenburg BP,
Peralta MC, et al. Postural deformities
41. Katz M, Kilbane C, Rosengard J, et al. in Parkinson’s disease. Lancet Neurol
Referring patients for deep brain 2011;10(6):538Y549. doi:10.1016/
stimulation: an improving practice. Arch S1474-4422(11)70067-9.
Neurol 2011;68(8):1027Y1032.
51. Mahler LA, Ramig LO, Fox C. Evidence-based
doi:10.1001/archneurol.2011.151.
treatment of voice and speech disorders in
42. Castrioto A, Lozano AM, Poon YY, et al. Parkinson disease. Curr Opin Otolaryngol
Ten-year outcome of subthalamic stimulation Head Neck Surg 2015;23(3):209Y215.
in Parkinson disease: a blinded evaluation. doi:10.1097/MOO.0000000000000151.

Continuum (Minneap Minn) 2016;22(4):1064–1085 www.ContinuumJournal.com 1085

Copyright © American Academy of Neurology. Unauthorized reproduction of this article is prohibited.


Review Article

Neuropsychiatric Issues
Address correspondence to
Dr Jennifer G. Goldman, Rush
University Medical Center,
Department of Neurological
Sciences, 1725 W Harrison St,
Suite 755, Chicago, IL 60612,
Jennifer_Goldman@rush.edu.
in Parkinson Disease
Relationship disclosure: Jennifer G. Goldman, MD, MS, FAAN
Dr Goldman has received
personal compensation for
serving as a consultant for
Acadia Pharmaceuticals Inc, ABSTRACT
Pfizer Inc, and Teva
Pharmaceutical Industries Ltd. Purpose of Review: This article reviews the recognition and management of
Dr Goldman receives research neuropsychiatric issues in Parkinson disease (PD), including mood disorders,
support from Acadia cognitive impairment, and behavioral disturbances.
Pharmaceuticals Inc, Biotie
Therapies, the Michael J. Fox Recent Findings: Patients with PD frequently develop neuropsychiatric issues, and
Foundation, the National these issues can greatly affect their quality of life. In recent years, mood, cognitive, and
Institute of Neurological behavioral issues in PD have received greater recognition, with increasing atten-
Disorders and Stroke/National
Institutes of Health, and Teva tion directed toward improved screening and therapeutic interventions for symptom-
Pharmaceutical Industries Ltd. atic treatment. Taken together as a group, neuropsychiatric issues can be found
Unlabeled Use of throughout the whole course of PD, from early in the disease, potentially even in
Products/Investigational
Use Disclosure:
a premotor stage, to the time of diagnosis and later in the course with more
Dr Goldman discusses the advanced disease.
unlabeled/investigational use Summary: In the comprehensive care of patients with PD, recognition of
of antidepressants,
antipsychotics, anxiolytics,
neuropsychiatric issues is critical. Advances in therapeutics for the different neuropsy-
and cognitive impairment chiatric symptoms are still needed, although several pharmacologic and
medications for the treatment nonpharmacologic options are available. Patient management frequently requires a
of Parkinson disease, none of
which are approved by the US
multidisciplinary approach, with collaboration of neurologists with neuropsychologists,
Food and Drug Administration psychologists, psychiatrists, and other health professionals.
except rivastigmine.
* 2016 American Academy Continuum (Minneap Minn) 2016;22(4):1086–1103.
of Neurology.

INTRODUCTION of PD, from premotor phases (eg,


Neuropsychiatric issues are frequently depression) to advanced disease (eg,
encountered in Parkinson disease (PD) dementia), although they may vary in
and substantially impact the lives of pa- their timing, type, and severity. Some
tients and their caregivers. While the of the neuropsychiatric symptoms
neuropsychiatric symptoms encoun- relate to the underlying neurobiology
tered in PD are diverse, they typically of PD itself, with neuroanatomical,
encompass mood, cognitive, and be- neurochemical, or neuropathologic
havioral issues, specifically depression changes, whereas others may be side
and anxiety; mild cognitive impairment effects of or exacerbated by dopami-
and dementia; and apathy, impulse con- nergic replacement therapies. This
trol disorders, and psychosis. In recent article highlights the recognition and
years, the importance of neuropsychi- management of commonly encoun-
atric issues in PD has been increasingly tered mood, cognitive, and behavioral
recognized, which has resulted in great- issues in PD.
er attention directed toward diagnostic
tools and symptom management along MOOD DISORDERS
with advances in research regarding Mood symptoms are frequent in PD,
pathophysiology and therapeutics. Fur- with depression and anxiety being
thermore, neuropsychiatric symptoms the most commonly encountered.
may be found throughout the course Neurodegenerative and neurochemical
1086 www.ContinuumJournal.com August 2016

Copyright © American Academy of Neurology. Unauthorized reproduction of this article is prohibited.


KEY POINTS
changes in mesocortical and mesolimbic anhedonia, timing of dopaminergic h Neuropsychiatric issues
dopaminergic neurons, brainstem se- medications).6 are common in
rotonergic and noradrenergic neu- Clinical symptoms of depression in Parkinson disease
rons, and cortical-subcortical and limbic PD are generally similar to those in the and include mood,
circuits suggest an intrinsic neuro- non-PD population and include mood cognitive, and
biological basis for depression and changes (eg, depressed mood, de- behavioral symptoms
anxiety in PD. However, reactive de- creased interest and pleasure) as well that can occur at all
pression and situational stressors as somatic and vegetative features. Some stages of the disease.
may also occur, whether at the time of features of major depression, however, h Depression in Parkinson
PD diagnosis, with advancing symptoms, may overlap with other symptoms of PD, disease is likely
or in other circumstances. Mood changes including motor symptoms (slowness underdiagnosed and
may occur as effects of medical or sur- or psychomotor retardation, blunted may manifest as milder
gical interventions, such as with dopa- affect), somatic symptoms (muscle ten- forms of depression
mine agonists and mania/hypomania sion, gastrointestinal problems), vege- rather than meeting
Diagnostic and
or postoperative depression, suicidal tative symptoms (changes in weight or
Statistical Manual of
ideation, or mania after deep brain appetite, changes in sleep [insomnia or
Mental Disorders criteria
stimulation (DBS).1,2 hypersomnia], fatigue, or loss of energy), definitions for
and cognitive symptoms (impaired major depression.
Depression attention and concentration, indeci-
h Some features of
While estimates of depression in PD in siveness). Anxiety and apathy may ac-
Parkinson disease
the literature have ranged from 2.7% company depression in PD, although depression may overlap
to 90%, it is thought that about 40% to these may also be separate and distinct with other symptoms of
50% of patients with PD have clinically entities. Patients with PD who have de- Parkinson disease,
significant depression.3 Depression in pression are less likely than patients including motor
PD, however, may be underdiagnosed without PD who have depression to features (slowness or
because of the various definitions and have excessive guilt; feelings of worth- psychomotor retardation,
criteria used for identifying depression lessness, hopelessness, or helplessness; blunted affect), somatic
and the failure of patients with PD to delusions; or suicidality. While suicide symptoms (muscle
meet Diagnostic and Statistical Man- is relatively infrequent in PD, suicidal tension, gastrointestinal
ual of Mental Disorders, Fourth Edi- ideation has been reported in 11% of problems), vegetative
symptoms (changes in
tion (DSM-IV) or Fifth Edition (DSM-5) patients with PD.7 A unique aspect of
weight or appetite,
criteria for major depression.4,5 Pa- depression in PD, along with anxiety, is
changes in sleep
tients with PD may experience milder its occurrence as a nonmotor fluctua- [insomnia or
minor forms of depression or situa- tion, occurring at the nadir of dopami- hypersomnia], fatigue,
tional depression. In a meta-analysis, nergic stimulation (off period).8 Similar or loss of energy), and
the weighted prevalence of major de- to motor fluctuations, with the return cognitive symptoms
pressive disorder in PD was 17%, minor of increased tremor, bradykinesia, or (impaired attention
depression 22%, and dysthymia 13%3; gait difficulty, while off or wearing off, and concentration,
however, when the definitions involved patients with PD can exhibit depressed indecisiveness).
clinically significant symptoms rather mood, tearfulness, anxiety, or panic
than DSM criteria, depressive symp- symptoms in the off period. Recogni-
toms occurred in 35%. As such, a tion of the timing or cyclical occurrence
National Institute of Neurological Dis- of mood symptoms in this context is
orders and Stroke/National Institute of important as treatment strategies may
Mental Health working group recom- target off or wearing off periods.
mended a more inclusive approach to Depressive symptoms can occur
detecting depression in PD regardless throughout the disease course in PD,
of etiology, severity, information source, from premotor to advanced stages,
and relationship with other features (eg, and contribute to reduced quality of life
Continuum (Minneap Minn) 2016;22(4):1086–1103 www.ContinuumJournal.com 1087

Copyright © American Academy of Neurology. Unauthorized reproduction of this article is prohibited.


Neuropsychiatric Issues in PD

KEY POINTS
h It is important to ask and daily function. Neurobiological and ing overlapping symptoms (PD versus
patients with Parkinson epidemiologic data support the hypoth- depression, depression versus apathy),
disease about esis that depression, along with anxiety, difficulty in assessing cognitively im-
depressive symptoms, may represent a prodromal feature of paired patients with PD, differences in
even when mild in PD. Consistent with the hypothesized depression in PD versus the general
severity, as well as about pathologic Braak staging of PD, de- population, different purposes of the
symptoms pression and anxiety may reflect brain- scales, lack of collateral information re-
of hopelessness stem degeneration affecting the locus quired, and failure to specify the timing
and suicidality. coeruleus and dorsal raphe nuclei and of the assessment, in particular in rela-
h Depression in Parkinson resultant noradrenergic and seroto- tion to PD medications. Despite these
disease may also occur nergic dysfunction, which may occur concerns, the task force recommended
as a nonmotor even prior to motor dysfunction. Epi- the Hamilton Depression Scale, Beck
fluctuation or off period demiologic studies support an over two- Depression Inventory, Hospital Anxiety
phenomenon, which fold increased prevalence of depression and Depression Scale, MontgomeryY
necessitates management
diagnosis in patients later diagnosed )sberg Depression Rating Scale, and
strategies that target
with PD.9 Depression is also commonly Geriatric Depression Scale for screen-
cyclical fluctuations.
reported in early, de novo, or untreated ing purposes and the Hamilton Depres-
h Treatment of Parkinson patients with PD, along with other non- sion Scale, Beck Depression Inventory,
disease depression, as
motor symptoms and neuropsychiatric MontgomeryY)sberg Depression Rat-
well as several other
issues, with de novo PD patients expe- ing Scale, and Zung Self-Rating Depres-
neuropsychiatric issues
in Parkinson disease,
riencing significantly more depression, sion Scale for severity measurements.
involves both fatigue, apathy, and anxiety than healthy Treatment of PD depression involves
pharmacologic and controls at baseline and over 2 years of both pharmacologic and nonpharma-
nonpharmacologic follow-up.10 Depression may be associ- cologic approaches. The pharmacologic
strategies. ated with a number of PD-related fac- rationale is rooted in neurochemical al-
tors, such as longer disease duration, terations of serotonin, norepinephrine,
younger age at onset, worsened motor and, to some degree, dopamine, which
severity and motor complications, more are associated with depression. The
advanced disease stage, greater disabil- evidence base, however, is limited for
ity, and the postural instability gait antidepressants in PD because few large
disturbance phenotype, as well as other randomized double-blind placebo-
neuropsychiatric comorbidities affect- controlled studies have been con-
ing cognition, psychosis, and sleep.11 ducted.13,14 Selective serotonin reuptake
The clinical assessment for PD de- inhibitors (SSRIs) are frequently used
pression may incorporate interview since they are better tolerated than
questions, diagnostic clinical examina- other antidepressant classes, such as
tions and criteria, and self- or clinician- tricyclic antidepressants (TCAs). In the
administered depression rating scales Study of Antidepressants in Parkinson’s
for screening or measuring severity. An Disease comparing paroxetine, venlafax-
International Parkinson and Movement ine, and placebo, both drug treatment
Disorder Society (MDS) task force re- groups showed similar improvement
viewed depression rating scales in PD, in Hamilton Depression Scale scores
critiquing the scales’ psychometric prop- at 12 weeks.15 Two other randomized
erties, validation in PD populations, controlled trials compared SSRIs and
availability of cutoff scores for patients TCAs (one with desipramine versus
with and without PD, sensitivity to citalopram, the other with paroxetine
change, and inclusion of somatic and controlled release versus nortripty-
psychological items.12 Several problems line); in both studies, the drug treat-
with the scales were identified, includ- ment groups did better than placebo,
1088 www.ContinuumJournal.com August 2016

Copyright © American Academy of Neurology. Unauthorized reproduction of this article is prohibited.


KEY POINTS
but TCAs were superior to SSRIs.16,17 panic disorder, phobias, obsessive- h Anxiety in Parkinson
However, TCAs should be used cau- compulsive disorder, and situational disease can occur both
tiously, particularly in the elderly and anxiety. Similar to depression in PD, by itself and
those with neuropsychiatric and auto- DSM-IV criteria fail to capture many with depression.
nomic problems, due to their anticho- patients with PD who have anxiety h Patients with Parkinson
linergic effects. Dopamine agonists (eg, disturbances.20 Prevalence estimates disease who have
pramipexole) improved depressive range from 11% to 21% for general- extreme anxiety when
symptoms in some studies and provided ized anxiety disorder, from 4% to 30% wearing off or in off
motor benefit. Serotonin syndrome is a for panic disorder, from 8% to 50% for periods can have panic
rare risk with TCAs, SSRIs, and seroto- social phobia, and from 11% to 25% for attacks with marked
nin norepinephrine reuptake inhibitors anxiety disorders not otherwise spec- anxiety, shortness of
(SNRIs) when combined with mono- ified, although these estimates may be breath, diaphoresis,
amine oxidase B inhibitors. In a random- confounded not only by the diagnostic and autonomic and
ized controlled study, nonpharmacologic criteria selected, but also by differences gastrointestinal
symptoms, which can
therapies, such as cognitive-behavioral in populations studied, the rating scales
even lead to emergency
therapy, demonstrated greater improve- used, and the presence of comorbid
department visits and
ment in Hamilton Depression Scale depression. Patients with PD may have evaluations for
scores than clinical monitoring as well multiple anxiety disorders, and even cardiopulmonary reasons.
as modest gains in verbal memory and those without a formally diagnosed
h Specific anxiety states in
executive function scores.18 Transcranial anxiety disorder frequently endorse
Parkinson disease can
magnetic stimulation, counseling, and clinically significant anxiety symptoms. include generalized
psychotherapy have gained attention Using an inclusive diagnostic and anxiety disorder, panic
as potential nonpharmacologic treat- data-driven approach, four specific disorder, phobias (eg,
ments of PD depression, although they syndromal profiles of anxiety were social, agoraphobia),
require further study. identified: (1) no anxiety or depression, obsessive-compulsive
(2) episodic anxiety without depres- disorder, and situational
Anxiety sion, (3) persistent anxiety with de- or stress-related anxiety.
Anxiety is a common neuropsychiatric pression, and (4) both persistent and h Some symptoms of
issue in PD, occurring in 20% to 50% episodic anxiety with depression.20 This anxiety can overlap
of patients with PD,14,19 and is likely underscores a prominent relationship be- Parkinson disease
underrecognized and undertreated. tween anxiety and depression, and thus symptoms, such as
Anxiety in PD may occur within the it can be difficult to identify whether muscle tension, poor
context of depression, but also may separate risk factors for these two mood concentration, fatigue,
occur by itself as an anxiety syndrome disorders exist. Younger age and female sleep problems
(insomnia), and
or as a nonmotor fluctuation during gender have been variably reported as
autonomic dysfunction
off periods. Patients with PD who have risk factors, and a prior history of anxiety
(cardiovascular, respiratory,
extreme anxiety when in off periods seems to be a strong predictor of sub- gastrointestinal).
can have panic attacks with marked sequent anxiety disorders in PD.19,21
anxiety, shortness of breath, diapho- Some symptoms of anxiety can over-
resis, and autonomic and gastrointes- lap with motor and nonmotor symp-
tinal symptoms, which can even lead toms of PD, such as muscle tension,
to emergency department visits and poor concentration, fatigue, sleep prob-
evaluations for cardiopulmonary rea- lems (insomnia), and autonomic dys-
sons. Patients with PD may have situa- function. Like depression, anxiety may
tional anxiety related to their tremors be due to the underlying neurobiology
and experience freezing or “turning off,” of PD, with neurochemical alterations
a fear of falling, or discomfort in social and neurodegeneration in the brain-
situations. Some specific anxiety states stem and subcortical regions.19 Similarly,
include generalized anxiety disorder, anxiety symptoms can occur throughout
Continuum (Minneap Minn) 2016;22(4):1086–1103 www.ContinuumJournal.com 1089

Copyright © American Academy of Neurology. Unauthorized reproduction of this article is prohibited.


Neuropsychiatric Issues in PD

the disease course, from premotor An MDS task force reviewed anxiety
phases to advanced stages (Case 3-1). rating scales in PD, critiquing six scales
Anxiety also has been proposed as a risk in a similar manner to those reviewed
factor for PD, with a 1.5-fold increased for depression.25 None of the reviewed
risk of PD associated with phobic scales met the task force’s criteria for
anxiety and antianxiety medication pre- “recommended” but a few scales were
scriptions. The concept of a “PD per- classified as “suggested.” Among those
sonality type” that is more anxious, were the Beck Anxiety Inventory, Hos-
lower novelty seeking, and higher in pital Anxiety and Depression Scale,
harm avoidance has been proposed Zung Self-Rating Anxiety Scale, Anxiety
over the years.22 Compared to healthy Status Inventory, Spielberger State-
controls, early and de novo PD patients Trait Anxiety Inventory, and Hamilton
had more anxiety and worse health- Anxiety Rating Scale. Problems identi-
related quality of life.23 Anxiety symp- fied with the anxiety rating scales
toms have been endorsed by over 50% included substantial overlap (anxiety
of patients with PD on nonmotor symp- versus depressive symptoms, anxiety
tom surveys.24 versus motor and other PD symptoms)
Evaluations for anxiety in PD may and failure to capture episodic anxiety
incorporate interview questions, self- disturbances and the heterogeneity of
report or clinician-rated scales, and diag- anxiety disorders in PD (eg, general-
nostic clinical examinations and criteria. ized anxiety, panic attacks).

Case 3-1
A 68-year-old man presented with a 2-year history of a left hand resting
tremor, small handwriting, and shuffling gait. He reported having
increased anxiety that was unusual for him for several years prior to the
onset of motor symptoms. He also had developed insomnia and lost about
9 kg (20 lbs) over the past 3 years. He denied depressed mood or loss of
interest. Both he and his wife felt that he could not handle stress at work
as well as previously. His tremor was worse when he was anxious. He did
not take any medications. His examination revealed an anxious man with
normal cognition, moderate left hand resting tremor, mild postural and
action tremors (more so on the left), mild to moderate bradykinesia (more
so on the left), and a mildly slow gait with decreased left arm swing but no
postural instability.
Comment. This case demonstrates the typical features of early
idiopathic Parkinson disease (PD). This patient described new-onset anxiety
and stress prior to the onset of his motor symptoms. He did not endorse
depressive symptoms, although the clinician should inquire further about
depressed mood, loss of interest, and somatic and vegetative symptoms. Several
epidemiologic studies, along with Braak staging of PD neurodegeneration and
neurochemical alterations in brainstem areas, suggest that anxiety and
depressive symptoms may precede the onset of motor symptoms of PD. In some
patients with PD, such as this de novo PD patient, motor and mood symptoms
may respond to dopaminergic treatment. Selective serotonin reuptake
inhibitors (SSRIs) and judicious use of benzodiazepines may be used to
treat PD anxiety and may be needed in conjunction with dopaminergic
replacement therapies. It is not uncommon for a patient’s tremors to
worsen with anxiety or when under stress.

1090 www.ContinuumJournal.com August 2016

Copyright © American Academy of Neurology. Unauthorized reproduction of this article is prohibited.


KEY POINTS
To date, no controlled therapeutic distinct from Alzheimer disease (AD), h To date, randomized
trials have been conducted specifically have occurred.29,30 controlled trials of
for anxiety disorders in PD.13 Medica- Clinical features of PD cognitive im- anxiety medications and
tions used to treat anxiety disorders in pairment, whether PD-MCI or PD de- therapies are lacking in
PD are similar to those used in the non- mentia, encompass a broad range of Parkinson disease.
PD population, such as SSRIs, SNRIs, deficits and severity. Most commonly, h For anxiety symptoms
benzodiazepines, and, in some, buspi- cognitive symptoms reflect dysfunction related to off periods,
rone. Benzodiazepines should be given in attention, working memory, execu- modification of the
cautiously in elderly patients and those tive function, memory, and visuospatial dopaminergic regimen
with cognitive impairment or psychosis, function. Cognitive deficits in PD-MCI may be effective, such as
poor balance or falls, and daytime may occur in isolation (single domain) with use of longer-acting
sleepiness. Anxiety treatments are un- or in combination, with multiple cogni- dopaminergics, higher
derused even in movement disorder tive domains impaired. Some studies doses or adjunctive
neurologic practices. One study showed suggest two different PD cognitive phe- dopaminergic treatment,
or more frequent and
that PD anxiety was more likely to be notypes, one with frontostriatal-based
smaller doses.
treated when comorbid depression or deficits on attention and executive func-
motor fluctuations occurred, and 53% tion tasks and the other with posterior h Mild cognitive
of patients with PD who had anxiety cortical deficits on semantic fluency, impairment in Parkinson
disease is recognized as a
disorders (n = 38) were medically memory, and visuospatial tasks. 31
potential prodromal
untreated.26 For off period anxiety Symptoms of PD cognitive impairment
stage to Parkinson
symptoms, the dopaminergic medica- include slowed thinking, trouble with disease dementia.
tion regimen may require adjustments. paying attention and concentration,
Nonpharmacologic treatments, such as problems with multitasking or plan- h Parkinson disease
dementia has been
counseling, psychotherapy, cognitive- ning, difficulty switching tasks or
defined as a syndrome
behavioral therapy, relaxation, stress starting new ones, forgetfulness or with an insidious onset
reduction therapies, and exercise, may short-term memory problems, or dif- and impairment in more
reduce anxiety in PD but require further ficulty with sense of direction. While than one cognitive
study in large controlled trials. language (ie, confrontational naming) domain that has a
is generally less affected, patients with significant effect on
COGNITIVE DISORDERS PD often report difficulty finding the activities of daily living.
Cognitive impairment is a frequent “right words.” While both nonamnestic
complication of PD, ranging from mild deficits (eg, attention, executive func-
dysfunction to a marked dementia. tion, visuospatial abilities) and memory
Mild cognitive impairment in PD (PD- or amnestic deficits occur in PD, their
MCI) has been increasingly recognized frequencies vary across studies based
as a distinct entity or potential precur- on differences in the population stud-
sor stage to dementia, with an esti- ied and the neuropsychological tests
mated prevalence of 19% to 38% (mean and definitions of impairment used.27
27%).27 PD dementia is a frequent PD dementia has been defined as a
complication of long-standing PD, syndrome with an insidious onset and
with cross-sectional estimates of about impairment in more than one cogni-
40% and longitudinal studies reveal- tive domain that has a significant effect
ing that 80% of patients with PD on activities of daily living. In contrast to
develop dementia after 8 or more years the DSM-IV definition for PD dementia,
of disease.28 Recognition of PD-MCI the International Parkinson and MDS
and PD dementia has increased over criteria for PD dementia propose that
the years, and concerted efforts to memory does not have to be impaired
develop uniform criteria for both MCI in PD dementia and that dementia can
and dementia, specifically in PD and be present if other cognitive domains
Continuum (Minneap Minn) 2016;22(4):1086–1103 www.ContinuumJournal.com 1091

Copyright © American Academy of Neurology. Unauthorized reproduction of this article is prohibited.


Neuropsychiatric Issues in PD

KEY POINTS
h In contrast to the (eg, executive function, visuospatial reasoning, visuospatial abilities, and
definition for Parkinson function) are affected, including be- praxis). Cognitive tests vary in their
disease dementia in the havioral problems (eg, psychosis, sleep sensitivity to PD cognitive deficits and
Diagnostic and Statistical disturbances, mood disturbances), optimal use (eg, screening, response to
Manual of Mental which are common in PD dementia. Pa- change or intervention). To date, no
Disorders, Fourth Edition, tients with parkinsonism who develop consensus exists regarding specific neu-
the International an early dementia (within 1 year of the ropsychological test batteries or even
Parkinson and Movement motor symptoms) are diagnosed as global tests, although the MDS PD-MCI
Disorder Society criteria having dementia with Lewy bodies, and PD dementia diagnostic criteria,
for Parkinson disease although this condition has overlap of along with reviews and literature, pro-
dementia propose that
neuropathology with PD dementia. vide recommendations for global and
memory does not have
Cognitive impairment in PD is associated domain-specific tests.
to be impaired in
Parkinson disease and
with poor outcomes, negatively impact- In the evaluation of patients with PD
that dementia can be ing patients’ quality of life, contributing to who have acute or new-onset cognitive
present if other cognitive morbidity and mortality, and increasing issues, infections (eg, urinary tract infec-
domains (eg, executive nursing home placement. Cognitive tion, pneumonia), metabolic derange-
function, visuospatial deficits, including PD-MCI, occur in de ments, dehydration, new neurologic
function) are affected novo PD patients with estimates of problems (eg, stroke, subdural hema-
and include behavioral about 20%.32Y34 Thus it is important toma), new medical problems (eg,
problems (eg, psychosis, to ask patients and caregivers about vitamin B12 deficiency, thyroid disor-
sleep disturbances, cognitive symptoms even at this early ders), and medication effects (eg, pain,
mood disturbances), stage. Neurochemical abnormalities in bladder, sedating medications) should
which are common
cholinergic, dopaminergic, serotoner- be excluded. It is important to inquire
in Parkinson
gic, and noradrenergic systems; neuro- about other comorbid neuropsychiat-
disease dementia.
pathologic changes with cortical, ric problems, including poor nighttime
h Patients with limbic, and brainstem Lewy bodies; sleep, excessive daytime sleepiness,
parkinsonism who
comorbid AD pathology and vascular psychosis, depression, anxiety, and ap-
develop an early
disease; biological markers including ge- athy, which may influence cognitive
dementia (within 1 year
of the motor symptoms)
netics such as "-synuclein duplication/ function. Medications, including PD
are diagnosed as having triplication, MAPT, GBA and APOE *4 medications such as anticholinergics
dementia with Lewy mutations; and comorbid nonmotor and dopamine agonists, should be care-
bodies, although it symptoms may influence phenotypic fully reviewed so that agents that may
has overlap differences and rates of cognitive de- contribute to impaired cognition, con-
of neuropathology cline.35 Risk factors for PD cognitive fusion, or sleepiness can be reduced or
with Parkinson impairment include older age, ad- discontinued. Clinical, laboratory, and
disease dementia. vanced disease, longer disease duration, imaging investigations should be or-
h Parkinson disease baseline cognitive impairment, and dered, as indicated. Management also
cognitive impairment is neuropsychiatric comorbidities (eg, includes addressing safety and driv-
associated with poor hallucinations and depression). ing issues.
outcomes, negatively Cognitive function in PD can be ex- Treatment trials for cognitive im-
impacting patients’ amined in numerous ways, ranging pairment in PD have primarily focused
quality of life, contributing from bedside tests by the clinician to on PD dementia but in recent years
to morbidity and
formal evaluations by a neuropsycholo- have also addressed PD-MCI or exec-
mortality, and increasing
gist. Cognition can be assessed globally utive dysfunction. Many of the trials
nursing home placement.
and with tests targeting individual have studied drugs developed for AD,
cognitive domains and functions (eg, such as cholinesterase inhibitors (eg,
orientation, attention, working mem- rivastigmine, donepezil, galantamine)
ory, processing speed, executive func- and the N-methyl-D-aspartate (NMDA)
tion, memory, language, abstract antagonist memantine.36 To date, only
1092 www.ContinuumJournal.com August 2016

Copyright © American Academy of Neurology. Unauthorized reproduction of this article is prohibited.


KEY POINTS
rivastigmine has been approved by syndromes. Both impulse control dis- h In the evaluation for
the US Food and Drug Administration orders and psychosis can occur as acute or new-onset
(FDA) for the treatment of PD demen- adverse effects of dopaminergic medi- cognitive issues,
tia based on a randomized double-blind cations, particularly those dopamine infections (eg, urinary
placebo-controlled 24-week trial of agonists with D2 and D3 mesocortico- tract infection,
over 500 subjects with mild to moder- limbic activity, whereas dopamine ago- pneumonia), metabolic
ate PD dementia and a modest but nists may improve apathy in some derangements,
statistically significant improvement patients with PD. dehydration, new
on the Alzheimer’s Disease Assessment neurologic problems
ScaleYCognitive Subscale.37 Donepezil Apathy (eg, stroke, subdural
hematoma), new
was studied in an equally large double- Apathy is a syndrome that incorporates
medical problems (eg,
blind placebo-controlled trial, with the behavioral, cognitive, and emotional
vitamin B12 deficiency,
Alzheimer’s Disease Assessment ScaleY aspects, unified by the primary loss of thyroid disorders),
Cognitive Subscale demonstrating sim- motivation and decreased goal-directed and medication effects
ilar effects at 24 weeks at 10 mg/d (after behavior.14,43 Apathy can occur alone but (eg, pain, bladder,
removing a treatment-by-country inter- also along with depression and cogni- sedating medications)
action) and improvement on the Clini- tive impairment, especially executive dys- should be excluded.
cian Interview-Based Impression of function, and may impact patients’ h It is important to inquire
Change plus caregiver input.38 Side ef- involvement in activities and social events about other comorbid
fects with cholinesterase inhibitors of and contribute to poor outcomes. neuropsychiatric
particular relevance to patients with Lack of interest, motivation, energy, problems, including poor
PD include nausea and gastrointestinal and enthusiasm, or anhedonia (ie, the nighttime sleep, excessive
problems (less so with the transdermal inability to experience pleasure) in ap- daytime sleepiness,
formulation of rivastigmine39), increased athy can overlap clinically with depres- psychosis, depression,
tremor and parkinsonism, and brady- sion. Several studies, however, suggest anxiety, and apathy,
cardia and syncope. Studies with mem- that apathy and depression can be which may influence
cognitive function.
antine and PD dementia revealed mixed dissociable in PD, as the lack of motiva-
results.40,41 More recently, cholinester- tion in PD apathy does not result from h Medications, including
ase inhibitors and other agents with emotional distress, apathy in PD does Parkinson disease
noradrenergic or dopaminergic mech- not manifest with the negativity associ- medications such as
anticholinergics and
anisms (eg, atomoxetine and mono- ated with depression, and the mood of
dopamine agonists,
amine oxidase B inhibitors) have been the apathetic patient with PD is blunted
should be carefully
studied in PD-MCI. Studying the effects rather than sad per se.44 Patients with reviewed so that agents
of mental and physical exercise on PD PD who have executive dysfunction that may contribute to
cognitive impairment is a growing field may have difficulty with initiating activ- impaired cognition,
as is the study of brain stimulation ities, switching tasks, planning, and confusion, or sleepiness
techniques (eg, transcranial magnetic processing, all of which may be neces- can be reduced
stimulation, DBS). 36,42 Improved sary for goal-directed behaviors. Pa- or discontinued.
symptomatic treatments and disease- tients with PD who are apathetic also h It is important to discuss
modifying agents for PD-MCI and PD perform worse on cognitive tasks, and driving and home safety
dementia are greatly needed. apathy may herald cognitive decline issues with patients
and dementia in PD.45 Four different, who have Parkinson
BEHAVIORAL DISORDERS but related, subdomains of apathy have disease with cognitive
Behavioral syndromes including apa- been proposed, reflecting behavioral, impairment as well as
thy, impulse control disorders, and cognitive, and affective components with their caregivers.
psychosis are common in PD. Altered and underlying dysfunction of subcorti-
cognitive or decision-making processes cal and cortical brain regions and their
and underlying neurobiological changes connections (eg, anterior cingulate,
in PD may contribute to these behavior amygdala, ventral striatum, frontal
Continuum (Minneap Minn) 2016;22(4):1086–1103 www.ContinuumJournal.com 1093

Copyright © American Academy of Neurology. Unauthorized reproduction of this article is prohibited.


Neuropsychiatric Issues in PD

KEY POINTS
h Apathy in Parkinson cortex).43 These include a reward Evaluation Scale, Apathy Scale, Apathy
disease may manifest as deficiency syndrome, depression, dys- Inventory, and Lille Apathy Rating Scale,
decreased goal-directed executive syndrome, and an auto- as well as single-item questions from
behaviors, loss of activation deficit in which patients lack the Unified PD Rating Scale and Neuro-
motivation, decreased spontaneous activation of mental pro- psychiatric Inventory.49 Of these, the
interest, or blunted cesses without external stimulation. Apathy Scale and Lille Apathy Rating
emotional responses. The prevalence of apathy ranges Scale were specifically developed for
h Proposed subdomains or from 15% to 75% depending on the PD and validated in PD populations,
types of apathy in PD population studied, with lower rates but only the Apathy Scale was recom-
Parkinson disease in early PD (20% to 36%) and higher mended by the task force. Problems
include: (1) a reward rates (over 60%) in more advanced PD. identified with apathy rating scales
deficiency syndrome in Apathy is also associated with poor out- included substantial overlap of apathy
which patients lack comes, decreased quality of life, and symptoms with depression, cognitive
emotional resonance, increased caregiver burden, similar to decline, and PD symptoms; lack of clear
(2) depression, (3)
many of the other neuropsychiatric definitions; failure of the scales to ad-
dysexecutive syndrome
symptoms encountered in PD. Thus it dress motor or nonmotor fluctuations
of cognitive deficits, and
(4) auto-activation deficit
is important to ask about apathy at all in PD; and limited clinimetric testing
in which patients stages of PD. In several de novo PD in PD.
lack spontaneous cohorts, apathy has been reported and, The pharmacologic treatment of
activation of mental in some, depressive symptoms, anhe- apathy has been anchored in medica-
processes without donia, and fatigue have also been tions for depression, dementia, or PD,
external stimulation. reported.46,47 In patients with more despite a limited evidence base.13 In
h Apathy is also associated advanced PD, apathy co-occurs with the context of comorbid depression or
with poor outcomes, cognitive impairment and dementia.45 dementia, management strategies may
decreased quality of life, Apathy may occur after DBS, often in need to address these issues. The use
and increased caregiver the first months to 1 year, and with- of antidepressants for PD apathy has
burden, similar to drawal of dopamine agonists may con- been reported mostly in case reports
many of the other tribute to postoperative apathy.48 with mixed outcomes. In a small
neuropsychiatric Several versions of diagnostic criteria randomized double-blind placebo-
symptoms encountered for apathy have been proposed over controlled trial of transdermal riva-
in Parkinson disease.
the years, with some defining a “pure” stigmine versus placebo in PD apathy
apathy syndrome distinct from de- without depression and dementia
pression or dementia. Others cover a (n = 31), a significant improvement
broader context, including comorbid in the Lille Apathy Rating Scale was
neuropsychiatric features; spanning seen after 6 months.50 Dopamine ago-
multiple neurodegenerative diseases; nists, particularly those with D2 and D3
or integrating apathy’s behavioral, receptor stimulation and involved in
cognitive, and emotional features.43 the “reward system,” have been studied
Recognition of apathy can also be chal- in PD apathy. In a randomized double-
lenging because of a lack of consensus blind placebo-controlled trial of
for diagnosis and its overlapping fea- piribedil in patients with PD with
tures with anhedonia, depression, and apathy who had DBS of the subtha-
cognitive impairment. Moreover, in the lamic nucleus (n = 37), significant
clinic, symptoms often are brought to improvement was shown in the
attention by the caregiver rather than Apathy Scale in the piribedil group
reported by the patient. Using method- with a trend toward improvement of
ology similar to other rating scale re- quality of life.51 Other dopamine ago-
views, an MDS task force reviewed nists have been studied in PD apathy
scales for apathy, including the Apathy in open-label studies or as secondary
1094 www.ContinuumJournal.com August 2016

Copyright © American Academy of Neurology. Unauthorized reproduction of this article is prohibited.


KEY POINTS
outcome measures for motor or non- Impulse Control Disorders in Parkinson’s h Punding is a stereotyped,
motor PD clinical trials. The stimulant Disease Patients Treated With Mirapex repetitive, purposeless
methylphenidate, used for attention (Pramipexole) and Other Anti-Parkinson behavior performed by
deficit hyperactivity disorder, has been Agents (DOMINION) of 3090 treated the patient. In Parkinson
considered for PD apathy, although to patients with PD, one or more impulse disease, patients with
date, only a case report is available. control disorders were identified in punding will exhibit an
13.6% of patients: compulsive buying excessive and intense
Impulse Control Disorders in 5.7%, gambling in 5.0%, binge eating preoccupation with
The impulsive-compulsive behaviors disorder in 4.3%, and compulsive sexual items or activities (eg,
identified in PD include impulse con- behavior in 3.5%.53 In one de novo PD disassembling and
reassembling objects or
trol disorders, punding, and dopamine study, impulse control disorders were
rearranging papers).
dysregulation syndrome.52 Punding is reported in 18%, although another study
a stereotyped, repetitive, purposeless did not confirm this.34,54 h Dopamine dysregulation
behavior performed by the patient. Risk factors for impulse control dis- syndrome typically
manifests in younger
While the term was originally used to orders may be extrinsic (ie, dopami-
male patients with
describe these behaviors in chronic nergic medications) or intrinsic (ie,
Parkinson disease who
amphetamine users, high doses of do- disease-related and other). Impulse develop an addictivelike
pamine replacement therapy can lead control disorders are more common regimen of self-
to similar behaviors in PD. In PD, pa- in patients with PD treated with dopa- medicating with high
tients with punding will exhibit an minergic medications, particularly do- doses of dopaminergic
excessive and intense preoccupation pamine agonists (17.1% treated with a medications, especially
with items or activities (eg, disas- dopamine agonist versus 6.9% not short-acting medications
sembling and reassembling objects or taking a dopamine agonist).53 This ap- and levodopa. Many of
rearranging papers). Dopamine dys- pears to be a drug class effect with no these patients have very
regulation syndrome, previously known significant differences between differ- minimal motor
as hedonic homeostatic dysregulation ent dopamine agonists. Levodopa has symptoms on
examination, even in
syndrome, typically manifests in youn- also been associated with impulse con-
off periods, but may
ger male patients with PD who develop trol disorders, especially at higher doses,
have dyskinesia.
an addictivelike regimen of self- and reports regarding amantadine are
medicating with high doses of dopa- mixed. Several premorbid features have h Punding and dopamine
dysregulation syndrome
minergic medications, especially been associated with impulse control
share repetitive excessive
short-acting medications and levodopa. disorders in PD: male sex, early onset
compulsive features and
Many of these patients have very mini- of PD, unmarried status, past or current a relationship with
mal motor symptoms on examination, cigarette smoking, personal or family dopaminergic stimulation.
even in off periods, but may have history of alcoholism or gambling, and
h Impulse control disorders
dyskinesia. Punding and dopamine high impulsive or novelty-seeking
in Parkinson disease
dysregulation syndrome share repeti- traits.52 It is important to inquire about frequently include
tive excessive compulsive features these personal or family history fea- pathologic gambling,
and a relationship with dopaminergic tures, in particular prior to starting a compulsive shopping,
stimulation. patient with PD on a dopamine ago- compulsive eating,
Prevalence rates of impulse control nist. The nature of the impulse control and hypersexuality.
disorders vary depending on the PD disorder may differ in males and
population studied, medications used, females, with males having higher
and behaviors included in impulse rates of compulsive sexual behavior
control disorder definitions and assess- and females having higher rates of
ments. In the study A Cross-sectional, compulsive buying and binge eating
Retrospective Screening and Case- (Case 3-2). Intrinsic factors asso-
control Study Examining the Frequency ciated with impulse control disor-
of, and Risk Factors Associated With, ders may relate to the underlying
Continuum (Minneap Minn) 2016;22(4):1086–1103 www.ContinuumJournal.com 1095

Copyright © American Academy of Neurology. Unauthorized reproduction of this article is prohibited.


Neuropsychiatric Issues in PD

neurobiology of PD. Patients with PD diction, and mood disorders, with one
who have impulse control disorders study detecting genetic polymorphisms
may have impaired decision making, in the D3 dopamine receptor (DRD3)
altered risk-reward processing, and cog- and glutamate ionotropic receptor
nitive and executive function deficits. NMDA type subunit 2B (GRIN2B) in
Neuroimaging studies suggest altered patients with PD with impulse control
dopaminergic binding, structural ab- disorders compared to those without
normalities, or disrupted networks in impulse control disorders.55
regions involved with cognitive pro- Impulse control disorders in PD are
cesses and reward (eg, ventral striatum, underrecognized, partly because of lack
orbitofrontal, anterior cingulate, and of routine screening; stigma or embar-
dorsolateral prefrontal cortex regions). rassment of symptoms; and limited
Genetic susceptibilities to impulse con- awareness of impulse control disorders
trol disorders have been considered and their association with dopami-
given their association with positive nergic medications by patients, care-
family histories of alcoholism, drug ad- givers, and health care providers.

Case 3-2
A 54-year-old woman with a 5-year history of Parkinson disease (PD)
with right arm and leg tremor and slowness came with her husband for
her clinic visit. She had been started on carbidopa/levodopa 25 mg/100 mg
initially since she was having marked tremor and difficulty at work, and
this dose was gradually increased to 8 tablets daily. She began to
develop wearing off between doses and then was started on ropinirole,
which helped her tremor, cramping, and slowness. She experienced some
anxiety when her medications were wearing off. However, she had
developed some sleepiness during the daytime and began to shop
compulsively in stores and online. She also bought lottery tickets. Her
husband was very concerned that she had exceeded their credit card limits
and that she could not focus on anything else. She felt embarrassed
about it but could not stop. Her examination revealed that she was very
mildly bradykinetic, without tremors or rigidity, and had mild to
moderate dyskinesia.
Comment. This case demonstrates a young-onset patient with PD who
has developed an impulse control disorder on high-dose levodopa and
a dopamine agonist. The impulse control disorder symptoms of compulsive
shopping and purchasing lottery tickets are disruptive and bothersome,
having personal and financial consequences in this patient. The patient was
embarrassed to discuss this. It is important for clinicians to educate patients
and caregivers about impulse control disorders and their association with
dopaminergic medications, particularly dopamine agonists, and to
inquire about these symptoms at clinic visits. Management strategies for
this patient involved reducing her ropinirole dose while monitoring for
reemergence of motor and neuropsychiatric symptoms. In addition, the
patient and her husband agreed to limit her access to money, computer time,
and lottery ticket stores. After discontinuation of her ropinirole, her
impulse control disorder symptoms resolved. She was managed on
carbidopa/levodopa with entacapone for several more years without
recurrence of her impulse control disorder before considering surgical
interventions for her PD motor symptoms.

1096 www.ContinuumJournal.com August 2016

Copyright © American Academy of Neurology. Unauthorized reproduction of this article is prohibited.


KEY POINTS
Impulse control disorders are important mood stabilizers (valproate, topiramate, h Treatment of impulse
to recognize as they can profoundly zonisamide), donepezil, finasteride, control disorders in
affect patients and caregivers and result and naltrexone. Cognitive-behavioral patients with Parkinson
in poor financial, personal, employment, therapy may have a role, but further disease typically
and health outcomes. PD-specific ques- study is needed. includes a decrease or
tionnaires and rating scales for impulse withdrawal of the
control disorders, including the Ques- Psychosis dopamine agonist, which
tionnaire for Impulsive-Compulsive Psychosis is frequent, affecting over may cause worsening
Disorders in Parkinson’s Disease and one-third of patients with PD treated of motor symptoms.
Questionnaire for Impulsive-Compulsive with chronic dopaminergic therapy h According to the
Disorders in Parkinson’s DiseaseYRating and over 50% in longitudinal studies of criteria for Parkinson
Scale, which can be administered by the 10 years or more. PD psychosis contrib- disease psychosis from
patient or a rater, or dopamine dys- utes to poor outcomes, with increased the National Institute of
regulation syndrome item on the MDS mortality, nursing home placement, Neurological Disorders
and Stroke/National
Unified PD Rating Scale may help iden- and caregiver stress.58
Institute of Mental
tify impulse control disorders.56 Pa- The clinical spectrum of PD psycho-
Health, the diagnosis of
tients with PD and their caregivers sis ranges from illusions to hallucina- Parkinson disease
should be screened for impulse control tions to delusions.59 According to the psychosis requires at least
disorders at regular clinical care visits. criteria for PD psychosis from the Na- one of the following:
Treatment of impulse control disor- tional Institute of Neurological Disorders illusions, false sense of
ders generally includes a reduction in and Stroke/National Institute of Mental presence, hallucinations,
dose or discontinuation of the dopa- Health, the diagnosis requires at least or delusions. These
mine agonist or other dopaminergic one of the following: illusions, false features should occur after
medication. The patient may require a sense of presence, hallucinations, or the onset of Parkinson
support system so as to avoid triggering delusions. These features should occur disease and be present
features (eg, money, casino) and mon- after the onset of PD and be present for for at least 1 month,
either as a recurrent or
itor progress. For many patients, im- at least 1 month, either as a recurrent or
continuous symptom;
pulse control disorder symptoms remit continuous symptom; other medical,
other medical, neurologic,
upon discontinuation of the dopami- neurologic, or psychiatric causes and or psychiatric causes
nergic agonist, although worsening of acute delirium should be excluded. and acute delirium
motor and neuropsychiatric symptoms Associated features for assessment in- should be excluded.
can occur in the withdrawal process.57 clude the presence or absence of
h Minor psychotic
In about 15% to 20% of patients with insight, dementia, or treatment for PD. phenomena and
PD, withdrawal of the dopamine ago- These criteria allow for a broader spec- nonvisual hallucinations,
nist can lead to neuropsychiatric symp- trum of PD psychosis and include some features not often
toms of withdrawal, including dysphoria, “minor” hallucinations such as illusions, included in older
anxiety, pain, and cravings. Dopami- presence hallucinations, passage hallu- epidemiologic studies,
nergic medications for PD motor symp- cinations, and simple hallucinations. represent an
toms in the setting of impulse control When criteria from the National Insti- important part of the
disorders typically include levodopa, tute of Neurological Disorders and Parkinson disease
monoamine oxidase inhibitors, and Stroke/National Institute of Mental psychosis spectrum.
possibly amantadine. To date, few ran- Health were applied to a cross-
domized controlled trials of medica- sectional PD cohort, the prevalence of
tions in PD impulse control disorders PD psychosis was 60%, compared to
have been conducted; several smaller 43% when defined by only the pres-
open-label studies, case reports, and ence of hallucinations and/or delu-
retrospective reviews describe the use sions.60 Minor psychotic phenomena
of antipsychotics (quetiapine, cloza- and nonvisual hallucinations, features not
pine), SSRIs, antiepileptic drugs or often included in older epidemiologic
Continuum (Minneap Minn) 2016;22(4):1086–1103 www.ContinuumJournal.com 1097

Copyright © American Academy of Neurology. Unauthorized reproduction of this article is prohibited.


Neuropsychiatric Issues in PD

KEY POINTS
h Illusions or studies, represent an important part of Simple hallucinations in PD lack form
misperceptions of real the PD psychosis spectrum. (eg, photopsia, flashes of light or color).
stimuli include the Most hallucinations in PD are visual, Complex hallucinations include clearly
interpretation of although they also can occur in audi- defined or formed visions that may take
inanimate objects as tory, tactile, olfactory, and gustatory the shape of animals, humans, or other
living beings (eg, a chair modalities. Illusions or mispercep- objects. Common complex visual hallu-
mistaken for a dog, a tions of real stimuli include the inter- cinations may include mice scurrying
lamppost mistaken pretation of inanimate objects as living on the floor; children playing in the
for a tree). beings (eg, a chair mistaken for a dog, house; or sometimes distorted, gro-
h Many, but not all, visual a lamppost mistaken for a tree). tesque, or bizarre figures. Many visual
hallucinations in “Presence” hallucinations evoke the hallucinations in PD are nonthreaten-
Parkinson disease are sense that someone is nearby when no ing, brief (lasting seconds to minutes),
nonthreatening, brief one is really there, whereas “passage” and worse at night or in instances of
(lasting seconds to hallucinations involve the sensation compromised or low vision.
minutes), and worse
of a person or animal passing in the Hallucinations in nonvisual modali-
at night or in instances
person’s peripheral visual field. These ties are frequently accompanied by
of compromised or
low vision.
minor hallucinations may not be trou- visual hallucinations and occur in
blesome for the patient or caregiver more advanced PD (Case 3-3). Once
and, thus, may not necessitate treat- present in multiple sensory modali-
ment interventions with antipsychotics. ties, a high risk exists of multimodal

Case 3-3
A 72-year-old man with a 6-year history of Parkinson disease (PD), with
bradykinesia, right hand resting tremor, occasional imbalance and
falls, and mild dyskinesia, reported the feeling of someone standing
behind him. He kept looking behind him, but found no one was there.
He was somewhat disturbed by this as it first began at night, but now also
occurred during the day. He had had fleeting shadows in his peripheral
vision for 1 year and intermittently saw furry animals scurrying across
the kitchen floor, which bothered him, as he kept his house neat and tidy.
He tried to reason with himself as he knew that the animals were not
really there. He was treated with five tablets of carbidopa/levodopa
25 mg/100 mg a day, pramipexole 1 mg 3 times a day, and amantadine
100 mg 3 times a day.
Comment. This case demonstrates psychosis in PD with illusions, passage
and presence hallucinations, and formed visual hallucinations. In this
case, the hallucinations initially occurred only at night but progressed to
daytime occurrence and more involved visual hallucinations. Hallucinations
in PD are often worse in settings of reduced sensory stimulation (eg,
low light, evening). This patient’s features suggest a progressive nature to
his course of hallucinations and are a potential harbinger of further
hallucinations. Since he is bothered by the hallucinations, an intervention
may be considered. The first consideration would likely be reducing or
discontinuing his amantadine or pramipexole dose. The timing of his
carbidopa/levodopa doses may also be adjusted to minimize doses in
the evening hours when psychosis may be worse. Adjustments to
dopaminergic medications may require careful balancing of motor and
nonmotor effects. Nonpharmacologic strategies, such as night-lights
in the house, may be helpful.

1098 www.ContinuumJournal.com August 2016

Copyright © American Academy of Neurology. Unauthorized reproduction of this article is prohibited.


KEY POINTS
hallucinations continuing.61 Auditory efforts regarding its occurrence and by h Auditory hallucinations
hallucinations in PD are often vague regularly asking about these symptoms in Parkinson disease are
and, in contrast to schizophrenia, are at clinic visits. Patients and caregivers often vague and, in
less common and less likely to be may be reluctant to volunteer these contrast to schizophrenia,
threatening or directly interact with symptoms because of stigma or their are less common and less
the patient. potentially bizarre nature. In many likely to be threatening
Delusions occur less often than hal- cases, caregivers are unaware of the or directly interact with
lucinations, affecting about 5% to 10% symptoms until the psychosis becomes the patient.
of patients.58 Common themes of de- problematic. Several rating scales for h Acute Parkinson disease
lusions include jealousy, spousal infi- PD psychosis have been developed for psychosis should prompt
delity, paranoia, abandonment, or research or clinical purposes, but most an evaluation for
somatic illnesses, in contrast to grandi- have been borrowed from schizophre- medical and neurologic
osity, reference, and bizarre beliefs seen nia assessment and, therefore, do not conditions other than
in schizophrenia. Misidentification syn- necessarily address the psychotic phe- Parkinson disease and a
review of the patient’s
dromes are specific types of delusions nomena in PD. The MDS Task Force on
medication regimen.
frequently associated with PD dementia Rating Scales systematically critiqued
and commonly include Capgras syn- 12 scales used for assessing PD psy-
drome (eg, patient thinks that his chosis62; of these, four scales met the
recognizable spouse is an imposter) criteria for “recommended” status, the
and Frégoli syndrome (eg, patient be- Neuropsychiatric Inventory, Brief Psy-
lieves that familiar people are disguised chiatric Rating Scale, Positive and Neg-
as strangers). ative Syndrome Scale, and Scale for
PD psychosis may be caused by ex- Assessment of Positive Symptoms, but
trinsic (ie, dopaminergic medications) none was considered to be a definitive
or intrinsic (ie, disease-related) factors. rating tool for assessing PD psychosis.
Intrinsic, or PD-related, factors include A nine-item Scale for Assessment of
abnormalities in visual, sleep, mood, Positive Symptoms for PD was created
and cognitive processes. Other risk from relevant items of the Scale for As-
factors associated with PD psychosis sessment of Positive Symptoms from
include older age, greater axial rigidity, PD clinical trials63 and used in a trial of
advanced disease, and potentially ge- pimavanserin for PD psychosis.64
netic susceptibilities. Compared to Treatments for PD psychosis include
patients with PD who do not have pharmacologic and nonpharmacologic
hallucinations, patients with PD who strategies. Factors such as temporal
hallucinate exhibit greater cognitive course (acute versus chronic), patient
deficits, particularly in attention, execu- age, presence of dementia, mood or
tive function, and visuospatial abilities. sleep disturbances, and motor severity
The pathophysiology of PD psychosis may influence treatment decisions.
has focused on three primary neuro- Acute PD psychosis should prompt an
anatomical areas, the visual system, evaluation for medical and neurologic
brainstem, and cortex, as well as accom- conditions other than PD and a review
panying neurochemical and neuropath- of the patient’s medication regimen.
ologic changes in these areas. PD Causes of PD psychosis may include
psychosis may result from interactions dopaminergic medication effects, but
between behavioral and cognitive phe- other causes, such as toxic-metabolic
nomena and abnormalities in “top- encephalopathy, urinary or pulmonary
down” or “bottom-up” processing. infections, stroke, subdural hematoma,
Recognition of PD psychosis can be or medication interactions, should be
facilitated by increasing educational excluded. Acute management of PD
Continuum (Minneap Minn) 2016;22(4):1086–1103 www.ContinuumJournal.com 1099

Copyright © American Academy of Neurology. Unauthorized reproduction of this article is prohibited.


Neuropsychiatric Issues in PD

KEY POINT
h Causes of Parkinson psychosis may require cautious short- receptor 2A (5-HT2A) receptor inverse
disease psychosis may term use of low-dose benzodiazepines. agonist, recently demonstrated improve-
include dopaminergic Specific medical causes, such as infec- ment on the Scale for Assessment of
medication effects, but tions, should be treated. Non-PD med- Positive Symptoms for PD without wors-
other causes, such as ications with centrally acting properties ened parkinsonism in patients with PD
toxic-metabolic (eg, anticholinergics for bladder hyper- psychosis and was approved by the FDA
encephalopathy, urinary activity, TCAs for depression, benzodi- for treatment of PD psychosis in 2016.64
or pulmonary infections, azepines for anxiety or sleep, hypnotics Once antipsychotic therapy is initiated,
stroke, subdural for sleep, and opioids for pain) should continued treatment may be necessary
hematoma, or be reduced or stopped. Dopaminergic to maintain control of psychosis. Cho-
medication interactions,
medications for PD motor symptoms linesterase inhibitors may play a role in
should be excluded.
may need to be reduced or discon- the treatment of patients with PD who
tinued, in some cases while monitoring have dementia and hallucinations.
for worsened motor function.
Antipsychotic medications may be CONCLUSION
required for acute and chronic PD Neuropsychiatric issues in PD repre-
psychosis, particularly if dopaminergic sent a growing area of focus in the
medications cannot be reduced without multidisciplinary care of patients with
worsening parkinsonism. To date, few PD at all stages of their disease. While
randomized double-blind placebo- increased recognition of neuropsychi-
controlled trials of atypical antipsy- atric symptoms exists, there remains a
chotics in PD have been conducted.11 great need for patient, caregiver, and
Clozapine is supported by double- health care provider education; inquiry
blind placebo-controlled trials in about these symptoms at clinical visits;
PD demonstrating improvement in and development of assessment tools
psychosis without worsening of motor and rating scales. Advances are needed
function. However, its use requires for symptomatic treatment and disease-
blood count monitoring given the rare modifying agents.
idiosyncratic risk of agranulocytosis
as well as monitoring for side effects REFERENCES
of sedation or hypotension. Quetiapine 1. Maier F, Merkl J, Ellereit AL, et al. Hypomania
and mania related to dopamine replacement
demonstrates comparable efficacy to therapy in Parkinson’s disease. Parkinsonism
clozapine in comparison studies, Relat Disord 2014;20(4):421Y427. doi:10.1016/
frequent and easy administration, and j.parkreldis.2014.01.001.
a relatively low-risk side effect profile, 2. Voon V, Howell NA, Krack P. Psychiatric
but it has not been successful in double- considerations in deep brain stimulation for
Parkinson’s disease. Handb Clin Neurol
blind placebo-controlled randomized 2013;116:147Y154. doi:10.1016/B978-0-444-
trials. Doses for clozapine and quetiapine 53497-2.00012-7.
to treat PD psychosis are typically lower 3. Reijnders JS, Ehrt U, Weber WE, et al.
than those used for schizophrenia or A systematic review of prevalence studies of
other psychiatric disorders. In contrast depression in Parkinson’s disease. Mov
Disord 2008;23(2):183Y189; quiz 313.
to the serotonergic clozapine and que- doi:10.1002/mds.21803.
tiapine, medications with greater dopa- 4. American Psychiatric Association. Diagnostic
mine blockade, including olanzapine, and statistical manual of mental disorders,
should be avoided because of risk of fifth edition. Washington, DC: American
Psychiatric Publishing, 2013.
increased parkinsonism. Antipsychotics
5. American Psychiatric Association. Diagnostic
also carry a black box warning for use in
and statistical manual of mental disorders,
elderly patients with dementia. Pimavan- fourth edition. Washington, DC:
serin, a 5-hydroxytryptamine, serotonin American Psychiatric Publishing, 2000.

1100 www.ContinuumJournal.com August 2016

Copyright © American Academy of Neurology. Unauthorized reproduction of this article is prohibited.


6. Marsh L, McDonald WM, Cummings J, treatments for depression in Parkinson’s
Ravina B; NINDS/NIMH Work Group on disease: a double-blind, randomized,
Depression and Parkinson’s Disease. placebo-controlled study. Mov Disord
Provisional diagnostic criteria for depression 2008;23(6):850Y857. doi:10.1002/mds.21966.
in Parkinson’s disease: report of an
18. Dobkin RD, Menza M, Allen LA, et al.
NINDS/NIMH Work Group. Mov Disord
Cognitive-behavioral therapy for depression
2006;21(2):148Y158. doi:10.1002/mds.20723.
in Parkinson’s disease: a randomized,
7. Nazem S, Siderowf AD, Duda JE, et al. controlled trial. Am J Psychiatry
Suicidal and death ideation in Parkinson’s 2011;168(10):1066Y1074. doi:10.1176/
disease. Mov Disord 2008;23(11):1573Y1579. appi.ajp.2011.10111669.
doi:10.1002/mds.22130.
19. Dissanayaka NN, White E, O’Sullivan JD,
8. Leentjens AF, Dujardin K, Marsh L, et al. et al. The clinical spectrum of anxiety in
Anxiety and motor fluctuations in Parkinson’s disease. Mov Disord
Parkinson’s disease: a cross-sectional 2014;29(8):967Y975. doi:10.1002/mds.25937.
observational study. Parkinsonism Relat
20. Starkstein SE, Dragovic M, Dujardin K, et al.
Disord 2012;18(10):1084Y1088. doi:10.1016/
Anxiety has specific syndromal profiles in
j.parkreldis.2012.06.007.
Parkinson disease: a data-driven approach.
9. Fang F, Xu Q, Park Y, et al. Depression and Am J Geriatr Psychiatry 2014;22(12):
the subsequent risk of Parkinson’s disease 1410Y1417. doi:10.1016/j.jagp.2013.09.006.
in the NIH-AARP Diet and Health Study.
21. Sagna A, Gallo JJ, Pontone GM. Systematic
Mov Disord 2010;25(9):1157Y1162.
review of factors associated with depression
doi:10.1002/mds.23092.
and anxiety disorders among older adults
10. de la Riva P, Smith K, Xie SX, Weintraub D. with Parkinson’s disease. Parkinsonism
Course of psychiatric symptoms and global Relat Disord 2014;20(7):708Y715.
cognition in early Parkinson disease. doi:10.1016/j.parkreldis.2014.03.020.
Neurology 2014;83(12):1096Y1103.
22. Ishihara L, Brayne C. What is the evidence
doi:10.1212/WNL.0000000000000801.
for a premorbid parkinsonian personality:
11. Dissanayaka NN, Sellbach A, Silburn PA, et al. a systematic review. Mov Disord 2006;
Factors associated with depression in 21(8):1066Y1072. doi:10.1002/mds.20980.
Parkinson’s disease. J Affect Disord 2011;
23. Duncan GW, Khoo TK, Yarnall AJ, et al.
132(1Y2):82Y88. doi:10.1016/j.jad.2011.01.021.
Health-related quality of life in early
12. Schrag A, Barone P, Brown RG, et al. Parkinson’s disease: the impact of nonmotor
Depression rating scales in Parkinson’s symptoms. Mov Disord 2014;29(2):195Y202.
disease: critique and recommendations. doi:10.1002/mds.25664.
Mov Disord 2007;22(8):1077Y1092.
24. Barone P, Antonini A, Colosimo C, et al. The
doi:10.1002/mds.21333.
PRIAMO study: a multicenter assessment of
13. Seppi K, Weintraub D, Coelho M, et al. nonmotor symptoms and their impact on
The Movement Disorder Society quality of life in Parkinson’s disease. Mov Disord
Evidence-Based Medicine Review Update: 2009;24(11):1641Y1649. doi:10.1002/mds.22643.
treatments for the non-motor symptoms of
25. Leentjens AF, Dujardin K, Marsh L, et al.
Parkinson’s disease. Mov Disord 2011;
Anxiety rating scales in Parkinson’s disease:
26(suppl 3):S42YS80. doi:10.1002/mds.23884.
critique and recommendations. Mov
14. Gallagher DA, Schrag A. Psychosis, apathy, Disord 2008;23(14):2015Y2025.
depression and anxiety in Parkinson’s doi:10.1002/mds.22233.
disease. Neurobiol Dis 2012;46(3):581Y589.
26. Pontone GM, Williams JR, Anderson KE,
doi:10.1016/j.nbd.2011.12.041.
et al. Pharmacologic treatment of anxiety
15. Richard IH, McDermott MP, Kurlan R, et al. disorders in Parkinson disease. Am J Geriatr
A randomized, double-blind, placebo-controlled Psychiatry 2013;21(6):520Y528. doi:10.1016/
trial of antidepressants in Parkinson disease. j.jagp.2012.10.023.
Neurology 2012;78(16):1229Y1236. doi:10.1212/
27. Litvan I, Aarsland D, Adler CH, et al. MDS
WNL.0b013e3182516244.
Task Force on mild cognitive impairment in
16. Menza M, Dobkin RD, Marin H, et al. A Parkinson’s disease: critical review of
controlled trial of antidepressants in PD-MCI. Mov Disord 2011;26(10):1814Y1824.
patients with Parkinson disease and doi:10.1002/mds.23823.
depression. Neurology 2009;72(10):886Y892.
28. Aarsland D, Kurz MW. The epidemiology of
doi:10.1212/01.wnl.0000336340.89821.b3.
dementia associated with Parkinson’s
17. Devos D, Dujardin K, Poirot I, et al. disease. Brain Pathol 2010;20(3):633Y639.
Comparison of desipramine and citalopram doi:10.1111/j.1750-3639.2009.00369.x.

Continuum (Minneap Minn) 2016;22(4):1086–1103 www.ContinuumJournal.com 1101

Copyright © American Academy of Neurology. Unauthorized reproduction of this article is prohibited.


Neuropsychiatric Issues in PD

29. Litvan I, Goldman JG, TrPster AI, et al. placebo-controlled trial. Lancet Neurol
Diagnostic criteria for mild cognitive 2010;9(10):969Y977. doi:10.1016/S1474-
impairment in Parkinson’s disease: 4422(10)70194-0.
Movement Disorder Society Task Force
41. Aarsland D, Ballard C, Walker Z, et al.
guidelines. Mov Disord 2012;27(3):349Y356.
Memantine in patients with Parkinson’s
doi:10.1002/mds.24893.
disease dementia or dementia with Lewy
30. Emre M, Aarsland D, Brown R, et al. Clinical bodies: a double-blind, placebo-controlled,
diagnostic criteria for dementia associated multicentre trial. Lancet Neurol 2009;
with Parkinson’s disease. Mov Disord 2007;22(12): 8(7):613Y618. doi:10.1016/S1474-
1689Y1707; quiz 837. doi:10.1002/mds.21507. 4422(09)70146-2.
31. Williams-Gray CH, Foltynie T, Brayne CE, 42. Hindle JV, Petrelli A, Clare L, Kalbe E.
et al. Evolution of cognitive dysfunction in Nonpharmacological enhancement of
an incident Parkinson’s disease cohort. Brain cognitive function in Parkinson’s disease:
2007;130(pt 7):1787Y1798. doi:10.1093/ a systematic review. Mov Disord 2013;
brain/awm111. 28(8):1034Y1349. doi:10.1002/mds.25377.

32. Aarsland D, Brønnick K, Larsen JP, et al. 43. Pagonabarraga J, Kulisevsky J, Strafella AP,
Cognitive impairment in incident, untreated Krack P. Apathy in Parkinson’s disease:
Parkinson disease: the Norwegian ParkWest clinical features, neural substrates, diagnosis,
study. Neurology 2009;72(13):1121Y1126. and treatment. Lancet Neurol 2015;14(5):
doi:10.1212/01.wnl.0000338632.00552.cb. 518Y531. doi:10.1016/S1474-4422(15)00019-8.

33. Yarnall AJ, Breen DP, Duncan GW, et al. 44. Kirsch-Darrow L, Marsiske M, Okun MS,
Characterizing mild cognitive impairment in et al. Apathy and depression: separate
incident Parkinson disease: the ICICLE-PD factors in Parkinson’s disease. J Int
study. Neurology 2014;82(4):308Y316. Neuropsychol Soc 2011;17(6):1058Y1066.
doi:10.1212/WNL.0000000000000066. doi:10.1017/S1355617711001068.
34. Weintraub D, Simuni T, Caspell-Garcia C, et al. 45. Dujardin K, Sockeel P, Delliaux M, et al.
Cognitive performance and neuropsychiatric Apathy may herald cognitive decline and
symptoms in early, untreated Parkinson’s dementia in Parkinson’s disease. Mov Disord
disease. Mov Disord 2015;30(7):919Y927. 2009;24(16):2391Y2397. doi:10.1002/
doi:10.1002/mds.26170. mds.22843.
35. Halliday GM, Leverenz JB, Schneider JS, 46. Dujardin K, Langlois C, Plomhause L, et al.
Adler CH. The neurobiological basis of Apathy in untreated early-stage Parkinson
cognitive impairment in Parkinson’s disease. disease: relationship with other non-motor
Mov Disord 2014;29(5):634Y650. symptoms. Mov Disord 2014;29(14):
doi:10.1002/mds.25857. 1796Y1801. doi:10.1002/mds.26058.
36. Goldman JG, Weintraub D. Advances in the 47. Erro R, Picillo M, Vitale C, et al. Non-motor
treatment of cognitive impairment in symptoms in early Parkinson’s disease:
Parkinson’s disease. Mov Disord 2015; a 2-year follow-up study on previously
30(11):1471Y1489. doi:10.1002/mds.26352. untreated patients. J Neurol Neurosurg
Psychiatry 2013;84(1):14Y17. doi:10.1136/
37. Emre M, Aarsland D, Albanese A, et al.
jnnp-2012-303419.
Rivastigmine for dementia associated with
Parkinson’s disease. N Engl J Med 2004; 48. Thobois S, Ardouin C, Lhommée E, et al.
351(24):2509Y2518. doi:10.1056/ Non-motor dopamine withdrawal syndrome
NEJMoa041470. after surgery for Parkinson’s disease:
predictors and underlying mesolimbic
38. Dubois B, Tolosa E, Katzenschlager R, et al. denervation. Brain 2010;133(pt 4):1111Y1127.
Donepezil in Parkinson’s disease dementia: doi:10.1093/brain/awq032.
a randomized, double-blind efficacy and
safety study. Mov Disord 2012;27(10): 49. Leentjens AF, Dujardin K, Marsh L, et al. Apathy
1230Y1238. doi:10.1002/mds.25098. and anhedonia rating scales in Parkinson’s
disease: critique and recommendations. Mov
39. Emre M, Poewe W, De Deyn PP, et al. Disord 2008;23(14):2004Y2014. doi:10.1002/
Long-term safety of rivastigmine in Parkinson mds.22229.
disease dementia: an open-label, randomized
50. Devos D, Moreau C, Maltête D, et al.
study. Clin Neuropharmacol 2014;37(1):9Y16.
Rivastigmine in apathetic but dementia and
doi:10.1097/WNF.0000000000000010.
depression-free patients with Parkinson’s
40. Emre M, Tsolaki M, Bonuccelli U, et al. disease: a double-blind, placebo-controlled,
Memantine for patients with Parkinson’s randomised clinical trial. J Neurol Neurosurg
disease dementia or dementia with Lewy Psychiatry 2014;85(6):668Y674. doi:10.1136/
bodies: a randomised, double-blind, jnnp-2013-306439.

1102 www.ContinuumJournal.com August 2016

Copyright © American Academy of Neurology. Unauthorized reproduction of this article is prohibited.


51. Thobois S, Lhommée E, Klinger H, et al. Sci 2010;289(1Y2):12Y17. doi:10.1016/
Parkinsonian apathy responds to j.jns.2009.08.014.
dopaminergic stimulation of D2/D3
59. Ravina B, Marder K, Fernandez HH, et al.
receptors with piribedil. Brain 2013;136(pt 5):
Diagnostic criteria for psychosis in
1568Y1577. doi:10.1093/brain/awt067.
Parkinson’s disease: report of an NINDS,
52. Weintraub D, David AS, Evans AH, et al. NIMH work group. Mov Disord 2007;
Clinical spectrum of impulse control 22(8):1061Y1068. doi:10.1002/mds.21382.
disorders in Parkinson’s disease. Mov Disord 60. Fénelon G, Soulas T, Zenasni F, et al.
2015;30(2):121Y127. doi:10.1002/mds.26016. The changing face of Parkinson’s
53. Weintraub D, Koester J, Potenza MN, et al. disease-associated psychosis: a cross-sectional
Impulse control disorders in Parkinson study based on the new NINDS-NIMH criteria.
disease: a cross-sectional study of 3090 Mov Disord 2010;25(6):763Y766. doi:10.1002/
patients. Arch Neurol 2010;67(5):589Y595. mds.22839.
doi:10.1001/archneurol.2010.65. 61. Goetz CG, Stebbins GT, Ouyang B. Visual
54. Antonini A, Siri C, Santangelo G, et al. plus nonvisual hallucinations in Parkinson’s
Impulsivity and compulsivity in drug-naı̈ve disease: development and evolution over
patients with Parkinson’s disease. Mov Disord 10 years. Mov Disord 2011;26(12):2196Y2200.
2011;26(3):464Y468. doi:10.1002/mds.23501. doi:10.1002/mds.23835.

55. Napier TC, Corvol JC, Grace AA, et al. Linking 62. Fernandez HH, Aarsland D, Fénelon G, et al.
neuroscience with modern concepts of Scales to assess psychosis in Parkinson’s
impulse control disorders in Parkinson’s disease: critique and recommendations. Mov
disease. Mov Disord 2015;30(2):141Y149. Disord 2008;23(4):484Y500. doi:10.1002/
doi:10.1002/mds.26068. mds.21875.

56. Weintraub D, Mamikonyan E, Papay K, et al. 63. Voss T, Bahr D, Cummings J, et al.
Questionnaire for Impulsive-Compulsive Performance of a shortened Scale for
Disorders in Parkinson’s Disease-Rating Scale. Assessment of Positive Symptoms for
Mov Disord 2012;27(2):242Y247. doi:10.1002/ Parkinson’s disease psychosis. Parkinsonism
mds.24023. Relat Disord 2013;19(3):295Y299.
doi:10.1016/j.parkreldis.2012.10.022.
57. Rabinak CA, Nirenberg MJ. Dopamine
agonist withdrawal syndrome in Parkinson 64. Cummings J, Isaacson S, Mills R, et al.
Pimavanserin for patients with Parkinson’s
disease. Arch Neurol 2010;67(1):58Y63.
doi:10.1001/archneurol.2009.294. disease psychosis: a randomised,
placebo-controlled phase 3 trial. Lancet
58. Fénelon G, Alves G. Epidemiology of 2014;383(9916):533Y540. doi:10.1016/S0140-
psychosis in Parkinson’s disease. J Neurol 6736(13)62106-6.

Continuum (Minneap Minn) 2016;22(4):1086–1103 www.ContinuumJournal.com 1103

Copyright © American Academy of Neurology. Unauthorized reproduction of this article is prohibited.


Review Article

Treatment of Advanced
Address correspondence to
Dr Janis M. Miyasaki, 7-133
Clinical Sciences Building,
11350 83rd Ave, University of
Alberta, Edmonton, AB T6G
2G3, Canada,
miyasaki@ualberta.ca.
Parkinson Disease and
Relationship Disclosure:
Dr Miyasaki received personal
compensation as a consultant
Related Disorders
for Merz Pharma Group and
as a lecturer for Teva Janis M. Miyasaki, MD, MEd, FRCPC, FAAN
Pharmaceuticals Industries
Ltd, has received research/
grant support as principal
investigator of studies for
ABSTRACT
Parkinson Alberta and Purpose of Review: Parkinson disease often spans decades of a patient’s lifetime.
Patient-Centered Outcomes Over time, nonmotor symptoms predominate and may limit dopaminergic therapy.
Research Institute, and
receives royalties from Neurologists continue to play a vital role in treatment. In addition to balancing
UpToDate Inc. neurobehavioral complications of Parkinson disease with motor benefit, addressing
Unlabeled Use of nonmotor symptoms common in the advanced stage may improve quality of life
Products/Investigational
Use Disclosure:
and reduce symptom burden. Symptoms such as dysphagia, constipation, urinary
Dr Miyasaki reports dysfunction, orthostatic hypotension, and pain respond to nonpharmacologic and
no disclosure. pharmacologic therapies.
* 2016 American Academy Recent Findings: Evidence for treatment of many nonmotor symptoms is weak or
of Neurology.
lacking. The evidence for treatment of the atypical parkinsonian syndromes (pro-
gressive supranuclear palsy, multiple system atrophy, and corticobasal degeneration) in
advanced stages is even more scant.
Summary: Engaging palliative care physicians in the joint care of patients can provide
patients with access to expertise in end-of-life issues. Neurologic illnesses have specific
hospice criteria to guide clinicians for referrals. Evidence supports that assisting patients
with advance directives can result in improved satisfaction with care and improved
quality of life in the last weeks of life. Neurologists can remain engaged in their
patients’ care throughout the course of illness.

Continuum (Minneap Minn) 2016;22(4):1104–1116.

INTRODUCTION system atrophy, and corticobasal de-


Parkinson disease (PD) often spans de- generation have similar challenges but
cades of a patient’s lifetime. In addi- less evidence exists to guide care in
tion to complications in motor response advanced stages. Therefore, adapting
to treatment, nonmotor symptoms evidence from PD and applying this
begin to predominate in the later stages evidence to other neurodegenerative
of illness.1 A patient’s response to diseases is appropriate.
levodopa often wanes, resulting in im-
mobility, reduced quality of life, and PROGRESSION OF ILLNESS
increased caregiver burden. 2 Ap- A study of patients who had PD for at
proaches to treatment for those in least 10 years found that the average
the last 5 years of life require incorpo- Montreal Cognitive Assessment (MoCA)
ration of psychosocial interventions, score was 23.5 (plus or minus 3.8
Supplemental digital content: support for the spouse, and careful points), and 12% of patients were on
Direct URL citations appear in discussions to determine the goals of cognitive-enhancing medications.3 Fur-
the printed text and are pro-
vided in the HTML, PDF, and care for the patient and family. Pro- thermore, those who survived 20 years
app versions of this article. gressive supranuclear palsy, multiple or more were termed elite patients

1104 www.ContinuumJournal.com August 2016

Copyright © American Academy of Neurology. Unauthorized reproduction of this article is prohibited.


KEY POINT
with young-onset illness and levodopa caregiver burden. 4,6,10Y13 As noted h Although individual
responsiveness.4 Despite the elite sta- in Table 4-2, the majority of patients progression is hard to
tus, these patients had motor fluctua- with advanced PD live in a long-term predict in patients with
tions, cognitive decline with problems care facility. Parkinson disease,
in verbal fluency and delayed recall, Perhaps more reflective of the overall there is predictable
and the median Hoehn and Yahr scale average patient with PD in the United progression over time to
stage was 3. (See Table 4-15 for a States is the retrospective cohort more nonmotor
listing of the stages of the Hoehn and study of Medicare beneficiaries diag- symptoms such as
Yahr scale.) Quality of life was mildly to nosed with PD.14 Of the 469,055 dysphagia, cognitive
moderately reduced. Caregivers re- individuals receiving Medicare bene- decline, and often
fits in 2002, 25% lived in a long-term diminished dopaminergic
ported more strain, particularly with
responsiveness.
social isolation. Despite their disability, care facility. Women were more likely
58% of patients still participated in to live in long-term care (1.34 odds
some form of exercise or physical ratio). Hip fracture and dementia were
therapy, and 89% lived at home. This the strongest clinical predictors of
cohort consisted of patients attending placement (2.1 and 4.06 odds ratios,
a National Parkinson Foundation Cen- respectively). Few received outpatient
ters of Excellence participating in the neurologist care (33%). By 3 years
Quality Improvement Initiative re- follow-up, 84% of patients had died.
search project and, therefore, may not The greatest predictor of receiving
reflect all US people with PD. Indeed, hospice care was outpatient neurolo-
the National Parkinson Foundation gist care (2.35 odds ratio).
Quality Improvement Initiative co- Tertiary care centers often follow
hort seems to have fared better than smaller numbers of patients who are
other studies from Australia and China stages 4 and 5 of the Hoehn and Yahr
(Table 4-2).6Y9 Mild cognitive impair- scale, yet data from Medicare confirms
ment and dementia are important that patients who are bed bound
complications in PD and can result in increasingly occupy hospice registries.
hallucinations and the necessity of Why have these patients stopped
altering antiparkinsonian medications seeing their neurologists? An obvious
to reduce delirium, which then results explanation is the logistics behind a
in reduced quality of life and increased simple visit outside the home or

a
TABLE 4-1 Stages of Hoehn and Yahr Scale

1.0 Unilateral involvement only


1.5 Unilateral and axial involvement
2.0 Bilateral involvement without impairment of balance
2.5 Mild bilateral disease with recovery on pull test
3.0 Mild to moderate bilateral disease; some postural instability;
physically independent
4.0 Severe disability; still able to walk or stand unassisted
5.0 Wheelchair dependent or bedridden unless aided
a
Reprinted with permission from Goetz CG, et al, Mov Disord.5 B 2004 Movement Disorder
Society. onlinelibrary.wiley.com/doi/10.1002/mds.20213/abstract.

Continuum (Minneap Minn) 2016;22(4):1104–1116 www.ContinuumJournal.com 1105

Copyright © American Academy of Neurology. Unauthorized reproduction of this article is prohibited.


Advanced Parkinson Disease

TABLE 4-2 Progression of Parkinson Disease in Various Cohorts

Follow-up Living
Study N= Period Alive Mortality Rate Independently Psychosis Choking
Hely et al, 136 20 years 36 3.1 times that of 3% 74% 48%
20087 age-matched controls
Auyeung 171 10 years 121 1.1 times that of 32% 60% 60%
et al, 20128 age-matched controls
Forsaa et al, 230 12 years 25 60%
20109

skilled nursing facility when a person patients’ needs, the available re-
is wheelchair dependent. Another con- sources in the system to support them
sideration is whether patients feel utility (eg, home care, hospice, day pro-
exists in continued connection. grams), and improve the safety and
Qualitative studies confirm that quality of patient care.
patients and caregivers feel alone in What does this mean for neurolo-
their struggle with PD, find the health gists caring for patients with PD?
care system confusing, and care is Giving information regarding what to
often provided in silos without com- expect in the next stage of illness is
munication between segments of the important for patients as they prepare
health care system, which is challeng- for their work, home, and family lives.
ing to navigate and financially For instance, patients with postural
straining.15Y17 One group proposed instability will require adjustments to
four stages for physicians to follow their home to reduce the risk of falls,
(Table 4-3).18 This framework empha- may require the use of a walker for
sizes the role of physician as educator safe ambulation, and may require a
and advocate and requires integration wheelchair for unpredictable or longer
of systems-based practice. That is, distances, which can be facilitated by a
neurologists need to be aware of their home safety assessment. The home

a
TABLE 4-3 Four Stages of Information Sharing in Parkinson Disease

Stage Physician’s Role


1 Provide information and education on Parkinson disease
2 Educate patients regarding medication and importance of
adherence to schedules and developing a social support system
3 Highlight safe living environment and encourage participation in
support groups
4 Assist patients in finding the meaning of life and the value
of existence

a
Data from Liao YC, et al, J Neurosci Nurs.18 journals.lww.com/jnnonline/pages/articleviewer.aspx?
year=2013&issue=12000&article=00007&type=abstract.

1106 www.ContinuumJournal.com August 2016

Copyright © American Academy of Neurology. Unauthorized reproduction of this article is prohibited.


KEY POINT
setting, including where the bedroom member of society, and financial ben- h Questions about
and full bathroom are located, be- efits, there begins to be a physical cost function at work and
comes important when stairs cannot (including fatigue) to working. Moni- the ability to participate
be negotiated independently. Al- toring the balance between the bene- in activities with the
though postural instability begins with fits and physical costs of working is family can help patients
Hoehn and Yahr scale stage 2.5, how important for patients with PD. Al- gauge whether work is
often do neurologists counsel patients though neurologists cannot make the becoming too difficult
and families to begin planning changes decisions for patients, physicians can to manage. Similarly,
in the home setting for safety? Where pose questions to help patients self- questions about
stair lifts are not possible, moving to an monitor. The ability to do activities managing stairs, the
home setup, or
apartment, condominium, or bungalow after work, engage in family activities,
organizing a home
may soon be required. This involves a and performance appraisals (assessment
safety visit can nudge
patient not only leaving his or her home by an employer regarding the em- patients and caregivers
that has been cherished for decades, ployee’s ability to complete tasks for a to consider moving to a
but may require leaving familiar neigh- job or profession correctly and in a more accessible home.
borhoods and the social network timely manner) are often good indica-
established over a lifetime. In the tors of the ability to continue working.
author’s experience, it can take 2 years
for a patient and spouse to actually CAREGIVERS AS PART OF THE
move from their home after having had CARE TEAM
the conversation on many occasions. Caregivers play in integral role in the
Some may not move until a crisis occurs. care of patients with PD. Initially,
Similarly, employment for patients caregivers are an informant, providing
with PD is challenging. While employ- another witness to the patient’s symp-
ment provides intellectual stimulation, toms (Case 4-1). For example, dyski-
social engagement, a routine, mainte- nesia is often not obvious to the
nance of self-image as a contributing patient, but spouses or family members

Case 4-1
A 59-year-old man presented for an evaluation of his Parkinson disease
(PD). He had been diagnosed with PD 8 years ago and had retired from his
occupation as a store clerk 2 years ago due to disability from his PD. He
had difficulty giving the details of his medication history. He was
dyskinetic during the visit but described that he was in the “off” stage for
much of the day and wanted his medications significantly increased since
this had worked for him in the past. His wife stated that her husband was
often confused, did not seem to take his medications in a predictable
manner, often forgot doses, acted impulsively, and was very dyskinetic to
the point that he was off balance due to dyskinesia. She expressed that her
husband had not wanted her to accompany him to appointments in the
past, but she wanted information and to ensure that the information
given to the new neurologist was accurate.
Comment. It is possible to engage family members or spouses in the
interview process and maintain patient autonomy and dignity. Patients
may not have insight to their illness. Obtaining collateral history from
family members is important. Asking the patient’s permission to get the
spouse’s or family member’s viewpoint can allow valuable information to
surface. This can strengthen the physicianYfamily caregiver bond and
improve patient and caregiver care.

Continuum (Minneap Minn) 2016;22(4):1104–1116 www.ContinuumJournal.com 1107

Copyright © American Academy of Neurology. Unauthorized reproduction of this article is prohibited.


Advanced Parkinson Disease

KEY POINT
h Assessing caregiver will observe and report dyskinesia or as advocates for patients remaining at
burden with the Zarit dystonia. Insights to cognitive func- home with support programs in place.
Burden Interview will tion, behavioral changes, or hopeless-
help identify those ness often originate from caregiver PALLIATIVE CARE APPROACH
caregivers requiring reports. In later stages, caregivers add The scenario in Case 4-2 is a common
assistance. Small care coordination to their role. They representation of patients who have
changes may enable must often assume decision making had PD for more than 10 years.3,4
patients to remain at and, therefore, are responsible for Cognitive decline and dementia can
home longer by engaging outside help, community result in antiparkinsonian medication
providing respite to organizations, and medical care for reduction. The presence of nonmotor
the caregiver.
the person with PD. Added to these symptoms presents a challenge to the
organizational tasks is the physical practitioner at this stage of illness.
burden of caring for someone requir- Explaining to families that neuro-
ing increasing assistance with activities behavioral symptoms are frequent
of daily living. Often, family caregivers consequences of neurodegenerative
are either spouses of a similar age with disease can help family members link
their own health and cognitive prob- patients’ inexplicable behavior to a neu-
lems or adult children with families to rologic diagnosis. Including spouses
raise in addition to providing care for and other family members as an impor-
their parent. Assessing caregiver strain tant part of the care team by eliciting
can help identify those patient- their observations and care goals can be
caregiver dyads requiring assistance. valuable in management of the disease.
A useful tool is the Zarit Burden In the author’s experience with the first
Interview.19,20 Items in the scale in- interdisciplinary neurologist-led pallia-
clude caregiving burden, anger, rela- tive care program for PD and related
tionships with others, fear for the disorders, families often were unaware
future, health of the caregiver, impact that in advanced stages they had a
on social life, privacy, financial strain, choice between optimal motor function
and other items reflecting strain. Care- or cognitive clarity or that this balancing
givers score items as never, sometimes, act would dominate future care conver-
quite frequently, and nearly always. sations. Additional discussion of the
Even minimal changes such as in- diagnosis and treatment of neuropsy-
home care by paid caregivers can chiatric complications of PD is discussed
provide welcome respite for the family. in the article “Neuropsychiatric Issues in
Continued contact with patients and Parkinson Disease” by Jennifer G.
families even when improvement is no Goldman, MD, MS, FAAN21 in this issue
longer the treatment goal can be valu- of Continuum.
able, and neurologists often have the Recently, palliative care approaches
best insights into the nature and degree to late-stage PD and parkinsonian
of disability of neurodegenerative dis- disorders have been implemented in
eases and can help educate the home earlier stages.20,22,23 Previous studies
care providers about the patients. have examined place of death and
Day programs can be helpful, partic- cause of death in PD.24,25 In brief, these
ularly if transportation to the program is studies demonstrate that the majority
available. Volunteer organizations can of patients die in an acute care hospi-
provide friendly visitors to intellectually tal, and few patients have discussions
challenge patients and provide care- with their physician regarding prefer-
givers with time away from the patient. ences for care delivery or place of
Neurologists can provide a valuable role death. Establishing palliative care for

1108 www.ContinuumJournal.com August 2016

Copyright © American Academy of Neurology. Unauthorized reproduction of this article is prohibited.


Case 4-2
A 77-year-old man was first diagnosed with Parkinson disease 17 years
ago. He was an active participant in the local support group. Up until the
past year, he exercised several times a week, and his response to levodopa
was well maintained. In the past year, his ability to provide details of his
history and medication schedule had deteriorated. His wife attended visits
with him, and she reported that since medications were reduced due to
hallucinations, he had severe pain at night. This lasted for hours, and she
admitted to occasionally giving him extra doses of levodopa with good
response. However, the next day he was drowsier and at times confused.
Due to slowness getting to the bathroom, the patient limited water
intake. He was also constipated, and it was suspected that this was
contributing to poor levodopa response due to delayed gastric emptying
and poor absorption. The patient frequently coughed when drinking
liquids and had choked on soup several times in the past month. He
became anxious if his wife left him on his own. His wife felt strained caring
for him and was concerned about the future. They planned to sell their
two-story home and move to a condominium.
Comment. Advanced Parkinson disease and related disorders result in a
series of losses for the patient and caregiver. Patients lose their
employment, sense of identity, and often cognitive function. Their
disability often results in the need for patients to move from their homes,
resulting in loss of friends and social networks. Helping patients and
caregivers anticipate needs can allow them to come to terms with
impending losses so that decisions are not deferred until crises occur.

PD can help treat patients and families tive is further affected by the admitting
in a holistic manner, focus on non- service for in-hospital care, with at least
motor symptoms, and establish pa- one study finding that surgical services
tients’ goals of care and their wishes were more likely to have an advance
regarding place of death. A formal directive determined at admission.27
palliative care team consists of physi- However, the correct documentation
cians, social workers, nurses, and spiri- of the presence of an advance directive
tual or pastoral care. However, many was zero out of 100 reviewed charts,28
aspects of palliative care can be incor- and another study indicated that phy-
porated into individual practice. Neu- sicians would disregard the patient’s
rologic follow-up does not need to end wishes in the face of a request by a
upon admission to a long-term care relative to continue resuscitation ef-
facility (Case 4-3). forts or intensive care treatment.29 A
Studies demonstrate that physicians’ further study found that of 306 study
initiation of the conversation, the pa- participants, only 25% had a durable
tient’s higher education level, and ex- power of attorney, and 15% had an
posure to advance directive campaigns advance directive.30 Although 74% of
positively influence patients to com- study participants felt that it was
plete an advance directive.26 Despite important to make their wishes
this, among those over 65 years of age known to their doctor, only 16% had
attending a tertiary care hospital, only done so. Discussing their wishes with
63% had completed an advance direc- family was important to 87% of par-
tive. Completion of an advance direc- ticipants, and 60% had done so.

Continuum (Minneap Minn) 2016;22(4):1104–1116 www.ContinuumJournal.com 1109

Copyright © American Academy of Neurology. Unauthorized reproduction of this article is prohibited.


Advanced Parkinson Disease

KEY POINTS
h Advance care planning
or advance directives are Case 4-3
often completed by the A 79-year-old woman with a 6-year history of Parkinson disease presented
patient without a for follow-up. Her response to levodopa had been definite but modest and
physician providing complicated by dementia. She was on a complicated regimen of levodopa,
context for interventions quetiapine, midodrine, polyethylene glycol (PEG) 3350, and senna glycoside.
or the likelihood that an Her husband had recently passed away and her children were overwhelmed
intervention will help with her care. She was on a list for placement in a long-term care facility. Her
the patient meet their family asked if the patient could continue to see the neurologist for
care goals. Neurologists follow-up visits.
have the information Comment. This scenario is common. Although families may be willing
that can help patients and able to bring patients to appointments, the misconception often exists
make informed choices that visits to specialists must stop upon admission to long-term care facilities.
to complete their If the physician is willing, continuing to provide care for these patients offers
advance directives. the skilled nursing facilities or long-term care facilities with the opportunity to
h Advance directives learn about Parkinson disease and related disorders, the nuances of behavior
should be reviewed
changes, and how to appropriately manage medication changes. Patients
periodically. They are a
often speak of giving up their neurologist as the last of a long series of losses
living document that
for them and is one that they associate with abandonment and the time to
requires revision
give up hope.
according to the
patient’s condition Although some patients may fear related disorders are admitted to hos-
and experiences “death panels,” a study found that pital, and our experience of outcomes
with hospitalizations. patients with do not resuscitate orders from intensive care unit interven-
had better quality of life in the last tions is important to give perspec-
weeks of life than those requesting tive to patients and their families. An
intensive or invasive treatment.31 A advance directive or goals of care
study specifically for those with PD document should be viewed as a
found that patients consistently “living document” so that the conver-
overestimated the effectiveness of sation is reviewed periodically. Follow-
interventions such as cardiopulmo- ing recent hospitalizations, aspiration
nary resuscitation (CPR) and resusci- pneumonia or change in functional
tation and ventilation.32 If patients status are often good opportunities to
discussed advance directive planning reflect with the patient about their
with their physician, 72% chose pain goals of care.
and symptom management compared Beyond the role of helping patients
to 47% wishing CPR, 16% wishing with advance care planning, neurolo-
ventilator support, and 20% wanting gists can also play a role in their pal-
a feeding tube. Proxy decision makers liative care. Although palliative care is
often opted for more intensive in- often equated with imminent death,
terventions if the patient had not recent evidence demonstrates that
discussed wishes with them. The early palliative care in oncology pa-
authors of the study concluded that tients results in improved quality of
coordinating advance care planning life and prolonged survival.33 A trial of
with patients and families is more palliative care in patients with PD,
effective. Neurologists can and leaving referral open to those Hoehn
should play an active role in their and Yahr scale stage 3 or greater, with
patients’ decision making. Neurolo- caregiver strain or expressed desire to
gists are often the specialists called talk about advance directives, showed
upon when patients with PD and improved symptom burden using the

1110 www.ContinuumJournal.com August 2016

Copyright © American Academy of Neurology. Unauthorized reproduction of this article is prohibited.


KEY POINTS
Edmonton Symptom Assessment Sys- abdominal massage improved constipa- h Pain is often not
tem Revised: Parkinson Disease (ESAS- tion scores but not other outcome recognized in Parkinson
rPD) scale (Supplemental Digital measures.36 disease. Use of a pain
Content 4-1, links.lww.com/CONT/ Urinary dysfunction is common in scale and physical
A174).22 In addition to assessing care- PD. Possible etiologies include detrusor measures (range-of-
giver burden, neurologists can also hyperreflexia. Proposed therapies have motion exercises, heat
assess nonmotor symptoms of PD included the use of apomorphine and or cold compresses, and
and their severity. In the study of coincident improvement of urinary topical analgesics) can
palliative care for PD, symptoms function with deep brain stimulation be effective.
responding most were constipation, (DBS) of the subthalamic nucleus.34 h Constipation reduces
rigidity, confusion, choking, drowsi- Neither, however, had sufficient evi- the effectiveness
ness, and pain. The improvement in dence (Class IV studies only) to make a of dopaminergic
symptoms was similar to patients with recommendation for use. Prior to medications. Ensure
metastatic cancer. Thus, palliative medications, trying a bladder routine patients have a bowel
care approaches to PD can improve movement daily or, at
(going to the bathroom and attempt-
least, every 2 days.
patients’ symptom burden and should ing to void every 2 to 3 hours), may be
be available to them. The ESAS-rPD effective. Anticholinergic agents are h Orthostatic hypotension
provides information regarding severity commonly used to treat urinary fre- can limit treatment in
and therefore is more useful than a up to 30% of patients
quency; however, they are associated
with Parkinson disease
yes/no survey instrument to guide with confusion and delirium in this
and significantly higher
treatment and can be completed by population in addition to possibly in multiple system
the patient or caregiver. worsening constipation. A new treat- atrophy where
ment for urinary urgency and fre- dysautonomia is a
NONMOTOR SYMPTOM quency is mirabegron, a $3-adrenergic diagnostic criterion.
MANAGEMENT IN ADVANCED agonist. It does not cause delirium and
PARKINSON DISEASE has the possibly positive side effect of
Evidence is lacking for the treatment of hypertension. To date, no studies are
pain in PD. Most studies focus on reported for this agent in PD.
optimizing dopaminergic therapy. In Orthostatic hypotension can limit
patients with advanced PD, increasing treatment in up to 30% of patients
dopaminergic therapy may not be an with PD and significantly higher in
option. Therefore, addressing pain with multiple system atrophy where dysau-
range-of-motion exercises and topical tonomia is a diagnostic criterion.37
treatments or acetaminophen should Obtaining adequate standing systolic
be first-line therapy. If patients report blood pressure is often associated
pain on an ordinal scale or using the with supine hypertension, and thus
Faces Pain Scale of a score greater than 6, symptomatic control must be balanced
despite the previously mentioned with the risks. Orthostatic hypotension
conservative therapies, opioid medi- is defined as at least 20 mm Hg systolic
cations can be helpful. drop or 10 mm Hg diastolic blood
Constipation can respond to dietary pressure drop within 3 minutes of
changes, increased water intake, exer- standing. Longer periods of standing
cise, and polyethylene glycol (PEG) will have a higher yield and may more
3350 17 g diluted in 8 oz of liquid.34,35 accurately represent daily activities.
A Cochrane review found polyethyl- Orthostatic hypotension can lead to
ene glycolYbased (PGB) bisacodyl sup- falls, confusion, fatigue, and shoulder
pository significantly reduced the pain in addition to syncope. Absolute
mean defecation period (20 minutes hypotension is less than 90 mm Hg
versus 36 minutes for control), and systolic. Critical times for orthostatic
Continuum (Minneap Minn) 2016;22(4):1104–1116 www.ContinuumJournal.com 1111

Copyright © American Academy of Neurology. Unauthorized reproduction of this article is prohibited.


Advanced Parkinson Disease

KEY POINTS
h Orthostatic hypotension hypotension are: first thing in the 600 mg 3 times a day. Common side
can lead to falls, morning, following meals, following effects include changing color of the
confusion, fatigue, sitting or lying for prolonged periods, urine; difficult, burning, or painful uri-
and shoulder pain in 30 minutes to 1 hour after levodopa or nation; and urinary frequency.
addition to syncope. dopaminergic medication dose, higher If patients develop supine hyperten-
h Appropriate goals for ambient temperatures, and following a sion at bedtime, first elevate the head
blood pressure are at warm or hot bath or shower. Appropri- of the bed at 30 degrees (ie, 4 inches)
least 90 mm Hg systolic ate goals for blood pressure are at least at night. If unsuccessful, medication
while standing and less 90 mm Hg systolic while standing and options are losartan 50 mg in the
than 180 mm Hg less than 180 mm Hg systolic supine. evening, nifedipine 30 mg at night,
systolic supine. Physical methods for orthostatic hypo- clonidine 0.1 mg to 0.2 mg at night, or
tension treatment have not been tested hydralazine 25 mg in the evening.37
in PD or related disorders but include Aspiration and choking on food or
an abdominal binder (which is essen- liquids is a burden for patients and
tially compression of the abdomen to families, resulting in fear of meal times
increase venous return and reduce and weight loss. Detecting aspiration
splanchnic-mesenteric pooling and is is challenging. A study of 35 patients
easier than compression stockings), with PD found 40% had swallowing
water bolus (two 8-oz glasses of cold complaints and 20% of patients had
water in rapid succession will raise airway food penetration on video-
systolic blood pressure by 20 mm Hg fluoroscopy.38 Of those without swal-
for 1 to 2 hours), contracting bilateral lowing complaints, 22% had evidence
groups of muscles for 30 seconds of aspiration on videofluoroscopy.
(crossing their legs and squeezing, Respiratory parameters for patients
standing on their toes or squeezing were impaired compared to controls.
their buttocks), and avoiding lying flat. Behavioral steps to improve swal-
If these fail, medications may be used. lowing include: removing distractions
Midodrine is a selective agonist at during meals (do not watch television
peripheral !1 receptors with a duration while eating), not talking while eating,
of 2 to 4 hours. The maximum dose is putting the fork or spoon down be-
10 mg 3 times a day (often 1 hour prior tween bites to reduce rushing food,
to meals, and no later than 6:00 PM). and clearing the mouth with small
Midodrine tablets come in 2.5 mg, amounts of water between each mouth-
5 mg, and 10 mg dosages. Fludrocor- ful. Mincing or pureeing foods can help,
tisone expands plasma volume and in- and patients should avoid foods with
creases sensitivity of !-adrenoceptors. multiple consistencies (soup or stews
Typical dose is 0.1 mg to 0.2 mg daily, with large solids), and ensure the chin
usually given in the morning. Potassium and neck are in a neutral position. Swal-
should be monitored as hypokalemia lowing exercises have been studied in
and constipation are common with flu- PD.39 These exercises focused on in-
drocortisone. At higher doses, corti- creasing the strength and range of
costeroidlike complications may occur. motion of the mouth, larynx, and
The newest addition to the arma- pharyngeal structures and coordination
mentarium of orthostatic hypotension between breathing and swallowing.
treatment is droxidopa, an oral norepi- Exercises were to be performed twice
nephrine precursor. Starting droxidopa a day, 5 days a week. Fifteen subjects
dose is 100 mg 3 times a day (avoiding demonstrated improved bolus control
3 hours prior to bedtime to reduce and swallow as well as reduced residue
supine hypertension) to a maximum of on structures following a swallow.
1112 www.ContinuumJournal.com August 2016

Copyright © American Academy of Neurology. Unauthorized reproduction of this article is prohibited.


KEY POINT
Patients reported reduction in fear require more support from the hospice h Hospice care is available
with eating or drinking and reduced team, lasting 5 to 7 days. Respite care to patients with
symptom frequency. If dysphagia is allows caregivers to have a break from Parkinson disease and
marked and cognition is relatively caregiving. Respite care may last for related disorders. Use
intact, enteral feeding is an option. A 5 days every benefit period and will the hospice guidelines
Cochrane review found no evidence, be delivered in a skilled nursing facility, to refer patients for
however, that enteral feeding improves assistive living, hospital, or inpatient in-home care, inpatient
quality of life, prolongs life, or is hospice unit. Referral to palliative care care, or respite care.
superior to hand feeding using safe services when symptom burden is in-
techniques.40 When enteral feeding is creasing beyond the expertise of a neu-
initiated, it is always wise to discuss the rologist is another option. Neurologists
possibility of discontinuing feeding in should be aware that many palliative
the future. care physicians are uncomfortable with
Telemedicine is increasing the op- neurodegenerative disease and there-
tions for patients to remain connected fore may not accept these referrals.
with their specialists. There are many
possible models of care at this time. CONCLUSION
Patients may continue with their neu- Advanced PD and related disorders are
rologist in conjunction with a palliative challenging and, to some, dishearten-
care specialist or admission to hospice ing. Neurologists can play a valuable
if they meet criteria. As found in the role until the end of life. Neurologists
large Medicare cohort, those patients have the knowledge and clinical expe-
with PD requiring long-term care were rience that can shine a light for those
two times more likely to receive less familiar with neurodegenerative
hospice care if they continued to see illnesses, thereby advocating for ac-
their neurologist. The criteria for hos- cess to resources needed to support
pice admission for neurologic diagno- our patients and their families.
ses and generic criteria are seen in The MoCA should be administered
Table 4-4.23 Of note, PD has overtaken once per year to detect cognitive de-
many cancers as a cause of death of cline and dementia. Once dementia is
hospice enrollees. Therefore, US hos- diagnosed, the MoCA no longer needs
pices are becoming more familiar to be administered annually. The Zarit
with this patient population. Hospice Burden Interview, including questions
can provide physician and nurse visits about physical and sexual aggression,
in the home, in-home assistance, and should be administered once a year to
care until the time of death. Levels of caregivers to detect caregiver burden
care in hospice are routine home care, and identify measures that can mitigate
continuous care, general inpatient care, strain. For those with advanced PD or
and respite care.41 Routine home care related disorders, the ESAS-rPD can
provides multidisciplinary home care, identify symptoms and track response
necessary supplies, and nurse visits. to interventions. It is the only scale for
Continuous care is for patients with advanced PD that has demonstrated
severe symptoms needing temporary these important characteristics. Ortho-
extra support. Care can be provided for static hypotension should be treated
8 hours a day in this situation and is for with reducing/discontinuing antihyper-
short periods. General inpatient care is tensive medications and then admin-
provided in a hospice bed or a unit in a istering midodrine or droxidopa if
hospital. This is typically reserved for physical maneuvers are not effective.
patients who are imminently dying and Constipation should be first treated
Continuum (Minneap Minn) 2016;22(4):1104–1116 www.ContinuumJournal.com 1113

Copyright © American Academy of Neurology. Unauthorized reproduction of this article is prohibited.


Advanced Parkinson Disease

TABLE 4-4 Hospice Guidelines for Neurologic Disorders

Disorder Guideline
Dementia Stage 7C or higher on the Functional Assessment Staging Test (FAST) scale
AND
One or more of the following in the past year: aspiration pneumonia,
pyelonephritis, septicemia, stage 3 or 4 pressure ulcers, recurrent fevers, other
conditions suggesting limited prognosis, or inability to maintain sufficient fluid/
caloric intake in past 6 months (10% weight loss or serum albumin G2.5 g/dL)
Stroke or coma Palliative Performance Scale score e40%
AND
Poor nutritional status with inability to maintain sufficient fluid/caloric
intake (10% weight loss in 6 months, 7.5% weight loss in 3 months,
serum albumin e2.5 g/dL, or pulmonary aspiration resistant to speech
therapy interventions)
Other neurologic disease Critically impaired breathing including dyspnea at rest, vital capacity
including amyotrophic G30%, oxygen need at rest, AND refusal of artificial ventilation
lateral sclerosis, Parkinson
OR
disease, muscular
dystrophy, myasthenia gravis, Rapid disease progression (to bed-bound status, unintelligible speech,
or multiple sclerosis need for pureed diet, and/or major assistance needed for activities of daily
living) with either:
Critical nutrition impairment in the prior year (inability to maintain
sufficient fluid/caloric intake, continuing weight loss, dehydration, AND
refusal of artificial feeding methods)
OR
Life-threatening complications in the prior year (recurrent aspiration
pneumonia, pyelonephritis, sepsis, recurrent fever, OR stage 3 or 4
pressure ulcers)
Generic criteria Terminal condition (can be multiple conditions)
AND
Rapid decline over past 3 to 6 months as evidenced by progression of
disease signs, symptoms, and test results; decline in Palliative Performance Scale
score e40%, and involuntary weight loss 910%, and/or serum albumin G2.5 g/dL

a
Reprinted with permission from Boersma I, et al, Neurology.23 B 2014 American Academy of Neurology. www.neurology.org/content/
83/6/561.full.

with diet and fluid intake with or tatic hypertrophy, the use of mir-
without abdominal massage. If neces- abegron can be effective without
sary, the use of PEG 3350 or PGB lowering blood pressure or causing
bisacodyl have been shown to be effec- delirium. Dysphagia with silent aspira-
tive. Urinary dysfunction is complex in tion is common in PD. Videofluoroscopy
PD. Urodynamics may be helpful to remains the gold standard for diagno-
determine which interventions will be sis of aspiration. Beyond dietary mo-
most helpful. For those with urinary dification and effective hand feeding,
frequency and urgency without pros- there are no effective treatments for

1114 www.ContinuumJournal.com August 2016

Copyright © American Academy of Neurology. Unauthorized reproduction of this article is prohibited.


KEY POINT
dysphagia in PD. There is no evi- 20 years. Mov Disord 2008;23(6):837Y844.
doi:10.1002/mds.21956. h Aspiration is
dence that percutaneous endo- underestimated by
scopic gastrostomy feeding in PD 8. Auyeung M, Tsoi TH, Mok V, et al. Ten year
survival and outcomes in a prospective cohort bedside testing. The
improves mortality or improves quality of new onset Chinese Parkinson’s disease gold standard remains
of life. In selected cases, percuta- patients. J Neurol Neurosurg Psychiatry 2012; videofluoroscopic
neous endoscopic gastrostomy for nu- 83(6):607Y611. doi:10.1136/jnnp-2011-301590. swallow study. Changes
trition or medication administration 9. Forsaa EB, Larsen JP, Wentzel-Larsen T, et al. to diet andmodifying
A 12-year population-based study of the meal routine
may be effective; however, discussing psychosis in Parkinson disease. Arch Neurol
the possibility of discontinuing en- can improves
2010;67(8):996Y1001. doi:10.1001/
teral feeding may be helpful prior to archneurol.2010.166. swallowing safety.
placing the percutaneous endoscopic 10. Haahr A, Kirkevold M, Hall EO, Ostergaard
gastrostomy tube. K. Living with advanced Parkinson’s
disease: a constant struggle with
When we can fulfill this role for our unpredictability. J Adv Nurs 2011;
patients, we are doing what drew us to 67(2):408Y417. doi:10.1111/
medicine: relieving suffering and car- j.1365-2648.2010.05459.x.
ing for our patients. 11. McLaughlin D, Hasson F, Kernohan WG,
et al. Living and coping with Parkinson’s
REFERENCES disease: perceptions of informal carers.
Palliat Med 2011;25(2):177Y182. doi:10.1177/
1. Weerkamp NJ, Tissingh G, Poels PJ, et al.
0269216310385604.
Nonmotor symptoms in nursing home
residents with Parkinson’s disease: prevalence 12. Vossius C, Larsen JP, Janvin C, Aarsland D.
and effect on quality of life. J Am Geriatr Soc The economic impact of cognitive
2013;61(10):1714Y1721. doi:10.1111/jgs.12458. impairment in Parkinson’s disease. Mov Disord
2011;26(8):1541Y1544. doi:10.1002/
2. Ganga G, Alty JE, Clissold BG, et al.
mds.23661.
Longitudinal study of levodopa in Parkinson’s
disease: effects of the advanced disease 13. Leiknes I, Tysnes OB, Aarsland D, Larsen JP.
phase. Mov Disord 2013;28(4):476Y481. Caregiver distress associated with
doi:10.1002/mds.25335. neuropsychiatric problems in patients with
early Parkinson’s disease: the Norwegian
3. Hassan A, Wu SS, Schmidt P, et al. What are
ParkWest study. Acta Neurol Scand
the issues facing Parkinson’s disease patients
2010;122(6):418Y424. doi:10.1111/
at ten years of disease and beyond? Data
j.1600-0404.2010.01332.x.
from the NPF-QII study. Parkinsonism Relat
Disord 2012;18(suppl 3):S10YS14. 14. Safarpour D, Thibault DP, DeSanto CL, et al.
doi:10.1016/j.parkreldis.2012.06.014. Nursing home and end-of-life care in
Parkinson disease. Neurology 2015;85(5):
4. Hassan A, Wu SS, Schmidt P, et al.
413Y419. doi:10.1212/WNL.0000000000001715.
The profile of long-term Parkinson’s disease
survivors with 20 years of disease duration 15. Williamson C, Simpson J, Murray CD.
and beyond. J Parkinsons Dis 2015;5(2): Caregivers’ experiences of caring for a husband
313Y319. doi:10.3233/JPD-140515. with Parkinson’s disease and psychotic
5. Goetz CG, Poewe W, Rascol O, et al; symptoms. Soc Sci Med 2008;67(4):583Y589.
Movement Disorder Society Task Force on doi:10.1016/j.socscimed.2008.04.014.
Rating Scales for Parkinson’s Disease. 16. Habermann B, Hines D, Davis L. Caring for
Movement Disorder Society Task Force parents with neurodegenerative disease:
report on the Hoehn and Yahr staging scale: a qualitative description. Clin Nurse Spec
status and recommendations. Mov Disord 2013;27(4):182Y187. doi:10.1097/
2004;19(9):1022Y1028. NUR.0b013e318295576b.
6. Reid WG, Hely MA, Morris JG, et al. 17. Giles S, Miyasaki J. Palliative stage Parkinson’s
Dementia in Parkinson’s disease: a 20-year disease: patient and family experiences of
neuropsychological study (Sydney Multicentre health-care services. Palliat Med 2009;23(2):
Study). J Neurol Neurosurg Psychiatry 120Y125. doi:10.1177/0269216308100773.
2011;82(9):1033Y1037. doi:10.1136/
jnnp.2010.232678. 18. Liao YC, Wu YR, Tsao LI, Lin HR. The
experiences of Taiwanese older individuals
7. Hely MA, Reid WG, Adena MA, et al. at different stages of Parkinson disease.
The Sydney multicenter study of Parkinson’s J Neurosci Nurs 2013;45(6):370Y377.
disease: the inevitability of dementia at doi:10.1097/JNN.0b013e3182a3cd5c.

Continuum (Minneap Minn) 2016;22(4):1104–1116 www.ContinuumJournal.com 1115

Copyright © American Academy of Neurology. Unauthorized reproduction of this article is prohibited.


Advanced Parkinson Disease

19. O’Rourke N, Tuokko HA. Psychometric technology. Am J Hosp Palliat Care 2009;26(4):
properties of an abridged version of The 270Y276. doi:10.1177/1049909109331886.
Zarit Burden Interview within a representative
31. Garrido MM, Balboni TA, Maciejewski PK,
Canadian caregiver sample. Gerontologist 2003;
et al. Quality of life and cost of care at the
43(1):121Y127. doi:10.1093/geront/43.1.121.
end of life: the role of advance directives.
20. Miyasaki JM, Kluger B. Palliative care for J Pain Symptom Manage 2015;49(5):828Y835.
Parkinson’s disease: has the time come? doi:10.1016/j.jpainsymman.2014.09.015.
Curr Neurol Neurosci Rep 2015;15(5):26.
32. Kwak J, Wallendal MS, Fritsch T, et al.
doi:10.1007/s11910-015-0542-4.
Advance care planning and proxy decision
21. Goldman JG. Neuropsychiatric issues in making for patients with advanced Parkinson
Parkinson disease. Continuum (Minneap disease. South Med J 2014;107(3):178Y185.
Minn) 2016;22(4 Movement doi:10.1097/SMJ.0000000000000075.
Disorders):1086Y1103.
33. Zimmermann C, Swami N, Krzyzanowska M,
22. Miyasaki JM, Long J, Mancini D, et al. et al. Early palliative care for patients with
Palliative care for advanced Parkinson advanced cancer: a cluster-randomised
disease: an interdisciplinary clinic and new controlled trial. Lancet 2014;383(9930):
scale, the ESAS-PD. Parkinsonism Relat 1721Y1730. doi:10.1016/S0140-6736(13)62416-2.
Disord 2012;18(suppl 3):S6YS9. doi:10.1016/
34. Zesiewicz TA, Sullivan KL, Arnulf I, et al.
j.parkreldis.2012.06.013.
Practice parameter: treatment of nonmotor
23. Boersma I, Miyasaki J, Kutner J, Kluger B. symptoms of Parkinson disease: report of
Palliative care and neurology: time for a the Quality Standards Subcommittee of the
paradigm shift. Neurology 2014;83(6): American Academy of Neurology. Neurology
561Y567. doi:10.1212/WNL.0000000000000674. 2010;74(11):924Y931. doi:10.1212/
WNL.0b013e3181d55f24.
24. Walker RW, Churm D, Dewhurst F, et al.
Palliative care in people with idiopathic 35. Zangaglia R, Martignoni E, Glorioso M, et al.
Parkinson’s disease who die in hospital. Macrogol for the treatment of constipation
BMJ Support Palliat Care 2014;4(1):64Y67. in Parkinson’s disease. A randomized
doi:10.1136/bmjspcare-2012-000412. placebo-controlled study. Mov Disord 2007;
22(9):1239Y1244. doi:10.1002/mds.21243.
25. Sleeman KE, Ho YK, Verne J, et al. Place
of death, and its relation with underlying 36. Coggrave M, Norton C, Cody JD.
cause of death, in Parkinson’s disease, Management of faecal incontinence and
motor neurone disease, and multiple sclerosis: constipation in adults with central
a population- based study. Palliat Med 2013; neurological diseases. Cochrane Database
27(9):840Y846. doi:10.1177/0269216313490436. Syst Rev 2014;1:CD002115. doi:10.1002/
14651858.CD002115.pub5.
26. Alano GJ, Pekmezaris R, Tai JY, et al. Factors
influencing older adults to complete advance 37. Low PA, Tomalia VA. Orthostatic hypotension:
directives. Palliat Support Care 2010;8(3): mechanisms, causes, management. J Clin
267Y275. doi:10.1017/S1478951510000064. Neurol 2015;11(3):220Y226. doi:10.3988/
jcn.2015.11.3.220.
27. Anumobi E, Detweiler MB, Sethi R, et al.
Comparison of advance medical directive 38. Monteiro L, Souza-Machado A, Pinho P,
inquiry and documentation for hospital et al. Swallowing impairment and pulmonary
inpatients in three medical services: dysfunction in Parkinson’s disease: the silent
implications for policy changes. J Aging Soc threats. J Neurol Sci 2014;339(1Y2):149Y152.
Policy 2015;27(2):156Y172. doi:10.1080/ doi:10.1016/j.jns.2014.02.004.
08959420.2014.983356.
39. Argolo N, Sampaio M, Pinho P, et al.
28. Cheang F, Finnegan T, Stewart C, et al. Do swallowing exercises improve
Single-centre cross-sectional analysis of swallowing dynamic and quality of life in
advance care planning among elderly Parkinson’s disease? NeuroRehabilitation
inpatients. Intern Med J 2014;44(10): 2013;32(4):949Y955. doi:10.3233/NRE-130918.
967Y974. doi:10.1111/imj.12550.
40. Sampson EL, Candy B, Jones L. Enteral tube
29. Burkle CM, Mueller PS, Swetz KM, et al. feeding for older people with advanced
Physician perspectives and compliance with dementia. Cochrane Database Syst Rev
patient advance directives: the role external 2009;15(2):CD007209. doi:10.1002/
factors play on physician decision making. 14651858.CD007209.pub2.
BMC Med Ethics 2012;13:31. doi:10.1186/
41. Forman WB, Kopchak Sheehan D, Kitzes JA
1472-6939-13-31.
editors. Hospice and palliative care: concepts
30. Clements JM. Patient perceptions on the use and practice. 2nd ed. Sudbury, MA: Jones
of advance directives and life prolonging & Bartlett Publishers, 2003.

1116 www.ContinuumJournal.com August 2016

Copyright © American Academy of Neurology. Unauthorized reproduction of this article is prohibited.


Review Article

Diagnostic Approach to
Address correspondence to
Dr Nikolaus R. McFarland, 1149
South Newell Dr, L3-100,
PO Box 100236, Gainesville,

Atypical Parkinsonian FL 32610, nikolaus.mcfarland@


neurology. ufl.edu.
Relationship Disclosure:

Syndromes Dr McFarland is supported by


a career development grant
from the National Institutes of
Health/National Institute of
Nikolaus R. McFarland, MD, PhD Neurological Disorders and
Stroke (K09 NS067024) and
the Michael J. Fox Foundation.
ABSTRACT Unlabeled Use of
Products/Investigational
Purpose of Review: Although increasingly recognized, atypical parkinsonian syn- Use Disclosure:
dromes remain challenging to diagnose and are underrecognized due to overlap with Dr McFarland reports
no disclosure.
other parkinsonisms. This article provides a diagnostic approach to atypical parkinso-
* 2016 American Academy
nian syndromes, including progressive supranuclear palsy (PSP), multiple system atrophy of Neurology.
(MSA), corticobasal degeneration (CBD), and dementia with Lewy bodies. The goal of
this review is to aid the clinician in recognizing key clinical and pathologic features and
to raise awareness of recent advances in diagnostics and treatment.
Recent Findings: Diagnostic criteria for atypical parkinsonian syndromes are evolving
to encompass increasingly recognized heterogeneity in the presentation of these dis-
orders and information gleamed from clinicopathologic correlations. PSP and CBD in
particular now share similar pathologic clinical features and include a number of
phenotypic variants. Pathologic diagnoses are increasingly used in clinical practice, and
there is frequent reference now by clinicians to tauopathies, including PSP and CBD,
and the synucleinopathies, which include MSA and dementia with Lewy bodies (as well
as Parkinson disease). Research into biomarkers, including both tissue and imaging
modalities and genetics, has the potential to increase disease recognition and make
earlier diagnosis and treatment possible. Although novel therapeutics are being
studied for atypical parkinsonian syndromes such as PSP, no new breakthrough inter-
ventions have emerged for the treatment of PSP, CBD, and MSA. Current therapeutic
management for these disorders frequently uses a multidisciplinary team approach.
Summary: The approach to atypical parkinsonian syndromes requires recognition of
a constellation of overlapping but distinct clinical features that help with identifying
and distinguishing them from Parkinson disease and other similar disorders.

(Minneap Minn) 2016;22(4):1117–1142.

INTRODUCTION such as early dementia, frequent falls,


Parkinsonism is defined as a hypo- ocular dysmotility, prominent dysau-
kinetic syndrome and is characterized tonomia, or ataxia. These syndromes
by the presence of resting tremor, mus- typically involve multisystem degen-
cular rigidity, bradykinesia or akinesia, eration and are referred to as atypical
and postural instability. While many parkinsonian syndromes. They com-
secondary or acquired causes of parkin- monly include progressive supra- Supplemental digital content:
Videos accompanying this ar-
sonism exist, the most common pri- nuclear palsy (PSP), multiple system ticle are cited in the text as
mary or neurodegenerative cause of atrophy (MSA), corticobasal degener- Supplemental Digital Content.
Videos may be accessed by
parkinsonism is Parkinson disease ation (CBD), and dementia with Lewy clicking on links provided in the
(PD). A smaller but significant number bodies (DLB), as well as other rarer HTML, PDF, and app versions
of this article; the URLs are
of patients present with a parkinsonian causes. It is critical to distinguish these provided in the print version.
syndrome that has atypical features disorders from classic PD as disease Video legends begin on page 1139.

(Minneap Minn) 2016;22(4):1117–1142 www.ContinuumJournal.com 1117

Copyright © American Academy of Neurology. Unauthorized reproduction of this article is prohibited.


Atypical Parkinsonian Syndromes

progression and subsequent functional nian syndromes with PD, secondary


decline is often more rapid than in PD. parkinsonisms, and heredodegen-
In these syndromes, treatment with erative disorders makes clinical diag-
standard PD therapies frequently lacks nosis challenging (Figure 5-1). These
efficacy and is fraught with complica- issues may lead to underrecognition,
tions. Patients often have complex care delay in diagnosis, and even misdiag-
needs that necessitate a multidis- nosis. Table 5-11 includes a list of pri-
ciplinary approach. This Continuum mary causes of atypical parkinsonisms.
article focuses on the diagnostic ap- Despite the diversity of these condi-
proach to atypical parkinsonian syn- tions, there has been an evolution of
dromes and aims to help the clinician our understanding of the pathophysi-
recognize key clinical and pathologic ology of these disorders. Increasingly,
features as well as recent advances in clinicopathologic terms are being
diagnostics and treatment. used to describe atypical parkinsonian
The considerable overlap of signs syndromes in the clinic because of the
and symptoms for atypical parkinso- diagnostic uncertainty and overlap of

FIGURE 5-1 Overlap of parkinsonian syndromes. Atypical parkinsonisms have common


features with Parkinson disease, secondary parkinsonisms, and
heredodegenerative disorders with parkinsonism.

1118 www.ContinuumJournal.com August 2016

Copyright © American Academy of Neurology. Unauthorized reproduction of this article is prohibited.


a
TABLE 5-1 Primary Causes of Atypical Parkinsonism

b Multisystem Disease
Progressive supranuclear palsy
Corticobasal syndrome
Multiple system atrophy
Dementia with Lewy bodies
ParkinsonismYdementiaYamyotrophic lateral sclerosis
b Heredodegenerative Disorders
Huntington disease
Spinocerebellar ataxias (especially types 2, 3, and 17)
Wilson disease
Hereditary ceruloplasmin deficiency
Neuronal brain iron accumulation disorders (eg, PKAN2)
X-linked dystonia-parkinsonism (Lubag disease)
Gerstmann-Sträussler-Scheinker syndrome
Neuronal ceroid lipofuscinoses
Mitochondrial cytopathies
a
Modified with permission from Jankovic J, Lang AE, Saunders.1 B 2008 Saunders, an imprint
of Elsevier.

symptoms (Figure 5-2). One common- guish atypical parkinsonian syndromes


ality among neurodegenerative disor- from PD.2 These include rapid disease
ders is the presence of abnormal progression, early gait instability and
proteinaceous deposits in pathologic falls, absence or paucity of tremor, auto-
brain tissue that have been linked to nomic failure, and poor or absent re-
disease mechanisms, giving rise to the sponse to levodopa, including pain/
term neuroproteinopathy. Table 5-2 in- dysesthesia. Additional features may
cludes examples of various proteinop- include oculomotor abnormalities, pyr-
athies and diseases linked to proteins amidal tract or cerebellar signs (ataxia),
that accumulate in intracellular inclusions prominent dysautonomia, severe dysar-
or extracellular plaques (eg, !-synuclein, thria or dysphonia, laryngeal stridor,
ubiquitin, tau, and "-amyloid). myoclonus, alien limb, apraxia, and
Despite recent research and diag- early dementia (Table 5-3). This article
nostic advances, the diagnosis of atyp- focuses on some of the most common
ical parkinsonian syndromes still relies atypical parkinsonian syndromes and
primarily on clinical evaluation. Ancil- their identifying features.
lary tests such as brain imaging can be
supportive, but there are no identified, PROGRESSIVE SUPRANUCLEAR
reliable, and specific biomarkers that PALSY
have been established as diagnostic. PSP was described over 50 years ago by
However, certain features or ‘‘red flags’’ Drs Steele, Richardson, and Olszewski.3
have been identified that help distin- It is the most common form of atypical

(Minneap Minn) 2016;22(4):1117–1142 www.ContinuumJournal.com 1119

Copyright © American Academy of Neurology. Unauthorized reproduction of this article is prohibited.


Atypical Parkinsonian Syndromes

KEY POINT
h Key features of
progressive supranuclear
palsy include early gait
instability, unexplained
falls, supranuclear gaze
palsy, axial rigidity,
dysarthric speech,
and dementia.

FIGURE 5-2 Clinicopathologic overlap of neurodegenerative proteinopathies. Atypical


parkinsonian syndromes share abnormal accumulation of proteins such as
!-synuclein, tau, amyloid, and TDP-43.
ALS = amyotrophic lateral sclerosis; FTD = frontotemporal dementia; FTLD-U = frontotemporal
lobar degeneration with ubiquitin; MND = motor neuron disease; TDP-43 = TAR DNA binding
protein 43.

parkinsonism, comprising about 5% to Gait instability and early falls are key
6% of those patients presenting with features of PSP and distinguish it from
parkinsonism. The estimated preva- other parkinsonian syndromes. Relative
lence and annual incidence of PSP is to other parkinsonian syndromes, falls
about 5 per 100,000 in individuals be- occur early, within the first year or two,
tween the ages of 50 and 99 years, but and often lead to significant injury and
is likely higher due to misdiagnosis and fractures.4 Gait in PSP is characteristically
underrecognition. The average age of stiff, broad based, with knees extended
onset is typically in the sixties (average and arms abducted. It is often described
age of 63 to 66 years), and the mean as clumsy like a ‘‘drunken sailor’’ or
survival from diagnosis is reported be- ‘‘dancing bear,’’ and includes large
tween 5 to 8 years. Hallmarks of the dis- lateral deviations and step asymmetry.
ease include prominent, early postural When turning, persons with PSP tend
instability, unexplained falls, vertical supra- to pivot rather than turn en bloc as is
nuclear palsy, and progressive dementia. more typical in PD. The cause of falls is

1120 www.ContinuumJournal.com August 2016

Copyright © American Academy of Neurology. Unauthorized reproduction of this article is prohibited.


KEY POINT

TABLE 5-2 Classification of Atypical Parkinsonism h The rocket sign occurs in


as Proteinopathies patients with progressive
supranuclear palsy who
Proteinopathy Disorder have lost insight into their
postural instability and
Amyloidoses Alzheimer disease
‘‘rocket’’ out of their
Ubiquitin-proteasome disorders Parkinson disease, parkin mutation chair without assistance,
Synucleinopathies Parkinson disease, dementia with Lewy bodies, resulting in a high risk
multiple system atrophy for falling.

Tauopathies Progressive supranuclear palsy, corticobasal


degeneration, Alzheimer disease,
frontotemporal dementia (with parkinsonism)
Polyglutamine expansion Huntington disease, spinocerebellar ataxias
Prion disease Creutzfeldt-Jakob disease, Gerstmann-Sträussler-
Scheinker syndrome

often multifactorial and includes axial unchecked sitting (falling into their
rigidity, bradykinesia, loss of postural chair) are also common.
reflexes, freezing, a visual-vestibular com- A key feature of PSP includes inabil-
ponent, and decreased insight. ‘‘Rocket ity to perform volitional saccades and
sign’’ occurs in patients with PSP who progressive supranuclear ophthalmo-
have lost insight into their postural paresis. Although limitation of upgaze
instability and ‘‘rocket’’ out of their is often described as a sign of PSP, it is
chair without assistance, resulting in a nonspecific and can be seen in other
high risk for falling. Retropulsion and neurodegenerative disorders as well

TABLE 5-3 Red Flags for Differentiating Atypical Parkinsonism From


Parkinson Disease

b Features Predictive of Atypical Parkinsonism


Rapid disease progression
Early gait instability, falls
Absence or paucity of tremor
Irregular jerky tremor, myoclonus
Poor/absent response to levodopa
b Additional Features of Atypical Parkinsonism (and Associated Disorder)
Abnormal eye movements (progressive supranuclear palsy)
Pyramidal tract/cerebellar signs (multiple system atrophy)
Dysautonomia (multiple system atrophy)
Severe dysarthria, dysphonia, or stridor (multiple system atrophy)
Apraxia, alien limb, myoclonus (corticobasal syndrome)
Early, prominent dementia (dementia with Lewy bodies, progressive
supranuclear palsy/corticobasal syndrome)

(Minneap Minn) 2016;22(4):1117–1142 www.ContinuumJournal.com 1121

Copyright © American Academy of Neurology. Unauthorized reproduction of this article is prohibited.


Atypical Parkinsonian Syndromes

KEY POINTS
h Early signs of as in aging. Limitation of downgaze is thria that is often spastic or hypernasal,
supranuclear gaze most sensitive, and the syndrome hypokinetic, and monotonous. Speech
palsy in patients frequently progresses to include can be slow and can include stuttering,
with progressive upgaze and lateral gaze palsies. echolalia, and occasional involuntary
supranuclear palsy Slowed saccades and reduced optoki- vocalizations. An apraxia of phonation
include slowed vertical netic nystagmusVvertical more af- has also been reported. Most concern-
saccades and reduced fected than horizontalVare also ing, however, is progressive dysphagia
optokinetic nystagmus. frequently observed and are early pre- that can lead to aspiration, pneumonia,
Square-wave jerks, or dictors of progression to supranuclear and early death. As with other parkin-
minute saccadic eye gaze palsy. Another common finding is sonisms, mood disturbance is fre-
movements, may also
square-wave jerks, characterized by quently present and characterized by
be present, representing
rapid back and forth mini saccadic apathy, depression, or both.6 Disinhi-
fixation instability.
intrusions during fixation. Complete bition, dysphoria, anxiety, and irritabil-
h To assess for the gaze palsy and involuntary ocular ity are also possible. Emotional lability
applause sign, a clinician
fixation (as well as lack of eye blink) (also called emotional incontinence),
can demonstrate three
contribute to the classic ‘‘Mona Lisa’’ referred to as pseudobulbar affect, also
claps to the patient and
ask him or her to copy.
stare or stone face. Ability to overcome can occur and cause significant distress
The applause sign is gaze limitation with vestibular or for both the patient and caregivers.
present if the patient cervical-ocular reflex maneuvers de- PSP has the highest rate of pseudo-
claps more than fines supranuclear palsy, but can be bulbar laughing or crying across all
three times and difficult to assess in patients with PSP parkinsonian syndromes.7 Cognitive
continues (perseverates). who have significant axial rigidity. Visual decline and dementia occur with dis-
concerns are also common. Blurred ease progression. A frontal subcortical
vision occurs due to a combination of dementia is typical, with slowed pro-
factors including decreased blink and cessing, or bradyphrenia, reduced
tear production, drying of the cornea, verbal fluency, and executive dysfunc-
and ocular irritation. Diplopia is com- tion. In PSP there is often perseveration
mon not only in PSP but also in other of automatic behaviors, as exemplified
parkinsonisms and is most frequently by the three-clap test or applause sign.
due to convergence insufficiency. Read- (The clinician should demonstrate to
ing is affected and may be helped by the patient three claps and ask him or
prisms in select cases. Despite visual her to copy. The applause sign is pres-
aids, patients often continue to expe- ent if the patient claps more than three
rience blurred vision and difficulty scan- times and continues [perseverates]).
ning text due to progressive gaze palsy. Grasping, imitative behaviors may also
In addition, patients may also experience be present.
photosensitivity (wearing sunglasses
inside), decreased eye blink, blepharo- Diagnosis, Heterogeneity,
spasm, and eye-opening apraxia, leading and Progressive Supranuclear
to eyebrow furrowing (procerus con- Palsy Variants
traction), and vertical wrinkling of the In 1996 the National Institute of Neuro-
forehead, referred to as procerus sign.5 logical Disorders and Stroke (NINDS)
Additional symptoms include a char- and the Society for PSP (SPSP) inter-
acteristic facial appearance from the national workshop proposed criteria
rigid bradykinesia and dystonia in facial for the diagnosis of classic PSP
musculature. The hypertonic facial mus- (Richardson syndrome).8 The criteria
cles produce facial folds and a wor- for possible or probable PSP included
ried, astonished expression. Bulbar a progressive disorder with onset after
features include a progressive dysar- the age of 40 with postural instability,
1122 www.ContinuumJournal.com August 2016

Copyright © American Academy of Neurology. Unauthorized reproduction of this article is prohibited.


significant falls, slowing of vertical sac- phenotypic variants have recently
cades, or vertical gaze palsy. Definite been described: PSP-parkinsonism,
PSP added the requirement of patho- PSPYpure akinesia with gait freezing,
logic evidence. Supportive findings in- PSPYcorticobasal syndrome (PSP-CBS)
cluded symmetric rigidity, diminished (or primary nonfluent aphasia), PSPY
response to levodopa, and early cogni- behavioral variant of frontotemporal
tive impairment. Factors excluding the dementia (FTD), and two other pos-
diagnosis of PSP were encephalitis, sible PSP variants with features that over-
focal brain lesion, hallucinations, dysau- lap with either primary lateral sclerosis
tonomia, and alien limb syndrome. (PLS) or cerebellar ataxia. Table 5-4 lists
Cerebellar features were also previ- the clinical and pathologic features that
ously included as exclusionary, but a best differentiate these PSP variants.
recent description of a cerebellar var- Although each PSP variant may pres-
iant of PSP has called this exclusion ent differently, they are united by later
into question.9 onset of the more typical PSP features
The diagnosis of PSP is further com- of postural instability, significant falls,
plicated by its heterogeneous presenta- and supranuclear gaze palsy. Addi-
tion, resulting in increasing recognition tionally, although the distribution of
of clinical variants or phenotypes.10 In tau pathology may differ, each variant
classic presentations, the diagnosis shares pathognomonic PSP-related tau
of Richardson syndrome is relatively pathology. The definition of PSP is con-
straightforward. However, at least five tinually undergoing revision, and new

TABLE 5-4 Progressive Supranuclear Palsy Syndromes

Progressive Supranuclear
Palsy (PSP) Syndrome Clinical Features Regional Pathology
Classic PSP Early gait instability, falls, supranuclear Dentate nucleus, globus pallidus,
(Richardson syndrome) gaze palsy, axial rigidity, dysarthria, striatum, midbrain, and superior
dysphagia, progressive dementia cerebellar peduncle
PSP-parkinsonism Tremor, rigid bradykinesia, levodopa Substantia nigra, subthalamic nucleus
responsive,a late cognitive decline,
longer life expectancy (9 years)
PSPYpure akinesia with Early gait difficulty, freezing of Motor cortex, pons, cerebellum
gait freezing gait/motor block, micrographia, speech
impairment, hypophonia, longer
disease duration (11Y15 years)
PSPYcorticobasal syndrome Dystonia, dyspraxia, cortical sensory Frontal and parietal cortex
loss, apraxia of speech
PSPYbehavioral variant of Predominant cognitive, personality Frontotemporal cortex
frontotemporal dementia change, late parkinsonism
PSPYprimary lateral sclerosis Bulbar, limb weakness, upper motor Frontal predominant, corticospinal tract
neuron signs/spasticity
PSP-cerebellar Cerebellar ataxia Deep cerebellar nuclei
a
Levodopa response for PSP-parkinsonism wanes later in disease.

(Minneap Minn) 2016;22(4):1117–1142 www.ContinuumJournal.com 1123

Copyright © American Academy of Neurology. Unauthorized reproduction of this article is prohibited.


Atypical Parkinsonian Syndromes

criteria are likely in the near future. dominance types and atypical PSP var-
In 2014, because of the overlap in fea- iants is ongoing.
tures, Respondek and colleagues10 Progressive supranuclear palsyY
proposed a more comprehensive parkinsonism. PSP-parkinsonism is
approach to describing the pheno- the most common variant of PSP with
typic spectrum of PSP based on the features of tremor, early asymmetric
predominant clinical feature seen bradykinesia, and axial rigidity that
within the first 2 years of presenta- mimic PD.11 As illustrated in Case 5-1,
tion. Predominance types were de- a striking feature is relatively normal
fined including classic Richardson eye movements early on, although
syndrome, postural instability predom- slowed saccades and reduced optoki-
inant, oculomotor predominant, Par- netic nystagmus may be present, sug-
kinsonism, CBS, FTD, and those gesting atypical disease. Patients are
unclassified. Validation of these pre- also frequently levodopa responsive,

Case 5-1
A 65-year-old man presented with a 5-year history of reported Parkinson
disease. He was diagnosed 2 years ago, but had noticed onset of a mild
right hand resting tremor 3 years earlier and progressive slowing and
stiffness of his right arm and gait. After his initial diagnosis, he was started
on carbidopa/levodopa 25 mg/100 mg with initial positive response.
However, gait instability progressed with onset of falls, slurred speech, and
mild dysphagia. His wife also noticed a progressive change in his facial
expression. He seemed to stare more and rarely initiated conversation.
On examination (Supplemental Digital Content 5-1, links.lww.com/
CONT/A178), he was fully oriented. His speech was hypophonic and dysarthric,
but language and cognition were preserved. His facial expression was
masked, and he tended to squint. Vertical saccades (upgaze and downgaze)
were reduced, and horizontal saccades were slowed. Off levodopa he had
moderate bradykinesia of hand movements, finger tapping, and rigidity in
the limbs, on the right more than the left. Mild left hand dystonic posturing
was also noted. He had marked neck rigidity, and he found it difficult to
achieve full range of motion. He stood without assistance, and his gait was
stooped, slow, and his stance was narrowed and unstable with a tendency to
hold onto the examiner or walls. Pull test revealed minimal to no compensation.
He would have fallen if not caught. Brain MRI was notable for age-related
cerebral atrophy and mild prominence of basal cisterns. Further increase in
carbidopa/levodopa dose was unhelpful. A year later, he continued to slow
in all of his movements, his dysarthria worsened, and his vertical gaze
palsy became more apparent. A diagnosis of progressive supranuclear
palsyYparkinsonism was suspected.
Comment. This case illustrates several features suggestive of probable
progressive supranuclear palsyYparkinsonism. Early on, the patient’s
features were typical of Parkinson disease, including tremor, asymmetric
rigid bradykinesia, and response to levodopa. Later, he developed
oculomotor abnormalities, bulbar features (eg, dysarthria, mild dysphagia),
increased postural instability, and waning levodopa response. There is an
increased awareness of levodopa-responsive cases of progressive supranuclear
palsy, especially early in the disease course.

1124 www.ContinuumJournal.com August 2016

Copyright © American Academy of Neurology. Unauthorized reproduction of this article is prohibited.


KEY POINT
and that response can be sustained cognitive issues, these patients may be h Atrophy of the midbrain
for years until the later onset of PSP misdiagnosed with FTD or another pri- and superior cerebellar
symptoms are manifest. Life expec- mary dementia.15 peduncles correlates
tancy in PSP-parkinsonism is also Progressive supranuclear palsyY with disease progression
typically longer than in Richardson primary lateral sclerosis. Several re- in progressive
syndrome, averaging 9 or more years ports have suggested possible overlap supranuclear palsy.
from diagnosis. of a PSP syndrome with PLS.16,17 In-
Pure akinesia with gait freezing. In deed, there are common features that
pure akinesia with gait freezing, pa- overlap. PLS is similarly progressive and
tients characteristically present with typically manifests bulbar symptoms,
primary freezing of gait, usually with spastic gait, and sometimes ocular dys-
initiation, and frequent falls.12 Micro- motility, although usually not supra-
graphia, masked face, and apraxia of nuclear palsy. Purported PSP-PLS cases
eyelids may also be present. However, are reported to have a tauopathy
tremor, rigidity, and ophthalmoplegia mainly in motor areas linked by the
are not typical. In 2007, diagnostic cri- corticospinal tract.18
teria were developed that require Cerebellar variant of progressive
gradual onset of freezing of gait or supranuclear palsy. Recently, a number
speech, without tremor, a lasting re- of reports have described a novel var-
sponse to levodopa, or imaging sugges- iant of PSP with features of cerebellar
tive of Binswanger disease. In the first ataxia either as the presenting or pre-
5 years, there should be no evidence dominant symptom and tau pathology
of supranuclear palsy, dementia, or in the deep cerebellar nuclei.9,19 This
limb rigidity.12 cerebellar subtype of PSP appears rare,
Progressive supranuclear palsyY but may be more prevalent in Asian
corticobasal syndrome. PSP-CBS is one populations given recent reports.
of the rarest presentations of PSP. In
Pathology
contrast to classic PSP, early gait dif-
ficulty and postural instability are not The hallmark of PSP is abnormal de-
typical.13 The clinical features of PSP- position of tau and associated pathol-
CBS include asymmetrical dyspraxia, ogy (Figure 5-3). The tufted astrocyte
alien limb phenomenon, and dystonia. is pathognomonic, but other features
Rigidity and bradykinesia are common include coiled bodies, neuropil threads,
and do not respond to levodopa. These pretangles, and neurofibrillary tangles.20
patients additionally have difficulty with Tau pathology generally spares the
saccadic eye movements, most often cortex and involves the basal ganglia,
to the side affected by apraxia, as op- dentate, pontine, and oculomotor nuclei.
posed to the typical vertical gaze palsy. There is associated gliosis and degen-
Progressive supranuclear palsyY eration that is marked by midbrain at-
frontotemporal dementia. PSP-FTD is rophy, loss of pigmented cells in the
a rare variant of PSP. These patients may substantia nigra, and atrophy of the
present with behavioral and cognitive subthalamic nucleus, superior cerebel-
dysfunction either as the only feature or lar and middle cerebellar peduncles,
as the predominant feature of disease. dentate nucleus, and frontal cortex.
Other typical symptoms of PSP follow The tauopathies (PSP, CBS, Alzheimer
later in the course, including falls, disease, and FTD) are characterized by
supranuclear gaze palsy, and levodopa- abnormal tau hyperphosphorylation and
unresponsive parkinsonism.14 Because deposition. Tau is encoded by MAPT
of the predominant behavioral and and normally functions to stabilize

(Minneap Minn) 2016;22(4):1117–1142 www.ContinuumJournal.com 1125

Copyright © American Academy of Neurology. Unauthorized reproduction of this article is prohibited.


Atypical Parkinsonian Syndromes

FIGURE 5-3 Typical neuropathologic findings in progressive supranuclear palsy. The


macroscopic photo shows atrophy of the superior cerebellar peduncle (SCP) and
midbrain structures including the subthalamic nucleus (STN) and loss of
pigmented cells in the substantia nigra (SN) (A).The photomicrographs show classic pathologic
findings including neurofibrillary tangles (B), neuropil threads (C, arrows), coiled bodies (D),
and tufted astrocyte (E, arrow) stained with PHF1 antibody to human tau.
Panel A is reprinted with permission from Dickson DW, et al, Curr Opin Neurol.18 B 2010 Lippincott Williams &
Wilkins, Inc. journals.lww.com/co-neurology/Abstract/2010/08000/Neuropathology_of_variants_of_progressive.9.aspx.

microtubules. The protein has six Brain imaging such as MRI, however,
isoforms based on splice variants, in- remains most helpful. In PSP, atrophy
cluding three or four microtubule bind- of the midbrain and superior cerebel-
ing repeat (R) domains. In PSP the 4R:3R lar peduncles can help distinguish PSP
tau ratio in brain is increased (lower in from other neurodegenerative disor-
PSP-parkinsonism) compared to con- ders. Measurement of suprapontine an-
trols. By contrast, the opposite is ob- teroposterior diameter with MRI has
served in FTD. Although MAPT is not been used for analyzing midbrain atro-
mutated in sporadic PSP, it is closely phy, but remains controversial. Another
linked to parkinsonisms including PSP. proposed method of assessing mid-
Mutation in MAPT is most commonly brain atrophy is by measurement of
associated with FTD with parkinsonism the midsagittal area of the midbrain
linked to chromosome 17 (FTDP-17). tegmentum and ratio to area of the
A recent genome-wide association study pons, which is significantly reduced in
was performed and identified three ad- PSP compared to that in PD, MSA-
ditional putative genes associated with parkinsonism, and healthy controls.22
PSP: STX6 (encodes syntaxin 6), EIF2AK3 As a result, the brainstem is often
(PERK, which involves the endoplasmic beaked and takes on the appearance
reticulum unfolded response pathway), of a hummingbird or penguin body
and MOBP (myelin-associated oligoden-
(Figure 5-4). The width of the supe-
drocytic basic protein, which is concen-
rior versus the middle cerebral peduncle
trated in pathologic regions).21
has also been used.23 Diffusion-tensor
Diagnostics imaging (DTI) shows both gray and
To date there is no available blood, CSF, white matter reduction, with predilec-
or single imaging marker for PSP. tion for the anterior and medial thalamic

1126 www.ContinuumJournal.com August 2016

Copyright © American Academy of Neurology. Unauthorized reproduction of this article is prohibited.


KEY POINT
h The approach to
treatment of patients
with progressive
supranuclear palsy
should include a
multidisciplinary team
and involve physical,
occupational, and speech
therapy; psychiatry;
neuropsychology; social
work; and palliative care.

FIGURE 5-4 Sagittal T1-weighted MRI of a patient with


progressive supranuclear palsyYRichardson
syndrome. Marked midbrain atrophy is
present, suggesting the appearance of a hummingbird or
penguin sign. Inset shows atrophy of the superior cerebellar
peduncles (arrows).

nuclei.24 Consistent with that found and to develop an exercise program to


pathologically, positron emission to- maintain mobility. Occupational ther-
mography (PET)/single-photon emis- apy can help patients (and caregivers)
sion computed tomography (SPECT) with activities of daily living. Speech
scans of patients with PSP reveal therapy is important early on to treat
hypometabolism of bilateral frontal dysarthria. More advanced patients may
cortex, caudate heads, thalami, cingu- need cognitive therapy for speech
late gyri, and midbrain.25 Recent tau apraxia or to discuss alternative com-
PET ligands, such as 18F-T807/8 and munication modes if markedly dys-
18 arthric or anarthric. Dysphagia and
F-THK523, that have been developed
for Alzheimer disease also show aspiration are the leading causes of
promise for detecting tau in PSP (and morbidity and mortality in PSP. The
other tau disorders), but will require author recommends early and regular
further validation.26 swallowing evaluations (6-month in-
tervals) to assess risk and to modify
Therapeutic Strategies the diet. Often in later stages of the
Currently there is no specific treatment disease, the question of a feeding tube
or cure for PSP. Treatment is focused arises. Discussion with the patient,
primarily on symptomatic improve- family/caregiver, and speech patholo-
ment and, ideally, should involve a gist regarding the benefits and risks
multidisciplinary team approach includ- of a feeding tube is recommended. A
ing physical and occupational therapy, percutaneous gastrostomy tube can
speech pathology, neuropsychology, help maintain nutrition, hydration, and
psychiatry, social work, and palliative medication administration, but does
care. Physical therapy is critical for gait not reduce the risk of aspiration. The
and postural instability, fall prevention, decision regarding gastrostomy should
(Minneap Minn) 2016;22(4):1117–1142 www.ContinuumJournal.com 1127

Copyright © American Academy of Neurology. Unauthorized reproduction of this article is prohibited.


Atypical Parkinsonian Syndromes

KEY POINTS
h Levodopa therapy be carefully approached and discussed antidepressant, or an SSRI may be help-
should be tried in most as it raises ethical, cultural, and personal ful. Treatments for other symptoms are
progressive supranuclear considerations regarding quality of summarized in Table 5-5.
palsy cases, with a life. A social worker or case manager
levodopa dose of up to is often critical to help family and care- CORTICOBASAL DEGENERATION
1200 mg/d in divided givers plan for care needs. Palliative CBD is an atypical parkinsonian syn-
doses as tolerated. Partial care also should be considered, and drome with predominant involvement
response is possible in frank discussion of advance directives of the cortex and basal ganglia that
early progressive should be pursued. presents with varied phenotypes.
supranuclear palsy, Although no one medication treats The classic presentation with asym-
particularly in progressive
all the symptoms of PSP, specific treat- metric rigidity, dystonia, and ideomo-
supranuclear
ments may be helpful. For parkinson- tor apraxia is now referred to as CBS,
palsyYparkinsonism.
ism, a levodopa trial up to 1200 mg/d but CBD is increasingly also recognized
h The most common (up to 300 mg per dose if it can be to present with features that may over-
presenting features for
tolerated) for 1 month is recommended lap with FTD, primary progressive
corticobasal degeneration
to determine responsiveness. PSP- aphasia, Alzheimer disease, posterior
are asymmetric hand
clumsiness or apraxia
parkinsonism in particular is often levo- cortical atrophy, and PSP. Typically,
followed by early dopa responsive initially and contrasts marked asymmetry of involvement is
bradykinesia, frontal with that observed in classic Richardson the most striking feature and helps dif-
syndrome, tremor, syndrome and other variants. Dopa- ferentiate CBD from other degenerative
and rigidity. mine agonists, however, often provide disorders. The most common present-
minimal benefit. Amantadine has been ing feature is asymmetric hand clumsi-
reported to have benefit for gait and ness followed by early bradykinesia, a
dysphagia, but studies to support this frontal syndrome, tremor, and rigidity.30
treatment have been limited.27 Anticho- The mean onset of disease occurs in the
linergics should be avoided due to the sixth decade, and prognosis is generally
potential for confusion. Coenzyme Q10 poor with a mean survival of about
is a well-tolerated supplement and has 7 years from diagnosis. Typical features
recently been shown in a small ran- include marked asymmetry, focal ri-
domized clinical trial to provide mod- gidity, coarse rest/action tremor, limb
est benefit in PSP, including slight dystonia (followed by contractures),
improvement in cognitive function.28 alien limb phenomenon, hand, limb,
Painful dystonic posturing of the neck gait, or speech apraxia, myoclonus, cor-
and limbs, blepharospasm, and eye- tical sensory loss, language deficits,
opening apraxia may be treated with frontal/cortical dementia, oculomotor
botulinum toxin. For cognitive decline dysfunction (gaze palsy, impaired con-
and dementia, cholinesterase inhibi- vergence), bulbar impairment, postural
tors such as rivastigmine may provide instability, gait difficulty, hyperreflexia,
modest benefit. Mood disturbance in- and extensor plantar response. Poor
cluding depression, anxiety, and irrita- levodopa response tends to occur, but
bility should be identified and treated high-dose trials are warranted early in
with antidepressants. For apathy, acti- the disease as in PSP (levodopa dose of
vating antidepressants such as bupro- up to 1200 mg/d for 2 to 3 months as
pion, venlafaxine, sertraline, and tolerated) and sometimes provide par-
fluoxetine are preferred over other se- tial benefit. Case 5-2 illustrates a typical
lective serotonin reuptake inhibitors case of CBS.
(SSRIs), which may worsen apathy. If Ideomotor apraxia, myoclonus,
pseudobulbar affect is marked, asymmetric rigid-bradykinetic syndrome,
dextromethorphan-quinidine, a tricyclic and later-onset gait/balance disturbance
1128 www.ContinuumJournal.com August 2016

Copyright © American Academy of Neurology. Unauthorized reproduction of this article is prohibited.


TABLE 5-5 Overlapping Symptomatic Treatment Options for Progressive Supranuclear Palsy
and Corticobasal Degeneration

Symptom Treatment Options


Parkinsonism/rigidity Levodopa trial is appropriate to determine responsiveness of motor symptoms,
especially progressive supranuclear palsyYparkinsonism3
Dopamine agonists (minimal benefit)
Amantadine (possible benefit in one study27)
Gait imbalance, falls Physical therapy and assistive devices
Dystonia, blepharospasm Botulinum toxin
Spasticity, contractures Muscle relaxants (eg, baclofen)
Dysarthria Speech therapy
Dysphagia Swallow evaluation (with fluoroscopy) and therapy; modified diet,
aspiration precautions (eg, head of bed up 45 degrees, possible percutaneous
gastrostomy tube)
Sialorrhea Anticholinergics (eg, glycopyrrolate, scopolamine); may worsen cognition
Botulinum toxin in parotid and submandibular glands; use caution as botulinum
toxin can worsen dysphagia
Atropine 1% ophthalmic drops administered sublingually (for palliative use)
Ocular symptoms Zolpidem (short-term benefit/improvement in
voluntary saccades)29
Balance and eye movement therapy to help with better control of
downward saccades
Ophthalmic lubricants for dry eyes
Sunglasses for photosensitivity
Ophthalmology referral and prisms for diplopia
Eyelid crutches or myomectomy are not recommended for eye-opening apraxia
Urinary frequency Antispasmodics; due to risk of central nervous system side effects (eg, dementia)
agents such as tolterodine, trospium, solifenacin, and darifenacin are
preferred as they are more selective and less likely to cross blood-brain barrier
Dementia Cholinesterase inhibitors (eg, rivastigmine, donepezil)
Depression and anxiety Tricyclic antidepressants, selective serotonin reuptake inhibitors (SSRIs),
serotonin norepinephrine reuptake inhibitors (SNRIs)
Pseudobulbar affect Dextromethorphan plus quinidine

have been reported as the best pre- certain task (eg, use a screwdriver or
dictors for CBS diagnosis.31 Ideomotor cut with a pair of scissors). This type of
apraxia is defined by an inability to apraxia can be difficult to distinguish
perform a skilled motor task despite from limb-kinetic apraxia, which is
having intact language, motor, and frequently seen in parkinsonisms, but
sensory function. Examples include in- is independent of modality (imitation
ability to imitate gestures or mime a versus miming). Peculiar to CBS, alien

(Minneap Minn) 2016;22(4):1117–1142 www.ContinuumJournal.com 1129

Copyright © American Academy of Neurology. Unauthorized reproduction of this article is prohibited.


Atypical Parkinsonian Syndromes

KEY POINT
h Ideomotor apraxia is
defined by an inability to
Case 5-2
An 80-year-old right-handed man with history of hypertension, prostatic
perform a skilled
hypertrophy, and deep venous thrombosis presented with symptoms of
motor task despite having
progressive gait difficulty, falls, increasing fatigue, tremulousness, and left
intact language, motor,
hand dysfunction that he had experienced over the past 2 years. The
and sensory function.
patient’s wife reported that his gait and movement had slowed several
Examples include inability
years prior, but had worsened further in the setting of deep venous
to imitate gestures or
thrombosis and leg edema. Recently, he had begun dragging his left leg
mime a certain task (eg,
more. The patient noted increasing stiffness in his left arm and tended to
use a screwdriver or cut
hold it flexed at his side. He felt that his symptoms were beginning to
with a pair of scissors).
spread to his right side and reported symptoms of micrographia.
This type of apraxia can
Additional symptoms included hoarse, slowed speech, a decrease in facial
be difficult to distinguish
expression, and mild difficulty swallowing. If he ate too quickly he would
from limb-kinetic apraxia,
hiccup, and swallowing pills had become difficult. The patient denied
which is frequently seen
changes in cognition, memory, or mood. He had previously been evaluated
in parkinsonisms, but is
by a local neurologist, diagnosed with parkinsonism, and treated with a
independent of modality
dopamine agonist with some subjective symptomatic benefit. The addition
(imitation versus miming).
of carbidopa/levodopa did not provide additional benefit.
On examination (Supplemental Digital Content 5-2, links.lww.com/
CONT/A204), the patient was fully oriented but displayed mild disorganized
thought processes, and his cognition and language were otherwise intact.
The patient’s speech was characterized by mild hypokinetic dysarthria.
He had slight facial hypomimia, reduced vertical saccades, and slowed
optokinetic nystagmus. His left upper extremity tone was rigid, and he tended
to hold the arm flexed at his side, but without tremor. Ideomotor apraxia
and reduced graphesthesia were noted in his left hand, but no neglect. The
patient’s gait was slow and stiff with the left arm held flexed, and his left leg
dragged. Postural reflex was reduced on pull testing. Review of his brain MRI
suggested possible asymmetric cerebral atrophy, right greater than left, and
‘‘beaking’’ of his midbrain.
After his initial presentation, the patient’s symptoms rapidly progressed
with increasing left-sided rigidity, dystonic posturing, alien limb
phenomenon, limb-kinetic apraxia, and cortical sensory deficits. He had
increasing symptoms of vertical gaze palsy, gait difficulty, falls, and he
required a wheelchair and assistance with activities of daily living. His speech
remained mildly dysarthric, but was characterized by some grandiosity and
nonsensical statements. He remained on a dopamine agonist (due to
subjective benefit over levodopa) and received botulinum toxin injections
for left upper extremity dystonia and mild antecollis.
Comment. This case demonstrates progressive asymmetric limb
dysfunction, rigid bradykinesia, dystonia, apraxia, and cortical sensory
deficits consistent with probable corticobasal syndrome. Later progression
of cognitive deficits, axial rigidity, falls, and supranuclear palsy raise the
possibility of a corticobasalYprogressive supranuclear palsy syndrome.

limb phenomenon appears as abnor- lateral limb. Dementia in CBS is actually


mal grasping, posturing, or spontane- a late feature with typically preserved
ous levitation of an arm or leg, but can semantic memory.32 Neuropsychiatric
also include pursuit or avoidance of a testing often shows a fronto-striatal-
tactile stimulus in the opposite or contra- parietal predominance with deficits in

1130 www.ContinuumJournal.com August 2016

Copyright © American Academy of Neurology. Unauthorized reproduction of this article is prohibited.


attention, concentration, verbal fluency, sion of more than 1 year, age of onset
language, praxis, and executive and vi- of 50 years or more, with or without a
suospatial function.33 Cortical findings similar family history or known tau
such as aphasia, limb apraxia, and mutations (Table 5-6).
graphesthesia depend on the hemisphere
predominantly affected. Pathology
Given the varied presentations and Although classically defined by as a dis-
evolving clinicopathologic under- tinct clinicopathologic entity, in recent
standing of CBD, clinical criteria for di- studies CBD has increasingly been
agnosis have recently been revisited. shown to present with other distinct
Armstrong and colleagues34 proposed pathologies. Clinical diagnosis of CBD
new diagnostic criteria for four CBD correlates with CBD pathology in only
phenotypes including CBS, frontal 25% to 56% of cases.34 Pathologically,
behavioral-spatial syndrome, nonfluent/ CBD is characterized by symmetrical
agrammatic primary progressive apha- cerebral atrophy, which is typically pre-
sia, and a PSP syndrome. Probable and sent despite the asymmetric clinical
possible criteria are described and in- presentation usually seen in CBD. Neu-
clude insidious onset, gradual progres- rodegeneration is widespread and, in

a
TABLE 5-6 Corticobasal Syndrome Phenotypes

Syndrome Key Features


Corticobasal syndrome (CBS) Asymmetric limb rigidity, akinesia,
(classic corticobasal degeneration) dystonia, or myoclonus
PLUS
Orobuccal or limb apraxia, cortical sensory
deficit, or alien limb phenomenon
Probable CBS is two features in each of
the categories above; possible CBS is one
feature in each of the categories above
and may be symmetric
Frontal behavioral (frontotemporal Executive dysfunction, behavioral or
dementia) variant personality changes
Posterior cortical atrophy syndrome Visuospatial disturbance, apraxia,
myoclonus, association with Alzheimer
disease pathology
Progressive nonfluent/ Effortful, agrammatic speech; impaired
agrammatic aphasia grammar/sentence comprehension or
groping, or distorted speech production
(apraxia of speech)
Progressive supranuclear Axial or symmetric limb rigidity/akinesia,
palsy syndrome postural instability, falls, urinary
incontinence, behavioral changes,
supranuclear vertical gaze palsy
a
Modified with permission from Armstrong MJ, et al, Neurology.34 B 2013 American Academy of
Neurology. www.neurology.org/content/80/5/496.short.

(Minneap Minn) 2016;22(4):1117–1142 www.ContinuumJournal.com 1131

Copyright © American Academy of Neurology. Unauthorized reproduction of this article is prohibited.


Atypical Parkinsonian Syndromes

addition to the cortex, includes the basal PSP, rehabilitative services and mul-
ganglia, thalamus, substantia nigra, sub- tidisciplinary approaches are critical to
thalamic nucleus, and red nucleus. Path- maintain function as long as possible
ologic diagnosis is characterized by despite progressive decline. Regular
widespread but topographic deposi- exercise and activity early in the dis-
tion of 4R-predominant, hyperpho- ease are most beneficial and may delay
sphorylated tau in neurons and glia, disability. Despite the limitations in
astrocytic plaques, and corticobasal in- treatment, a levodopa dose trial up to
clusions.35 In subcortical nuclei, such 800 mg/d to 1200 mg/d should be at-
as the substantia nigra, intranuclear in- tempted; however, dopaminergic ther-
clusions are reminiscent of globose apy rarely provides improvement and
neurofibrillary tangles, but those in may induce dyskinesia or intolerance.
cortex are granular, crescent shaped, Clonazepam or levetiracetam may im-
or globular.36 Ballooned neurons with prove myoclonus, and muscle relaxants
diffuse cytoplasmic immunoreactivity (eg, baclofen) and botulinum toxin can
to !"-crystallin and ubiquitin are com- help painful rigidity and dystonic con-
mon, but not specific to CBD. ‘‘Coiled tractures that limit function. Experi-
bodies’’ are frequent in oligodendrog- mental therapeutics have focused on
lia, but, unlike PSP, tufted astrocytes tau pathology (see previous section on
are absent. PSP therapeutics).

Diagnostics MULTIPLE SYSTEM ATROPHY


Neuroimaging is most useful in CBD. MSA is characterized by variable pre-
Asymmetric frontoparietal atrophy can sentations of parkinsonism, cerebellar
be identified on CT/MRI and can some- and pyramidal signs, and autonomic dys-
times be helpful in differentiating CBD function. It was first described in the
from other parkinsonian syndromes.37 1960s under the subheadings of Shy-
Fludeoxyglucose positron emission Drager syndrome, olivopontocerebellar
tomography (FDG-PET) similarly can atrophy, and striatonigral degeneration,
sometimes reveal asymmetric cortical depending on the predominant pre-
metabolism. Uptake on levodopaY senting symptoms.39 However, the dis-
positron emission tomography (DOPA- covery of oligodendroglial cytoplasmic
PET) likewise is reduced in the inclusions and associated multisystem
striatum and highly asymmetric in the neurodegeneration in patients with
cortex. [ 123 I]"-CIT (iodine-123-2"- MSA, regardless of the clinical pheno-
carbomethoxy-3"-[4-iodophenyl] type, suggested a common pathology
tropane) SPECT shows a similar pat- and has led to the terminology used
tern of reduced asymmetric striatal today. Two clinical phenotypes are
binding, but is nonspecific to CBD generally distinguished by predominant
(similar to dopamine transporter parkinsonism (MSAYparkinsonian type
SPECT [DAT-SPECT]). Both tau and [MSA-P]) or predominant cerebellar
"-amyloid ligand are also being ex- ataxia (MSAYcerebellar type [MSA-C])
plored for use in distinguishing CBD (Table 5-7). Median age of onset for
from other related disorders.38 MSA is 58 years of age, which is
younger than that of PSP and CBD.
Therapeutic Strategies No MSA cases have been identified
Effective pharmacologic therapies for younger than age 30, whereas for PSP
CBD are still lacking, and treatment the cutoff age is 50 years. Disease
remains mainly supportive. Similar to progression is faster than in PD and
1132 www.ContinuumJournal.com August 2016

Copyright © American Academy of Neurology. Unauthorized reproduction of this article is prohibited.


KEY POINTS

TABLE 5-7 Multiple System Atrophy Phenotypes h Multiple system


atrophyYparkinsonian
Clinical Presentation Featuresa type may be differentiated
from Parkinson disease
Multiple system atrophyYparkinsonian Onset 940 years, duration G10 years,
by its more symmetrical
type (MSA-P) (previously referred to as progressive parkinsonism poorly
Shy-Drager syndrome or striatonigral responsive to levodopa, with autonomic appearance, atypical
degeneration) failure (including orthostatic hypotension, tremor, dystonia
impotence, bladder dysfunction) (antecollis), early
dysarthria/dysphonia,
Multiple system atrophyYcerebellar Ataxia; degeneration of ventral pons,
gait and postural
type (MSA-C) (previously referred to olives, cerebellum; mild parkinsonism
instability, dysautonomia,
as olivopontocerebellar atrophy) and cognitive decline
and rapid progression.
a
Autonomic dysfunction, respiratory symptoms, and sleep disturbance can precede motor signs by h Multiple system
months to years.
atrophyYcerebellar type
is one of the most
common causes of
sporadic, adult-onset
mean survival is approximately 6 to toms. Levodopa-induced dyskinesia is ataxia and is distinguished
9 years, consistent with more wide- also possible; the development of early by parkinsonism,
spread neurodegeneration. orofacial dystonia is a red flag for MSA-P. dysautonomia, and
There are, however, select cases of rapid progression.
Clinical Features MSA-P that respond well to levodopa
In the Western hemisphere, MSA-P is for many years. Case 5-3 illustrates
more common than MSA-C. Patients many of the features of MSA-P.
present with features of bradykinesia, In MSA-C, patients present with
rigidity, and tremor and manifest a cerebellar signs such as gait and
more symmetric appearance than in balance impairment, limb ataxia, and
PD. The pill-rolling type of tremor typ- staccato speech or dysarthria. Oculo-
ically seen in PD is uncommon in motor disturbances (nystagmus, jerky
patients with MSA-P. By contrast, trem- pursuits, and hypometric/hypermetric
or in MSA-P is often higher frequency, saccades) may be present. The gait
lower amplitude, and sometimes has a ataxia in MSA-C may be indistinguish-
jerky, stimulus-sensitive, myoclonic able from other sporadic, adult-onset
component.40 Postural instability is a cerebellar ataxias. Other clues in the
later feature in MSA-P compared to that history, such as the presence of par-
in PSP. Speech is often characterized by kinsonism, dysautonomia, or rapid
a mixed spastic, hypokinetic dysarthria progression, should point toward the
or dysphonia as opposed to that ob- diagnosis of MSA-C.40 Notably, the
served in PSP. Dysphagia is not uncom- predominant motor features of the dis-
mon, earlier, and more marked than ease can change with disease progres-
observed in PD. Some patients also sion so that patients who present with
develop respiratory or laryngeal stridor. cerebellar features may eventually also
Other suggestive features include develop parkinsonism.
hyperreflexia, Babinski signs, dystonia, Features common to both subtypes
anterocollis, and early striatal deformi- of MSA include sleep disturbance,
ties (Table 5-8). MSA-P is often poorly autonomic failure, and respiratory dys-
responsive to dopaminergic therapy. function, which can precede motor signs
Patients who do respond may do so by several months to years.41 Ortho-
transiently, and therapy can be limited static hypotension is a frequent, debil-
by exacerbation of orthostatic symp- itating symptom, often complicated by

(Minneap Minn) 2016;22(4):1117–1142 www.ContinuumJournal.com 1133

Copyright © American Academy of Neurology. Unauthorized reproduction of this article is prohibited.


Atypical Parkinsonian Syndromes

TABLE 5-8 Differentiating Multiple System AtrophyYParkinsonian


Type and Multiple System AtrophyYCerebellar Type
from Idiopathic Parkinson Disease

b Multiple System AtrophyYParkinsonian Type (MSA-P)


Symmetrical onset
Rapid progression
Tremor (distal, myoclonic)
Frequent rigidity, hypokinesia
Dystonia (axial), anterocollis (dropped head)
Early falls
Dysarthria, dysphonia
Sleep apnea, rapid eye movement (REM) sleep behavior disorder
Respiratory/laryngeal stridor
Hyperreflexia, Babinski signs
Dysautonomia (69% versus 5% in Parkinson disease)
Poor/unsustained levodopa response (~30%)
Dyskinesia (orofacial common)
b Multiple System AtrophyYCerebellar Type (MSA-C)
Cerebellar limb and gait ataxia
Early falls
Dysarthria (scanning, ataxic)
Dysphagia
Gaze impairment (hypokinetic/hyperkinetic saccades)
Lower and upper motor neuron signs
Emotionality, depression, anxiety
Progressive dementia

treatment. Urogenital dysfunction, tory insufficiency.42 These red flags


such as incomplete bladder emptying may help to distinguish MSA from PD
or urinary incontinence, is common in (Table 5-8). Although dementia histor-
women, whereas erectile dysfunction ically has not been included as a
is common in men. Urogenital dys- feature of MSA, there is increasing
function tends to occur early in MSA recognition of cognitive dysfunction
compared to PD, in which symptoms and estimates of dementia in 14% to
present only at more advanced disease 16% of patients with MSA.43
stages. Other MSA features include Pisa
syndrome, a form of axial dystonia that Pathology
manifests as lateral bending of the MSA is characterized by oligodendrog-
trunk; camptocormia, or abnormal for- lial cytoplasmic inclusions and multi-
ward flexion of the trunk; and respira- system neurodegeneration, including

1134 www.ContinuumJournal.com August 2016

Copyright © American Academy of Neurology. Unauthorized reproduction of this article is prohibited.


KEY POINT

Case 5-3 h Glial cytoplasmic


inclusions that are
A 66-year-old man presented with a 4-year history of progressive bilateral
enriched with
upper extremity and lower facial tremors, rare myoclonus, bradykinesia,
!-synuclein are a
and speech and gait difficulty with recent falls and freezing. Over the past
pathologic hallmark of
couple of years he had been treated with carbidopa/levodopa and
multiple system atrophy.
required a high dose (up to 500 mg every 4 hours for effect). However, on
presentation off medication, he reported little change in symptoms except
mild decrease in tremor. Additional symptoms included dysphagia,
reduced appetite, urinary frequency and urgency, erectile dysfunction,
constipation, dream enactment, depression, and anxiety.
On examination (Supplemental Digital Content 5-3, links.lww.com/CONT/
A179) while seated in a wheelchair, the patient had abnormal posture with
mild right head tilt, hypokinetic dysarthric speech with mild tremulousness,
and jaw-opening dystonia. Facial hypomimia and reduced/slowed volitional
saccades vertically were noted. Jerky tremor was apparent only with sustained
posture and intention. He had moderate axial and bilateral limb rigidity and
bradykinesia, and his gait was slow and marked by hesitation and freezing.
On pull testing he took several steps and was caught by the examiner. Brain
MRI demonstrated moderate cerebral atrophy and reduced T2 signal in the
posterior putamen bilaterally and slit hyperintensity.
Comment. This case illustrates features of multiple system atrophyY
parkinsonism including rapid progression, dysarthria, dystonia, abnormal
posture, atypical tremor, axial and limb-rigid bradykinesia that is
reasonably symmetric, early gait and postural instability, rapid eye movement
(REM) sleep behavior disorder, and dysautonomia with urinary frequency and
urgency, erectile dysfunction, and probable gastrointestinal dysmotility. Note
also the presence of a wheelchair (the ‘‘wheelchair sign’’), which can be a
red flag for atypical parkinsonism and indicate significant, early postural
instability. Symptoms were partially responsive to high-dose levodopa (500 mg
5 times daily). REM sleep behavior disorder was treated with clonazepam.

neuronal loss and gliosis involving the Diagnostics


putamen, substantia nigra, pons, inferior Although the diagnosis of MSA is pri-
olivary nucleus, cerebellum, and inter- marily based on clinical criteria, several
mediolateral cell column of the thoracic diagnostic studies may be helpful. So
and sacral spinal cord.44 The degree of far no blood test exists to distinguish
involvement (nigrostriatal system, MSA from other parkinsonian syn-
pons, cerebellum) determines the pre- dromes. Biomarkers such as !-synuclein
dominant presentation or subtype of are being explored, but remain contro-
MSA. In patients with autonomic fea- versial. Neuroimaging such as MRI is
tures, the dorsal motor nucleus of the often helpful. MRI findings supportive
vagus, locus coeruleus, and ventrolat- of the diagnosis include bilateral T2
eral medulla are affected. Oligoden- hypointensity in the posterolateral puta-
droglial cytoplasmic inclusions are men, representing iron deposition, and
argyrophilic, sickle-shaped, or oval slit hyperintensity in the lateral margin
cytoplasmic aggregates that are of the putamen (Figure 5-6). Olivo-
enriched with !-synuclein and corre- pontocerebellar atrophy is consistent
late with the severity of neuronal loss with MSA-C. Pontine atrophy and gliosis
and disease duration (Figure 5-5).45 may be apparent on T2-weighted

(Minneap Minn) 2016;22(4):1117–1142 www.ContinuumJournal.com 1135

Copyright © American Academy of Neurology. Unauthorized reproduction of this article is prohibited.


Atypical Parkinsonian Syndromes

urinary dysfunction. If a sleep distur-


bance is suspected, polysomnogram
should be performed to identify sleep
apnea, periodic limb movements, and
rapid eye movement (REM) sleep be-
havior disorder.

Therapeutic Strategies
Treatment in MSA focuses mainly on
supportive therapy and should involve
allied health care and rehabilitation
services because of the multisystem
nature of disease. Some 30% to 60% of
patients with MSA initially respond to
dopaminergic therapy. In the absence
of other treatments, a trial of up to
1000 mg/d to 1200 mg/d of levodopa
(300 mg per dose if tolerated) for a
period of 3 months is supported and
FIGURE 5-5 Glial cytoplasmic inclusions in multiple system
atrophy. Oligodendroglial cytoplasmic inclusions is the mainstay of initial therapy for
here are stained with antibody to human parkinsonism in MSA. Dopamine ago-
!-synuclein.
nists, because of autonomic and other
issues, are generally inferior to levodopa
images in a hot cross bunYlike pattern. for MSA and have not been studied
Additionally, PET scans often show carefully in this setting. The challenge is
decreased striatal and frontal metabo- to balance the benefits of therapy with
lism. DAT (125I-ioflupane) SPECT also the development of motor complica-
typically shows asymmetric reduced tions that may appear earlier in MSA as
striatal binding.46 Autonomic testing also compared to classic PD. These MSA fea-
can be helpful and include tilt-table tures include dyskinesia, frequently in-
testing, 24-hour ambulatory blood volving the jaw and face, and orthostatic
pressure and heart rate monitoring, hypotension. Alternative therapies such
and baroreceptor sensitivity (ie, the as deep brain stimulation are not recom-
baroreflex mechanism or ability to mended due to poor response.
regulate blood pressure by controlling Autonomic symptoms such as ortho-
heart rate, contractility, and peripheral static hypotension are treated first with
resistance) for orthostatic hypotension. conservative measures including oral
In the office, testing should include hydration, increased salt intake, and
supine blood pressure with heart rate, compression stockings or abdominal
then standing blood pressure and binder. If still symptomatic, pharmaco-
heart rate after 3 minutes. A drop in logic therapy with fludrocortisone or
systolic blood pressure of more than desmopressin, which increase blood
20 mm Hg or drop in diastolic blood volume, may be helpful but are con-
pressure of more than 10 mm Hg with traindicated in patients with heart fail-
minimal rise in heart rate is diagnostic. ure. Midodrine, an !1 agonist, can also
Other tests may include sweat testing be used but has the potential to cause
(eg, quantitative sudomotor axon reflex supine hypertension. Recently, the US
test [QSART]) and gastric emptying study Food and Drug Administration (FDA)
(for gastroparesis), and urodyamics for approved the use of droxidopa for
1136 www.ContinuumJournal.com August 2016

Copyright © American Academy of Neurology. Unauthorized reproduction of this article is prohibited.


KEY POINTS
h Pharmacologic
treatment of orthostatic
hypotension may
include enhancing
blood volume with
fludrocortisone or
desmopressin or adding
drugs that increase
vascular resistance such
as midodrine, droxidopa,
or pyridostigmine.
h Dementia with Lewy
bodies is characterized
by rapid-onset dementia,
parkinsonism (coincident
or following cognitive
decline), mental status
fluctuations, and
hallucinations.

FIGURE 5-6 Axial T2-weighted MRI of a patient with multiple system atrophyYparkinsonian type
demonstrating slitlike hyperintensity (A, arrows) at the rim of the right putamen,
fluid-attenuated inversion recovery (FLAIR) hypointensity of posterior putamen (B,
arrows), and typical hot cross bun sign representing atrophy and gliosis of the pons (C).

neurogenic orthostatic hypotension. rapidly progressive dementia that is part


Droxidopa is a synthetic amino acid pre- of the spectrum of PD. It is the second
cursor for norepinephrine and epineph- most common form of neurodegenera-
rine. Pyridostigmine is an alternative that tive dementia, after Alzheimer disease,
provides a mild boost in blood pressure and has similar features to other demen-
without supine hypertension but most tias, including PD. The clinical criteria for
of the data supportive of its use come DLB in addition to early dementia in-
from patients with neuropathy. For neu- clude: (1) parkinsonism that is coinci-
rogenic bladder, antispasmodics or bot- dent with or follows dementia onset; (2)
ulinum toxin injections are often helpful. fluctuating cognition, awareness, or alert-
Alpha-blockers for benign prostatic
ness; and (3) recurrent visual hallucina-
hypertrophy, however, have the poten-
tions.47 Additional features include gait
tial to cause hypotension. Some patients
instability, falls, syncope or transient loss
require intermittent self-catheterization
or placement of a suprapubic catheter. of consciousness, delusions/paranoia,
depression, REM sleep behavior disor-
PARKINSON DEMENTIAS: der, and neuroleptic sensitivity. A com-
DEMENTIA WITH LEWY BODIES bination of dementia and psychosis in
DLB (also known as [diffuse] Lewy body general is considered a poor prognostic
dementia or disease) is an early-onset, predictor in this population.

(Minneap Minn) 2016;22(4):1117–1142 www.ContinuumJournal.com 1137

Copyright © American Academy of Neurology. Unauthorized reproduction of this article is prohibited.


Atypical Parkinsonian Syndromes

Pathology dementia syndromes are generally lack-


Diffuse Lewy bodies are the hallmark ing, and diagnosis remains primarily
in DLB as well as in PD dementia clinical. Imaging studies have been
(Figure 5-7). Incidental Lewy bodies, promising for development of potential
however, are also found at autopsy in biomarkers to distinguish DLB.51 Brain
individuals with no clinical signs of MRI generally shows diffuse cerebral
parkinsonism, but may indicate pre- atrophy with relative preservation of
clinical disease.48 The degree of Lewy the occipital and mesial temporal lobes
body involvement of the cortex corre- when compared to Alzheimer disease.
lates with the severity of dementia.49 SPECT studies focused on examining
Spreading Lewy body pathology, or occipital hypoperfusion have reported
synucleinopathy, from brainstem to some specificity/sensitivity in discrimi-
neocortex is described by Braak and nating DLB from Alzheimer disease.
colleagues50 and is thought to contrib- FDG-PET has also been used to exam-
ute to progressive cognitive impairment. ine occipital (and parietal) lobe changes
Coincident Alzheimer disease pathology in DLB. There is emerging evidence for
is also frequently found in cases and the cingulate island sign, or preservation
contributes to dementia. of FDG-PET metabolism in the poste-
rior cingulate relative to the cuneus and
Diagnostics precuneus, that correlates with DLB
Disease-specific biomarkers for DLB/PD versus Alzheimer pathology.52 Dopami-
dementia and related parkinsonian- nergic imaging (ie, DAT-SPECT) likewise

FIGURE 5-7 Typical Lewy bodies are found in the


pigmented cells of the substantial nigra in
both Parkinson disease and dementia with
Lewy bodies. Here Lewy bodies (arrows) are stained with an
antibody specific to the pathologic phosphorylated form
(serine 129) of !-synuclein found enriched in Lewy bodies.
They have a characteristic round appearance with lighter
core and dark halo.

1138 www.ContinuumJournal.com August 2016

Copyright © American Academy of Neurology. Unauthorized reproduction of this article is prohibited.


may be useful if there is question re- monitoring for the risk of agranulocy-
garding the diagnosis of DLB, but is not tosis.55 Other potential side effects of
specific. Pittsburg compound B (PiB) clozapine include orthostatic hypotension
scanning also can be helpful to assess and depression. Recently pimavanserin,
amyloid burden, given the association a selective 5-hydroxytryptamine, seroto-
with DLB more than in PD dementia.53 nin receptor 2 (5-HT2) inverse agonist,
In Alzheimer disease, CSF total and phos- has been studied in phase 3 trials and
phorylated tau levels are increased and holds promise for controlling psychosis
amyloid-" level is reduced. By contrast, without exacerbating parkinsonism.56
CSF tau (and possibly also !-synuclein) Treatment of cognitive impairment
is significantly lower in DLB than in begins with formal neuropsychiatric as-
Alzheimer disease.54 sessment and cognitive therapy. Often,
comorbid depression, apathy, and anx-
Therapeutic Strategies iety are found and should be treated.
As with other atypical parkinsonian For cognitive dysfunction, cholinester-
syndromes, treatment of DLB is symp- ase inhibitors such as rivastigmine and
tomatic and involves coordination of donepezil have been used, and some
caregivers and allied health personnel. evidence supports improvements in
Treatment of dementia-related psy- cognition.57 Memantine also may pro-
chosis in parkinsonisms often requires vide mild benefit.58
balancing use of dopaminergic medi-
cations, such as levodopa, and use of CONCLUSION
antipsychotics. Levodopa is usually The approach to atypical parkinsonian
beneficial for parkinsonism but in syndromes requires careful attention to
some cases can exacerbate psychosis details of the patient’s history, such as
and hallucinations. Conversely, reduc- mode of presentation and disease pro-
tion in levodopa dose, particularly at gression, as well as identification of
night, may benefit hallucinations, agi- signs and symptoms that may provide
tation, and psychosis. Dopaminergic clues to diagnosis. Growing recognition
agonists are typically not used or are of the clinical heterogeneity, overlap,
weaned, and nondopaminergic agents and varied phenotypes of these syn-
such as monoamine oxidase inhibitors dromes may further aid in diagnosis
(MAOIs), amantadine, and anticholin- and will be particularly important as
ergics are typically avoided due to their novel therapies emerge. Our increasing
potential to worsen cognition and psy- understanding of the clinicopathologic
chosis. If not improved with medication correlates for these syndromes will aid
changes, hallucinations and psychosis in the development of diagnostic bio-
are often treated with atypical antipsy- markers and will likely bolster efforts
chotics. Of note, use of antipsychotics aimed at the development of disease-
for dementia-related psychosis carries modifying therapeutics.
risk of increased mortality (mainly car-
diovascular or infectious) and should be VIDEO LEGENDS
used judiciously. Although quetiapine is Supplemental Digital
often the drug chosen by many clinicians Content 5-1
and may be helpful in mild cases, Progressive supranuclear palsyY
studies have not supported its efficacy, parkinsonism. Video shows the 65-year-
particularly for control of hallucinations. old man in Case 5-1 demonstrating
In contrast, clozapine has proven efficacy hypophonic and dysarthric speech with
but requires frequent routine blood preserved language and cognition. His
(Minneap Minn) 2016;22(4):1117–1142 www.ContinuumJournal.com 1139

Copyright © American Academy of Neurology. Unauthorized reproduction of this article is prohibited.


Atypical Parkinsonian Syndromes

facial expression is masked, and he tends Psychiatry 1989;(suppl):78Y89. doi:10.1136/


jnnp.52.Suppl.78.
to squint. Vertical saccades (upgaze and
3. Richardson JC, Steele J, Olszewski J.
downgaze) are reduced, and horizontal Supranuclear ophthalmoplegia, pseudobulbar
saccades are slowed. Off levodopa he palsy, nuchal dystonia and dementia. A clinical
demonstrates moderate bradykinesia report on eight cases of ‘‘heterogenous
of hand movements, finger tapping, system degeneration.’’ Trans Am Neurol Assoc
1963;88:25Y29.
and rigidity in the limbs, on the right
4. Williams DR, Watt HC, Lees AJ. Predictors of
more than the left. His gait is stooped falls and fractures in bradykinetic rigid
and slow, and his stance is narrowed syndromes: a retrospective study. J Neurol
and unstable with a tendency to hold Neurosurg Psychiatry 2006;77(4):468Y473.
doi:10.1136/jnnp.2005.074070.
onto the examiner or walls. Pull test
reveals minimal to no compensation. 5. Batla A, Nehru R, Vijay T. Vertical wrinkling
of the forehead or procerus sign in
He would have fallen if not caught. progressive supranuclear palsy. J Neurol Sci
2010;298(1-2):148Y149. doi:10.1016/
links.lww.com/CONT/A178 j.jns.2010.08.010.
B 2016 American Academy of Neurology.
6. Litvan I, Mega MS, Cummings JL, Fairbanks L.
Neuropsychiatric aspects of progressive
Supplemental Digital supranuclear palsy. Neurology 1996;
Content 5-2 47(5):1184Y1189. doi:10.1212/WNL.47.5.1184.
Corticobasal degeneration. Video 7. Strowd RE, Cartwright MS, Okun MS, et al.
Pseudobulbar affect: prevalence and quality
shows the 80-year-old man in Case 5-2 of life impact in movement disorders.
demonstrating asymmetric limb dys- J Neurol 2010;257(8):1382Y1387. doi:10.1007/
function, rigidity, bradykinesia, dysto- s00415-010-5550-3.
nia, apraxia, and cortical sensory deficits 8. Litvan I, Hauw JJ, Bartko JJ, et al. Validity
consistent with probable corticobasal and reliability of the preliminary NINDS
neuropathologic criteria for progressive
degeneration. supranuclear palsy and related disorders.
links.lww.com/CONT/A204 J Neuropathol Exp Neurol 1996;55(1):97Y105.

B 2016 American Academy of Neurology. 9. Kanazawa M, Shimohata T, Toyoshima Y,


et al. Cerebellar involvement in progressive
supranuclear palsy: a clinicopathological
Supplemental Digital study. Mov Disord 2009;24(9):1312Y1318.
Content 5-3 doi:10.1002/mds.22583.
Multiple system atrophyYparkinsonism. 10. Respondek G, Stamelou M, Kurz C, et al.
The phenotypic spectrum of progressive
Video shows the 66-year-old man in
supranuclear palsy: a retrospective multicenter
Case 5-3 demonstrating hypokinetic study of 100 definite cases. Mov Disord 2014;
dysarthric speech, jaw-opening dysto- 29(14):1758Y1766. doi:10.1002/mds.26054.
nia, atypical tremor, symmetric rigid 11. Williams DR, de Silva R, Paviour DC, et al.
bradykinesia, and early postural insta- Characteristics of two distinct clinical
bility consistent with multiple system phenotypes in pathologically proven
progressive supranuclear palsy: Richardson’s
atrophyYparkinsonism. syndrome and PSP-parkinsonism. Brain
2005;128(pt 6):1247Y1258. doi:10.1093/
links.lww.com/CONT/A179 brain/awh488.
B 2016 American Academy of Neurology.
12. Williams DR, Holton JL, Strand K, et al. Pure
akinesia with gait freezing: a third clinical
REFERENCES phenotype of progressive supranuclear
1. Jankovic J, Lang AE. Movement disorders: palsy. Mov Disord 2007;22(15):2235Y2241.
diagnosis and assessment. In: Bradley WG, doi:10.1002/mds.21698.
Daroff RB, Fenichel GM, Jankovic J, eds.
13. Williams DR, Lees AJ. Progressive
Neurology in clinical practice. 5th edition.
supranuclear palsy: clinicopathological
Philadelphia, PA: Saunders; 2008.
concepts and diagnostic challenges. Lancet
2. Quinn N. Multiple system atrophyVthe Neurol 2009;8(3):270Y279. doi:10.1016/
nature of the beast. J Neurol Neurosurg S1474-4422(09)70042-0.

1140 www.ContinuumJournal.com August 2016

Copyright © American Academy of Neurology. Unauthorized reproduction of this article is prohibited.


14. Hassan A, Parisi JE, Josephs KA. Autopsy-proven 2010;74(14):e60. doi:10.1212/
progressive supranuclear palsy presenting as WNL.0b013e3181d7d871.
behavioral variant frontotemporal dementia.
26. Ariza M, Kolb HC, Moechars D, et al.
Neurocase 2011;18(6):478Y488. doi:10.1080/
Tau positron emission tomography (PET)
13554794.2011.627345.
imaging: past, present, and future. J Med
15. Donker Kaat L, Boon AJ, Azmani A, et al. Chem 2015;58(11):4365Y4382.
Familial aggregation of parkinsonism in doi:10.1021/jm5017544.
progressive supranuclear palsy. Neurology
27. Norris FH, Panitch HS, Denys EH, et al.
2009;73(2):98Y105. doi:10.1212/
The treatment of subacute sclerosing
WNL.0b013e3181a92bcc.
panencephalitis with interferon: a case
16. King A, Curran O, Al-Sarraj S. Atypical report. J Neurol 1986;233(2):102Y107.
progressive supranuclear palsy presenting as doi:10.1007/BF00313855.
primary lateral sclerosis. J Neurol Sci 2013;
28. Stamelou M, Reuss A, Pilatus U, et al.
329(1Y2):69. doi:10.1016/j.jns.2013.03.015.
Short-term effects of coenzyme Q10 in
17. Nagao S, Yokota O, Nanba R, et al. progressive supranuclear palsy: a randomized,
Progressive supranuclear palsy presenting as placebo-controlled trial. Mov Disord
primary lateral sclerosis but lacking 2008;23(7):942Y949. doi:10.1002/mds.22023.
parkinsonism, gaze palsy, aphasia, or
29. Cotter C, Armytage T, Crimmins D. The use
dementia. J Neurol Sci 2012;323(1Y2):147Y153.
of zolpidem in the treatment of progressive
doi:10.1016/j.jns.2012.09.005.
supranuclear palsy. J Clin Neurosci
18. Dickson DW, Ahmed Z, Algom AA, et al. 2010;17(3):385Y386. doi:10.1016/
Neuropathology of variants of progressive j.jocn.2009.05.038.
supranuclear palsy. Curr Opin Neurol
30. Wenning GK, Litvan I, Jankovic J, et al.
2010;23(4):394Y400. doi:10.1097/
Natural history and survival of 14 patients
WCO.0b013e32833be924.
with corticobasal degeneration confirmed
19. Jellinger K. Cerebellar involvement in at postmortem examination. J Neurol
progressive supranuclear palsy. Mov Disord Neurosurg Psychiatry 1998;64(2):184Y189.
2010;25(8):1104Y1105. doi:10.1002/mds.23045. doi:10.1136/jnnp.64.2.184.
20. Stamelou M, de Silva R, Arias- Carrión O, 31. Litvan I, Agid Y, Goetz C, et al. Accuracy
et al. Rational therapeutic approaches to of the clinical diagnosis of corticobasal
progressive supranuclear palsy. Brain degeneration: a clinicopathologic study.
2010;133(pt 6):1578Y1590. doi:10.1093/ Neurology 1997;48(1):119Y125. doi:10.1212/
brain/awq115. WNL.48.1.119.
21. Höglinger GU, Melhem NM, Dickson DW, 32. Graham NL, Bak TH, Hodges JR. Corticobasal
et al. Identification of common variants degeneration as a cognitive disorder. Mov
influencing risk of the tauopathy Disord 2003;18(11):1224Y1232. doi:10.1002/
progressive supranuclear palsy. Nat Genet mds.10536.
2011;43(7):699Y705. doi:10.1038/ng.859.
33. Pillon B, Gouider-Khouja N, Deweer B, et al.
22. Oba H, Yagishita A, Terada H, et al. New Neuropsychological pattern of striatonigral
and reliable MRI diagnosis for progressive degeneration: comparison with Parkinson’s
supranuclear palsy. Neurology disease and progressive supranuclear
2005;64(12):2050Y2055. doi:10.1212/ palsy. J Neurol Neurosurg Psychiatry 1995;
01.WNL.0000165960.04422.D0. 58(2):174Y179. doi:10.1136/jnnp.58.2.174.
23. Longoni G, Agosta F, Kostić VS, et al. MRI 34. Armstrong MJ, Litvan I, Lang AE, et al.
measurements of brainstem structures in Criteria for the diagnosis of corticobasal
patients with Richardson’s syndrome, degeneration. Neurology 2013;80(5):
progressive supranuclear palsy-parkinsonism, 496Y503. doi:10.1212/WNL.0b013e31827f0fd1.
and Parkinson’s disease. Mov Disord
35. Dickson DW, Bergeron C, Chin SS, et al.
2011;26(2):247Y255. doi:10.1002/mds.23293.
Office of rare diseases neuropathologic
24. Knake S, Belke M, Menzler K, et al. In vivo criteria for corticobasal degeneration.
demonstration of microstructural brain J Neuropathol Exp Neurol
pathology in progressive supranuclear palsy: 2002;61(11):935Y946.
a DTI study using TBSS. Mov Disord 2010;25(9):
36. Kouri N, Whitwell JL, Josephs KA, et al.
1232Y1238. doi:10.1002/mds.23054.
Corticobasal degeneration: a pathologically
25. Renard D, Collombier L, Castelnovo G, distinct 4R tauopathy. Nat Rev Neurol
Labauge P. Teaching NeuroImages: FDG-PET 2011;7(5):263Y272. doi:10.1038/
in progressive supranuclear palsy. Neurology nrneurol.2011.43.

(Minneap Minn) 2016;22(4):1117–1142 www.ContinuumJournal.com 1141

Copyright © American Academy of Neurology. Unauthorized reproduction of this article is prohibited.


Atypical Parkinsonian Syndromes

37. Whitwell JL, Jack CR Jr, Boeve BF, et al. 65(8):1074Y1080. doi:10.1001/
Imaging correlates of pathology in archneur.65.8.1074.
corticobasal syndrome. Neurology
49. Hurtig HI, Trojanowski JQ, Galvin J, et al.
2010;75(21):1879Y1887. doi:10.1212/
Alpha-synuclein cortical Lewy bodies
WNL.0b013e3181feb2e8.
correlate with dementia in Parkinson’s
38. Murray ME, Kouri N, Lin WL, et al. disease. Neurology 2000;54(10):1916Y1921.
Clinicopathologic assessment and imaging doi:10.1212/WNL.54.10.1916.
of tauopathies in neurodegenerative
50. Braak H, Rüb U, Jansen Steur EN, et al.
dementias. Alzheimers Res Ther 2014;
Cognitive status correlates with
6(1):1. doi:10.1186/alzrt231.
neuropathologic stage in Parkinson disease.
39. Stefanova N, Bücke P, Duerr S, Wenning GK. Neurology 2005;64(8):1404Y1410.
Multiple system atrophy: an update. Lancet doi:10.1212/01.WNL.0000158422.41380.82.
Neurol 2009;8(12):1172Y1178. doi:10.1016/
51. Sinha N, Firbank M, O’Brien JT. Biomarkers
S1474-4422(09)70288-1.
in dementia with Lewy bodies: a review.
40. Köllensperger M, Geser F, Seppi K, et al. Int J Geriatr Psychiatry 2012;27(5):443Y453.
Red flags for multiple system atrophy. Mov doi:10.1002/gps.2749.
Disord 2008;23(8):1093Y1099. doi:10.1002/
52. Graff-Radford J, Murray ME, Lowe VJ,
mds.21992.
et al. Dementia with Lewy bodies: basis
41. Jecmenica-Lukic M, Poewe W, Tolosa E, of cingulate island sign. Neurology
Wenning GK. Premotor signs and symptoms 2014;83(9):801Y809. doi: 10.1212/
of multiple system atrophy. Lancet Neurol WNL.0000000000000734.
2012;11(4):361Y368. doi:10.1016/ 53. Gomperts SN, Rentz DM, Moran E,
S1474-4422(12)70022-4. et al. Imaging amyloid deposition in Lewy
42. Köllensperger M, Geser F, Ndayisaba JP, et al. body diseases. Neurology 2008;71(12):
Presentation, diagnosis, and management of 903Y910. doi:10.1212/01.wnl.
multiple system atrophy in Europe: final 0000326146.60732.d6.
analysis of the European multiple system 54. Mollenhauer B, Bibl M, Trenkwalder C, et al.
atrophy registry. Mov Disord 2010;25(15): Follow-up investigations in cerebrospinal
2604Y2612. doi:10.1002/mds.23192. fluid of patients with dementia with Lewy
43. Brown RG, Lacomblez L, Landwehrmeyer BG, bodies and Alzheimer’s disease. J Neural
et al. Cognitive impairment in patients with Transm (Vienna) 2005;112(7):933Y948.
multiple system atrophy and progressive doi:10.1007/s00702-004-0235-7.
supranuclear palsy. Brain 2010;133(pt 8): 55. Hack N, Fayad SM, Monari EH, et al.
2382Y2393. doi:10.1093/brain/awq158. An eight-year clinic experience with
44. Ahmed Z, Asi YT, Sailer A, et al. The clozapine use in a Parkinson’s disease
neuropathology, pathophysiology and clinic setting. PloS One 2014;9(3):e91545.
genetics of multiple system atrophy. doi:10.1371/journal.pone.0091545.
Neuropathol Appl Neurobiol 2012;38(1):
56. Cummings J, Isaacson S, Mills R, et al.
4Y24. doi:10.1111/j.1365-2990.2011.01234.x.
Pimavanserin for patients with Parkinson’s
45. Wenning GK, Jellinger KA. The role of disease psychosis: a randomised,
alpha-synuclein in the pathogenesis of placebo-controlled phase 3 trial. Lancet
multiple system atrophy. Acta Neuropathol 2014;383(9916):533Y540. doi:10.1016/
2005;109(2):129Y140. doi:10.1007/ S0140-6736(13)62106-6.
s00401-004-0935-y.
57. Rolinski M, Fox C, Maidment I, McShane R.
46. Perju-Dumbrava LD, Kovacs GG, Pirker S, Cholinesterase inhibitors for dementia with
et al. Dopamine transporter imaging in Lewy bodies, Parkinson’s disease dementia
autopsy-confirmed Parkinson’s disease and and cognitive impairment in Parkinson’s
multiple system atrophy. Mov Disord disease. Cochrane Database Syst Rev
2012;27(1):65Y71. doi:10.1002/mds.24000. 2012;3:CD006504. doi:10.1002/
14651858.CD006504.pub2.
47. McKeith IG, Dickson DW, Lowe J, et al.
Diagnosis and management of dementia with
58. Emre M, Tsolaki M, Bonuccelli U, et al.
Lewy bodies: third report of the DLB Consortium.
Memantine for patients with Parkinson’s
Neurology 2005;65(12):1863Y1872. doi:10.1212/
disease dementia or dementia with Lewy
01.wnl.0000187889.17253.b1.
bodies: a randomised, double-blind,
48. DelleDonne A, Klos KJ, Fujishiro H, et al. placebo-controlled trial. Lancet Neurol
Incidental Lewy body disease and preclinical 2010;9(10):969Y977. doi:10.1016/
Parkinson disease. Arch Neurol 2008; S1474-4422(10)70194-0.

1142 www.ContinuumJournal.com August 2016

Copyright © American Academy of Neurology. Unauthorized reproduction of this article is prohibited.


Review Article

Diagnosis and
Address correspondence to
Dr Elan D. Louis, Yale School
of Medicine, Department of
Neurology, LCI 710, 15 York St,

Management of Tremor PO Box 20818, New Haven,


CT 06520-8018,
elan.louis@yale.edu.
Elan D. Louis, MD, MS, FAAN Relationship Disclosure:
Dr Louis reports no disclosure.
Unlabeled Use of
Products/Investigational
ABSTRACT Use Disclosure:
Dr Louis discusses the
Purpose of Review: Tremor, which is a rhythmic oscillation of a body part, is among unlabeled/investigational use
the most common involuntary movements. Rhythmic oscillations may manifest in a of acetazolamide, alprazolam,
variety of ways; as a result, a rich clinical phenomenology surrounds tremor. For this baclofen, carbamazepine,
clonazepam, cyclophosphamide,
reason, diagnosing tremor disorders can be particularly challenging. The aim of this ethosuximide, gabapentin,
article is to provide the reader with a straightforward approach to the diagnosis and phenobarbital, phenytoin,
management of patients with tremor. pregabalin, primidone, and
topiramate for the treatment of
Recent Findings: Scientific understanding of the pathophysiologic basis of tremor tremor as well as carbidopa/
disorders has grown considerably in recent years with the use of a broad range of levodopa for the treatment of
neuroimaging approaches and rigorous, controlled postmortem studies. The basal orthostatic tremor and
trihexyphenidyl for the
ganglia and cerebellum are structures that seem to play a prominent role. treatment of dystonic tremor.
Summary: The diagnosis of tremor disorders is challenging. The approach to tremor * 2016 American Academy
involves a history and a neurologic examination that is focused on the nuances of of Neurology.
tremor phenomenology, of which there are many. The evaluation should begin with a
tremor history and a focused neurologic examination. The examination should attend
to the many subtleties of tremor phenomenology. Among other things, the history and
examination are used to establish whether the main type of tremor is an action tremor
(ie, postural, kinetic, or intention tremor) or a resting tremor. The clinician should then
formulate two sets of differential diagnoses: disorders in which action tremor is the
predominant tremor versus those in which resting tremor is the main tremor. Among
the most common of the former type are essential tremor, enhanced physiologic
tremor, drug-induced tremor, dystonic tremor, orthostatic tremor, and cerebellar
tremor. Parkinson disease is the most common form of resting tremor, along with
drug-induced resting tremor. This article details the clinical features of each of these as
well as other tremor disorders.

Continuum (Minneap Minn) 2016;22(4):1143–1158.

INTRODUCTION activating the arms during movement),


Tremor, which is the rhythmic oscilla- the brain region affected (cerebellum,
tion of a body part, is among the most midbrain), the presence of associated
common forms of involuntary move- medical conditions (eg, hyperthyroidism),
ment. These oscillations may manifest and the presence of associated neurologic
in a broad variety of ways. Given this conditions (eg, dystonia, Parkinson dis-
richness of clinical phenomenology, ease [PD]). As a result, a sizable nomen-
tremor can be classified in numerous clature surrounds tremor, and this can Supplemental digital content:
Videos accompanying this ar-
ways. For example, tremors can be clas- be daunting and confusing to the clini- ticle are cited in the text as
sified based on their topographic dis- cian. The aim of this article is to pro- Supplemental Digital Content.
Videos may be accessed by
tribution (eg, arm, neck, chin, or voice), vide the reader with a straightforward clicking on links provided in the
their frequency in Hz, their activation approach to the diagnosis and manage- HTML, PDF, and app versions
of this article; the URLs are pro-
condition (ie, whether the tremor is ment of the patient with tremor. This vided in the print version. Video
brought on by resting the arms versus approach includes a medical history and legends begin on page 1155.

Continuum (Minneap Minn) 2016;22(4):1143–1158 www.ContinuumJournal.com 1143

Copyright © American Academy of Neurology. Unauthorized reproduction of this article is prohibited.


Diagnosing Tremor

KEY POINT
h Tremors are rhythmic a neurologic examination followed by focused neurologic examination. First,
oscillations of a diagnosis and, ultimately, treatment. the examiner should try to elicit postural
body part. tremor in the arms (ie, tremor observed
APPROACH TO THE PATIENT during sustained arm extension). The
WITH TREMOR examiner should assess the following
The approach to the patient with tremor, features about the postural tremor:
as with any neurologic patient, begins & Are the movements regularly
with a history and then includes a neu- recurrent and oscillatory?
rologic examination. & Across which joints are the
movements occurring and in
Medical History what directions?
The first step is to elicit a medical his- & Are the tremors in each arm
tory. This history should include an occurring synchronously (ie,
initial set of questions used to determine in phase)?
whether the patient has an action or a & Does the postural tremor have a
resting tremor. Open-ended questions reemergent quality (ie, is it
such as, ‘‘What type of tremor do you initially absent)?
have?’’ may be followed by a series of & Is the tremor accompanied by
more specific questions about tremor abnormal postures in the same
such as, ‘‘Do you have your tremor when limb (eg, flexed posturing or
you are holding a cup?’’ This is then dystonic postures)?
followed by additional questions that & Is the tremor characterized by
elicit information on the following: distractibility (ie, a decrease or
& The body regions that are involved cessation of tremor when
& The limb and body positions or volitionally performing a task [eg,
the specific maneuvers that elicit finger tapping with opposite
or suppress the tremor hand]), entrainment (ie, the tremor
& The age and time course of onset may be brought into a specific
and evolution of symptoms over time rhythm), or suggestibility (ie, the
& Whether the patient is aware or tremor may be induced with
unaware of the tremor certain stimuli)?
& The presence of any accompanying Next, the examiner should try to eli-
pulling sensations or pain cit kinetic tremor (ie, tremor that oc-
& The presence of associated curs during movement) by asking the
movements or additional patient to perform the finger-nose-
neurologic symptoms finger maneuver, draw spirals or write
& The current use of potential a sentence, pour water between cups,
tremor-inducing medications
or drink from a cup. The examiner
& The current use of tremor- should assess the following features:
exacerbating substances
(eg, caffeine) & Does the tremor have an
& Experiences of diarrhea, weight intentional component (ie, does it
loss, or heat intolerance worsen as the limb approaches
& Family history of tremor in first- or a target)?
second-degree relatives & Are dystonic postures present (eg,
during the finger-nose-finger
Neurologic Examination maneuver)?
After obtaining a patient’s medical his- & What is the relative severity of
tory, the next step is a detailed and kinetic tremor versus postural

1144 www.ContinuumJournal.com August 2016

Copyright © American Academy of Neurology. Unauthorized reproduction of this article is prohibited.


KEY POINTS
tremor (or resting tremor if this is tremor has an intentional component, h The cardinal feature of
also present)? worsening during the finger-nose-finger essential tremor is
This should be followed by an as- maneuver as the patient approaches kinetic tremor.
sessment of tremor at rest in any of the target (ie, the finger or the nose)
h In essential tremor, the
the limbs (while the patient is seated, (Supplemental Digital Content 6-1, amplitude of kinetic
standing, walking, and lying down) as links.lww.com/CONT/A180). Inten- tremor is generally
well as an assessment of cranial tremors: tion tremor is not limited to the arms greater than that of
head (ie, neck) (while seated and while and is detectable in the neck in 10% postural tremor.
lying), jaw (with mouth closed and of patients when the patient’s head h Resting tremor may
then while open), face, chin, tongue, and approaches a target (eg, as the pa- occur in advanced
voice (during sustained phonation). tient lowers his or her head to meet a essential tremor but is
cup or a spoon), and it can be restricted to the arms
CONSTRUCTING A DIFFERENTIAL observed in the legs as well.2 rather than the legs.
DIAGNOSIS Aside from kinetic tremor, postural
tremor often occurs in patients with
The history and physical examination
essential tremor, although it is not the
are used to establish whether the main
predominant tremor. This tremor is
type of tremor is an action tremor (ie,
generally worse in the wing-beat posi-
postural, kinetic, or intention tremor)
tion (ie, while the arms are extended
or a resting tremor. This point of diver-
and facing one another in the midline)
gence is useful in order to ascertain two
than when the arms are held straight
sets of differential diagnoses: those in
in front of the patient. The tremor in
which action tremor is the predom-
the two arms is generally out of phase,
inant form of tremor versus those in
creating a seesaw effect when the arms
which resting tremor is the main tremor.
are held in a wing-beat position. This
accounts for the observation that func-
ACTION TREMOR tionality may improve when two hands
The following sections discuss the are used, rather than one hand (eg,
more commonly encountered disor- while holding a cup). The postural
ders as well as those that, while less tremor of essential tremor is generally
common, have a particularly distinc- greatest in amplitude at the wrist joint,
tive set of clinical features. rather than more proximal or distal
joints, and generally involves wrist
Essential Tremor flexion-extension rather than rotation-
The cardinal feature of essential tremor supination. As a general rule, the am-
is kinetic tremor, which may be ob- plitude of kinetic tremor exceeds that
served during a variety of activities on of postural tremor, and the converse
neurologic examination (eg, spiral pattern should raise serious questions
drawing, pouring water between two about the essential tremor diagnosis.3
cups, finger-nose-finger maneuver) Tremor at rest, without other cardinal
(Supplemental Digital Content 6-1, features of parkinsonism such as rigid-
links.lww.com/CONT/A180). In gen- ity or bradykinesia, occurs in approx-
eral, the tremor is mildly asymmetric, imately 20% of patients with essential
affecting one arm more than the other, tremor attending a specialty clinic but,
with an approximate 30% difference in contrast to that of PD, is a late fea-
on average between sides. In approx- ture and has only been observed in the
imately 5% of patients, the tremor is arm (ie, does not involve the leg).4
markedly asymmetric or unilateral.1 In Aside from tremor, another motor
approximately 50% of patients, the feature of essential tremor is gait ataxia,
Continuum (Minneap Minn) 2016;22(4):1143–1158 www.ContinuumJournal.com 1145

Copyright © American Academy of Neurology. Unauthorized reproduction of this article is prohibited.


Diagnosing Tremor

KEY POINTS
h Given its progressive which is in excess of that seen in into a more complex, multidirectional
nature and propensity similarly aged controls. In most pa- tremor. Unless it is severe, the neck
to result in functional tients, this is mild, although in some tremor, which is a postural tremor,
disabilities, the term it may be of moderate severity. Some should subside and then resolve when
benign essential tremor evidence indicates that this gait ataxia the patient is lying on his or her back
is no longer favored. is more pronounced in patients who with the head fully at rest. Isolated
h The jaw tremor of have midline cranial tremors (eg, tremors neck tremor, with minimal or no ac-
essential tremor typically of the neck, jaw, and voice).5 companying arm tremor, is extremely
occurs when the Initially in essential tremor, the rare and should raise suspicion that
mouth is open rather tremor may be mild and asymptomatic the diagnosis is dystonia rather than
than closed. and may not worsen for many years. essential tremor. A curious feature of
However, in most individuals, the the neck tremor of essential tremor is
tremor worsens with time. Several pat- that patients are often unaware of its
terns of progression have been de- presence, particularly when it is mild
scribed. The two most common are: (Case 6-1). The presence of dystonic
(1) late-life onset (after the age of 60) posturing in essential tremor cases is
with progression and (2) early-life controversial, although it is likely that
onset (before the age of 40) with mild, the presence of a mild dystonic posture
stable tremor for many years that in the tremulous arm in some cases
then has a late-life progression. The does not preclude a diagnosis of essen-
least common pattern is that of early- tial tremor, especially when the dys-
life onset with marked worsening over tonic posture is a late finding in a
the ensuing years to decade.6 Few case with long-standing and severe
prospective, longitudinal natural his- essential tremor.
tory studies exist, but the best esti- Thirty percent to 50% of ‘‘essential
mates of rate of change indicate that tremor’’ cases are misdiagnosed, with
arm tremor worsens by 2% to 5% per many of these patients having PD or
year.7 With the progression of time, dystonia rather than essential tremor.
the tremor tends to spread beyond Differentiation from PD may be made
the hands and arms so that patients readily, however, by the absence of ri-
develop cranial tremors (neck, voice, gidity, the absence of other signs of
jaw). These are particularly prevalent parkinsonism (eg, hypomimia), the ab-
in women with essential tremor, sence of bradykinesia that is in excess
among whom the risk of neck tremor of age (ie, absence of slowness that is
is several times higher than that of men above and beyond that seen in normal
with essential tremor (Supplemental aging), and the absence of bradykinesia
Digital Content 6-1, links.lww.com/ accompanied by decrement (ie, absence
CONT/A180). The prevalence of neck of a sequential decrement in amplitude
tremor is the highest of these cranial during finger taps).8 The characteristics
tremors, with voice tremor being less of the tremor are also important in dis-
so, and jaw tremor being even less tinguishing a patient with essential
common. The jaw tremor is more often tremor from one with PD. The presence
one that is seen when the mouth is of isolated postural tremor (ie, postural
held open or during speech rather tremor with minimal kinetic tremor),
than at rest; the latter is more a feature a postural tremor predominantly in-
of PD-related jaw tremor. Neck tremor volving the metacarpophalangeal
often begins as a unidirectional tremorV joints rather than the wrist, or a postural
‘‘no no’’ (ie, horizontal) or ‘‘yes yes’’ tremor characterized by greater wrist ro-
(ie, vertical)Vbut with time can evolve tation than wrist flexion and extension,
1146 www.ContinuumJournal.com August 2016

Copyright © American Academy of Neurology. Unauthorized reproduction of this article is prohibited.


KEY POINTS

Case 6-1 h Flexed posturing of the


hand during arm
A 72-year-old woman presented with a tremor that affected her hands
extension is an early sign
when she wrote and when she held eating utensils. The tremor had been
of Parkinson disease.
present for many years and perhaps even as early as her midtwenties, but
she felt it had begun to worsen considerably during the past 4 years. Her h Neck tremor should
mother had had a similar tremor, as did one of her brothers. Although she not occur in patients
did not notice it herself, her husband told her that for the past year, he with enhanced
had sometimes noticed a head tremor when he sat across from her in a physiologic tremor.
restaurant. On examination, the tremor was noticeable when she
performed the finger-nose-finger maneuver and worsened considerably as
she approached her nose. There was some postural tremor as well, but it
was less severe than the kinetic tremor. There was an occasional horizontal
head tremor, which was not present while she lay down. She was treated
with propranolol but could not tolerate more than 80 mg/d without
becoming bradycardic (ie, heart rate of less than 60 beats/min). This dose
resulted in a 20% reduction in her arm tremor.
Comment. This case illustrates the progressive and often familial nature
of essential tremor, highlights the fact that the central defining clinical
feature of essential tremor is kinetic tremor, and exemplifies the presence
of head tremor in some cases, particularly in women.

are all indicators that the likely diagno- essential tremor from the spinocere-
sis is emerging PD rather than essential bellar ataxias. Hyperthyroidism can be
tremor. Reemergent tremor is a pos- assessed by clinical history (eg, diar-
tural tremor that commences after a rhea, weight loss, or heat intoler-
brief latency of several seconds, rather ance) as can the use of medications
than immediately, and is a feature of PD. (eg, lithium, valproate) or other sub-
Flexed posturing of the hand during stances (eg, caffeine, tobacco) that
arm extension is an additional marker produce or exacerbate tremor. A dif-
of parkinsonism. Dystonia in the arm is ficult differential is between a mild
characterized by dystonic posturing case of essential tremor and an indi-
(eg, spooning of the fingers, referring vidual with enhanced physiologic
to the tendency during arm extension tremor, although the presence of neck
to flex the wrist and hyperextend the tremor should exclude the latter.
metacarpophalangeal and phalangeal Computerized tremor analysis, with
joints) (Supplemental Digital Content inertial loading (ie, placing a weight
6-1, links.lww.com/CONT/A180) or the on the hand), can assist with this
presence of a tremor that is neither differential. In patients with a tremor
rhythmic nor oscillatory. The possibility of central origin (eg, essential tremor),
of neck dystonia should be assessed and the primary tremor frequency should
is characterized by head tilt or rotation not change with inertial loading. Other
with head tremor, hypertrophy of the features that support the essential
sternocleidomastoid muscle, the pres- tremor diagnosis are the presence of
ence of a tremor null point, or a sensory essential tremor in one or more first-
trick by history (ie, a maneuver such as degree relatives. Reduction in tremor
touching the chin that lessens the with ethanol use is often used as a
tremor). Absence of cerebellar speech diagnostic marker; however, this is not
(ie, either scanning or dysarthric) and very specific and of limited utility.
absence of nystagmus distinguishes Patients with most tremor disorders
Continuum (Minneap Minn) 2016;22(4):1143–1158 www.ContinuumJournal.com 1147

Copyright © American Academy of Neurology. Unauthorized reproduction of this article is prohibited.


Diagnosing Tremor

KEY POINT
h With enhanced often experience a reduction in tremor agents that have been used include
physiologic tremor, no following ethanol consumption. topiramate, gabapentin, and benzodi-
intentional component The tremor of essential tremor may azepines (alprazolam or clonazepam)
occurs on the be severe enough to result in embar- (Table 6-1). High-frequency thalamic
finger-nose- rassment and functional disability, and stimulation (ie, deep brain stimulation
finger maneuver. these are the main motivations for [DBS]) markedly reduces the severity
treatment.9 Beta-blockers (especially of the tremor, as does thalamotomy, in
propranolol) and primidone, alone or patients with severe pharmacologically
in combination, are the most effective refractory tremor.
pharmacologic therapies, although many
patients choose to discontinue these Enhanced Physiologic Tremor
medications due to their limited effi- Enhanced physiologic tremor is an ac-
cacy (Table 6-1). Propranolol has been tion tremor of the hands that occurs
used in doses up to 320 mg/d, although to some extent in all people.10 The pos-
doses in excess of 100 mg/d are rarely tural and kinetic components are gen-
tolerated in the elderly, with the main erally several Hz faster and generally
issue being slowed heart rate. Asthma have a lower amplitude than those seen
is a relative contraindication to the use in essential tremor,10 with the caveat
of propranolol but is not an absolute that at disease onset, patients with es-
contraindication and must be assessed sential tremor have a low-amplitude
on a case-by-case basis. Primidone is tremor that may be difficult to dis-
given in doses up to 750 mg/d, although tinguish from enhanced physiologic
lower doses are often effective. An tremor.11 Unlike essential tremor, no
acute nausea and ataxia is observed in intentional component occurs on the
approximately 25% of patients who are finger-nose-finger maneuver. Voice and
prescribed this medication, and pre- limb tremor may be present but not
loading the patient with phenobarbital head (ie, neck) tremor. Cogwheeling
(eg, 30 mg 2 times a day for 3 days) is may be present, but it is not accom-
one method that is used to avoid this panied by rigidity. On quantitative
unwanted side effect. These drugs re- computerized tremor analysis, inertial
sult in a mild to moderate reduction in loading reveals a pattern that is con-
the amplitude of tremor in 30% to 70% sistent with a peripherally generated
of patients, but they do not abolish rather than a centrally generated tremor
it unless the tremor is mild. Other (ie, a reduction in the primary tremor
frequency occurs with inertial loading).
Medications and Treatment should begin with reas-
TABLE 6-1
Dosage Ranges for surance that the patient does not have
Essential Tremor essential tremor or PD. Many of the
individuals who present to a clinician
Medication Dosage Range do so because they are also anxious.
Propranolol 10Y320 mg/d Beta-blockers, at low dose (eg, pro-
Primidone 25Y750 mg/d
pranolol up to 60 mg/d or used in
a 10 mg to 60 mg dose on an as-
Topiramate 25Y300 mg/d needed basis), and benzodiazepines
Gabapentin 100Y1800 mg/d may be effective.
Alprazolam 0.125Y3 mg/d
Drug-Induced Action Tremor
Clonazepam 0.5Y4 mg/d
A variety of medications may produce
or exacerbate action tremor, and the
1148 www.ContinuumJournal.com August 2016

Copyright © American Academy of Neurology. Unauthorized reproduction of this article is prohibited.


KEY POINTS
severity of this tremor can range from who have been diagnosed with dystonia h While limb tremor may
mild to severe.10,12 These medica- may occur both in limbs that exhibit be present, head tremor
tions are inclusive of but not limited dystonic postures or movements as should not be a feature
to bronchodilators, lithium, valproic well as limbs that do not exhibit these. of drug-induced
acid, methylphenidate, prednisone, Furthermore, the tremor may occur in action tremor.
and pseudoephedrine. A number of limbs that appear to be at rest as well h The tremor in dystonia is
features help to differentiate drug- as those that are active (ie, during sus- often neither rhythmic
induced action tremor from other tained posture or during movement). nor oscillatory.
forms of action tremor. First, by history, What complicates matters is that, as
the onset of tremor follows the use noted previously, patients with long-
of the putatively causative medication. standing and clinically advanced essen-
One caveat is that the onset may not tial tremor may develop mild dystonic
be immediate; it may occur gradually posturing of the hand during arm ex-
over several months. Second, there may tension. Hence, considerable debate
be a dose-response relationship such exists as to where dystonia as a disease
that higher doses of medications are ends and where essential tremor as a
associated with greater tremor ampli- disease begins and vice versa. With all
tude. Third, discontinuing the medi- this in mind, when considering a par-
cation should result in the complete ticular patient, several issues should be
resolution of tremor.12 Furthermore, taken into consideration. First, what are
while limb tremor may be present, head the features of the tremor itself? Sec-
tremor should not be a feature of ond, does the patient exhibit dystonic
drug-induced action tremor. Finally, postures or movements? On closer in-
with a stable medication dose, the spection, the tremor itself in patients
tremor should not worsen progres- with dystonia may be neither rhythmic
sively, in contrast to the tremor of es- (ie, it is not regularly recurrent) nor
sential tremor or PD. oscillatory (ie, it does not rotate around
The mechanism that underlies these a central plane); these features distin-
drug-induced action tremors is not guish the tremor from that of essential
fully known, although such tremors are tremor. In terms of the second ques-
thought to represent a form of enhanced tion, patients with dystonia may have
physiologic tremor.10 In addition, some a variety of sustained postures or twist-
evidence suggests that drug-induced ing movements involving the orbicu-
action tremor may also be mediated laris muscles, neck, and other muscles,
through central mechanisms.13 which do not occur in essential tremor.
The treatment of these tremors is Some patients with dystonia only have
to remove or lower the dose of the these postures and movements in their
causative medication. When this is hands. These patients exhibit one or
not possible, beta-blockers (eg, pro- more of a variety of dystonic postures
pranolol, 10mg/d to 320 mg/d) may during arm extension (eg, spooning of
be beneficial. the hands as defined previously [Sup-
plemental Digital Content 6-2, links.
Dystonic Tremor lww.com/CONT/A181], difficulty
A variety of tremors may occur in pa- maintaining both hands strictly parallel
tients who have been diagnosed with while outstretched in a karate-chop posi-
dystonia, and a particularly challeng- tion, thumb flexion during arm ex-
ing differential is that of essential tension, or other dystonic postures).
tremor versus dystonia with dystonic These should not occur in essential
tremor.14Y16 The tremors in patients tremor unless the disease is advanced;
Continuum (Minneap Minn) 2016;22(4):1143–1158 www.ContinuumJournal.com 1149

Copyright © American Academy of Neurology. Unauthorized reproduction of this article is prohibited.


Diagnosing Tremor

KEY POINT
h Voice tremor in patients furthermore, they should be of mild to 60 mg/d), benzodiazepines, or
with dystonia is often severity relative to the tremor itself. beta-blockers. For dystonic neck
associated with Tremor in the neck or voice is tremor, other options include IM botu-
voice breaks. another issue worth discussing. Patients linum toxin injections, selective dener-
with neck dystonia (ie, torticollis) may vation, or DBS.
also have neck tremor (Case 6-2).17
This tremor is generally neither strictly Primary Writing Tremor
rhythmic nor oscillatory and may be Primary writing tremor is a task-specific,
accompanied by twisting or tilting of kinetic hand tremor that occurs pri-
the neck, jerklike or sustained neck marily or only during writing and does
deviation, hypertrophy of neck mus- not occur or is milder during other
cles, or a symptom of pulling sensations tasks that involve the use of the
or pain in the neck. These features do hands.10,19 The current definition of
not occur in essential tremor. Also, in primary writing tremor excludes pa-
contrast to the head tremor of essential tients who have dystonic postures with
tremor, which generally resolves when hand tremor while writing (ie, dystonic
the patient lies down, dystonic head writing tremor).10 Primary writing tremor
tremor often persists while the patient has a similar frequency to that seen in
is recumbent.18 Voice tremor may also patients with essential tremor (ie, 4 Hz
be present in patients with vocal cord to 8 Hz) and is relieved by ethanol con-
dystonia (ie, spasmodic dysphonia), but sumption in 30% to 50% of cases.19 The
in contrast to the voice tremor of es- mechanisms that underlie primary writ-
sential tremor, is often associated with ing tremor are unclear, and it is debated
voice breaks or strangulated speech. whether the condition represents a
The treatment of dystonic tremor variant of essential tremor or a variant
includes the use of medications used of dystonia, and in some families all
to treat dystonia (ie, trihexyphenidyl three conditions may be present.20
up to 10 mg/d in adults, baclofen up Tremor in patients with essential tremor

Case 6-2
A 43-year-old woman presented for a neurologic consultation because of
neck pain with tremor. She stated that her neck pain had begun about
5 years previously, mainly on the left side. For the past 3 years, she had felt
a pulling sensation and increased pain in the neck region. She also
described that her head had been turning to one side and sometimes it
even felt shaky. She had not noticed any hand tremor. On examination,
there was a mild postural tremor of the left arm with a little bit of spooning
of the hand on that side. No tremor occured during the finger-nose-finger
maneuver. Her left sternocleidomastoid muscle was slightly hypertrophic,
and her head tended to preferentially turn to the right and shake
intermittently. The shakiness was irregular. This head tremor persisted even
when she lay down on the examining table. She was treated with IM
botulinum toxin injections to several neck muscles, which helped diminish all
of her symptoms, although they did not resolve completely.
Comment. This case illustrates a number of key features of dystonic
head (neck) tremor, including the fact that the tremor is often irregular
and that it often persists in the recumbent position. Furthermore, it may
be accompanied by pain or pulling sensations.

1150 www.ContinuumJournal.com August 2016

Copyright © American Academy of Neurology. Unauthorized reproduction of this article is prohibited.


KEY POINTS
may be present to a differing degree levodopa (levodopa doses of 100 mg/d h Orthostatic tremor may
during various activities; sometimes to 1000 mg/d). A large number of be more easily felt or
tremor may be most prominent during other agents have also been tried, in- auscultated than seen.
writing, which is a further challenge cluding propranolol, primidone, phe-
h In patients with diseases
when attempting to arrive at the correct nytoin, carbamazepine, ethosuximide, of the cerebellum, it is
diagnosis (ie, essential tremor versus baclofen, and acetazolamide. DBS can important to distinguish
primary writing tremor).21 This obser- provide benefit.25 tremor from dysmetria.
vation about essential tremor also raises
the question as to whether primary writ- Cerebellar Tremor
ing tremor is an overdiagnosed entity. The term cerebellar tremor has clas-
Treatment of primary writing tremor sically been used to describe a tremor
includes the use of propranolol, pri- that can occur in patients with spin-
midone, or anticholinergic medications ocerebellar ataxia as well as other clas-
(see the previous sections on essential sic disorders of the cerebellum.10 There
tremor and dystonic tremor) as well as is some overlap in clinical phenome-
the use of writing and hand orthotic nology with that which is seen in
devices. Local IM injections of botuli- patients with essential tremor (ie, a
num toxin have exhibited some benefit kinetic tremor with intentional com-
as well.22 ponent), as the latter is increasingly
being viewed as a disorder that in-
Orthostatic Tremor volves cerebellar system dysfunc-
Patients with this rare, although debil- tion.6 The classic cerebellar tremor
itating, entity often report feelings of is an action tremor and has both ki-
unsteadiness while standing.23 In addi- netic components as well as a terminal
tion, there is a clear complaint of worsening (ie, an intentional compo-
tremor while standing (ie, orthostatic nent).26 The tremor is quite slow (3 Hz
tremor) rather than while walking, to 4 Hz), and other cerebellar signs,
sitting, or lying down. Because of such as overshoot, may also be present
these symptoms, patients with ortho- on the finger-nose-finger maneuver.
static tremor typically avoid situations Moreover, the tremor may occur in
that require standing still (eg, standing several planes (ie, X, Y, and Z axes)
in lines).24 On examination, physi- during this and other maneuvers. Pa-
cians may see, feel, or be able to tients with classically defined cerebel-
auscultate a rapid (14 Hz to 16 Hz) lar tremor often have other cerebellar
fine tremor in the calves. The EMG signs, including saccadic eye movement
indicates the presence of a 14 Hz to abnormalities, dysarthric or scanning
16 Hz tremor in leg (especially calf) speech, gait ataxia, and hypotonia.10 It
muscles. A slower, larger-amplitude is important, although often difficult,
tremor may also be superimposed on to try to separate the tremor (ie, rhyth-
top of this tremor and can be more mic oscillatory movements) from prob-
disabling for the patients than the lems with force and timing of motion
faster tremor. (ie, dysmetria), as only the former gen-
The treatment for orthostatic tremor erally improves with DBS, whereas the
is challenging and rarely effective.23 latter might worsen. A number of med-
Numerous agents have been used, ications have been used to treat cere-
often to little avail. The most com- bellar tremor, although efficacy is
monly used agents have been clonaz- extremely limited.27 The most effec-
epam (0.5 mg/d to 4 mg/d), gabapentin tive treatment for cerebellar tremor
(300 mg/d to 1800 mg/d), and carbidopa/ is thalamic DBS.27
Continuum (Minneap Minn) 2016;22(4):1143–1158 www.ContinuumJournal.com 1151

Copyright © American Academy of Neurology. Unauthorized reproduction of this article is prohibited.


Diagnosing Tremor

KEY POINTS
Psychogenic Tremor The treatment of psychogenic trem-
h The presence of
entrainment, distractibility, This type of tremor occurs in patients or usually begins with a discussion of
and suggestibility all with specific psychiatric conditions, the diagnosis, recognition of the pa-
point to a diagnosis of especially conversion disorder, and tient’s suffering, and a referral to a
psychogenic tremor. malingering.28 By history, the tremor psychiatrist in order to explore the
h Wing-beat tremor is often has a sudden (ie, abrupt) start underlying psychiatric issues.
considered a classic with maximal tremor at onset rather
tremor in Wilson than an insidious onset followed by a Wilson Disease With
disease, but it is not the slowly progressive course. On examina- Associated Tremor
most common type of tion, the tremor often has nonphysiologic Patients with Wilson disease may pres-
tremor in that disease. or unusual features (eg, may exhibit ent with a wide range of involuntary
variable frequency or change direction, movements, including tremor.31,32 In-
and an unusual combination of rest, deed, tremor ranks among the eight
postural, and kinetic tremors may major symptoms reported by patients
be seen) (Case 6-3). On examination, with Wilson disease who have neuro-
the clinician may see signs that are logic features.33 Tremor is usually ac-
suggestive of psychogenic tremor, companied by other neurologic signs,
including entrainment (ability of the although there are rare reports of iso-
examiner to alter the rhythm of the lated tremor and even rarer reports of
patient’s tremor by having it match isolated action tremor.34 Although the
the rhythm of a tremor the examiner classic tremor associated with Wilson
produces), distractibility (ability of disease is the wing-beat tremor (pres-
the examiner to stop the tremor by ent on abduction of the shoulder and
asking the patient to perform certain flexion of the elbow), it is not the most
mental or physical tasks), and sug- commonly observed type of tremor.32,35
gestibility (ability of the examiner to Overall, the tremor phenomenology
bring on the tremor with the power of is quite varied. Indeed, across patients,
suggestion).29 Interestingly, on quan- a wide range of tremors may occur, in-
titative computerized tremor analysis, cluding kinetic tremor as well as resting
inertial loading can result in a para- tremor, postural and intention tremors,
doxical increase in tremor amplitude tremors that can be symmetric or
rather than the expected decrease in asymmetric, those that are low ampli-
amplitude that should be seen with tude and others that are high ampli-
organic tremors.30 tude, and those that are intermittent,

Case 6-3
A 27-year-old woman presented for a neurologic evaluation with a reported
tremor that had begun suddenly 2 years previously after she had fallen down
the stairs. The tremor affected her ‘‘entire body’’ and was not relieved by
anything. She had been unable to work as a waitress since the onset of the
tremor and stated that her life was ‘‘falling apart.’’ On examination, she had
a head tremor, which sometimes involved a horizontal movement and, at
other times, involved a vertical movement. At other times it was complex
and rotatory. The tremor was virtually continuous. The examination was
otherwise normal. The examiner was able to stop the tremor for a few
seconds while distracting the patient.
Comment. A number of important features of psychogenic tremor are
illustrated in this case, including its sudden onset, its often variable quality,
and the fact that it may cease when the patient is distracted.

1152 www.ContinuumJournal.com August 2016

Copyright © American Academy of Neurology. Unauthorized reproduction of this article is prohibited.


KEY POINT
while others are constant and progres- ataxia, parkinsonism, or cognitive de- h Patients with fragile X
sive.32,35 Most of the large published cline.40 Tremor can be one of the ear- tremor-ataxia syndrome
case series describe the broad pano- liest signs,41 and it may occur in a may have a mixture
ply of neurologic signs, and a detailed variety of activation conditions, includ- of different tremor
modern characterization of the tremor ing with action (ie, postural, kinetic, or phenomenologies.
phenomenology is lacking. According intentional) or with rest. Action tremor
to one series, 32% of patients exhibited predominates, although the tremor has
tremor at the time of their first neuro- not been described in a nuanced and
logic evaluation at a tertiary care cen- systematic manner in the published lit-
ter35; in another retrospective review erature. While an occasional patient will
of patients seen in a tertiary referral have isolated action tremor, which re-
center, 60% of patients exhibited tremor sembles that seen in patients with essen-
at some point.36 Tremor most com- tial tremor,40 many patients have mixed
monly occurs in the hands, with 82% of tremor phenomenology and a constel-
patients having hand tremor, accord- lation of neurologic signs in addition
ing to one report.37 Most patients pres- to tremor.42 When present, the tremor
ent well before the age of 40 years, and may vary in severity from mild and
the laboratory workup may reveal a asymptomatic to severe and disabling,40
low serum ceruloplasmin, an abnor- although one retrospective cohort study
mal brain MRI (ie, lesions in the basal reported that tremor becomes consid-
ganglia), a high 24-hour urine copper erably disabling within 13 years of on-
concentration, an abnormal slit-lamp set of motor symptoms.41 Controlled
examination (ie, presence of Kayser- trials evaluating symptomatic therapies
Fleischer rings), elevated liver function have not been reported in FXTAS.43
tests, or abnormal liver biopsy results.33 However, therapies used for tremor in
Treatment of the underlying disease essential tremor and PD have been
with tetrathiomolybdate, zinc, or trien-
tried with variable success; surgical ther-
tine has been recommended38; little
apy is very effective for essential tremor
has been written about the specific
and PD tremor and is an option for pa-
treatment of the tremor as a neuro-
tients with FXTAS who have medication-
logic sign. For more information, refer
resistant and disabling tremor.43
to the article ‘‘Wilson Disease’’ by
Ronald F. Pfeiffer, MD, FAAN,39 in this Peripheral NeuropathyYRelated
issue of Continuum. Tremor
Several of the acquired and familial
Fragile X Tremor-Ataxia neuropathies are associated with pos-
Syndrome tural and kinetic tremors of the arms
Fragile X tremor-ataxia syndrome of mild to moderate severity.44Y46 For
(FXTAS) is an inherited degenerative some neuropathies (eg, IgM demyelin-
disorder that primarily affects older men ating paraproteinemic neuropathy), up
and is associated with a wide range of to 90% of patients are reported to have
neurologic symptoms and signs.40 The such tremor.47 By history, patients with
syndrome is caused by a CGG repeat this type of tremor have a coexist-
expansion in the premutation range in ing peripheral neuropathy in the same
the 5’ noncoding region of the fragile limbs that are tremulous. The neurop-
X mental retardation 1 (FMR1) gene. athy and the tremor should be tempo-
Classically, these patients are men in rally linked, with tremor accompanying
their sixties who develop some com- or following the neuropathy. On ex-
bination of action tremor, cerebellar amination, a peripheral neuropathy
Continuum (Minneap Minn) 2016;22(4):1143–1158 www.ContinuumJournal.com 1153

Copyright © American Academy of Neurology. Unauthorized reproduction of this article is prohibited.


Diagnosing Tremor

KEY POINT
h Midbrain (rubral) tremor characterized by sensory deficits, weak- the pontine-midbrain region, affecting
is generally a unilateral ness, wasting (Supplemental Digital Con- cerebellar outflow tracts and dopaminer-
tremor accompanied tent 6-3, links.lww.com/CONT/A182), gic nigrostriatal fibers.53 There are also
by rest, postural, and diminished/absent deep tendon re- reports of lesions occurring elsewhere
and kinetic/intentional flexes is readily apparent in the tremu- (eg, the thalamus),54 which is one of
components. lous limb or limbs,48,49 although the the motivations for referring to the
severity of the weakness does not ne- tremor as Holmes tremor rather than
cessarily correlate with the severity of rubral tremor. As the dopaminergic
the tremor.50 The tremor disappears if system is involved in most cases,
neuropathic weakness progresses to treatment with levodopa (100 mg/d to
the point of paralysis. The underlying 1000 mg/d) may be beneficial.55 In
mechanisms are likely to be diverse and addition, medications that are used
may involve central as well as peripheral for the treatment of essential tremor
components.44 If the tremor occurs may be effective. DBS has proven
in the setting of an immunoglobulin- beneficial in some cases as well.55
mediated disease, then immunosuppres-
sive therapies, such as corticosteroids, RESTING TREMOR
IV immunoglobulin, cyclophosphamide, The differential for resting tremor is
or plasma exchange may be used. There less extensive than that of kinetic
are several reports of the use of pre- tremor. The main items are PD and
gabalin (up to 450 mg/d) for the treat- drug-induced tremor.
ment of neuropathic tremor.46 The
tremor may respond to DBS.51 Parkinson Disease
The tremor in patients with PD is clas-
Midbrain (Rubral) Tremor sically a tremor at rest. The tremor may
Midbrain tremor has also been re- affect any limb but is generally asym-
ferred to as Holmes tremor or rubral metric, affecting one limb or one side
tremor.10,52 The tremor is generally of the body (ie, arm and leg) preferen-
unilateral and has three components: tially. In patients with arm tremor, the
tremor at rest, postural tremor, and tremor often involves distal joints
kinetic/intentional tremor, with the rel- (eg, fingers and wrist) rather than the
ative severity generally being such that elbow or shoulder. In addition, wrist
kinetic tremor is greater than postural pronation-supination rather than
tremor, which is greater than resting flexion-extension is more common
tremor (Supplemental Digital Con- (Supplemental Digital Content 6-5,
tent 6-4, links.lww.com/CONT/A183). links.lww.com/CONT/A184). In addi-
The tremor is often severe and dis- tion, limb postures (flexed posture of
abling and can render the affected the hand while walking or during arm
limb functionally useless. Patients may extension) is quite common, as are other
have other neurologic signs as well, cardinal signs of parkinsonism. A con-
including mild dystonia or ataxia. The fusing feature of the examination can
tremor may occur in a variety of clinical be the presence of reemergent tremor
settings (eg, in the setting of a brain during arm extension or during activi-
tumor, multiple sclerosis, or slowly ties (eg, pouring water). Tremor may
expanding vascular lesion) and, when affect the jaw, although in contrast to
occurring in the setting of a stroke, the essential tremor, is more often there
tremor may arise after a latency of when the mouth is closed and relaxed
months to years. On brain imaging, a rather than while speaking or holding
lesion is often but not always present in the mouth open.
1154 www.ContinuumJournal.com August 2016

Copyright © American Academy of Neurology. Unauthorized reproduction of this article is prohibited.


KEY POINTS
The treatment of parkinsonian history, there is a link between the use h In addition to the classic
resting tremor includes the use of anti- of a medication and the onset of the resting tremor, a large
cholinergic agents (including aman- tremor. Unless the patient has an proportion of patients
tadine up to 300 mg/d), as well as underlying disease of the basal ganglia, with Parkinson disease
carbidopa/levodopa (levodopa dose removal of the medication should re- also have postural or
up to 2000 mg/d) and dopamine sult in complete resolution of tremor, kinetic tremors of
agonists. Benzodiazepines play a role although this may take weeks to the arms.
as well. DBS is reserved for severe months. The treatment of such tremor h Drug-induced resting
cases in which patients are refractory first involves the removal of the causa- tremor may be either
to medications. tive drug or a reduction in dosage if unilateral or bilateral.
It is important to note that although this is possible, although this is often
resting tremor is one of the hallmark not possible in the setting of psycho-
features of PD, a large proportion of sis. Carbidopa/levodopa, amantadine,
patients also have postural or kinetic and anticholinergic agents may lessen
tremors of the arms.56 Sometimes the the severity of the tremor and may even
postural and kinetic tremor have a re- be used in combination with the tremor-
emergent quality; this so-called reemer- producing medication, if the latter
gent tremor surfaces after a latency of cannot be discontinued.
one or several seconds, has a frequency
that is similar to that of the resting CONCLUSION
tremor in PD, and often attains ampli- The diagnosis of tremor disorders is
tudes greater than that seen in patients challenging. The approach to tremor
with essential tremor.57 This tremor is involves a history and careful neuro-
often asymmetric and tends to increase logic examination, focused on the nu-
in severity (ie, crescendo) with sus- ances of clinical phenomenology. It
tained posture or during the course of is important, when generating the
repetitive movements during which differential diagnosis, to first consider
much of the limb is immobile (eg, while whether the primary type of tremor
pouring water between two cups, dur- is an action tremor or one at rest. As is
ing which much of the movement is the case with the diagnosis of many
proximal rather than distal). Reemergent movement disorders, arriving at the
tremor may at times occur in patients correct diagnosis is often based on
who do not have resting tremor.58 The pattern recognition.
treatment of this tremor is similar to the
treatment of the resting tremor of PD, VIDEO LEGENDS
although it is less responsive to medi- Supplemental Digital
cations than the latter. Content 6-1
Drug-Induced Resting Tremor Essential tremor. Video shows a
woman who exhibits a kinetic tremor
Several medications (eg, neuroleptic
with an intentional component on
agents) may cause resting tremor,
the finger-nose-finger maneuver. The
which is generally accompanied by
tremor is slightly asymmetric and is
other features of parkinsonism (eg,
worse on the left. A mild head tremor
bradykinesia or rigidity).12 The tremor
as also present. When she attempts
generally resembles a typical parkinso-
to pour water between two cups, the
nian resting tremor and may even be
kinetic tremor results in spillage.
asymmetric. As with other parkinso-
nian tremors, the tremor may have a links.lww.com/CONT/A180
reemergent component as well. By B 2016 American Academy of Neurology.

Continuum (Minneap Minn) 2016;22(4):1143–1158 www.ContinuumJournal.com 1155

Copyright © American Academy of Neurology. Unauthorized reproduction of this article is prohibited.


Diagnosing Tremor

Supplemental Digital Supplemental Digital


Content 6-2 Content 6-5
Dystonic postures with dystonic Parkinson disease. Video shows a
tremor. Video shows a man who ex- woman with Parkinson disease who
hibits mild kinetic tremor on the exhibits a resting tremor while stand-
finger-nose-finger maneuver. On arm ing with her hand at her side. The
extension, the man exhibits a mild tremor involves the distal joints (fin-
postural tremor with dystonic postur- gers and wrist) and is characterized by
ing of the hands, more so on the wrist pronation-supination and has a
left that on the right, where there is slight pill-rolling quality at times.
obvious spooning. links.lww.com/CONT/A184
links.lww.com/CONT/A181 B 2016 American Academy of Neurology.
B 2016 American Academy of Neurology.

Supplemental Digital REFERENCES


Content 6-3 1. Louis ED, Ford B, Pullman SL. Prevalence of
asymptomatic tremor in relatives of patients
Peripheral neuropathy with tremor. with essential tremor. Arch Neurol
Video shows a man with peripheral 1997;54(2):197Y200.
neuropathy with wasting of the intrin- 2. Louis ED, Frucht SJ, Rios E. Intention tremor in
sic hand muscles and some weakness essential tremor: prevalence and association
of finger extensors on arm extension. with disease duration. Mov Disord 2009;24(4):
626Y627. doi:10.1002/mds.22370.
When the man follows the examiner’s
3. Louis ED. The primary type of tremor
finger with his finger, a mild kinetic in essential tremor is kinetic rather
tremor is evident. than postural: cross-sectional observation
links.lww.com/CONT/A182 of tremor phenomenology in 369 cases. Eur
J Neurol 2013;20(4):725Y727. doi:10.1111/
B 2016 American Academy of Neurology. j.1468-1331.2012.03855.x.
4. Cohen O, Pullman S, Jurewicz E, et al. Rest
Supplemental Digital tremor in patients with essential tremor:
prevalence, clinical correlates, and
Content 6-4 electrophysiologic characteristics. Arch Neurol
Midbrain tremor. Video shows a 2003;60(3):405Y410.
woman with midbrain tremor who 5. Louis ED, Rios E, Rao AK. Tandem gait
exhibits some posturing of the right performance in essential tremor: clinical
correlates and association with midline
arm as well as reduced right arm swing tremors. Mov Disord 2010;25(11):1633Y1638.
while walking. Mild tremor is also doi:10.1002/mds.23144.
present while walking, although it is 6. Louis ED. Re-thinking the biology of
not clear whether the hand is fully essential tremor: from models to morphology.
at rest. While her right hand rests in Parkinsonism Related Disord 2014;
20(suppl 1):S88YS93. doi:10.1016/S1353-8020
her lap, a mild tremor appears, which (13)70023-3.
has a pill-rolling quality. The patient 7. Louis ED, Agnew A, Gillman A, et al.
exhibits a flapping postural tremor Estimating annual rate of decline:
on the right, with some posturing of prospective, longitudinal data on arm
that hand. On finger-nose-finger ma- tremor severity in two groups of essential
tremor cases. J Neurol Neurosurg Psychiatry
neuver (right), there is tremor with 2011;82(7):761Y765. doi:10.1136/
terminal worsening. There is no tremor jnnp.2010.229740.
on the left. 8. Jain S, Lo SE, Louis ED. Common misdiagnosis
links.lww.com/CONT/A183 of a common neurological disorder: how are
we misdiagnosing essential tremor? Arch
B 2016 American Academy of Neurology. Neurol 2006;63(8):1100Y1104.

1156 www.ContinuumJournal.com August 2016

Copyright © American Academy of Neurology. Unauthorized reproduction of this article is prohibited.


9. Zesiewicz TA, Elble RJ, Louis ED, et al. 24. Gerschlager W, Münchau A, Katzenschlager R,
Evidence-based guideline update: treatment et al. Natural history and syndromic
of essential tremor: report of the Quality associations of orthostatic tremor: a
Standards subcommittee of the American review of 41 patients. Mov Disord
Academy of Neurology. Neurology 2004;19(7):788Y795.
2011;77(19):1752Y1755. doi:10.1212/
25. Yaltho TC, Ondo WG. Thalamic deep brain
WNL.0b013e318236f0fd.
stimulation for orthostatic tremor. Tremor
10. Deuschl G, Bain P, Brin M. Consensus statement Other Hyperkinet Mov (N Y) 2011;1.
of the Movement Disorder Society on Tremor. pii:tre-01-26-56-2.
Ad Hoc Scientific Committee. Mov Disord
26. Bodranghien F, Bastien A, Casali C, et al.
1998;13(suppl 3):2Y23.
Consensus paper: revisiting the symptoms and
11. Elble RJ, Higgins C, Elble S. Electrophysiologic signs of cerebellar syndrome. Cerebellum
transition from physiologic tremor to essential 2016;15(3)369Y391. doi:10.1007/s12311-015-
tremor. Mov Disord 2005;20(8):1038Y1042. 0687-3.
12. Morgan JC, Sethi KD. Drug-induced tremors. 27. Labiano-Fontcuberta A, Benito-León J.
Lancet Neurol 2005;4(12):866Y876. Understanding tremor in multiple sclerosis:
prevalence, pathological anatomy, and
13. Raethjen J, Lemke MR, Lindemann M, et al.
pharmacological and surgical approaches
Amitriptyline enhances the central component
to treatment. Tremor Other Hyperkinet Mov
of physiological tremor. J Neurol Neurosurg
(N Y) 2012;2 pii:tre-02-109-765-2.
Psychiatry 2001;70(1):78Y82.
28. Bhatia KP, Schneider SA. Psychogenic tremor
14. Defazio G, Gigante AF, Abbruzzese G, et al.
and related disorders. J Neurol 2007;254(5):
Tremor in primary adult-onset dystonia:
569Y574.
prevalence and associated clinical features.
J Neurol Neurosurg Psychiatry 2013;84(4): 29. Thenganatt MA, Jankovic J. Psychogenic
404Y408. doi:10.1136/jnnp-2012-303782. tremor: a video guide to its distinguishing
15. Deuschl G. Dystonic tremor. Rev Neurol features. Tremor Other Hyperkinet Mov (N Y)
(Paris) 2003;159(10 pt 1):900Y905. 2014;4:253. doi:10.7916/D8FJ2F0Q.

16. Defazio G, Conte A, Gigante AF, et al. 30. Schwingenschuh P, Katschnig P, Seiler S, et al.
Is tremor in dystonia a phenotypic feature of Moving toward ‘‘laboratory-supported’’
dystonia? Neurology 2015;84(10):1053Y1059. criteria for psychogenic tremor. Mov Disord
doi:10.1212/WNL.0000000000001341. 2011;26(14):2509Y2515. doi:10.1002/
mds.23922.
17. Jankovic J, Leder S, Warner D, Schwartz K.
Cervical dystonia: clinical findings and 31. Soltanzadeh A, Soltanzadeh P, Nafissi S, et al.
associated movement disorders. Neurology Wilson’s disease: a great masquerader. Eur
1991;41(7):1088Y1091. Neurol 2007;57(2):80Y85.
18. Agnew A, Frucht SJ, Louis ED. Supine head 32. Lorincz MT. Neurologic Wilson’s disease.
tremor: a clinical comparison of essential Ann N Y Acad Sci 2010;1184:173Y187.
tremor and spasmodic torticollis patients. doi:10.1111/j.1749-6632.2009.05109.x.
J Neurol Neurosurg Psychiatry 2012;83(2):
33. Walshe JM, Yealland M. Wilson’s disease:
179Y181. doi:10.1136/jnnp-2011-300823.
the problem of delayed diagnosis. J Neurol
19. Bain PG, Findley LJ, Britton TC, et al. Primary Neurosurg Psychiatry 1992;55(8):692Y696.
writing tremor. Brain 1995;118(pt 6):1461Y1472.
34. Frucht S, Sun D, Schiff N, et al. Arm tremor
20. Cohen LG, Hallett M, Sudarsky L. A single secondary to Wilson’s disease. Mov Disord
family with writer’s cramp, essential tremor, 1998;13(2):351Y353.
and primary writing tremor. Mov Disord
1987;2(2):109Y116. 35. Starosta-Rubinstein S, Young AB, Kluin K,
et al. Clinical assessment of 31 patients
21. Louis ED, Ford B, Wendt KJ, et al. A comparison with Wilson’s disease. Correlations with
of different bedside tests for essential tremor. structural changes on magnetic resonance
Mov Disord 1999;14(3):462Y467. imaging. Arch Neurol 1987;44(4):
22. Rana AQ, Vaid HM. A review of primary 365Y370.
writing tremor. Int J Neurosci 2012;122(3): 36. Machado A, Chien HF, Deguti MM, et al.
114Y118. doi:10.3109/00207454. Neurological manifestations in Wilson’s
2011.635827. disease: report of 119 cases. Mov Disord
2006;21(12):2192Y2196.
23. Yaltho TC, Ondo WG. Orthostatic tremor: a
review of 45 cases. Parkinsonism Relat Disord 37. Saito T. Presenting symptoms and natural
2014;20(7):723Y725. doi:10.1016/j.parkreldis. history of Wilson disease. Eur J Pediatr
2014.03.013. 1987;146(3):261Y265.

Continuum (Minneap Minn) 2016;22(4):1143–1158 www.ContinuumJournal.com 1157

Copyright © American Academy of Neurology. Unauthorized reproduction of this article is prohibited.


Diagnosing Tremor

38. Brewer GJ. Neurologically presenting Wilson’s demyelinating symmetric neuropathy.


disease: epidemiology, pathophysiology and Mov Disord 2005;20(11):1529Y1530.
treatment. CNS Drugs 2005;19(3):185Y192.
49. Cardoso FE, Jankovic J. Hereditary
39. Pfeiffer RF. Wilson disease. Continuum motor-sensory neuropathy and movement
(Minneap Minn) 2016;22(4 Movement disorders. Muscle Nerve 1993;16(9):
Disorders):1246Y1261. 904Y910.
40. Leehey MA. Fragile X-associated tremor/ 50. Dalakas MC, Teräväinen H, Engel WK.
ataxia syndrome: clinical phenotype, diagnosis, Tremor as a feature of chronic relapsing and
and treatment. J Investig Med 2009;57(8): dysgammaglobulinemic polyneuropathies.
830Y836. doi:10.231/JIM.0b013e3181af59c4. Incidence and management. Arch Neurol
1984;41(7):711Y714.
41. Leehey MA, Berry-Kravis E, Min SJ, et al.
Progression of tremor and ataxia in male 51. McMaster J, Gibson G, Castro-Prado F, et al.
carriers of the FMR1 premutation. Mov Neurosurgical treatment of tremor in
Disord 2007;22(2):203Y206. anti-myelin-associated glycoprotein
42. Berry-Kravis E, Abrams L, Coffey SM, et al. neuropathy. Neurology 2009;73(20):1707Y1708.
Fragile X-associated tremor/ataxia syndrome: doi:10.1212/WNL.0b013e3181c1de66.
clinical features, genetics, and testing 52. Kiriyama T, Hirano M, Kitauchi T, et al.
guidelines. Mov Disord 2007;22(14):2018Y2030. A case of chemotherapy-responsive
43. Leehey MA, Hagerman PJ. Fragile paraneoplastic rubral tremor. Clin Neurol
X-associated tremor/ataxia syndrome. Neurosurg 2011;113(8):693Y695.
Handb Clin Neurol 2012;103:373Y386. doi:10.1016/j.clineuro.2011.04.011.
doi:10.1016/B978-0-444-51892-7.00023-1.
53. Goto S, Yamada K. Combination of thalamic
44. Pedersen SF, Pullman SL, Latov N, Vim stimulation and GPi pallidotomy
Brannagan TH 3rd. Physiological tremor synergistically abolishes Holmes’ tremor.
analysis of patients with anti-myelin-associated J Neurol Neurosurg Psychiatry
glycoprotein associated neuropathy and 2004;75(8):1203Y1204.
tremor. Muscle Nerve 1997;20(1):38Y44.
54. Tan H, Turanli G, Ay H, Saatçi I. Rubral
45. Saverino A, Solaro C, Capello E, et al. tremor after thalamic infarction in childhood.
Tremor associated with benign IgM Pediatr Neurol 2001;25(5):409Y412.
paraproteinaemic neuropathy successfully 55. Puschmann A, Wszolek ZK. Diagnosis and
treated with gabapentin. Mov Disord treatment of common forms of tremor.
2001;16(5):967Y968. Semin Neurol 2011;31(1):65Y77. doi:10.1055/
46. Alonso-Navarro H, Fernández-Dı́az A, s-0031-1271312.
Martı́n-Prieto M, et al. Tremor associated 56. Lance JW, Schwab RS, Peterson EA.
with chronic inflammatory demyelinating Action tremor and the cogwheel
peripheral neuropathy: treatment with phenomenon in Parkinson’s disease. Brain
pregabalin. Clin Neuropharmacol 1963;86:95Y110.
2008;31(4):241Y244. doi:10.1097/WNF.
57. Jankovic J, Schwartz KS, Ondo W. Re-emergent
0b013e3181585b71.
tremor of Parkinson’s disease. J Neurol
47. Bain PG, Britton TC, Jenkins IH, et al. Tremor Neurosurg Psychiatry 1999;67(5):646Y650.
associated with benign IgM paraproteinaemic
58. Louis ED, Pullman SL, Eidelberg D, Dhawan V.
neuropathy. Brain 1996;119(pt 3):789Y799.
Re-emergent tremor without accompanying
48. Budak F, Alemdar M, Kamaci S, Selekler M. rest tremor in Parkinson’s disease. Can J
Tremor in idiopathic distal acquired Neurol Sci 2008;35(4):513Y515.

1158 www.ContinuumJournal.com August 2016

Copyright © American Academy of Neurology. Unauthorized reproduction of this article is prohibited.


Review Article

Movement Disorders
Address correspondence to
Dr Manju A. Kurian, Room
111, Level 1 Clinical and
Molecular Genetics Unit,

Presenting in University College London,


Institute of Child Health, 30
Guilford Street, London WC1N

Childhood 1EH, United Kingdom,


manju.kurian@ucl.ac.uk.
Relationship Disclosure:
Dr Kurian receives
Manju A. Kurian, MBBChir, MRCPCH, PhD; research/grant support from
Russell C. Dale, MBChB, MRCP, PhD Gracious Heart Charity and
Rosetrees (IPAAK/515006),
Great Ormond Street Hospital
Children’s Charity (507837),
ABSTRACT Neurodegeneration With
Brain Iron Accumulation
Purpose of Review: This article provides an overview of movement disorders that Disorders Association
present in childhood. Key clinical features are discussed, and a brief guide to man- (5150081), and a Wellcome
agement strategies is provided. Recent advances in the field of pediatric movement Trust Intermediate Clinical
Fellowship (WT098524MA).
disorders are also a focus of the article. Dr Dale has received personal
Recent Findings: Advances in genetic technologies and cell biology have contributed compensation for serving on
greatly to the elucidation of underlying disease mechanisms in childhood movement the editorial advisory board of
Multiple Sclerosis and Related
disorders. This article discusses the expanding spectrum of both genetic and acquired Disorders and the editorial
movement disorders that present in childhood, including benign, acquired, genetic, boards of the European
and psychogenic movement disorders. Journal of Paediatric
Neurology and Neurology:
Summary: Movement disorders in childhood comprise a wide spectrum of both Neuroimmunology &
genetic and acquired diseases, ranging from benign self-limiting conditions to more Neuroinflammation, and on
progressive phenotypes associated with significant morbidity and mortality. Elucida- the scientific advisory board
for the Queensland Children’s
tion of the underlying cause is achieved through accurate history, detailed clinical Medical Research Institute.
examination, review of video footage (including home videos), and, where appro- Dr Dale has received honoraria
priate, neuroimaging and laboratory investigations. Early accurate diagnosis will for speaking engagements
as well as publishing royalties
facilitate the instigation of appropriate management strategies. from Biogen Inc and
Bristol-Myers-Squibb and has
received research support
Continuum (Minneap Minn) 2016;22(4):1159–1185.
from Multiple Sclerosis
Research Australia and the
National Health and Medical
Research Council.
INTRODUCTION etiology (molecular genetic features Unlabeled Use
of Products/Investigational
Movement disorders in childhood or neuropathology). Use Disclosure:
A wide range of movement dis- Drs Kurian and Dale report
comprise a heterogeneous group of no disclosures.
conditions that lead to impairment of orders are described in childhood, * 2016 American Academy
voluntary movement, abnormal pos- from benign self-limiting transient of Neurology.

tures, or inserted involuntary move- phenomena to more progressive dis-


ments. Movement disorders in children orders associated with significant
are often classified into two main cate- morbidity and mortality. Movement
gories: hyperkinetic/dyskinetic move- disorders in childhood may present
ment disorders (including stereotypies, acutely, subacutely, or in a more Supplemental digital content:
Videos accompanying this
tics, tremor, dystonia, and chorea)1 and chronic manner. The underlying etiol- article are cited in the text as
hypokinetic movement disorders, en- ogies for childhood movement disor- Supplemental Digital Content.
Videos may be accessed by
compassing parkinsonian phenotypes.2 ders are diverse and include both clicking on links provided in the
A number of other systems classify dis- acquired and genetic conditions. Sig- HTML, PDF, and app versions
of this article; the URLs are pro-
orders into subgroups according to nificant advances in genetic technolo- vided in the print version. Video
temporal and spatial patterns, clinical gies have facilitated the discovery of a legends begin on page 1182.
features, motor signs, and underlying number of novel disease-causing

Continuum (Minneap Minn) 2016;22(4):1159–1185 www.ContinuumJournal.com 1159

Copyright © American Academy of Neurology. Unauthorized reproduction of this article is prohibited.


Movement Disorders in Childhood

KEY POINTS
h Movement disorders genes and movement disorder syn- movements. The most common vocal
in childhood are dromes in childhood. tics are throat clearing and grunts.
heterogeneous and Accurate diagnosis of childhood- However, a remarkable breadth of
range from mild onset movement disorders requires motor and vocal tics may be observed
self-limiting phenotypes careful ascertainment of clinical history, from any body part. Tics are stereo-
to more severe detailed neurologic examination (for typical and repetitive for periods of
progressive recognition of phenomenology and time, but tend to change over time
neurodegenerative associated features), and a broad (months and years). The characteris-
disorders. tic features of tics are their repeti-
knowledge of potential differential di-
h A number of benign agnoses. This article provides an up- tive stereotypical nature, fluctuation
movement disorders of dated overview of movement disorders (waxing and waning), suppressibility,
childhood have been in childhood, with particular focus on and suggestibility.
identified, which are Tic duration is used to separate tics
newly recognized conditions.
transient in nature and into transient tic disorder, which occurs
usually have a good BENIGN MOVEMENT for less than 12 months, and chronic
long-term prognosis.
DISORDERS OF CHILDHOOD disorders, which occur for longer than
A number of relatively benign move- 12 months, and include: chronic motor
ment disorders of childhood have been tic disorder, chronic vocal tic disorder,
identified, relating to specific stages and when both motor and vocal tics
of normal development (Table 7-1).3 are present, Tourette syndrome. The
The underlying basis of such move- time of 12 months is used in the de-
ment disorders is not fully under- finition, as when tics last for longer
stood, but some are thought to be a than 12 months they are more likely to
manifestation of the developing brain, continue for much of childhood and
reflecting central nervous system im- adolescence. Despite being common
maturity. The majority of these con- (transient tics affect approximately 5%
ditions occur early in life, during the to 10% of children at some time, and
neonatal period, infancy, and early Tourette syndrome affects 0.5% of chil-
childhood, with resolution of symp- dren), these disorders remain poorly
toms over time. Furthermore, other understood and underrecognized by
neurologic symptoms are rarely pres- both clinicians and laypeople. Patients
ent, and most children are found to are often referred to opticians (for
have a normal neurologic examina- blinking and eye movements), otolar-
tion. Accurate recognition is crucial yngologists (for throat clearing), or re-
to avoid unnecessary investigations spiratory physicians (for repetitive
and provide families with reassur- cough) before a diagnosis is made. Tics
ance regarding long-term prognosis, are often mislabeled as habits.
where appropriate. Neurodevelopmental disorders are
also overrepresented in patients with
TICS, TOURETTE SYNDROME, Tourette syndrome, and a significant
AND STEREOTYPIES minority of patients may have autism
Tics are the most common movement spectrum disorder, sensory processing
disorder in childhood and can be disorder, or other dysexecutive prob-
separated into motor tics or vocal tics. lems, which can affect school and
Motor tics typically involve the face, general functioning. Tourette syn-
head, and shoulders, although they drome is not a disease but a symptom
can affect any body part. The most syndrome, and the etiology is likely
common motor tics are blinking, eye multifactorial with genetic vulnerabil-
movements, grimacing, and head ity factors plus environmental factors.4
1160 www.ContinuumJournal.com August 2016

Copyright © American Academy of Neurology. Unauthorized reproduction of this article is prohibited.


TABLE 7-1 Benign Movement Disorders of Childhood

Average Age
at Symptom Associated
Condition Symptom Onset Resolution Clinical Features Etiology
Jitteriness Neonatal period Midinfancy Generalized Hypoxic ischemic
rhythmic oscillation encephalopathy,
hypoglycemia,
hypocalcemia,
drug withdrawal
Benign neonatal Neonatal period Midinfancy Repeated myoclonic None known
sleep myoclonus jerks in sleep
Benign myoclonus Infancy Usually by 2 years Myoclonic spasms None known
of early infancy of the neck, trunk,
and limbs, similar
to infantile spasms
Spasmus nutans Infancy Within few months, Horizontal/vertical Rarely, cerebellar
nystagmus can head tremor with abnormalities on
persist pendular nystagmus neuroimaging
Infantile Infancy Early childhood Limb posturing, None known
masturbation but may persist rocking, thigh
adduction with
genital manipulation/
pressure over
pubic area
Shuddering Infancy to end of Usually gets Shivering None known
the first decade better with age movements of
head, shoulders,
and upper limbs,
often with facial
grimace
Benign idiopathic Infancy Usually by 1 year Segmental dystonia, None known
dystonia of infancy usually upper limb
Benign paroxysmal Infancy Midchildhood Paroxysmal head Mutation in
torticollis of infancy tilt, often with CACNA1A;
vomiting, irritability, mutation in PRRT2
pallor
Paroxysmal tonic Infancy Midchildhood Paroxysmal Mutation in
upgaze of infancy deviation of eyes CACNA1A;
upward/downward hypomyelination,
periventricular
leukomalacia,
vein of Galen
malformation,
pinealoma
Head nodding Usually by early Normally resolves Horizontal, vertical, Onchocerca
childhood in a few months, and oblique head volvulus infection
but may last movements (sub-Saharan
through childhood Africa)

Continuum (Minneap Minn) 2016;22(4):1159–1185 www.ContinuumJournal.com 1161

Copyright © American Academy of Neurology. Unauthorized reproduction of this article is prohibited.


Movement Disorders in Childhood

KEY POINT
h Tics and stereotypy are The most important management include rocking, flapping, and body-
the most common aspect of Tourette syndrome is to tensing. However, some stereotypies
movement disorders of make a confident diagnosis and edu- are extremely complex, including
childhood. Diagnosis is cate the child, family, and school that twisting, grimacing, and posturing.
dependent on careful Tourette syndrome is a neurologic The most important characteristics
history, observation, disorder and is not the child’s fault of stereotypy are: (1) the movements
and parental videos. so that the family understands and have a stereotypical nature (ie, the
Education of the family accepts the diagnosis. In addition, in- episodes are similar); (2) the patient is
and searching for forming the family that the tics typi- partially distractible (although the
neurodevelopmental cally improve between 11 and 20 years child may subsequently need to com-
and neuropsychiatric
of age can be reassuring. In ap- plete the movement after being dis-
comorbidity are
proximately 80% of patients, psy- tracted); and (3) the episodes are often
paramount, as
associated comorbidities
choeducation is all that is required. more likely to occur with certain stim-
such as anxiety and The need for more intervention is uli, such as excitement or anticipation
obsessive-compulsive dictated by impairment in social or (eg, waiting in a line), participation in a
disorder can exacerbate global functioning. When impair- favorite activity (eg, watching televi-
the movements and are ment occurs, treatment can include sion), or sometimes boredom (eg,
arguably more amenable psychological approaches including riding in the back of a car). Parental
to psychological or tic reversal therapy. When medical videos are highly informative and es-
medical treatments. treatments are required, the family sential for diagnosis. Stereotypy is
needs to understand that medication more common in children with devel-
will only reduce tics by 30% to 50% at opmental disorders, such as language
best, although this can be useful. In delay and autism spectrum disorder, but
general, clonidine or guanfacine is the complex motor stereotypy can also
first-line therapy, and antipsychotics occur in normally developed children.5
such as risperidone, are restricted to Detailed assessment for subtle neuro-
more severe cases. Other agents includ- developmental disorders is important
ing tetrabenazine, baclofen, topiramate, as this may present therapeutic op-
and benzodiazepine can be considered, tions, although the treatment of ste-
and deep brain stimulation (DBS) can reotypy itself is challenging and
be used for the most severely affected drugs are ineffective. Psychological
patients who have failed medical ther- approaches may be useful in well-
apy and who have failed to improve motivated cognitively able individuals.
in adolescence. Screening for comor- Many normally developed older chil-
bid diagnoses of attention deficit disor- dren with stereotypy have associated
der, obsessive-compulsive disorder, or neuropsychiatric conditions, such as
associated problems, is essential as attention deficit hyperactivity disorder,
these are frequently more important anxiety, or tics.5 Some children with
to treat than the tics. complex stereotypy describe complex,
Stereotypy is a common movement sometimes imaginary, thoughts during
phenotype and a normal feature of the movements.6
infant development, but when stereo-
typy continues beyond this period, ACQUIRED MOVEMENT
the episodes can become concerning DISORDERS
to parents (less so for the child) Acquired brain disorders such as trau-
because of the unusual nature of the matic brain injury, hypoxic ischemic
episodes and the potential for social injury, and infectious and autoim-
stigmatization. Stereotypies are re- mune processes can result in transient
petitive movements; classic examples or persistent movement disorders.
1162 www.ContinuumJournal.com August 2016

Copyright © American Academy of Neurology. Unauthorized reproduction of this article is prohibited.


KEY POINTS
Cerebral Palsy (Hypoxic Ischemic) Enterovirus, group A streptococcus, h Cerebral palsy is one of
The classic movement disorder cere- influenza, and Mycoplasma pneu- the most common
bral palsy phenotype is dystonic cere- moniae, among others. Whether causes of movement
bral palsy, although other terms such these encephalitides are due to direct disorders in children.
as dyskinetic cerebral palsy or choreo- invasion of the central nervous sys- Detailed clinical history,
athetoid cerebral palsy are sometimes tem by the microorganism or are examination, and
used depending on the movement postinfectious or immune mediated neuroimaging may
phenomenology. Classic teaching is is not always clear; both mechanisms aid diagnosis,
appear to be possible and may over- although alternative
that dystonic cerebral palsy occurs as
lap. One of the most important exam- diagnoses should be
a consequence of hypoxic ischemic
sought in cases that
injury, predominantly to the subcorti- ples of this overlap between infection
present atypically.
cal structures during early brain devel- and autoimmunity is the postYherpes
simplex encephalitis relapse syn- h Encephalitis may be
opment. Although a clear history of
drome. In the weeks after presenta- associated with a wide
brain injury is present in a proportion
spectrum of causative
of cases, no such history exists in a tion with typical herpes simplex
organisms and can
significant proportion, and many pa- encephalitis, 20% of patients will re-
manifest with both
tients with dystonic cerebral palsy lapse with encephalopathy and move- hyperkinetic and
have a normal MRI. It is increasingly ment disorders (typically chorea). hypokinetic phenotypes.
likely that some patients labeled as During the relapse, the CSF no longer
h N-Methyl-D-aspartate
having dystonic cerebral palsy in fact demonstrates positive herpes simplex
encephalitis is an
have a genetic movement disorder or polymerase chain reaction, but in- emerging important
another disease mechanism. A positive stead the CSF harbors autoantibod- cause of cerebral
family history of neurologic disease; ies against the N-methyl-D-aspartate autoimmune disease,
a fluctuating course; apparently normal (NMDA) receptor.8,9 These patients and both hyperkinetic
early development; a normal MRI; and benefit from immunosuppression. and hypokinetic
clinical features such as progressive This syndrome therefore represents movement disorders
dystonia, exercise-induced dystonia, or a prototypic example of infection- are reported as part
oculogyric crises are some features triggered autoimmunity.10 of the clinical
disease spectrum.
that should alert the clinician to the
possibility this is not typical dystonic Postinfectious/Autoimmune
cerebral palsy. Causes
The presence of movement disorders
Infectious Causes in the context of encephalitis is
Movement disorders do not com- strongly predictive of an autoimmune
monly result from direct infectious encephalitis, particularly antiYNMDA
causes of encephalitis but are more receptor encephalitis.11 Almost all
typically observed in autoimmune children with antiYNMDA receptor
encephalitis. However, a number of encephalitis have movement disor-
infections can affect subcortical struc- ders, and young children often have
tures, resulting in movement disor- generalized chorea and dystonia that
ders. The most important cause may be impairing, whereas older chil-
worldwide is Japanese B encephalitis, dren and adults are more likely to
which remains endemic in certain have more localized movements of the
parts of the world, especially Asia, face, such as orobuccal dyskinesia. In
and commonly results in a severe children, almost any movement disor-
dystonic-akinetic phenotype.7 Other der is possible in the course of disease,
infections that can affect subcortical and the movements typically evolve
structures and result in movement over the course of disease, with cho-
disorders include Epstein-Barr virus, rea, dystonia, stereotypy, akinesia, and
Continuum (Minneap Minn) 2016;22(4):1159–1185 www.ContinuumJournal.com 1163

Copyright © American Academy of Neurology. Unauthorized reproduction of this article is prohibited.


Movement Disorders in Childhood

tremor all observed. Arguably the most tures such as seizures and confusion
characteristic movement disorder of (Case 7-1). The syndrome is now very
antiYNMDA receptor encephalitis is recognizable, and, if suspected, immu-
the repetitive stereotypical clonic or nosuppressive therapy should be
tonic posturing that results in purpose- started while awaiting results of CSF
less, repetitive, and sometimes violent and serum NMDA receptor antibody
agitated movements.12 Some of the tests. Early treatment and the willing-
postures can be reminiscent of the syn- ness to use second-line immunother-
drome catatonia. The movements apy (rituximab or cyclophosphamide)
rarely occur in isolation but instead improve outcomes and reduce relapses.13
are part of a complex evolving enceph- Other autoimmune encephalopa-
alopathy syndrome with aphasia, psy- thies include basal ganglia encephalitis,
chosis, agitation, and dysautonomia which is a rare pure basal ganglia in-
as well as nonspecific encephalitis fea- flammatory encephalitis with restricted

Case 7-1
A previously healthy 7-year-old girl presented with a change in behavior and
restlessness. Over the following week, the illness progressed, with increasing
agitation and apparent delusional thoughts with repeated statements of
‘‘I’m going to die.’’ Examination revealed a generalized movement disorder
with repetitive restless movements, including scratching her face and cycling
movements of her legs. In the second week, she had three focal seizures
and became mute. MRI brain was normal, CSF showed 20 monocytes/mm3,
although infectious screening of CSF and blood was negative including
herpes simplex virus, and EEG showed nonspecific slowing. A clinical diagnosis
of antiYN-methyl-D-aspartate (NMDA) receptor encephalitis was made, and
a 5-day course of IV methylprednisolone was started in addition to IV
immunoglobulin (IVIg). Ten days later, she showed no apparent improvement
and remained mute and unresponsive to her parents and environment
despite apparently being awake. She only slept for 2 hours a night and had
ongoing purposeless stereotypical repetitive movements of her limbs and
face. Fourteen days after commencing immunotherapy, the CSF NMDA
receptor antibodies returned positive, and, given the lack of improvement,
rituximab 375 mg/m2 weekly for 4 weeks was started. Ten days after starting
rituximab, she started to make improvements. Her movement disorder
evolved to dystonic posturing, but she became more interactive and less
agitated. After a 2-month admission, she was discharged on a 6-month total
oral prednisolone taper. She remained B-cell depleted for 6 months and
continued to improve. At 12 months, all of her symptoms had resolved,
although her school performance lagged slightly behind.
Comment. This case exemplifies the movement phenomenology in
antiYNMDA receptor encephalitis, with restless stereotypical movements
later followed by dystonia. The case also exemplifies the fact that the
movement disorder rarely occurs in isolation and is almost always accompanied
by other changes in neurologic function, in this case, cognitive changes, loss
of speech, seizures, and agitated psychosis. The case also demonstrates that it
is possible to make a clinical diagnosis and start treatment before getting
confirmatory diagnostic results. A positive antibody result empowers the
clinician to use rituximab or other immunosuppression if the patient does
not adequately respond to first-line therapy.

1164 www.ContinuumJournal.com August 2016

Copyright © American Academy of Neurology. Unauthorized reproduction of this article is prohibited.


KEY POINTS
inflammation of the basal ganglia, typi- rare syndrome considered part of the h Sydenham chorea
cally presenting in young children with stiff person syndrome spectrum and is continues to be an
dystonia and akinesia or sometimes associated with glycine receptor anti- important cause of
chorea, plus change in behavior.14 Early bodies.17 More common in adults, immune-mediated
use of steroids, immunoglobulin, and this syndrome has been described in chorea worldwide
plasma exchange can improve outcomes young children who present with and can be associated
and reduce the chance of residual mild encephalopathy, irritability, rigid- with significant
atrophy and gliosis of the basal ganglia. ity, and stimulus-induced startle, long-term morbidity.
These patients may harbor autoanti- which is distressing. The syndrome is h Features of opsoclonus
bodies against dopamine-2 receptor. an autoimmune version of genetic myoclonus ataxia
Sydenham chorea is still endemic in hyperekplexia, and, given the immuno- syndrome should
the world and remains one of the more therapy responsiveness of PERM, it is prompt screening for
common causes of acute chorea. Trig- important to recognize. Autoantibodies neural crest tumors
gered by "-hemolytic streptococci as against DPPX, a subunit of potassium and initiation
of immunotherapy.
part of rheumatic fever, Sydenham channels, can be associated with an
chorea is a neuropsychiatric syndrome autoimmune encephalitis with symp-
with acute behavioral change in addi- toms of diarrhea, weight loss, enceph-
tion to chorea. Although the disease alopathy, and movement disorders
is usually monophasic and self- such as myoclonus, rigidity, and startle.18
resolving, a significant proportion of Autoantibodies against ,-aminobutyric
patients are left with minor chorea acid (GABA)-A receptors can sometimes
or residual neuropsychiatric disease. be associated with autoimmune en-
For this reason, it can be argued that cephalitis and movement disorders,
patients should be treated with acute although status epilepticus appears to
immunotherapy as some evidence ex- be the dominant phenotypic associa-
ists that this can improve short-term tion of this autoantibody.19
outcomes,15 although long-term data Opsoclonus myoclonus ataxia syn-
are lacking. drome is a recognizable entity that
Infection-mediated tics and emo- can be paraneoplastic (neural crest
tional lability, either as a poststrep- tumors), triggered by infection, or idio-
tococcal phenomenon (pediatric pathic. The diagnosis is clinical, and
autoimmune neuropsychiatric disor- no defining autoantibody is known.
ders associated with streptococcal Suspicion should result in screening
infections [PANDAS]) or other post- for neural crest tumors and initiation
infectious phenomenon (pediatric of immunotherapy. Some patients
acute neuropsychiatric syndrome appear to respond quickly to steroids
[PANS]), are uncommon. Although tics and IV immunoglobulin (IVIg), where-
are part of PANS, acute anxiety, separa- as others require rituximab or more
tion anxiety, and obsessive-compulsive chronic immunosuppression. General
symptoms with repetitive urination principles of treatment include early
are more characteristic features of intervention and induction of com-
PANS than PANDAS.16 Although these plete remission to avoid residual
entities are clearly infection associated symptoms as they may infer ongo-
and may benefit from antibiotics or ing inflammation and risk of perma-
immunomodulation, the evidence that nent neurocognitive disability.20 More
these disorders are definitely autoim- aggressive intervention appears to
mune is lacking at this time. have changed the natural history of
Progressive encephalopathy with this condition, with better outcomes
rigidity and myoclonus (PERM) is a now described.21
Continuum (Minneap Minn) 2016;22(4):1159–1185 www.ContinuumJournal.com 1165

Copyright © American Academy of Neurology. Unauthorized reproduction of this article is prohibited.


Movement Disorders in Childhood

KEY POINTS
h Acute presentation of a Although rare, the emergence of dyskinesia in (often genetic) epilep-
new movement disorder such autoimmune encephalopathies sies, amphetamine or other stimulant
with rapid evolution has radically changed the clinical ap- use or misuse, withdrawal movement
should prompt proach to the child with acute autoim- disorders such as with midazolam
neuroimaging to mune movement disorders. Rather infusion withdrawal, and cancer drug
exclude a structural or than observation and symptomatic use resulting in movement disorders,
vascular cause. therapy, increasing willingness exists often with leukoencephalopathies.23
h Iatrogenic etiologies to use immunotherapies, as early rec-
should be considered as ognition and intervention appear to Treatment Strategies for
a differential diagnosis improve outcomes. Acquired Movement Disorders
for movement disorders, The primary approach to the child with
and a detailed drug Stroke and Other an acquired movement disorder is
history and toxicology Space-Occupying Lesions defining the etiology and then treating
screen may aid in Any patient presenting with an acute the underlying cause. Defining and
the diagnosis of
movement disorder requires neuroim- treating the cause of disease in a timely
such disorders.
aging, and MRI is the optimal modal- fashion provides the best chance of
h Acquired movement ity. Movement disorders that affect a good outcome and reduces the
disorders have multiple one side of the body, referred to as potential destructive nature of disease.
potential causes.
hemidystonia or hemichorea, can be Symptomatic treatment of acquired
Although investigations
caused by acute vascular events, such as movement disorders is generally unsat-
such as MRI, CSF
analysis, and
stroke or vasculitis, and MRI with isfactory, and patients with autoim-
autoantibody testing are diffusion-weighted imaging is part of mune movement disorders appear to
important, it cannot be the diagnostic pathway. Moyamoya dis- be vulnerable to drug adverse events;
overemphasized that a ease and other vasculopathies may for example, neuroleptic drugs can
careful clinical history present with chorea, and surgical inter- induce akinesia and neuroleptic malig-
and examination for vention can improve the movement nant syndrome in Sydenham chorea
specific movement disorder and reduce the risk of ongoing and antiYNMDA receptor encephalitis,
disorder phenomenology stroke.22 Likewise, tumors or other respectively.24 Agents such as benzo-
is essential. The focus space-occupying lesions involving the diazepines or "2 -agonists may be
should be on defining subcortical structures may present with safer treatments for acquired move-
the etiology and
acute or subacute onset of movement ment disorders, but the evidence base
providing specific
disorders, and imaging is diagnostic. is limited.
targeted intervention.
Early recognition and
Toxic/Iatrogenic Causes GENETIC MOVEMENT DISORDERS
intervention are
increasingly emphasized. Iatrogenic or toxic mechanisms are Advances in molecular genetic tech-
important causes of movement disor- niques, such as chromosomal micro-
ders, but they are usually moderately array studies, targeted next-generation
straightforward to diagnose as a clear multiple gene panels, and whole
temporal association of drug usage exome/genome sequencing strategies,
and onset of movement disorder usu- have led to a significant increase in the
ally exists. Direct questioning regard- diagnosis of genetic movement disor-
ing potential drug ingestion is ders of childhood. Furthermore, a
therefore an important part of the large number of new genetic disor-
clinical history in children who acutely ders have been identified and clini-
present with involuntary movements cally characterized.
or postures. More common asso-
ciations include neuroleptic-induced Primary Genetic Dystonia
dystonia/neuroleptic malignant syn- Traditionally, genetic dystonias have
drome, antiepileptic drugYinduced been classified according to a DYT
1166 www.ContinuumJournal.com August 2016

Copyright © American Academy of Neurology. Unauthorized reproduction of this article is prohibited.


KEY POINT
number assigned to each disease locus. adolescence, with dystonia usually h Primary genetic
Paroxysmal disorders have also been beginning in one limb (Supplemental dystonias are defined
assigned DYT numbers and are Digital Content 7-1, links.lww.com/ as those generally
discussed later in this article. The most CONT/A185). In many patients, gener- associated with normal
common forms presenting early in life alized dystonia usually ensues within brain imaging. In
are DYT1, DYT5, and DYT11 dystonia, 5 years of presentation (Supplemental childhood, reported
but DYT12, DYT23, and DYT26 may Digital Content 7-2, links.lww.com/ forms include DYT1,
also present in childhood (Table 7-2). CONT/A186). DYT5, DYT11, DYT12,
DYT1 early-onset primary generalized DYT5 represents disorders of neu- DYT23, and DYT6.
torsion dystonia is inherited in an rotransmission secondary to inherited
autosomal dominant manner with in- dopa-responsive defects in the dopa-
complete penetrance (30% to 40%).25 mine synthesis pathway. Guanosine
It is caused by mutations in TOR1A triphosphate cyclohydrolase deficiency,
encoding torsinA, postulated to have also known as Segawa syndrome, is due
a role in the regulation of subcellular to heterozygous mutations in GCH1,
compartments such as the nuclear with incomplete penetrance.26 Affected
envelope and endoplasmic reticulum. patients present from early child-
The vast majority of patients harbor hood with limb dystonia, which can
the common GAG deletion. Symptom often mimic the symptoms and signs
onset is usually in childhood or early of diplegic cerebral palsy. Atypical

TABLE 7-2 DYT Classification of Primary Genetic Dystonia in Childhood

Paroxysmal/ Pattern of
DYT Number Nonparoxysmal Clinical Presentation Inheritance Causative Gene
DYT1 Nonparoxysmal Early-onset primary Autosomal dominant TOR1A
generalized idiopathic
torsion dystonia
DYT5a Nonparoxysmal Dopa-responsive Autosomal dominant GCH1
dystonia
DYT5b Nonparoxysmal Tyrosine hydroxylase Autosomal recessive TH
deficiency
DYT8 Paroxysmal Paroxysmal Autosomal dominant MR1
nonkinesogenic
dyskinesia
DYT10 Paroxysmal Paroxysmal kinesogenic Autosomal dominant PRRT2
dyskinesia
DYT11 Nonparoxysmal Myoclonus dystonia Autosomal dominant SGCE
DYT12 Nonparoxysmal Rapid-onset Autosomal dominant ATP1A3
dystonia-parkinsonism
DYT18 Paroxysmal Paroxysmal Autosomal dominant SLC2A1
exercise-induced
dyskinesia
DYT23 Nonparoxysmal Myoclonus dystonia Autosomal dominant CACNA1B
DYT26 Nonparoxysmal Myoclonus dystonia Autosomal dominant KCTD17

Continuum (Minneap Minn) 2016;22(4):1159–1185 www.ContinuumJournal.com 1167

Copyright © American Academy of Neurology. Unauthorized reproduction of this article is prohibited.


Movement Disorders in Childhood

disease presentations, including writer’s encoding the "3 subunit of the Na+/
cramp, dystonic tremor, and exercise- K+Yadenosine triphosphatase (ATPase)
induced dystonia, have all been de- pump. Affected patients usually present
scribed.27 Associations of GCH1 carriers acutely or subacutely (from over a few
manifesting early-onset Parkinson dis- minutes to months) with rostrocaudal
ease have been reported.28 DYT5b progression of dystonia-parkinsonism
tyrosine hydroxylase deficiency repre- and significant bulbar dysfunction.
sents another form of dopa-responsive It has become clear that the clinical
dystonia but is more complex, often spectrum of ATP1A3-related disease is
with concurrent features of parkinson- broad, encompassing a number of other
ism, other motor features, and neu- overlapping phenotypes, including al-
rodevelopmental delay.29 Although ternating hemiplegia of childhood as
response to levodopa can lead to sig- well as cerebellar ataxia, areflexia, pes
nificant clinical benefit in type B cavus, optic atrophy, and sensorineural
(hypokinetic rigid phenotype), the hearing loss (CAPOS) syndrome.31 The
earlier-onset type A subtype, associated reasons for this phenotypic pleiotropy
with neonatal encephalopathy, is more may be mutation dependent but are
drug resistant. Tyrosine hydroxylase yet to be fully elucidated.
deficiency is likely a phenotypic spec- Heterozygous mutations of CACNA1B
trum with a number of intermediate and KCTD17 have been described in
forms. Patients with tyrosine hydroxy- extended families with childhood-
lase deficiency often require much onset myoclonus dystonia.32,33 Car-
higher doses of levodopa to produce diac arrhythmias are additionally
a therapeutic response than in auto- reported in those with CACNA1B vari-
somal dominant GCH1 deficiency. ant p.R1389H. CaV2.2 channels car-
DYT11, or myoclonus dystonia syn- ried less current when compared with
drome, has clinical onset in childhood wild-type channels; it is postulated
(median age 5 years) and frequently that such altered channel properties
presents with early myoclonus affect- could affect neurotransmitter release at
ing the upper limbs and neck with excitatory and inhibitory channels.
predominantly upper (but also lower) Screening of another patient cohort
limb dystonia.30 More pronounced with myoclonus dystonia for this mis-
truncal involvement is evident later sense change has not identified fur-
in childhood and adolescence. Al- ther cases, with observation of this
cohol responsiveness is sometimes rare variant in control populations.
reported in older patients. Comorbid The significance of this mutation re-
neuropsychiatric features are com- mains yet to be fully determined.34
monly reported. Affected patients KCTD17Yassociated protein is pos-
have heterozygous mutations in SGCE, tulated to show significant putami-
encoding the *-sarcoglycan protein. nal expression, and KCTD17 gene
Disease mechanisms are not yet fully mutations result in impairment of en-
elucidated, but mutations are postulated doplasmic reticulumYdependent cal-
to impair targeting of *-sarcoglycan to cium signaling.
the plasma membrane, affecting pro-
teasomal degradation, protein glycosyl- Genetic Chorea
ation, and ectodomain shedding. A number of genes causing primarily
DYT12 is a rarer cause of genetic choreiform dyskinesia have been re-
dystonia-parkinsonism due to autoso- ported. Often children with such con-
mal dominant mutations in ATP1A3, ditions manifest chorea as part of a
1168 www.ContinuumJournal.com August 2016

Copyright © American Academy of Neurology. Unauthorized reproduction of this article is prohibited.


KEY POINT
complex neurologic disorder, as exem- disorder.38 MICU1 is a regulator of h The spectrum of genetic
plified by FOXG1 syndrome.35 mitochondrial calcium uniporter choreas is expanding
Benign hereditary chorea is a (MCU), a Ca(2+)-selective ion chan- and now includes
childhood-onset hyperkinetic move- nel located within the inner mitochon- disorders due to
ment disorder due to mutations in drial membrane Ca(2+) uniporter NKX2-1-AS1and
NKX2-1-AS1 (also known as NKX2-1 complex, and mutations disrupt mito- ADCY5 mutations
and TITF1).36 Hypotonia is commonly chondrial Ca (2+) signaling. as well as those
described in infancy, with onset of Mutations in ADCY5 are the most associated with more
chorea occurring from 2.5 to 3 years recently recognized significant cause complex neurologic
of age. Chorea is usually generalized of progressive chorea in children. disorders such as
FOXG1-related disease.
and may be exacerbated by excite- Originally reported in patients
ment or stress. Patients frequently presenting with familial dyskinesia
manifest neonatal/infantile respiratory with facial myokymia,39 ADCY5-related
symptoms (including respiratory dis- disease comprises a wide clinical spec-
tress syndrome, obstructive small air- trum of disorders, including those
ways disease, and chronic interstitial that mimic dyskinetic cerebral palsy
lung disease) and thyroid abnormali- and conditions resembling TITF1
ties (congenital hypothyroidism). benign hereditary chorea (Case 7-2).40
NKX2-1-AS1, from the natural killer A relapsing-remitting disease course
(NK) gene family of highly conserved is often evident, and severe sleep-
homeodomain-containing transcrip- disrupting movements are frequently
tion factors, is postulated to have a reported. Unlike TITF1 benign he-
role in embryonic development of reditary chorea, motor symptoms
the brain, lung, and thyroid gland. are typically progressive. Eliciting a
The combination of chorea with thy- history of sleep disturbance and dis-
roid abnormalities is also described ease progression can thus aid in clini-
in Allan-Herndon-Dudley syndrome, cal differentiation from benign
an X-linked disorder associated hereditary chorea and cerebral palsy.
with mutations in SLC16A2 encod- ADCY5 is a membrane-bound adenylyl
ing the monocarboxylate transporter cyclase that generates adenosine 3¶,5¶-
8 (MCT8).37 Mutations lead to loss of cyclic monophosphate (cAMP) from
function of this thyroid transporter, an adenosine triphosphate (ATP).
thereby impairing triiodothyronine ADCY5 shows high expression in the
transport capacity. Triiodothyronine brain striatum and in the nucleus
has an important role in oligodendro- accumbens, where it is responsible
cytes, inducing differentiation of the for 80% of adenylate cyclase activation.
precursor as well as acting as a survival "-Adrenergic agonists stimulate ADCY5
factor in these cells and influencing through a G-proteinYcoupled receptor,
the distribution of myelin-related pro- which leads to a conformational change
teins. Individuals with Allan-Herndon- in the protein, facilitating formation
Dudley syndrome present with pro- of a catalytic pocket into which ATP
found hypotonia and early limb spas- binds. Pathogenic variants are pre-
ticity. A complex movement disorder dicted to lead to gain of function,
evolves over time, with prominent cho- i n c re a s i n g A D C Y 5 e n z y m e a c -
rea and dystonia. tivity, either through "-adrenergic
In 2014, mutations in MICU1 were stimulation of the signal transduc-
identified in patients with cognitive tion pathway or by the interaction
impairment, proximal myopathy, and of the protein with other regula-
a progressive choreiform movement tory molecules.
Continuum (Minneap Minn) 2016;22(4):1159–1185 www.ContinuumJournal.com 1169

Copyright © American Academy of Neurology. Unauthorized reproduction of this article is prohibited.


Movement Disorders in Childhood

KEY POINTS
h Genetic paroxysmal
movement disorders
Case 7-2
Following an unremarkable pregnancy, a male infant was born by emergency
of childhood have been
cesarean delivery for fetal bradycardia/decelerations. His parents were fit and
increasingly
healthy, with no significant family history of note. He was born in good
recognized, and some
condition, with excellent Apgar scores (9/10 at 1 and 5 minutes), and had an
have important
uneventful early neonatal course. Abnormal involuntary movements were
treatment implications.
noted from midinfancy, which were initially paroxysmal in nature, often
Paroxysmal movement
occurring in clusters several times a day, with periods of relapse and remission.
disorders may often
Choreiform limb movements were reported, with orolingual dyskinesia
coexist with other
and mild truncal athetosis. The movement disorder progressed through early
episodic conditions,
childhood, eventually becoming nonparoxysmal and continuous in nature.
including epilepsy
Involuntary movements began to disrupt his sleeping pattern. He did not
and migraine.
show clinical response to a number of medications, including carbamazepine,
h Nonepileptic myoclonus levetiracetam, and tetrabenazine. Detailed neurologic investigation revealed
should prompt testing for normal brain and spine MRI, as well as normal blood, urine, and CSF screening
genetic and for metabolic disorders. Routine diagnostic testing for NKX2-1-AS1, SLC2A1,
neurometabolic and SGCE were negative. Whole exome sequencing undertaken on a
syndromes. research basis identified a previously reported mutation in ADCY5, which
was confirmed to be de novo.
Comment. This case clearly exemplifies the typical disease pattern of
ADCY5-related disease. Although dyskinetic cerebral palsy may be in the
differential diagnosis here, a number of features in this child’s history should
prompt investigation of an alternative cause, including the relatively uneventful
antenatal/postnatal course, evidence of normal neuroimaging, and clear
disease progression with worsening of symptoms. Testing for benign hereditary
chorea is reasonable, but negative sequencing for NKX2-1-AS1 should alert
the clinician to look for other causes of benign hereditary chorea mimics
and initiate ADCY5 testing. Further clinical clues toward this diagnosis
include the disruption of sleep and progressive nature of disease.

Genetic Paroxysmal Movement ders such as epilepsy and migraine.


Disorders Recognition of such syndromes is of
A number of genetic movement dis- significant clinical importance as many
orders may present episodically are amenable to treatment strategies
(Table 7-3). Such paroxysmal dyski- (Table 7-3).
nesias typically present in childhood,
often in children with normal neuro- Genetic Myoclonus
development and behavior. Paroxysmal Nonepileptic myoclonus is reported in
movements may be dystonic, chorei- a number of genetic conditions, often
form, or ataxic and are often associated in association with other movement
with specific triggers.41 Detailed clinical phenotypes. It is a prominent feature
history with direct questioning for of myoclonus dystonia syndrome, as
precipitants of the movement disorder discussed earlier in this article, but is
should be undertaken to aid in diag- also reported in a number of inherited
nosis. Many episodic movement dis- inborn errors of metabolism.
orders are caused by mutations in
genes encoding channels; therefore, Genetic Ataxia
it is not surprising that such genes also A number of predominantly ataxic
account for other paroxysmal disor- disorders of genetic origin have been

1170 www.ContinuumJournal.com August 2016

Copyright © American Academy of Neurology. Unauthorized reproduction of this article is prohibited.


TABLE 7-3 Clinical Features of Common Genetic Paroxysmal Disorders

Age of Reported Duration Causative


Disorder Onset Triggers of Episode Treatment Gene Allelic Disorders
Paroxysmal Infancy to Sudden Short: seconds Carbamazepine, PRRT2 Infantile convulsions
kinesogenic fourth decade movement to minutes phenytoin choreoathetosis, familial
dyskinesia Benign paroxysmal torticollis
of infancy
Familial migraine
Paroxysmal Infancy to Caffeine, alcohol, Longer: minutes Benzodiazepines MR1 None known
nonkinesogenic fourth decade stress/anxiety, to hours

Continuum (Minneap Minn) 2016;22(4):1159–1185


dyskinesia sleep deprivation
Paroxysmal Infancy to Exercise, stress, Longer: minutes Ketogenic diet SLC2A1 Glucose transporter deficiency
exercise-induced adulthood fasting to hours phenotypes like absence
dyskinesia epilepsy, myoclonic-atonic
epilepsy, generalized epilepsy,
early infantile epileptic
encephalopathy
Alternating Infancy Unclear Minutes Flunarizine ATP1A3 Rapid-onset
hemiplegia of to early dystonia-parkinsonism
childhood childhood
Cerebellar ataxia, areflexia,
pes cavus, optic atrophy, and
sensorineural hearing loss
syndrome
Episodic Childhood Sudden movement, Short: seconds Acetazolamide, KCNA1 None known
ataxia type 1 stress, exertion, to minutes carbamazepine
fatigue, illness
Episodic Childhood to Sudden movement, Very long: Flunarizine, CACNA1A Spinocerebellar ataxia 6,
ataxia type 2 adolescence stress, exertion, hours to days acetazolamide familial hemiplegic migraine
exercise, fatigue,
caffeine, alcohol,
cigarettes

Copyright © American Academy of Neurology. Unauthorized reproduction of this article is prohibited.


www.ContinuumJournal.com
1171
Movement Disorders in Childhood

KEY POINT
h A number of recessive described in childhood (Table 7-4), other cerebellar features of tremor,
genetic ataxias have and many are complex progressive head titubation, and nystagmus, as
been identified, neurologic disorders. well as myoclonus, dystonia, hypo-
including Friedreich Friedreich ataxia is the most com- reflexia, and retinitis pigmentosa.43
ataxia and ataxia monly reported, with clinical features Disease is caused by mutations in
telangiectasia. of a progressive, mainly sensory, cer- the "-tocopherol transfer protein, with
ebellar ataxia with speech difficulties, impaired incorporation of vitamin E
absent reflexes, pyramidal tract signs into very low density lipoproteins and
with weakness, hypertrophic cardio- subsequent excess loss of vitamin E.
myopathy, diabetes mellitus, and sen- Low vitamin E levels are detectable on
sorineural deafness. 42 Expanded blood testing.
guanine-adenine-adenine (GAA) re- Ataxia telangiectasia presents with
peats within intron 1 of frataxin (FXN) gait disturbance and truncal instability
are thought to impair exon splicing in early childhood, but choreiform
with reduced protein expression. movements and dystonia are also
Frataxin is postulated to have a role in described.44 Often, subtle abnormali-
mitochondrial iron metabolism, and ties in ambulation precede the onset
dyshomeostasis is thought to lead to of ocular and cutaneous telangiectasia.
increased free radical damage and oxi- Patients are at risk of immunodefi-
dative stress. Ataxia with vitamin E de- ciency (secondary to low immunoglob-
ficiency can mimic many of the features ulins) and lymphoreticular neoplasia.
seen in Friedreich ataxia and is charac- A raised "-fetoprotein can be a useful
terized by a progressive ataxia with disease marker. The disease is due to

TABLE 7-4 Childhood-Onset Genetic Disorders With Prominent Ataxia

Movement
Condition Inheritance Gene(s) Age of Presentation Symptomatology
Friedreich ataxia Autosomal FXN Childhood: 2Y15 years Ataxia
recessive
Ataxia with isolated Autosomal TTPA Childhood Ataxia, other cerebellar
vitamin E deficiency recessive features such as
tremor, nystagmus
Ataxia telangiectasia Autosomal ATM Early childhood: Ataxia, chorea,
recessive G5 years athetosis, dystonia,
eye movement
abnormalities
Ataxia with oculomotor Autosomal APTX (ataxia Mid childhood and later Ataxia, oculomotor
apraxia recessive with oculomotor childhood apraxia, chorea,
apraxia type 1) athetosis
SETX (AOA2)
Ataxia of Autosomal SACS Childhood to adulthood Ataxia
Charlevoix-Saguenay recessive
Spinocerebellar ataxia Autosomal Many genes, Childhood to adulthood Ataxia, dystonia, and
dominant or including ATXN1, chorea also sometimes
autosomal ATXN2, ATXN3, present, parkinsonism
recessive ATXN7

1172 www.ContinuumJournal.com August 2016

Copyright © American Academy of Neurology. Unauthorized reproduction of this article is prohibited.


KEY POINT
mutations in ATM, encoding a DNA cally, symptoms resemble those seen h Clinical parkinsonism
repair protein. Clinically similar to in idiopathic Parkinson disease, with in childhood should
ataxia telangiectasia are two forms of bradykinesia, resting tremor, rigidity, prompt diagnostic
autosomal recessive ataxia with oculo- and hypomimia. Pure parkinsonism is testing for underlying
motor apraxia due to mutations in rarely reported, with most patients genetic and
APTX (ataxia with oculomotor apraxia manifesting other movement disorders, neurometabolic etiologies.
type 1) and SETX (ataxia with oculo- most commonly dystonia (dystonia-
motor apraxia type 2).45,46 Both types parkinsonism) but also chorea and
present with a distinctive eye move- myoclonus. Cognitive impairment may
ment disorder in the context of ataxia, also be a comorbid feature. A number
chorea, distal sensory axonal neuro- of etiologies have been reported, in-
pathy, and progressive cerebellar atro- cluding disorders of monoamine syn-
phy on MRI. thesis, metabolism, and transport as
Ataxia of Charlevoix-Saguenay is a well as other inborn errors of metabo-
recently described condition originally lism (including mitochondrial disor-
in individuals originating from Quebec, ders), disorders of brain metal ion
Canada. Ataxia of Charlevoix-Saguenay accumulation, and juvenile-onset ge-
is caused by mutations in SACS, with netic parkinsonism due to mutations
over 100 mutations reported to date in the PARK genes (Table 7-3). Recog-
in patients of different ethnicities.47 nition of clinical features can be key to
Ataxia, dysarthria, spasticity, weakness, diagnosis, but adjunctive tests, such as
distal sensorimotor neuropathy, and CSF neurotransmitter analysis, CSF and
nystagmus are commonly reported clin- blood lactate, muscle biopsy, dopamine
ical symptoms. Distinctive neurora- transporter single-photon emission
diologic features include atrophy of computed tomography (DAT-SPECT)
the superior vermis and cerebellum nuclear imaging, and molecular genetic
with pontine linear hypointensities. studies will often be needed to confirm
A number of both dominant and re- the diagnosis.
cessive spinocerebellar ataxia disorders
may also present in childhood with Complex Inherited
prominent ataxia and gait instability, as Neurometabolic Syndromes
well as other movement phenotypes Movement disorders are often a prom-
(dystonia, chorea), speech distur- inent clinical feature in a number of
bance, and progressive motor dysfunc- inborn errors of metabolism that pres-
tion (Table 7-4). The increasing ent in childhood. Many of these
availability of targeted multiple gene conditions are complex, and patients
panel testing for the ever-expanding often manifest a number of different
list of spinocerebellar ataxia and other movement phenotypes. Frequently,
ataxia genes has improved the overall multisystem disease involvement ex-
diagnostic rate for these childhood- ists, with cognitive and motor delay.
onset ataxias,48 although detection of Dystonia, chorea, athetosis, ballismus,
certain mutations (eg, trinucleotide tremor, myoclonus, and parkinsonism
repeats, intronic variants) can still are all described in disorders of mo-
be challenging. noamine synthesis and transport
(Table 7-5) (Supplemental Digital
Genetic Parkinsonism Content 7-3, links.lww.com/CONT/
Rarely, genetic disorders that present A187). 27 The presence of eye move-
predominantly with infantile or juve- ment disorders (eg, oculogyric crises,
nile parkinsonism are reported.2 Typi- ocular flutter, saccade initiation failure)
Continuum (Minneap Minn) 2016;22(4):1159–1185 www.ContinuumJournal.com 1173

Copyright © American Academy of Neurology. Unauthorized reproduction of this article is prohibited.


Movement Disorders in Childhood

TABLE 7-5 Pediatric Parkinsonism and Complex Neurometabolic Syndromes

Group of Disorders Useful Diagnostic Causative Genes


and Syndrome/ Typical Age Reported Motor Tests/Disease (Molecular
Condition of Onset Phenotypes Biomarkers Confirmation)
Monogenic early-onset
parkinsonism
(PARK loci)
Juvenile Childhood/ Parkinsonism DAT-SPECT, PARK2, DNAJC6,
parkinsonism adolescence/ prolactin PINK1, DJ1,
early adulthood SNCA, LRRK2,
VPS35, EIF4G1,
SYNJ1, PDE8B
Monoamine neurotransmitter
disorders
Autosomal dominant Infancy/ Dystonia, CSF neurotransmitters, GCH1
GTP cyclohydrolase childhood/ parkinsonism, phenylalanine loading,
deficiency adulthood tremor trial of levodopa
Autosomal recessive Infancy/early Dystonia, CSF neurotransmitters GCH1
GTP cyclohydrolase childhood hypotonia
deficiency
6-Pyruvoyl Infancy Parkinsonism, CSF neurotransmitters PTS
tetrahydropterin dystonia, chorea,
synthase deficiency oculogyric crises
Sepiapterin reductase Infancy Dystonia, oculogyric CSF neurotransmitters, SPR
deficiency crises, dyskinesia, phenylalanine loading,
‘‘cerebral palsy mimic’’ serum prolactin
Dihydropteridine Infancy Dyskinesia, CSF neurotransmitters QDPR
reductase deficiency choreoathetosis,
dystonia, tremor
Aromatic L-amino Infancy Hypotonia, dystonia, CSF neurotransmitters, DDC
acid decarboxylase dyskinesia, aromatic L-amino
deficiency hypokinesia, acid decarboxylase
oculogyric crises plasma activity,
serum prolactin,
urine organic acids
decarboxylase
Dopamine transporter Infancy Dystonia, chorea, CSF neurotransmitters, SLC6A3
deficiency syndrome orolingual dyskinesia, DAT-SPECT
axial hypotonia,
later parkinsonism,
oculogyric crises and
eye movement
disorder, ‘‘cerebral
palsy mimic’’
Brain dopamine Infancy Dystonia, parkinsonism, None known SLC18A2
serotonin transport axial hypotonia,
disease dyskinesia,
oculogyric crises Continued on page 1175

1174 www.ContinuumJournal.com August 2016

Copyright © American Academy of Neurology. Unauthorized reproduction of this article is prohibited.


TABLE 7-5 Pediatric Parkinsonism and Complex Neurometabolic Syndromes Continued from page 1174

Group of Disorders Useful Diagnostic Causative Genes


and Syndrome/ Typical Age Reported Motor Tests/Disease (Molecular
Condition of Onset Phenotypes Biomarkers Confirmation)
Other inborn errors
of metabolism
Lysosomal disorders
GM1 gangliosidosis Infancy Ataxia, dystonia, Lysosomal enzyme GLB1
chorea, parkinsonism, assay
GM2 gangliosidosis Infancy GM2A
supranuclear gaze
Niemann-Pick C Infancy palsy, Gaucher NPC1, NPC2
Gaucher disease Infancy carrier (Parkinson GBA1
Neuronal ceroid Infancy disease risk) CLN1YCLN11
lipofuscinosis

Organic acidurias
Glutaric aciduria Infancy Hyperkinesia, Organic acids: GCDH
type 1 dystonia, Glutaric acid,
choreoathetosis 3-hydroxyglutaric acid
Fibroblast assay:
Reduced glutaryl-
coenzyme A
dehydrogenase
activity
Acylcarnitines:
Glutarylcarnitine
MRI: Enlarged
subdural spaces,
frontotemporal
atrophy, bat wing
dilatation of
sylvian fissures
Methylmalonic Infancy Dystonia, hypotonia Organic acids: MUT
aciduria Elevated
methylmalonic acid
Propionic acidemia Infancy Dystonia, Organic acids: PCCB, PCCA
choreoathetosis, Elevated
spasticity propionic acid
Aminoacidopathies
Homocystinuria Infancy Dystonia Abnormal plasma CBS, MTRR,
and urine MTR, MMACHC,
homocysteine MTHFR
Hartnup disease Infancy Ataxia, spasticity, SLC6A19
dystonia

Continued on page 1176

Continuum (Minneap Minn) 2016;22(4):1159–1185 www.ContinuumJournal.com 1175

Copyright © American Academy of Neurology. Unauthorized reproduction of this article is prohibited.


Movement Disorders in Childhood

TABLE 7-5 Pediatric Parkinsonism and Complex Neurometabolic Syndromes Continued from page 1175

Group of Disorders Useful Diagnostic Causative Genes


and Syndrome/ Typical Age Reported Motor Tests/Disease (Molecular
Condition of Onset Phenotypes Biomarkers Confirmation)
Disorders of purine
metabolism
Lesch-Nyhan Infancy Choreoathetosis, Hyperuricemia HPRT1
syndrome dystonia, hypotonia,
spasticity
Disorders of
creatine metabolism
Guanidinoacetate Infancy Dystonia, Magnetic resonance GAMT
N-methyltransferase choreoathetosis spectroscopy (MRS):
deficiency Low creatine peak
Urine: Low
creatine, high
guanidinoacetate
Glucose transporter Infancy/ Dystonia, Low CSF glucose, SLC2A1
deficiency childhood/ choreoathetosis, low CSF to plasma
adulthood ataxia, often glucose ratio
paroxysmal
symptoms
Cerebral folate Infancy Ataxia, dystonia, Low N5- FOLR1, DHFR,
deficiency chorea methyltetrahydrofolate MTHFR, SLC46A1
on CSF analysis
Mitochondrial
disorders
Numerous, including Infancy/ Dystonia, Lactate, organic Numerous
Leigh syndrome, childhood/ myoclonus, acids, muscle biopsy, mitochondrial
POLG1-related adulthood choreoathetosis, respiratory chain DNA point
disease, pyruvate parkinsonism, enzyme analysis mutations and
carboxylase deficiency, tremor, ataxia autosomal genes
Leber hereditary
optic neuropathy
plus dystonia
Striatal disorders
Bilateral striatal Infancy Progressive dystonia MRI: Bilateral ADAR1, ATP6,
necrosis striatal necrosis ATP5, MTND6,
ADAR1: Interferon MTND1, NDUFV1,
signature NUP62,SLC19A3,
SLC25A19

Continued on page 1177


Disorders of metal ion accumulation

1176 www.ContinuumJournal.com August 2016

Copyright © American Academy of Neurology. Unauthorized reproduction of this article is prohibited.


TABLE 7-5 Pediatric Parkinsonism and Complex Neurometabolic Syndromes Continued from page 1176

Group of Disorders Useful Diagnostic Causative Genes


and Syndrome/ Typical Age Reported Motor Tests/Disease (Molecular
Condition of Onset Phenotypes Biomarkers Confirmation)
Disorders of metal
ion accumulation
Iron
Pantothenate Childhood Early gait disturbance, MRI: Eye-of-the-tiger PANK2
kinaseYassociated progressive dystonia, sign in globus pallidus
neurodegeneration spasticity
Blood film
acanthocytes
Retinal pigmentation

Phospholipase Infancy/ Infantile MRI: Brain iron in PLA2G6


A2Yassociated childhood/ neuroaxonal globus pallidus and
neurodegeneration adulthood dystrophy (INAD): substantia nigra
Dystonia, spasticity (NAD, INAD)
Atypical neuroaxonal Cerebellar atrophy/
dystrophy (NAD): gliosis
Dystonia, spasticity
Optic atrophy
PLA2G6-related EEG: Fast rhythms
dystonia- (INAD)
parkinsonism:
subacute onset EMG/nerve
of dystonia- conduction studies:
parkinsonism, Distal sensory axonal
spasticity neuropathy

Dystonia, spasticity MRI: Brain iron in


globus pallidus and
substantia nigra

Mitochondrial Childhood Linear streaking of C19orf12


membrane medial medullary
proteinYassociated lamina between
neurodegeneration globus pallidus
externa and interna
MRI: Brain iron in
substantia nigra
and globus pallidus
and T1 halo in
substantia nigra
Beta propeller Infancy/ Early delay, MRI: Brain iron in WDR45
proteinYassociated childhood stereotypies, later substantia nigra
neurodegeneration development of and globus pallidus
dystonia-parkinsonism
Continued on page 1178

Continuum (Minneap Minn) 2016;22(4):1159–1185 www.ContinuumJournal.com 1177

Copyright © American Academy of Neurology. Unauthorized reproduction of this article is prohibited.


Movement Disorders in Childhood

TABLE 7-5 Pediatric Parkinsonism and Complex Neurometabolic Syndromes Continued from page 1177

Group of Disorders Useful Diagnostic Causative Genes


and Syndrome/ Typical Age Reported Motor Tests/Disease (Molecular
Condition of Onset Phenotypes Biomarkers Confirmation)
Fatty acid Childhood Focal dystonia, Cerebellar and FA2H
hydroxylaseYassociated spasticity with gait brainstem atrophy
neurodegeneration disturbance, later ataxia
White matter
changes
Dystonia, spasticity, MRI: Brain iron in
parkinsonism globus pallidus and
substantia nigra
Coenzyme A synthase Childhood Early-onset spastic- MRI: Brain iron in COASY
proteinYassociated dystonic paraparesis, globus pallidus and
neurodegeneration later parkinsonism substantia nigra,
reminiscent of, but
more subtle than,
pantothenate
kinaseYassociated
neurodegeneration
eye-of-the-tiger sign
Manganese
Manganese Childhood/ Dystonia, MRI: T1-weighted SLC30A10
transporter defect adulthood parkinsonism hyperintensity of
basal ganglia,
midbrain, and
cerebellum
Raised blood
manganese levels
Copper
Wilson disease Childhood/ Dystonia, MRI: T2-weighted ATP7B
adulthood parkinsonism, hyperintensity in
tremor, putamen and thalami,
choreoathetosis, midbrain face-of-
ataxia the-giant-panda sign
Kayser Fleischer
rings on slit-lamp
eye examination
Copper, ceruloplasmin,
urinary copper profile
Other
Hypomyelination Infancy/ Dystonia, MRI: TUBB4A
with atrophy of the childhood choreoathetosis Hypomyelination,
basal ganglia basal ganglia and
and cerebellum cerebellar atrophy
CSF = cerebrospinal fluid; DAT-SPECT = dopamine transporter single-photon emission computed tomography; DNA = deoxyribonucleic acid;
EMG = electromyography; GTP = guanosine triphosphate cyclohydrolase; MRI = magnetic resonance imaging.

1178 www.ContinuumJournal.com August 2016

Copyright © American Academy of Neurology. Unauthorized reproduction of this article is prohibited.


as well as autonomic features (eg, tant cause, accounting for a significant
sweating, nasal congestion, tempera- proportion of patients with bilateral
ture instability, ptosis) can be additional striatal necrosis who present with
clinical features that raise suspicion severe progressive medically intracta-
for these diseases, which can usually ble generalized dystonia.53 An interfer-
be diagnosed on CSF neurotransmit- on signature, measuring upregulation
ter analysis and confirmatory genetic of interferon-stimulated genes, can aid
studies. Patients with cerebral folate in the diagnosis of this bilateral striatal
deficiency, for example, secondary to necrosis subtype. The finding of bilat-
recessive mutations in the gene encod- eral striatal necrosis should prompt
ing the folate transporter FOLR1, can investigation for treatable causes, in-
present with a number of nonspe- cluding thiamine transporter deficiency
cific features, including gait instability, due to mutations in SLC19A3, where
hypotonia, seizures, and acquired micro- initiation of biotin and thiamine sup-
cephaly.49 Ataxia, chorea, and ballismus plementation can lead to marked im-
are also reported in a proportion of provement or even reversal of clinical
these patients. symptoms in tandem with radiologic
Inherited mineral deposition disor- improvement of the striatal features.54
ders, including Wilson disease, Structural abnormalities of the basal
neurodegeneration with brain iron ganglia may also be seen in some
accumulation, and SLC30A10 deficiency genetic hypomyelinating disorders. In
can all present with progressive neuro- 2013, TUBB4A was identified as the
logic phenotypes50Y52 and prominent genetic cause of hypomyelination
extrapyramidal features, including with atrophy of the basal ganglia and
dystonia, chorea, tremor, parkinson- cerebellum.55 Mutations are postu-
ism, and spasticity. Characteristic MRI lated to disrupt tubulin dimerization,
brain findings and molecular genetic microtubule polymerization, or micro-
testing can aid in the diagnosis of tubule stability. Extrapyramidal features,
these disorders of metal ion accumu- namely dystonia, rigidity, and, rarely,
lation (Table 7-5). choreoathetosis, are reported in af-
A number of other inborn errors of fected patients. A number of patients
metabolism can also present with also manifest ataxia and gait instability.
movement disorders, including lyso- TUBB4A-related diseases encompass a
somal disorders, mitochondrial dis- broad spectrum, from hypomyelination
eases, amino acidopathies, organic with atrophy of the basal ganglia and
acidurias, disorders of purine and cerebellum to other hypomyelinating
creatine metabolism, and glucose disorders and also whispering dyspho-
transporter deficiency. Detailed meta- nia (Table 7-5). The reasons for this
bolic screening is therefore often phenotypic pleiotropy remain to be
warranted in patients with complex fully elucidated.56
progressive neurologic syndromes
associated with motor phenotypes Treatment Strategies for
(Case 7-3) (Table 7-5). Genetic Childhood Movement
A number of genes have also been Disorders
identified in infantile and childhood-onset Treatment strategies for children
bilateral striatal necrosis (Table 7-5). with genetic movement disorders are
Mitochondriocytopathies are frequently usually dictated by disease etiology
associated with bilateral striatal necro- and movement disorder pheno-
sis. ADAR1 has emerged as an impor- menology. Children with dystonia of
Continuum (Minneap Minn) 2016;22(4):1159–1185 www.ContinuumJournal.com 1179

Copyright © American Academy of Neurology. Unauthorized reproduction of this article is prohibited.


Movement Disorders in Childhood

Case 7-3
A female infant was born to fit and healthy consanguineous parents. She
was born in good condition and did not require any resuscitation, nor was
she admitted to the neonatal intensive care unit. She was discharged on
day 2 of life. From the early neonatal period, she was irritable, with
difficulty in feeding and a disrupted sleep pattern. The parents reported
that often she would look as if she were half asleep, with her eyes half
closed. She was profoundly hypotonic and had delay in her motor milestones.
By 2 to 3 months of age, abnormal eye movements were noted, and the
family described periods when her eyes would deviate to the right and fixate
in an upward gaze lasting from seconds to several hours. Often during
these periods her body would be arched and hands fisted. The parents also
noted that she always seems to have a cold with nasal congestion, and even
during the winter, her hands and the back of her head were sweaty. During
infancy, she developed a complex movement disorder, with dystonic
posturing of her limbs, truncal athetosis, tremulous hand movements, and
occasional myoclonic jerks. A diagnostic lumbar puncture was undertaken,
and CSF neurotransmitter analysis revealed a low homovanillic acid, low
5-hydroxyindoleacetic acid, and normal pterin profile. CSF 3-O-methyldopa
and 5-hydroxytryptophan were markedly elevated. Further testing
revealed low aromatic L-amino acid decarboxylase plasma enzyme activity.
Molecular testing revealed that she was a compound heterozygote for two
mutations in the DDC gene, one reported in association with aromatic L-amino
acid decarboxylase deficiency and the other a novel 2-base pair deletion.
Comment. This case illustrates a child with a very-early-onset neurologic
disorder, characterized by severe motor delay, hypotonia, and mixed
movement symptomatology. The ptosis and severe hypotonia could indeed
be presenting features of neuromuscular disease, although the complex
movement disorder and presence of autonomic features (sweating, nasal
congestion, ptosis) should prompt CSF neurotransmitter analysis to look for
disorders of monoamine metabolism. This patient’s CSF findings are indeed
classic for aromatic L-amino acid decarboxylase deficiency, later confirmed
by enzyme assay and molecular genetic testing. This patient’s first mutation
has already been reported in the literature associated with aromatic L-amino
acid decarboxylase deficiency. The second mutation, although novel as a
2-base pair deletion, will lead to a frameshift with significant alteration to
the protein and likely premature stop.

undetermined cause will often have an with primary dystonia due to DYT158
early trial of levodopa to rule out a but has also been used as palliation in a
treatable dopa-responsive dystonia.27 number of other genetic disorders, such
Symptomatic treatment of dystonia as pantothenate kinaseYassociated
is managed with a number of differ- neurodegeneration. Chorea can often
ent agents, including baclofen, trihe- be difficult to manage from a therapeu-
xyphenidyl, the benzodiazepines,57 and, tic perspective, but tetrabenazine, ben-
more recently, clonidine and gabapentin. zodiazepines, and antiepileptic drugs (eg,
More invasive therapies, such as intra- levetiracetam, sodium valproate, carba-
thecal baclofen and DBS, are also used. mazepine) can be used.59 The paroxys-
DBS is particularly effective for children mal disorders have specific treatments,

1180 www.ContinuumJournal.com August 2016

Copyright © American Academy of Neurology. Unauthorized reproduction of this article is prohibited.


KEY POINTS
which can often be very effective commonly in adolescents and in fe- h Genetic movement
(Table 7-4). Genetic parkinsonian dis- males, and a diagnosis in a young child disorders are
orders are treated with either levodopa under 10 years of age should be made increasingly being
therapy or dopaminergic drugs such as with caution (although they do occur). recognized and
pramipexole, ropinirole, and rotigotine. The hallmarks of psychogenic move- diagnosed in children.
Neurotransmitter disorders and other ment disorders are an acute, often Often, chromosomal
complex neurometabolic syndromes explosive, onset with a short time to microarray is an early
require specific disease-targeted treat- maximum severity (often hours or investigation in such
ments, often necessitating consultation children to identify copy
days), incongruous characteristic (atyp-
with a specialist in the field. All children number variants
ical or odd) inconsistent findings, and accounting for disease
with complex genetic syndromes inconsistent performance relative to the phenotypes. Further
should be under the care of a multi- severity of movements (eg, the patient genetic testing is guided
disciplinary team, including a neurolo- is able to use a mobile phone despite by clinical history,
gist with expertise in movement violent movements). Pseudomyoclonus features on examination,
disorders with close links to physical, or pseudotremor are the most common and radiologic or
occupational, and speech and lan- psychogenic movement disorder move- metabolic biomarkers.
guage therapists. Regular input from ments. Pseudodystonia, akinesia, and The evolution of multiple
a general pediatrician to monitor gen- chorea are uncommon, although occa- gene panels and whole
eral health issues, vision, and hearing exome sequencing
sionally observed.23 The classic patient
and for orthopedic and secondary technologies has further
may have a history of high performance improved diagnostic
gastrointestinal complications is also in academics or sporting activities, a
an integral part of routine care. Ge- approaches for this
predisposition to anxiety, some pre- group of patients.
netic counseling to discuss the disease
ceding psychosocial stressors, and, h Early recognition and
risk in future pregnancies should also
sometimes, an acute, often minor, stress intervention as well
be offered to affected families.
(eg, dental procedure or infection). as involving a
It is essential to approach the prob- multidisciplinary team to
PSYCHOGENIC MOVEMENT lem with care and empathy and to rehabilitate the patient
DISORDERS IN CHILDREN perform necessary investigations to both physically and
Psychogenic presentations represent a reassure the patient and family that no psychologically are
significant proportion of acute presen- serious brain pathology, such as a essential in management
tumor, exists. Neuroimaging is almost of psychogenic
tations to neurology services and are
movement disorders.
important as they often represent a always necessary to exclude a structural
Treating psychogenic
significant and complex issue for the or metabolic cause for symptoms.
movement disorders can
child. Early recognition and treatment Explanation of the problem is impor- be rewarding to the
are considered important to provide tant to the child and family, and this child, family, and
the best chance of a good outcome. may require repeated consultations health practitioners.
The issue of whether these problems and discussions before the child and
should be termed psychogenic or family accept the likely explanation
functional is under debate but not (Case 7-4). Arguably the most useful
discussed in this article, and little test can be a second opinion from an
doubt exists that our understanding experienced colleague.60
of these disorders is changing. Care-
ful clinical history and observation is CONCLUSION
essential. Sometimes the diagnosis is Scientific advances have certainly
straightforward; however, sometimes improved diagnosis and available
prolonged observation is needed for treatments for children with move-
diagnosis of difficult cases. Psycho- ment disorders. Careful clinical history,
genic movement disorders occur more detailed examination, and review of

Continuum (Minneap Minn) 2016;22(4):1159–1185 www.ContinuumJournal.com 1181

Copyright © American Academy of Neurology. Unauthorized reproduction of this article is prohibited.


Movement Disorders in Childhood

Case 7-4
A 12-year-old girl with no significant medical problems presented with
shaking movements of the arms. She had attended school as usual the day
before but had onset of shaking movements the preceding evening, and
her condition had deteriorated dramatically over the course of the next
day. She had no other change in function. Examination revealed dramatic
coarse shaking of the upper limbs but no abnormal movements of the legs,
head, or trunk. Her walking revealed unsteadiness with significant wobbling
but without falls. During observation, it was noted that her symptoms
fluctuated significantly; she was observed to use her mobile phone normally
without shaking, and her shaking varied according to the people around
her. Her behavior, thinking, and memory were normal, and formal neurologic
examination was also unrevealing. On subsequent history, it became
apparent that she had been the victim of significant bullying at school.
The school described her as a kind, diligent, and sensitive girl who was
training 5 nights a week in state-level athletic competition. MRI brain and
EEG were normal.
Comment. The incongruous and inconsistent nature of this patient’s
movement disorder led to a diagnosis of a psychogenic movement disorder,
presumably secondary to the significant stressors. Psychological input
uncovered some baseline anxiety and fears of failure with perfectionism
and some obsessive-compulsive behavior. A confident diagnosis and
acceptance of the diagnosis by the family, followed by a multidisciplinary
rehabilitation with physical therapy and psychological input resulted in a
rapid improvement and return to normal function.

video recordings can aid diagnosis and Supplemental Digital


help direct further diagnostic strate- Content 7-2
gies. Genetic advances will allow elu- Complete resolution of DYT1 dysto-
cidation of new disease pathways that nia with trihexyphenidyl. Video shows
may inform future targeted therapeu- the same patient as in Supplemental
tic strategies. DBS and novel treat- Digital Content 7-1 6 years later.
ment approaches such as gene She is now 16 years old and has
therapy may also play an increasing complete control of her dystonia
role in the future for pharmaco- with trihexyphenidyl 6 mg 3 times
resistant and neurodegenerative child- per day as monotherapy.
hood movement disorders.
links.lww.com/CONT/A186
VIDEO LEGENDS B 2016 American Academy of Neurology.
Supplemental Digital
Supplemental Digital
Content 7-1
Content 7-3
DYT1 dystonia. Video shows a
Aromatic L-amino acid decarboxylase
10-year-old girl with genetically con-
deficiency. Video shows a 2.5-year-old
firmed DYT1 dystonia who presents
girl with aromatic L-amino acid decar-
with progressive limb dystonia, pre-
boxylase deficiency. While sitting dur-
dominantly affecting the right arm.
ing examination, the patient has an
links.lww.com/CONT/A185 oculogyric crisis with deviation of the
B 2016 American Academy of Neurology. eyes to the right and upward. Her right

1182 www.ContinuumJournal.com August 2016

Copyright © American Academy of Neurology. Unauthorized reproduction of this article is prohibited.


arm is held extended and pronated in a neurological relapse. Mov Disord
2014;29(1):90Y96. doi:10.1002/mds.25626.
dystonic posture. Examination reveals
four-limb hypotonia, bilateral ptosis, 10. Armangue T, Leypoldt F, Málaga I, et al.
Herpes simplex virus encephalitis is a trigger
mild mouth tenting, severe axial hypo- of brain autoimmunity. Ann Neurol
tonia, an overall paucity of spontaneous 2014;75(2):317Y323. doi:10.1002/ana.24083.
movement and left foot clawing. 11. Pillai SC, Hacohen Y, Tantsis E, et al. Infectious
and autoantibody-associated encephalitis:
links.lww.com/CONT/A187 clinical features and long-term outcome.
B 2016 American Academy of Neurology. Pediatrics 2015;135(4):e974Ye984.
doi:10.1542/peds.2014-2702.
ACKNOWLEDGMENTS 12. Mohammad SS, Fung VS, Grattan-Smith P,
Manju A. Kurian, MBBChir, MRCPCH, PhD, et al. Movement disorders in children with
anti-NMDAR encephalitis and other autoimmune
is funded by an Intermediate Wellcome encephalopathies. Mov Disord 2014;29(12):
Trust Fellowship (WT098524MA). 1539Y1542. doi:10.1002/mds.25999.
13. Titulaer MJ, McCracken L, Gabilondo I, et al.
REFERENCES Treatment and prognostic factors for long-term
1. Sanger TD, Chen D, Fehlings DL, et al. outcome in patients with anti-NMDA receptor
Definition and classification of hyperkinetic encephalitis: an observational cohort study.
movements in childhood. Mov Disord Lancet Neurol 2013;12(2):157Y165. doi:10.1016/
2010;25(11):1538Y1549. doi:10.1002/ S1474-4422(12)70310-1.
mds.23088.
14. Dale RC, Merheb V, Pillai S, et al.
2. Garcia-Cazorla A, Duarte ST. Parkinsonism Antibodies to surface dopamine-2 receptor
and inborn errors of metabolism. J Inherit in autoimmune movement and psychiatric
Metab Dis 2014;37(4):627Y642. doi:10.1007/ disorders. Brain 2012;135(pt 11):3453Y3468.
s10545-014-9723-6. doi:10.1093/brain/aws256.
3. Bonnet C, Roubertie A, Doummar D, et al. 15. Walker K, Brink A, Lawrenson J, et al.
Developmental and benign movement Treatment of sydenham chorea with
disorders in childhood. Mov Disord intravenous immunoglobulin. J Child Neurol
2010;25(10):1317Y1334. doi:10.1002/mds.22944. 2012;27(2):147Y155. doi:10.1177/
4. Ganos C, Martino D. Tics and tourette 0883073811414058.
syndrome. Neurol Clin 2015;33(1):115Y136.
16. Chang K, Frankovich J, Cooperstock M, et al;
doi:10.1016/j.ncl.2014.09.008.
PANS Collaborative Consortium. Clinical
5. Oakley C, Mahone EM, Morris-Berry C, et al. evaluation of youth with pediatric
Primary complex motor stereotypies in acute-onset neuropsychiatric syndrome
older children and adolescents: clinical (PANS): recommendations from the 2013
features and longitudinal follow-up. PANS Consensus Conference. J Child Adolesc
Pediatr Neurol 2015;52(4):398Y403.e1. Psychopharmacol 2015;25(1):3Y13.
doi:10.1016/j.pediatrneurol.2014.11.002. doi:10.1089/cap.2014.0084.
6. Robinson S, Woods M, Cardona F, et al. 17. Carvajal-González A, Leite MI, Waters P, et al.
Intense imagery movements: a common Glycine receptor antibodies in PERM and
and distinct paediatric subgroup of motor related syndromes: characteristics, clinical
stereotypies. Dev Med Child Neurol 2014; features and outcomes. Brain 2014;137(pt 8):
56(12):1212Y1218. doi:10.1111/dmcn.12518. 2178Y2192. doi:10.1093/brain/awu142.

7. Misra UK, Kalita J. Spectrum of movement 18. Tobin WO, Lennon VA, Komorowski L, et al.
disorders in encephalitis. J Neurol 2010;257(12): DPPX potassium channel antibody:
2052Y2058. doi:10.1007/s00415-010-5659-4. frequency, clinical accompaniments, and
outcomes in 20 patients. Neurology
8. Mohammad SS, Sinclair K, Pillai S, et al. Herpes 2014;83(20):1797Y1803. doi:10.1212/
simplex encephalitis relapse with chorea is WNL.0000000000000991.
associated with autoantibodies to
19. Petit-Pedrol M, Armangue T, Peng X, et al.
N-Methyl-D-aspartate receptor or
Encephalitis with refractory seizures, status
dopamine-2 receptor. Mov Disord 2014;29(1):
epilepticus, and antibodies to the GABAA
117Y122. doi:10.1002/mds.25623.
receptor: a case series, characterisation of
9. Hacohen Y, Deiva K, Pettingill P, et al. the antigen, and analysis of the effects of
N-methyl-D-aspartate receptor antibodies antibodies. Lancet Neurol 2014;13(3):276Y286.
in post-herpes simplex virus encephalitis doi:10.1016/S1474-4422(13)70299-0.

Continuum (Minneap Minn) 2016;22(4):1159–1185 www.ContinuumJournal.com 1183

Copyright © American Academy of Neurology. Unauthorized reproduction of this article is prohibited.


Movement Disorders in Childhood

20. Pranzatelli MR, Tate ED, Swan JA, et al. B 2013;136(pt 1):294Y303. doi:10.1093/brain/
cell depletion therapy for new-onset aws308.
opsoclonus-myoclonus. Mov Disord
2010;25(2):238Y242. doi:10.1002/mds.22941. 31. Sweney MT, Newcomb TM, Swoboda KJ.
The expanding spectrum of neurological
21. Mitchell WG, Wooten AA, O’Neil SH, et al. phenotypes in children with ATP1A3
Effect of increased immunosuppression on mutations, Alternating Hemiplegia
developmental outcome of opsoclonus of Childhood, Rapid-onset
myoclonus syndrome (OMS). J Child Neurol Dystonia-Parkinsonism, CAPOS and beyond.
2015;30(8):976Y982. doi:10.1177/ Pediatr Neurol 2015;52(1):56Y64. doi:10.1016/
0883073814549581. j.pediatrneurol.2014.09.015.
22. Ahn ES, Scott RM, Robertson RL Jr, Smith ER. 32. Groen JL, Andrade A, Ritz K, et al.
Chorea in the clinical presentation of CACNA1B mutation is linked to unique
moyamoya disease: results of surgical
myoclonus-dystonia syndrome. Hum Mol
revascularization and a proposed
Genet 2015;24(4):987Y993. doi:10.1093/
clinicopathological correlation. J Neurosurg
hmg/ddu513.
Pediatr 2013;11(3):313Y319. doi:10.3171/
2012.11.PEDS12199. 33. Mencacci NE, Rubio-Agusti I, Zdebik A, et al.
A missense mutation in KCTD17 causes
23. Dale RC, Singh H, Troedson C, et al. A
autosomal dominant myoclonus-dystonia.
prospective study of acute movement
Am J Hum Genet 2015;96(6):938Y947.
disorders in children. Dev Med Child
doi:10.1016/j.ajhg.2015.04.008.
Neurol 2010;52(8):739Y748. doi:10.1111/
j.1469-8749.2009.03598.x. 34. Mencacci NE, R’bibo L, Bandres-Ciga S, et al.
24. Mohammad SS, Wallace G, Ramanathan S, The CACNA1B R1389H variant is not
et al. Antipsychotic-induced akathisia and associated with myoclonus-dystonia in a
neuroleptic malignant syndrome in large European multicentric cohort. Hum
anti-NMDAR encephalitis. Ann Clin Mol Genet 2015;24(18):5326Y5329.
Psychiatry 2014;26(4):297Y298. doi:10.1093/hmg/ddv255.

25. Ozelius L, Lubarr N. DYT1 Early-onset primary 35. Cellini E, Vignoli A, Pisano T, et al; FOXG1
dystonia. In: Pagon RA, Adam MP, Ardinger Syndrome Study Group. The hyperkinetic
HH, et al, eds. GeneReviews. Seattle, WA: movement disorder of FOXG1-related
University of Washington, Seattle, 1999. epileptic-dyskinetic encephalopathy. Dev
www.ncbi.nlm.nih.gov/books/NBK1492/. Med Child Neurol 2016;51(1):93Y97.
Updated January 2, 2014. Accessed doi:10.1111/dmcn.12894.
June 15, 2016. 36. Peall KJ, Kurian MA. Benign hereditary
26. Segawa M. Dopa-responsive dystonia. chorea: an update. Tremor Other
Hyperkinet Mov (N Y) 2015;5:314.
Handb Clin Neurol 2011;100:539Y557.
doi:10.7916/D8RJ4HM5.
doi:10.1016/B978-0-444-52014-2.00039-2.
37. Kurian MA, Jungbluth H. Genetic disorders
27. Ng J, Heales SJ, Kurian MA. Clinical features
of thyroid metabolism and brain
and pharmacotherapy of childhood
development. Dev Med Child Neurol
monoamine neurotransmitter disorders.
2014;56(7):627Y634. doi:10.1111/
Paediatr Drugs 2014;16(4):275Y291.
dmcn.12445.
doi:10.1007/s40272-014-0079-z.
38. Logan CV, Szabadkai G, Sharpe JA, et al;
28. Mencacci NE, Isaias IU, Reich MM, et al;
UK10K Consortium. Loss-of-function mutations
International Parkinson’s Disease Genomics in MICU1 cause a brain and muscle disorder
Consortium and UCL-exomes consortium. linked to primary alterations in mitochondrial
Parkinson’s disease in GTP cyclohydrolase 1 calcium signaling. Nat Genet
mutation carriers. Brain 2014;137(pt 9): 2014;46(2):188Y193. doi:10.1038/ng.2851.
2480Y2492. doi:10.1093/brain/awu179.
39. Chen YZ, Friedman JR, Chen DH, et al.
29. Willemsen MA, Verbeek MM, Kamsteeg EJ, Gain-of-function ADCY5 mutations in
et al. Tyrosine hydroxylase deficiency: familial dyskinesia with facial myokymia.
a treatable disorder of brain Ann Neurol 2014;75(4):542Y549. doi:10.1002/
catecholamine biosynthesis. Brain ana.24119.
2010;133(pt 6):1810Y1822. doi:10.1093/
brain/awq087. 40. Mencacci NE, Erro R, Wiethoff S, et al.
ADCY5 mutations are another cause of
30. Peall KJ, Smith DJ, Kurian MA, et al. SGCE benign hereditary chorea. Neurology
mutations cause psychiatric disorders: 2015;85(1):80Y88. doi:10.1212/
clinical and genetic characterization. Brain WNL.0000000000001720.

1184 www.ContinuumJournal.com August 2016

Copyright © American Academy of Neurology. Unauthorized reproduction of this article is prohibited.


41. Erro R, Sheerin UM, Bhatia KP. Paroxysmal Expert Rev Neurother 2015;15(7):793Y802.
dyskinesias revisited: a review of 500 genetically doi:10.1586/14737175.2015.1055322.
proven cases and a new classification.
50. Bandmann O, Weiss KH, Kaler SG.
Mov Disord 2014;29(9):1108Y1116.
Wilson’s disease and other neurological
doi:10.1002/mds.25933.
copper disorders. Lancet Neurol
42. Bidichandani SI, Delatycki MB. Friedreich 2015;14(1):103Y113. doi:10.1016/
ataxia. In: Pagon RA, Adam MP, S1474-4422(14)70190-5.
Ardinger HH, et al, eds. GeneReviews.
51. Meyer E, Kurian MA, Hayflick SJ.
Seattle, WA: University of Washington,
Neurodegeneration with brain iron
Seattle, 1998. www.ncbi.nlm.nih.gov/books/
accumulation: genetic diversity and
NBK1281/. Updated July 24, 2014. Accessed
pathophysiological mechanisms. Annu Rev
June 15, 2016.
Genomics Hum Genet 2015;16:257Y279.
43. Schuelke M. Ataxia with vitamin E doi:10.1146/annurev-genom-090314-025011.
deficiency. In: Pagon RA, Adam MP,
52. Tuschl K, Mills PB, Clayton PT. Manganese
Ardinger HH, et al, eds. GeneReviews. Seattle,
and the brain. Int Rev Neurobiol 2013;110:
WA: University of Washington, Seattle,
277Y312. doi:10.1016/B978-0-12-410502-
2005. www.ncbi.nlm.nih.gov/books/NBK1241/.
7.00013-2.
Updated June 27, 2013. Accessed
June 15, 2016. 53. Rice GI, Kasher PR, Forte GM, et al.
Mutations in ADAR1 cause Aicardi-Goutières
44. Gatti R. Ataxia-telangiectasia. In: Pagon RA,
syndrome associated with a type I interferon
Adam MP, Ardinger HH, et al, eds.
signature. Nat Genet 2012;44(11):1243Y1248.
GeneReviews. Seattle, WA: University of
doi:10.1038/ng.2414.
Washington, Seattle, 1999. www.ncbi.nlm.
nih.gov/books/NBK26468/. Updated 54. Ortigoza-Escobar JD, Serrano M, Molero M,
March 11, 2010. Accessed June 15, 2016. et al. Thiamine transporter-2 deficiency:
outcome and treatment monitoring.
45. Coutinho P, Barbot C. Ataxia with
Orphanet J Rare Dis 2014;9:92. doi:10.1186/
oculomotor apraxia type 1. In: Pagon RA,
1750-1172-9-92.
Adam MP, Ardinger HH, et al, eds.
GeneReviews. Seattle, WA: University of 55. Simons C, Wolf NI, McNeil N, et al. A de novo
Washington, Seattle, 2002. www.ncbi.nlm. mutation in the "-tubulin gene TUBB4A
nih.gov/books/NBK1456/. Updated March 19, results in the leukoencephalopathy
2015. Accessed June 15, 2016. hypomyelination with atrophy of the basal
ganglia and cerebellum. Am J Hum Genet
46. Moreira MC, Koenig M. Ataxia with 2013;92(5):767Y773. doi:10.1016/
oculomotor apraxia type 2. In: Pagon RA, j.ajhg.2013.03.018.
Adam MP, Ardinger HH, et al, eds.
GeneReviews. Seattle, WA: University of 56. Erro R, Hersheson J, Ganos C, et al. H-ABC
Washington, Seattle, 2004. www.ncbi.nlm. syndrome and DYT4: variable expressivity
nih.gov/books/NBK1154/. Updated or pleiotropy of TUBB4 mutations? Mov
December 08, 2011. Accessed Disord 2015;30(6):828Y833. doi:10.1002/
June 15, 2016. mds.26129.

47. Vermeer S, van de Warrenburg BP, 57. Roubertie A, Mariani LL, Fernandez-Alvarez E,
Kamsteeg EJ. ARSACS. In: Pagon RA, Adam et al. Treatment for dystonia in childhood.
MP, Ardinger HH, et al, eds. GeneReviews. Eur J Neurol 2012;19(10):1292Y1299.
Seattle, WA: University of Washington, doi:10.1111/j.1468-1331.2011.03649.x.
Seattle, 2003. www.ncbi.nlm.nih.gov/books/ 58. Fox MD, Alterman RL. Brain stimulation for
NBK1255/. Updated October 11, 2012. torsion dystonia. JAMA Neurol
Accessed June 15, 2016. 2015;72(6):713Y719. doi:10.1001/
48. Németh AH, Kwasniewska AC, Lise S, et al; jamaneurol.2015.51.
UK Ataxia Consortium. Next generation 59. Hermann A, Walker RH. Diagnosis and
sequencing for molecular diagnosis of treatment of chorea syndromes. Curr Neurol
neurological disorders using ataxias as a Neurosci Rep 2015;15(2):514. doi:10.1007/
model. Brain 2013;136(pt 10):3106Y3118. s11910-014-0514-0.
doi:10.1093/brain/awt236.
60. Canavese C, Ciano C, Zibordi F, et al.
49. Molero-Luis M, Serrano M, O’Callaghan MM, Phenomenology of psychogenic movement
et al. Clinical, etiological and therapeutic disorders in children. Mov Disord 2012;
aspects of cerebral folate deficiency. 27(9):1153Y1157. doi:10.1002/mds.24947.

Continuum (Minneap Minn) 2016;22(4):1159–1185 www.ContinuumJournal.com 1185

Copyright © American Academy of Neurology. Unauthorized reproduction of this article is prohibited.


Review Article

Chorea
Address correspondence to
Dr Tiago A. Mestre,
Parkinson’s Disease and
Movement Disorders Centre,
Civic Campus, The Ottawa Tiago A. Mestre, MSc, MD
Hospital, 1053 Carling Ave,
Rm 2174, Ottawa,
ON K1Y 4E9, Canada,
tmestre@toh.on.ca. ABSTRACT
Relationship Disclosure: Purpose of Review: This article reviews the clinical approach to the diagnosis of
Dr Mestre receives personal
compensation for serving on adult patients presenting with chorea, using Huntington disease (HD) as a point of
the scientific advisory board reference, and presents the clinical elements that help in the diagnostic workup.
of AbbVie, for speaking Principles of management for chorea and some of the associated features of other
engagements for Teva
Pharmaceutical Industries Ltd, choreic syndromes are also described.
and for educational events Recent Findings: Mutations in the C9orf72 gene, previously identified in families
for WebMD. Dr Mestre with a history of frontotemporal dementia, amyotrophic lateral sclerosis, or both,
receives grant support from
the Parkinson Study have been recognized as one of the most prevalent causes of HD phenocopies in
Group/Parkinson Disease the white population.
Foundation. Summary: The diagnosis of chorea in adult patients is challenging. A varied
Unlabeled Use
of Products/Investigational
number of associated causes require a physician to prioritize the investigations, and
Use Disclosure: a detailed history of chorea and associated findings will help. For chorea presenting
Dr Mestre reports as part of a neurodegenerative syndrome, the consideration of a mutation in the
no disclosure.
C9orf72 gene is a new recommendation after excluding HD. There are no new
* 2016 American Academy
of Neurology. treatment options for chorea, aside from dopamine blockers and tetrabenazine.
There are no disease-modifying treatments for HD or other neurodegenerative
choreic syndromes.

Continuum (Minneap Minn) 2016;22(4):1186–1207.

INTRODUCTION ical diagnosis. Parkinsonism usually


Huntington disease (HD) is the most develops later in the course of the
frequent cause of a hereditary neuro- disease. The juvenile form of HD pre-
degenerative choreic syndrome. HD sents as a predominantly hypokinetic
has a worldwide distribution with some movement disorder with parkinsonism
geographic variability in its prevalence. but also myoclonus. Of importance, as
In North America and Europe, the many as 7% of subjects with a clinical
average prevalence is 5 per 100,000 presentation compatible with HD are
inhabitants.1 In patients with HD, the found to have a negative genetic test
initial symptoms occur more fre- for HD and have been coined ‘‘HD
2
quently between the ages of 30 and phenocopies.’’ Spinocerebellar ataxia
50, although onset can range from (SCA) type 17 (SCA17), also referred
childhood/adolescence (juvenile form, to as Huntington diseaseYlike (HDL)
also known as the Westphal variant) to syndrome type 4 (HDL4), and C9orf72-
individuals older than 70 years of age. related HD phenocopy in populations
HD can present to the clinician with of white individuals are considered the
3,4
Supplemental digital content: one of three symptom complexes class- most frequent alternative diagnoses.
Videos accompanying this ar-
ticle are cited in the text as ically described in this condition (mo- The motor and nonmotor elements that
Supplemental Digital Content. tor, cognitive, and neuropsychiatric) or help in the diagnostic workup of HD
Videos may be accessed by
clicking on links provided in the in combination. The motor symptoms phenocopies in the adult will be dis-
HTML, PDF, and app versions include chorea, dystonia, and tics. cussed later in this article, and a core de-
of this article; the URLs are
provided in the print version. Video
Chorea has been classically used as scription of neurodegenerative choreic
legends begin on page 1204. the reference manifestation for a clin- syndromes is provided in Table 8-1.
1186 www.ContinuumJournal.com August 2016

Copyright © American Academy of Neurology. Unauthorized reproduction of this article is prohibited.


TABLE 8-1 Clues for Diagnosis and Investigations of Adult-Onset Neurodegenerative
Diseases With Chorea

Neurodegenerative Relevant
Conditions Demographics Core Features Useful Investigations
Most common neurodegenerative choreic syndromes
Huntington Estimated prevalence Chorea (onset); Caudate atrophy (MRI),
disease (HD) is 5/100,000 (with depression, apathy, genetic test (HTT gene,
geographic variation); irritability, cognitive CAG repeat expansion
modal age group for impairment, parkinsonism, 935)
onset is 30Y50 years of age and dystonia (later
stages)
Huntington Common cause Ataxia in an Caudate and cerebellar
diseaseYlike (HDL) for HD-negative HD phenocopy atrophy, putaminal rim
syndrome neurodegenerative enhancement (MRI),
type 4 (HDL4)/ chorea genetic test (TBP gene,
Spinocerebellar (white populations) CAA/CAG repeat
ataxia (SCA) type expansion 942)
17 (SCA17)
C9orf72-related Common cause HD phenocopy, ataxia, Genetic test (C9orf72
HD phenocopy for HD-negative parkinsonism, and upper gene, GGGGCC/G4C2
neurodegenerative motor neuron signs, hexanucleotide repeat
chorea (white phenotypic heterogeneity expansion 960 are
populations) in families definitely pathogenic)
HDL2 Exclusive to HD phenocopy Caudate atrophy (MRI),
sub-Saharan acanthocytosis (fresh
African descendants blood film) in about
10% of patients, genetic
test (JPH3 gene, CTG
repeat expansion 940,
fully penetrant)
Rare neurodegenerative syndromes, chorea as a classic prominent feature
Chorea-acanthocytosis Estimated prevalence of Chorea, dystonia Acanthocytosis (fresh
(Levine-Critchley 1000 cases worldwide; (orofacial involvement, blood film), elevated
syndrome) young adult onset tongue protrusion), creatine kinase, chorein
tics, self-mutilation, absent in red blood cells,
seizures, sensorimotor predominant atrophy of
axonal polyneuropathy, the head of caudate (MRI),
hepatomegaly, splenomegaly genetic test (VPS13A gene)
McLeod syndrome Rarer than Chorea, dystonia, Acanthocytosis (fresh blood
chorea-acanthocytosis; parkinsonism, psychiatric film), antigen Kell absent/
exclusive to males and features, seizures, reduced in red blood cells,
with a later onset (after sensorimotor axonal elevated creatine kinase
fourth decade) neuropathy, cardiomyopathy, and liver function test,
hepatosplenomegaly, genetic test (XK gene)
hemolytic anemia
Continued on page 1188

Continuum (Minneap Minn) 2016;22(4):1186–1207 www.ContinuumJournal.com 1187

Copyright © American Academy of Neurology. Unauthorized reproduction of this article is prohibited.


Chorea

TABLE 8-1 Clues for Diagnosis and Investigations of Adult-Onset Neurodegenerative


Diseases With Chorea Continued from page 1187

Neurodegenerative Relevant
Conditions Demographics Core Features Useful Investigations
HDL1 Very rare (four Chorea, ataxia, prominent Atrophy of basal ganglia,
families reported5) psychiatric features, cerebellum, frontal and
myoclonus, seizures temporal lobes (MRI),
genetic test (PRNP gene,
eight octapeptide
repeat insertions)
Dentatorubral- Most common in Ataxia, chorea, myoclonus, Atrophy of cerebellum,
pallidoluysian atrophy Japan (1/208,000), rare dementia, seizures brainstem, cerebrum;
in non-Japanese hyperintensity in
populations periventricular white
matter (MRI), genetic test
(ATN1 gene, CAG
repeat expansion 947,
fully penetrant)
Neuroferritinopathy Reported in Cumbria, Chorea/dystonia with Hypointensity in dentate
United Kingdom, and orofacial involvement, nuclei, red nuclei, basal
France; estimated nontremulous parkinsonism ganglia, thalami, rolandic
prevalence of (less frequent), maintained cortex (T2*), bilateral
G1/1 million (very rare) asymmetrical features, pallidal necrosis with cystic
ataxia (rarer), late degeneration (MRI), low
neuropsychiatric and serum ferritin, genetic test
cognitive symptoms (FTL gene)
(later feature)
Aceruloplasminemia Estimated prevalence Dystonia (craniocervical), Low serum ceruloplasmin
of 1/1 million to parkinsonism, chorea and and high ferritin,
1/1.2 million (very rare); ataxia, depression, cognitive hypointensity in the
onset at middle age dysfunction, anemia, striatum, thalamus, and
diabetes mellitus, retinal dentate nucleus (T2* MRI),
degeneration preceding genetic test (CP gene)
neurologic manifestations
Neurodegenerative syndromes, chorea as a rare manifestation
SCA1 Rare chorea-athetosis, Cerebellar atrophy (MRI),
ataxia, dystonia, dementia, genetic test (ATXN1 gene,
early hyperreflexia, triplet repeat expansion
late neuropathy 939, fully penetrant)
SCA2 Rare chorea-athetosis, Cerebellar atrophy (MRI),
ataxia, hyperreflexia, genetic test (ATXN2 gene,
earlier slow saccades, CAG/CAA repeat expansion
levodopa-responsive, 932, fully penetrant)
parkinsonism, dystonia,
dementia, early neuropathy

Continued on page 1189

1188 www.ContinuumJournal.com August 2016

Copyright © American Academy of Neurology. Unauthorized reproduction of this article is prohibited.


TABLE 8-1 Clues for Diagnosis and Investigations of Adult-Onset Neurodegenerative
Diseases With Chorea Continued from page 1188

Neurodegenerative Relevant
Conditions Demographics Core Features Useful Investigations
SCA3 Rare chorea-athetosis, Cerebellar atrophy (MRI),
ataxia, levodopa-responsive, genetic test (ATXN3 gene,
parkinsonism, dystonia, triplet repeat expansion
hyperreflexia/spasticity, 951, fully penetrant)
late neuropathy
Wilson disease Chorea is relatively rare, Kayser-Fleischer rings
predominantly parkinsonism (slit-lamp examination),
and dystonia (with risus eye-of-the-panda sign,
sardonicus), ataxia, variable white matter
psychiatric symptoms, lesions in brainstem,
seizures can occur (G10%), cerebellum (MRI), low
KayserYFleischer rings, serum ceruloplasmin, high
arthropathy, hemolytic anemia urine copper (24-hour
urine), high liver copper
content on biopsy,
genetic test (CP gene)
Pantothenate Single case with Head and upper limb Bilateral hypointensity in
kinaseYassociated chorea reported, onset chorea with later the putamen, caudate,
neurodegeneration at age 766 generalization, imbalance substantia nigra, and
dentate nuclei,
eye-of-the-tiger signa (MRI),
acanthocytosis (fresh blood
film) in about 10% of
patients,a genetic test
(PANK2 gene)a
Friedreich ataxia Single case with Ataxia, chorea, cognitive Genetic test (FXN gene,
chorea reported3 impairment (at GAA repeat expansion 965,
presentation), dysphagia and fully penetrant)
dysarthria (later stages)
Pallidonigroluysian Very rare7 Chorea (20% of cases), gait Postmortem diagnosis
atrophy or balance disturbance
Lubag disease (TAF1) Filipino origin Dystonia-parkinsonism, Genetic test (TAF1 gene)
chorea is a very rare feature
(also reported in female
carriers)
MRI = magnetic resonance imaging.
a
Diagnostic features of pantothenate kinaseYassociated neurodegeneration, but not documented in the case report.

Acquired causes of chorea are im- chorea that a clinician should consider
portant to take into consideration since in the appropriate setting.8 Other spo-
a few can be treated. Certain elements radic forms of chorea are broadly
in the clinical presentation of acquired divided into immune-mediated, infec-
causes of chorea help in the differential tious, metabolic/endocrine, vascular, and
diagnosis. Chorea secondary to stroke others causes (Table 8-2). This article
and drug-induced chorea are among covers the approach to adult-onset cho-
the most prevalent causes of sporadic rea. As such, conditions with a classic

Continuum (Minneap Minn) 2016;22(4):1186–1207 www.ContinuumJournal.com 1189

Copyright © American Academy of Neurology. Unauthorized reproduction of this article is prohibited.


Chorea

TABLE 8-2 Nongenetic Causes of Chorea

b Drugs
Antiemetics (dopamine antagonists)
Antiepileptic drugs (eg, phenytoin, carbamazepine, valproic acid)
Antihistamines
Baclofen
Calcium channel blockers
Digoxin
Fluoroquinolones
Levodopa, dopamine agonists (levodopa-induced dyskinesia)
Lithium
Methotrexate, cyclosporine
Neuroleptics (tardive dyskinesia)
Oral contraceptives, estrogen replacement therapy (often with a history of
previous Sydenham chorea)
Psychostimulants (eg, cocaine, amphetamines)
Steroids
Theophylline
Tricyclic antidepressants
b Immune Mediated
Antibody associated (paraneoplastic or idiopathic)9
Associated with neoplasia: collapsin response mediator protein-5 (CRMP5)
(small cell lung carcinoma and thymoma), Hu (small cell lung carcinoma), Yo,
antineuronal nuclear antibody (ANNA) type 1 and type 2, N-methyl-D-
aspartate (NMDA) subunit NR1 (ovarian tumor)
Idiopathic: NMDA subunit NR1 (45% of cases), leucine-rich, glioma
inactivated 1 (LgI1), contactin-associated proteinlike 2 (CASPR2),
glutamic acid decarboxylase 65 (GAD65), IgLON family member 5 (IgLON5),
pediatric autoimmune neuropsychiatric disorders associated with
streptococcal infections (PANDAS)
Behçet disease
Celiac disease
Demyelinating disease (rare, hemiballismus reported)10
Sjögren syndrome
Systemic lupus erythematosus/antiphospholipid syndrome

Continued on page 1191

1190 www.ContinuumJournal.com August 2016

Copyright © American Academy of Neurology. Unauthorized reproduction of this article is prohibited.


TABLE 8-2 Nongenetic Causes of Chorea Continued from page 1190

b Infectious
Encephalitis (West Nile virus, mumps, measles, varicella zoster)
Human immunodeficiency virus (HIV) (eg, secondary focal lesion due to
toxoplasmosis, primary central nervous system lymphoma, HIV encephalitis)
Tuberculosis, cysticercosis, borreliosis, neurosyphilis, diphtheria
Variant Creutzfeldt-Jakob disease
b Metabolic/Endocrine
Acquired hepatolenticular degeneration (advanced liver disease)
Electrolyte imbalance (hypoglycemia/hypercalcemia, hypomagnesemia,
hyponatremia)
Hyperthyroidism
Hypoglycemia/hyperglycemia (nonketotic)
Vitamin B12 deficiency (more frequently found as a cause of chorea
in children)
b Vascular
Essential thrombocythemia (one case reported)11
Ischemic/hemorrhagic stroke
Polycythemia rubra vera (elderly, primarily females)
Posterior reversible encephalopathy syndrome
Postpump chorea (more frequent in children)
b Other Causes
Carbon monoxide intoxication
Hydrocephalus
Postanoxic/cerebral palsy
Psychogenic chorea

onset in the pediatric age group will CLINICAL APPROACH TO AN


not be discussed in detail, and brief ADULT PATIENT WITH CHOREA
references will be given when pertinent. Physicians treating adult patients pre-
Examples of these conditions include senting with chorea should carry out a
acquired postinfectious Sydenham detailed and focused history and ex-
chorea, benign hereditary chorea, pan- amination that consider the multiple
tothenate kinaseYassociated neuro- causes of chorea and its presentations.
degeneration, Lesch-Nyhan syndrome, The data collected by the physician
and autosomal recessive ataxias in which give important clues toward the most
chorea has also been described, such as likely cause of the presenting chorea
apraxia with oculomotor apraxia type 1 and the selection of the more perti-
and 2 and ataxia telangiectasia. nent investigations.

Continuum (Minneap Minn) 2016;22(4):1186–1207 www.ContinuumJournal.com 1191

Copyright © American Academy of Neurology. Unauthorized reproduction of this article is prohibited.


Chorea

KEY POINTS
h Chorea may present as Chorea as Symptom (Supplemental Digital Content 8-1,
an isolated symptom or Chorea can present as an isolated links.lww.com/CONT/A188).12 The ad-
as a syndrome with a symptom or a mixed movement dis- vent of a genetic diagnosis for HD
variable combination of order and can be associated with has shown that most of these cases
a mixed movement behavioral and cognitive symptoms, are, in fact, late-onset HD (Case 8-1).13
disorder, behavioral seizures, or symptoms suggestive of In terms of mode of onset, a rela-
and cognitive polyneuropathy. The different symp- tively acute onset of chorea can be
symptoms, seizures, tom complexes identified with a clin- the manifestation of stroke, and also
or polyneuropathy. other causes such as nonketotic hy-
ical interview can give the first clues
h Most of the cases of for a diagnosis and prioritization of perglycemia, chorea gravidarum, or
senile chorea investigations. Chorea is frequently drug-induced chorea (Table 8-2). A
correspond to cases noticed first by a third person and subacute course over a few days or
of late-onset weeks may be the manifestation of
not by the patient; consequently, the
Huntington disease.
information provided by a caregiver infectious process or autoimmune
or family member is particularly im- chorea, including a paraneoplastic
portant for a more rigorous history of syndrome, but does not exclude a
chorea. As with other neurologic symp- metabolic cause.14 In some cases of
toms, the age of onset (adult versus drug-induced chorea, a gradual onset
elderly), the acute versus progressive may occur, such as in levodopa-
mode of onset, and the type of dis- induced dyskinesia in Parkinson dis-
ease course (remitting, paroxysmal, or ease (Supplemental Digital Content
continuous) can favor a specific syn- 8-2, links.lww.com/CONT/A189) or
drome. In terms of age of onset, neuroleptic-induced dyskinesia.15 A
there is the particular case of senile more protracted course of months to
chorea defined as an idiopathic spo- years is usually associated with a neu-
radic form of chorea with onset in the rodegenerative condition. In terms of
elderly in the absence of dementia the time course of chorea, a slowly

Case 8-1
An 88-year-old woman presented with a 6- to 8-year history of progressive
involuntary movements and imbalance, for which she had to use a walker.
The involuntary movements were described as ‘‘shaking’’ and were
increasingly embarrassing to the patient. She had also noticed swallowing
air when eating. The patient did not endorse cognitive or mood symptoms,
and she lived independently at a retirement home. The patient did not have a
family history of chorea. Her examination was remarkable for pronounced
generalized chorea involving the limbs, trunk, and face, as well as an inability
to walk unaided. A diagnosis of senile chorea was made. The investigations at
that time included a head CT, renal and liver function tests, serum glucose,
thyroid-stimulating hormone (TSH), as well as anticardiolipin,
antiphospholipid, and antinuclear antibodies. All investigations were normal.
Genetic testing for Huntington disease (HD) showed a CAG repeat of 39,
confirmatory of a diagnosis of late-onset HD. Two years later, the patient
died at age 90 with complications from dysphagia and dehydration.
Comment. This case is demonstrative of how to approach a case of senile
chorea and how late in life a diagnosis of HD can be made. In a case of
senile chorea, a diagnosis of HD has implications for a patient’s children, who
are at risk of developing HD and may consider predictive genetic testing.

1192 www.ContinuumJournal.com August 2016

Copyright © American Academy of Neurology. Unauthorized reproduction of this article is prohibited.


KEY POINTS
remitting hemichorea is often a case years.18 The presence of chorea in h Chorea secondary to
of vascular chorea secondary to a specific body regions also provides stroke has an acute
stroke or nonketotic hyperglycemia. important clues for the differential onset and will, most
The particular case of paroxysmal diagnosis. In forms of neurodegenera- often, improve
recurrent chorea suggests a com- tive chorea, orofacial chorea frequently over time.
pletely different set of diagnoses with associated with dystonia is suggestive of h Recurrent episodes of
consideration of paroxysmal dyskine- a classic neuroacanthocytosis syndrome chorea, frequently in
sias when multiple and short-lived (ie, chorea-acanthocytosis (Case 8-2 association with dystonia,
episodes of chorea are reported. Three and Supplemental Digital Con- suggest a form of
forms are classically described: par- tent 8-4, links.lww.com/CONT/A191) paroxysmal dyskinesia.
oxysmal kinesigenic dyskinesia, par- and, less commonly, McLeod syn- h Orofacial chorea is
oxysmal nonkinesigenic dyskinesia, drome).19 Orofacial chorea can also suggestive of a classic
and paroxysmal exertion-induced (or be an early feature of the rare neuro- neuroacanthocytosis
exercise-induced) dyskinesia. Although ferritinopathy, a late-onset form of syndrome.
chorea can present in isolation, the neurodegeneration with brain iron ac-
paroxysmal dyskinesias manifest most cumulation.20 The dystonic protrusion
commonly with both dystonia and of the tongue with an associated
chorea in about 60% to 65% of genet- difficulty maintaining ingested food
ically proven cases of paroxysmal in the mouth (feeding dystonia) has
kinesigenic dyskinesia (Supplemental been classically described in chorea-
Digital Content 8-3, links.lww.com/ acanthocytosis, but can also be found
CONT/A190) and paroxysmal non- in other conditions considered in the
kinesigenic dyskinesia and in about differential diagnosis of chorea, such
95% of genetically proven cases of as McLeod syndrome, tardive dyskine-
paroxysmal exertion-induced dyski- sia, pantothenate kinaseYassociated
nesia.16 These conditions have been neurodegeneration, and Lesch-Nyhan
distinguished on a clinical basis, mainly syndrome.21 In addition, the presence
with respect to the trigger of the episodes, of head drops and truncal/cervical
and have been further characterized extension is recognized as a distinc-
by the age of onset, duration and tive feature of advanced chorea-
frequency of episodes, and response acanthocytosis, but is also described in
to treatment. In recent years, there McLeod syndrome and advanced
has been a significant breakthrough HD.22Y24 The presence of tics can be
in finding the genetic basis of these found in HD, but also in the classic
conditions (for more information, refer neuroacanthocytosis syndromes, par-
to the following section on investiga- ticularly in chorea-acanthocytosis. The
tions and Table 8-3). Psychogenic presence of significant behavioral
chorea is rarely reported as a cause symptoms in addition to cognitive de-
of psychogenic movement disorders terioration strongly suggests a neuro-
and typically presents in the form degenerative choreic syndrome, namely
of a paroxysmal movement dis- HD. In an HD population, apathy is
order.17 Recurrent chorea can also considered the most prevalent behav-
occur in patients with a history of ioral symptom (28.1%), followed by
chorea gravidarum or Sydenham cho- depression, irritability/aggressiveness,
rea who develop chorea when exposed and obsessive/compulsive behaviors.25
to drugs such as oral contraceptives or Psychosis is less prevalent and can affect
phenytoin, although these cases can as little as 1.2% of the HD population.25
assume a distinctive pattern of longer- Behavioral symptoms can precede the
lasting episodes separated, at times, by onset of motor symptoms and, at the

Continuum (Minneap Minn) 2016;22(4):1186–1207 www.ContinuumJournal.com 1193

Copyright © American Academy of Neurology. Unauthorized reproduction of this article is prohibited.


Chorea

a
TABLE 8-3 Clinical Features of Paroxysmal Dyskinesia

Paroxysmal
Paroxysmal Paroxysmal Exertion-Induced
Kinesigenic Dyskinesia Nonkinesigenic Dyskinesia Dyskinesia
Age of onset Childhood/adolescence Childhood/adolescence Variable (depending
(majority of cases) (majority of cases) on cause)
Episodes
Triggers Sudden movements or Coffee, tea, alcohol Prolonged exertion
intention to move (more consistent), anxiety, (more frequently),
excitement, fatigue fasting, stress, anxiety
Duration Brief, majority of episodes Typically more prolonged, Episode ends with rest
G1 minute 10 minutes to 4 hours
Frequency Up to hundreds of episodes Weekly episodes more Dependent on triggers
a dayb commonb
Treatment Good response to Avoid triggering factors; Avoid triggering factors,
antiepileptics; carbamazepine poor response to treat underlying cause
is drug of choice benzodiazepines, phenytoin, when applicable
acetazolamide, levodopa
Etiology
Gene associated Proline-rich transmembrane Myofibrillogenesis Glucose transporter 1
with primary protein 2 (PRRT2) regulator 1 (MR-1)d (SLC2A1)e (in 20Y25%
formc of cases)
Secondary causes Secondary to brain Secondary to brain injury, Parkinson disease,
injury (vascular, trauma, symptomatic cases are rare dopa-responsive dystonia
multiple sclerosis)
a
Data from Erro R, et al, Mov Disord.16 onlinelibrary.wiley.com/doi/10.1002/mds.25933/abstract.
b
Frequency has a tendency to decrease with age.
c
Genotype-phenotype correlation is incomplete, and overlap of forms of paroxysmal dyskinesia may exist.
d
Potassium calcium-activated channel, subfamily M alpha member 1 (KCNMA1) has been described in one family with cases of
paroxysmal nonkinesigenic dyskinesia and epilepsy.
e
Mutations in the gene SLC2A1 can also cause benign familial infantile seizures, familial infantile convulsions with paroxysmal
choreoathetosis, and hemiplegic migraine.

Case 8-2
A 28-year-old man presented with a 1-year history of difficulty chewing and
abnormal gait. In order to eat, the patient swallowed food in big chunks
without chewing it. There was no history of seizures, and he was otherwise
healthy. There was a known family history of chorea-acanthocytosis in his
brother and two paternal cousins. Examination was remarkable for a left
gum ulcer, dystonic protrusion of the tongue when ingesting food or
speaking, as well as jaw-opening dystonia and recurrent sniffing. He had mild
generalized chorea and slight weakness in his hands and feet. Ankle reflexes
were absent. Gait was abnormal with hesitations, namely with turns. The
investigations identified elevated creatine kinase (422 units/L) and
acanthocytes (more than 10%) in the blood. He was started on
Continued on page 1195

1194 www.ContinuumJournal.com August 2016

Copyright © American Academy of Neurology. Unauthorized reproduction of this article is prohibited.


KEY POINTS
Continued from page 1194 h In chorea-acanthocytosis,
trihexyphenidyl for jaw-opening dystonia with benefit. The patient did not seizures can occur in
return for follow-up. about 50% of cases.
Comment. This case shows features suggestive of chorea-acanthocytosis, h Huntington diseaseYlike
including the presence of oromandibular dyskinesia with feeding dystonia, syndrome type 2 is most
tics, as well as signs of peripheral neuropathy. The documented family history prevalent in patients
of chorea-acanthocytosis and the laboratory findings of elevated creatine with a sub-Saharan
kinase and presence of more than 10% of acanthocytes in the blood African ancestry.
facilitated the diagnosis.

time of a diagnosis based on motor in which the majority of the cases have
features, deficits in cognitive function seizures.5,19,30,32 In DRPLA, it is useful
can be elicited in formal testing.26 to recognize that when the age of
Suicidal ideation is relatively frequent onset overlaps the mean age of onset
(8% to 10%) and should be actively of the adult form of HD, seizures rarely
looked for, especially in patients with occur, and more common presenting
active depression and a previous features are chorea, in addition to dys-
suicidal attempt.27 tonia, parkinsonism, and psychosis.
HD phenocopies in which behav- When the clinician is considering
ioral or cognitive symptoms are part a genetic choreic syndrome, it is
of the clinical presentation include helpful to recognize that certain causes
HDL2, chorea-acanthocytosis, and have a higher incidence, in particular,
dentatorubral-pallidoluysian atrophy geographic areas or ethnic origins
(DRPLA).28Y30 In McLeod syndrome, and, thus, should be prioritized when-
behavioral problems are more com- ever applicable. For example, HDL2
mon and cognitive difficulties are has been reported almost exclusively
milder.29 Self-mutilation of the lip and in subjects with a sub-Saharan African
tongue is a clinical manifestation of ancestry, namely African Americans
chorea-acanthocytosis, but is uncom- and black South Africans, and in the
mon in McLeod syndrome.31 Lesch- rare cases of a presumed non-African
Nyhan syndrome is an X-linked genetic origin, an African ancestor could not
condition in which patients can have be ruled out.33 Neuroferritinopathy
self-mutilating behaviors but the onset has been mostly described in families
occurs in childhood. from the United Kingdom (Cumbria
Chorea gravidarum, antiphospho- region), with a few cases reported in
lipid syndrome, or systemic lupus France and one single case in North
erythematosus are among nondegen- America.34 DRPLA is most frequent in
erative causes of chorea that can Japan, although it is not exclusive to
present with concomitant behavioral that country, with rare cases reported
symptoms, including personality in European and North American pop-
changes, depression, psychotic symp- ulations of non-Japanese ancestry.35
toms, and cognitive impairment.18 The Haw River syndrome corresponds
The occurrence of seizures is a dif- to a family of African Americans orig-
ferentiating feature of some HD inally from North Carolina, and has
phenocopies, occurring in about 50% been reported as a form of DRPLA.36
of the cases in chorea-acanthocytosis
(30% as presenting feature) and Past Medical History
McLeod syndrome, as well as in DRPLA The presence of acute hemichorea
and HDL1, an exceedingly rare entity or hemiballismus in a patient with

Continuum (Minneap Minn) 2016;22(4):1186–1207 www.ContinuumJournal.com 1195

Copyright © American Academy of Neurology. Unauthorized reproduction of this article is prohibited.


Chorea

KEY POINTS
h In the Western world, significant vascular risk factors sug- SCA17, C9orf72-related HD pheno-
chorea gravidarum has gests stroke as a cause, while in a pa- copy, HDL2, as well as DRPLA, neuro-
become a rare cause tient with diabetes mellitus, a primary ferritinopathy, SCA1, SCA2, SCA3, SCA7,
of chorea. diagnosis of nonketotic hyperglycemia and HDL1.37 In SCA17, most of the
h In C9orf72-related has to be considered in addition. Re- cases reported are sporadic or isolated
Huntington disease, current miscarriages, migraine, and subjects in a family with an ataxic syn-
phenocopies present thrombotic events are a clue for the drome. Although a family with a SCA17
with significant presence of an antiphospholipid syn- mutation and multiple members pres-
phenotypic heterogeneity drome or systemic lupus erythema- enting an HD-like phenotype show
and have, in most of tosus. Hemodialysis, alcoholism, and that phenotypic homogeneity may
the cases, a positive malnutrition can point to extrapontine exist, this is exceedingly rare.38 For
family history. myelinolysis, and the presence of C9orf72-related HD phenocopies,
human immunodeficiency virus (HIV) available data suggest a higher preva-
risk factors provide clues to the clini- lence of a positive family history,
cian for HIV-associated causes of cho- although with phenotypic heteroge-
rea (eg, opportunistic infections neity within the known spectrum of
including toxoplasmosis, progressive C9orf72-related clinical presentations.4
multifocal leukoencephalopathy, and The prevalence of C9orf72-related
HIV encephalitis). HD phenocopies should be further
The occurrence of chorea in a preg- assessed as in the initial case series,
nant woman should raise the possibility only three of the 10 cases reported
of chorea gravidarum. In the Western had chorea in their presentation.4
world, chorea gravidarum is increas- An autosomal recessive inheritance
ingly rare; currently, its most common pattern is compatible with chorea-
causes are antiphospholipid syndrome acanthocytosis and aceruloplasminemia.
and systemic lupus erythematosus.18 In HDL3 also has an autosomal recessive
the past, rheumatic fever was the most inheritance pattern but has only been
prevalent cause for chorea gravidarum, described in a Saudi Arabian family.
but it decreased sharply with the wide- The onset of symptoms is in early
spread use of penicillin.18 childhood, and its presentation re-
sembles the HD Westphal variant.39
Family History An X-linked recessive inheritance pat-
A clear pattern of inheritance will help tern is compatible with McLeod syn-
the clinician prioritize genetic causes drome, although rare female cases
of chorea in the differential diagnosis. have been reported.40
However, it is important to recognize
that the absence of a family history EXAMINATION
does not exclude a genetic disorder The observation of chorea, other
for many possible reasons, including movement disorders, and additional
nonpaternity, de novo mutations (in- neurologic or systemic signs provides
cluding unstable trinucleotide repeats), clues for the differential diagnosis.
premature death of asymptomatic
carriers, as well as partial penetrance Phenomenology of Chorea and
and phenotypic variability. In fact, ge- Related Movement Disorders
netic causes for chorea are frequently Chorea is one of the hyperkinetic
found in sporadic cases (Case 8-1). movement disorders. Chorea (derived
In the presence of a family his- from the Greek word horos, meaning
tory, an autosomal dominant inheri- dance) is characterized by involuntary,
tance pattern is compatible with HD, brief, irregular, random movements
1196 www.ContinuumJournal.com August 2016

Copyright © American Academy of Neurology. Unauthorized reproduction of this article is prohibited.


KEY POINTS
(Supplemental Digital Content 8-5, particular chorea syndrome. Dystonic h Hemichorea or
links.lww.com/CONT/A192; Supple- features can be present in HD and in hemiballismus suggest
mental Digital Content 8-6, links. the most prevalent HD phenocopies the presence of a focal
lww.com/CONT/A193). Ballismus (de- such as HDL2, SCA17, but also in brain lesion.
rived from the Greek word ballismos, chorea-acanthocytosis, DRPLA, and h The combination of
meaning jumping about) is considered neuroferritinopathy (orofacial dysto- other movement disorders
a form of chorea characterized by high- nia), among others. Parkinsonism is a can help with the
amplitude movement of a limb in a late feature of adult-onset HD at a stage diagnosis of a particular
flinging or flailing motion, including the in which chorea is less intense. Non- chorea syndrome.
most proximal segments (Supplemen- tremulous parkinsonism has been de-
tal Digital Content 8-7, links.lww. scribed in neuroferritinopathy, but it
com/CONT/A194). Athetosis (derived is a less common presenting feature
from the Greek word athetos, meaning (7.5%) compared with chorea (50%)
not fixed) is classically a slow, random, and focal limb dystonia (47.5%).20 More
involuntary, writhing movement of the prominent ataxic features, including eye
distal region of the limbs and is now movement abnormalities (saccadic pur-
recognized to be a manifestation of suit, dysmetric saccades, gaze-evoked
underlying dystonia, namely in cases nystagmus), limb incoordination, and
of cerebral palsy, and will not be wide-based gait with imbalance should
addressed in this article (Supplemen- make the clinician suspect an SCA,
tal Digital Content 8-8, links.lww.
SCA17 being a more frequent cause.
com/CONT/A195).
DRPLA can also be considered, and,
Chorea Distribution more rarely, Wilson disease can be
considered when ataxia is part of a
Particular locations of chorea are sug-
chorea syndrome. In SCA17, although
gestive of specific etiologies; aside
ataxia is most often present, marked
from the diagnostic value of promi-
phenotypic heterogeneity occurs, and
nent orolingual and truncal/cervical
the presence of a pure choreic syn-
chorea-dystonia previously described,
the presence of hemichorea or hemi- drome does not exclude this diagno-
ballismus suggests a focal structural sis.38 It is worthwhile mentioning that
brain lesion secondary to a vascular choreic movements are exceptional
event, nonketotic hyperglycemia, and, in other SCAs, such as the example of
more rarely, an opportunistic infection SCA1, SCA2, SCA3, and SCA7.37,44 In
in HIV. Nevertheless, instances occur in these cases, ataxia is predominant, and
which hemichorea is a presenting fea- the description of chorea best fits with
ture of other conditions without a choreoathetosis. Very rare reports of
documentable focal brain lesion, ex- chorea in other SCAs can be found in
amples of which are autoimmune cho- the literature: hand choreic move-
rea including Sydenham chorea and ments have been described in SCA14
paraneoplastic syndromes as well as (PRKCG gene) and in SCA8 (ATXN8/
variant Creutzfeldt-Jakob disease.9,15,41 ATXN8OS gene) gene expansion, al-
Conversely, conditions such as non- though the pathologic role of an ex-
ketotic hyperglycemia can present as pansion in the ATXN8/ATXN8OS gene
generalized chorea42 (Case 8-3). remains a matter of discussion.45,46
Apart from eye movement abnormalities
Other Movement Disorders consistent with an ataxia syndrome, other
The presence of other movement dis- changes can be documented when ex-
orders can help with the diagnosis of a amining a patient with chorea. Difficulty

Continuum (Minneap Minn) 2016;22(4):1186–1207 www.ContinuumJournal.com 1197

Copyright © American Academy of Neurology. Unauthorized reproduction of this article is prohibited.


Chorea

Case 8-3
A 73-year-old woman presented to the
emergency department with an 8-day history
of generalized chorea with an acute onset.
There was no relevant past medical history, no
family history of chorea, and no known use
of neuroleptic drugs. Her examination was
remarkable for orofacial dyskinesias and
generalized chorea. The investigations for
vascular and metabolic cases of chorea were
remarkable for severe hyperglycemia on
admission (more than 500 mg/L). The brain
MRI showed bilateral putaminal hyperintensity
in T1-weighted imaging (Figure 8-143).
Chorea persisted beyond the normalization
of the patient’s hyperglycemia. She died
32 days after admission as a result of
unresolved sepsis.
Comment. This case shows that nonketotic
hyperglycemia may be a cause of acute-onset
chorea presenting in a generalized form. FIGURE 8-1 Brain MRI of the patient in Case 8-3 with
bilateral chorea-related nonketotic
The MRI findings highlighted in this case are hyperglycemia exhibiting putaminal
typical of the syndrome of chorea in the hyperintensity in T1-weighted imaging.
setting of nonketotic hyperglycemia. The Reprinted with permission from Mestre TA, et al. J Neurol Neurosurg
MRI findings can be unilateral or bilateral Psychiatry.43 B 2007 BMJ Publishing Group Ltd. jnnp.bmj.com/content/
78/5/549.extract.
consistent with the side(s) of the body
presenting with chorea.
Case modified with permission from Mestre TA, et al, J Neurol Neurosurg Psychiatry.43
B 2007 BMJ Publishing Group Ltd. jnnp.bmj.com/content/78/5/549.extract.

in initiating saccades is classically de- disturbances, and behavioral changes


scribed in HD. with a subacute presentation.9,48
Other associated neurologic signs
may suggest a particular diagnosis: In Systemic Features in Choreic
chorea-acanthocytosis (Case 8-2) and Syndromes
McLeod syndrome, areflexia (Achilles Classic neuroacanthocytosis syndromes
reflex absent in 90% of patients) and are recognized as multisystemic disor-
limb weakness together with muscle ders. McLeod syndrome is associated
wasting are suggestive of an axonal with a cardiomyopathy with atrial fib-
polyneuropathy and, in some cases, a my- rillation in 60% of the cases, in addition
opathy has also been documented.19,47 to hemolytic anemia, hepatomegaly,
Patients with SCA1, SCA2, and SCA3 and splenomegaly. 1 9 In chorea-
can also have a polyneuropathy. In acanthocytosis, hepatomegaly and
paraneoplastic syndromes, more com- splenomegaly are also found.32 The
monly there will be other neurologic presence of a photosensitive malar rash
signs, including peripheral neuropathy, and arthritis is suggestive of systemic
visual disturbances, ataxia, oculomotor lupus erythematosus. Signs of thyroid

1198 www.ContinuumJournal.com August 2016

Copyright © American Academy of Neurology. Unauthorized reproduction of this article is prohibited.


KEY POINTS
disease suggest the presence of under- considered, including cases without a h In neuroacanthocytosis
lying thyroid dysfunction. Anemia, dia- known family history for chorea and syndromes, the
betes mellitus, and retinal degeneration without an apparent cause after a first clinical multisystemic
are found in aceruloplasminemia. round of investigations. In the presence involvement is
Stigma of chronic liver failure can sug- of a symptom complex that points to a characteristic and has
gest acquired hepatocerebral degen- specific, albeit rare, cause of chorea, diagnostic and
eration, in which patients can present these neurodegenerative conditions management implications.
with orobuccal chorea resembling can also be considered in a first round h Huntington disease is
tardive dyskinesia, in addition to dys- of investigations (Table 8-1). the most frequent
tonia, parkinsonism, and ataxia, as well For HD, a trinucleotide cytosine- genetic cause of chorea.
as cognitive and behavioral problems.49 adenosine-guanine (CAG) equal to or
Kayser-Fleischer rings are observed in greater than 36 repeats is diagnostic in
Wilson disease. the presence of characteristic symptoms.
In the special case of asymptomatic
INVESTIGATIONS carriers, a result between 36 and 39
An initial panel of investigations should CAG repeats corresponds to incomplete
be used to consider treatable causes penetrance and uncertainty about
of chorea and include routine hema- phenoconversion in life. As with other
tology and blood biochemistry tests, neurodegenerative conditions caused
in addition to brain MRI and autoanti- by a trinucleotide expansion, a larger
body testing to identify antiphos- number of repeats is associated with
pholipid syndrome and systemic lupus an earlier age of onset, and a tendency
erythematosus (Table 8-4). Most of exists for the number of repeats to
these treatable causes have an acute increase from generation to genera-
or subacute onset, which is in striking tion, manifesting in the form of an
contrast with a more protracted course anticipation phenomenon for the age
of HD and HD phenocopies. As HD is of onset. CAG repeats in the interme-
the most prevalent genetic cause of diate range of 27 and 35 CAG repeats
chorea, testing for a pathologic repeat that have been described in rare case
expansion in the HTT gene should be reports compatible with symptomatic

TABLE 8-4 Initial Panel of Investigations in Patients With Chorea

1. Test for thyroid function, renal and liver function, electrolytes, erythrocyte
sedimentation rate, antinuclear antibodies, antiYdouble-stranded DNA
antibodies, anticardiolipin antibodies, and lupus anticoagulant.
2. Perform brain MRI.
3. If inconclusive or known family history of chorea, perform genetic test
for Huntington disease. If the latter genetic test is negative, consider
spinocerebellar ataxia type 17 and C9orf72 in white individuals, and
Huntington diseaseYlike syndrome type 2 in subjects with black
African ancestry.
4. Test for acanthocytes in peripheral fresh blood film. A single negative test is
not sufficient to rule out the presence of acanthocytes and should be done in
a laboratory with appropriate expertise; perform three assays.
DNA = deoxyribonucleic acid; MRI = magnetic resonance imaging.

Continuum (Minneap Minn) 2016;22(4):1186–1207 www.ContinuumJournal.com 1199

Copyright © American Academy of Neurology. Unauthorized reproduction of this article is prohibited.


Chorea

KEY POINTS
h The vast majority of HD are associated with significant phenocopies, and there was no diag-
Huntington disease behavioral abnormalities, with impli- nosis of SCA17. 53
phenocopies remain cations for genetic counseling.50,51 Among rare genetic entities that have
undiagnosed. Genetic testing in HD and other been reported to present as an HD phe-
h Spinocerebellar ataxia neurodegenerative conditions with an nocopy and have an available diagnostic
type 17 and adult onset not only impacts the life of test, Friedreich ataxia was reported as an
C9orf7-related the individual being tested, but also adult-onset choreic syndrome in the
Huntington disease his or her family, especially children. sixth decade of life, in association with
phenocopy are the most For family members at risk, the consid- cerebellar ataxia, and a single case of
frequent causes for a eration of predictive testing is usually adult-onset chorea was found to have
Huntington disease prompted by wishing to know the typical pathology of pantothenate
phenocopy. carrier status but also for a need to in- kinaseYassociated neurodegeneration.3,6
h Caudate atrophy can form life decisions including marriage, In more recent years, there has been
be found in Huntington parenthood, or a professional career. a significant breakthrough in the ge-
disease phenocopies, Genetic counseling is fundamental in netic basis of paroxysmal dyskinesia:
namely in Huntington order for patients to make a decision the myofibrillogenesis regulator 1 (MR-1)
diseaseYlike about predictive testing in a fully gene and the much less frequent
syndrome type 2, informed and free manner. Risks that potassium calcium-activated channel
chorea-acanthocytosis,
are associated with positive predictive subfamily M alpha 1 (KCNMA1) gene
and McLeod syndrome.
testing need to be discussed, includ- were identified in paroxysmal non-
ing the potential issues of discrimina- kinesigenic dyskinesia, the proline-
tion at work or for insurance purposes, rich transmembrane protein 2 (PRRT2)
tension in family and personal relations, gene was identified in paroxysmal
as well as stress on the patient. Legisla- kinesigenic dyskinesia, and the glucose
tion on protection against genetic dis- 1 transporter (SLC2A1 gene) was identi-
crimination exists but varies from fied in paroxysmal exertion-induced
country to country and, in the United (exercise-induced) dyskinesia.
States, can change from state to state. MRI can be helpful in the differential
A significant proportion of patients diagnosis of a suspected neurodegen-
presenting as an HD phenocopy cur- erative choreic syndrome, with atten-
rently remain undiagnosed. An alterna- tion to patterns of atrophy, presence
tive diagnosis is found in as low as of signal changes in T1, T2, and fluid-
2.8% of the cases in tertiary movement attenuated inversion recovery (FLAIR)
disorders centers.3 In an attempt to weighted images, and iron susceptibil-
prioritize investigations, elements related ity imaging. The hallmark feature in
with differentiating clinical features, clinical MRIs found in adult-onset HD
ethnic/geographic origin, and family is caudate atrophy (Figure 8-2). It is
history should be considered as de- important to emphasize that a few
scribed previously. Available clinical adult-onset neurodegenerative HD
data from case series in tertiary HD phenocopies may not be differentiated
centers from Western Europe suggest from HD based on MRI findings, exam-
that SCA17 and C9orf72 gene muta- ples of which include HDL2, chorea-
tions should be prioritized, particularly acanthocytosis, and McLeod syndrome.
in white individuals.4 HDL2 should be Nevertheless, in chorea-acanthocytosis
strongly considered in African Ameri- and McLeod syndrome, T2-weighted
cans. Of note, HDL2 may resemble HD hyperintensity in the striatum, mild gen-
more than any other known disease.52 eralized cortical atrophy (less in McLeod
In a Brazilian cohort, HDL2 and SCA2 syndrome), and hippocampal sclerosis
were the diagnoses found for HD and atrophy (in chorea-acanthocytosis)
1200 www.ContinuumJournal.com August 2016

Copyright © American Academy of Neurology. Unauthorized reproduction of this article is prohibited.


FIGURE 8-2 Caudate atrophy in a patient with Huntington
disease documented on T2-weighted MRI.

have been described.54 Cerebellar atro- be confirmatory of a diagnosis in spo-


phy occurs earlier and more severely in radic forms of chorea; in patients with
SCAs; in HD, cerebellar atrophy is a late chorea due to stroke, a vascular lesion in
and mild feature.55 In DRPLA, brain- the subthalamic nucleus is classically
stem and cerebellar atrophy can be described, but chorea can also be asso-
found in addition to diffuse T2-weighted ciated with strokes in other basal
hyperintensities in the deep subcortical ganglia locations, thalamus, and internal
white matter, and increased iron sus- capsule.59 In the variant Creutzfeldt-
ceptibility is found in the cerebellum, Jakob disease, the bilateral hyper-
dentate nuclei, and basal ganglia.56,57 intensity in the pulvinar of the thalamus
The presence of a distinctive pattern (hockey stick sign), more frequently
of iron deposition can suggest and, at found in FLAIR sequences, is almost
times, be diagnostic of neurodegen- pathognomonic.60 In nonketotic hyper-
eration with brain iron accumulation glycemia, T1-hyperintense lesions in
(Figure 8-358). In neuroferritinopathy, the putamen are observed.43 In ac-
hypointensities in the dentate nuclei, quired hepatocerebral degeneration, T1
red nuclei, basal ganglia, thalami, and hyperintense lesions are found in the
the rolandic cortex are found even in putamen and also in the internal cap-
nonmanifesting carriers.20 With disease sule, the mesencephalon, and the cere-
progression, a distinctive bilateral bellum, in addition to cavitations of the
pallidal necrosis with cystic degenera- basal ganglia, all thought to be second-
tion is observed (Figure 8-3).20 MRI ary to the deposition of manganese via
is also helpful to raise the suspicion or a portosystemic shunt.49 The presence
Continuum (Minneap Minn) 2016;22(4):1186–1207 www.ContinuumJournal.com 1201

Copyright © American Academy of Neurology. Unauthorized reproduction of this article is prohibited.


Chorea

KEY POINT
h The treatment of
chorea should aim at
reducing related
disability and improving
overall function.

FIGURE 8-3 MRI findings in neuroferritinopathy with hypointensity in T2*-weighted image


involving the globus pallidus, putamen, caudate nuclei, thalami (A), and dentate
nuclei (not shown). Hyperintensity in the putamen and pallidum in T2-weighted
fast spin echoYweighted image found at a later disease stage due to cystic degeneration (B).
58
Reprinted with permission from McNeill A, et al. Neurology. B 2008 American Academy of Neurology.
www.neurology.org/content/70/18/1614.short.

of brain calcification, better docu- step in the management of chorea. The


mented with a head CT, suggests idio- clinician should assess the degree of
pathic basal ganglia calcification once functional incapacity or social embar-
secondary causes are excluded; howev- rassment caused by chorea in a partic-
er, chorea is a rare presentation.61 Other ular individual. However, the lack of
investigations are to be specifically insight frequently shown by a patient
considered for certain sporadic and with chorea relative to the severity of
genetic causes (refer to Table 8-1 and his or her own symptoms can be a
Table 8-2), including an antibody panel limitation for an accurate assessment.
and search of an occult neoplasm in In fact, relatives often request treat-
paraneoplastic chorea. ment. Regardless of the decision, anti-
choreic medication should be used
MANAGEMENT judiciously as no drug has proven to
When chorea is the manifestation of a have a dramatic effect, and adverse
treatable condition, the main goal of effects may have an impact on the
management is treatment of the under- functional capacity of patients. Clas-
lying condition rather than of the sically, neuroleptics have been used
chorea itself. This includes removal of to treat chorea, although the evi-
the offending drug, normalization of dence supported by randomized con-
glycemia, immunomodulatory therapy trolled trials is scarce. The only treatment
for autoimmune chorea, or removal of shown to effectively treat chorea, in-
an underlying neoplasm. cluding HD, is the monoamine de-
When these approaches prove to pleter tetrabenazine.62 Due to the
be insufficient or a treatable cause is cost and tolerability profile of this
not found, symptomatic treatment is drug, it may be chosen as a second-
considered for chorea. Establishing line drug after the unsuccessful trial
clinical criteria for treatment is the first of an atypical neuroleptic. Monitoring

1202 www.ContinuumJournal.com August 2016

Copyright © American Academy of Neurology. Unauthorized reproduction of this article is prohibited.


KEY POINT
of depression, suicidal ideation, aka- No disease-modifying therapies are h In Huntington disease
thisia, and parkinsonism are manda- available for HD (although several and Huntington disease
tory with tetrabenazine. trials are ongoing) and other neuro- phenocopies, a
Pallidal deep brain stimulation (DBS) degenerative choreic syndromes. In multidisciplinary
has been considered as an option for aceruloplasminemia, anecdotal evi- management plan
the treatment of pharmacoresistant dence suggests that the use of iron is required.
chorea or ballismus in cases with sig- chelators may modify disease biology.65
nificant disability, as well as for head
drops in chorea-acanthocytosis, with CONCLUSION
a suggestion of efficacy in HD and Chorea is a hyperkinetic movement
chorea-acanthocytosis.22,63 disorder with a vast list of causes. Aside
The treatment of other symptoms from sporadic causes, the most preva-
in HD phenocopies deserves a special lent cause of chorea in the adult is HD.
comment. For severe tongue pro- The rigorous characterization of the
trusion, it is important to recognize phenomenology and associated neuro-
that swallowing and breathing difficul- logic and systemic features will help the
ties may be life-threatening. Botulinum clinician prioritize the investigations for
toxin injections have been considered the differential diagnosis of choreic
in these cases with success, in spite of syndromes. HD phenocopies represent
the risk of temporary dysphagia.21 a challenging diagnostic group; SCA17,
Tongue or lip biting can also be treated C9orf72 in white individuals, and HDL2
with botulinum toxin injections or in people of sub-Saharan African an-
with the use of mouth guards.21,64 In cestry, including African Americans, are
McLeod syndrome, orthoses may be the most frequent diagnoses, and apart
useful in patients with severe peripheral from a few other entities, the vast ma-
neuropathy. Patients with head drops jority of patients currently remain without
such as those found in chorea- a diagnosis. The management of chorea
acanthocytosis (but also in McLeod syn- represents a challenge, and the clinician
drome and advanced HD) can use head should be able to identify the disability
protective gear. secondary to chorea as the appropriate
Mood symptoms are generally indication for treatment. When a treat-
treated as in general psychiatry, since able cause of chorea is present, identi-
there are no medications specifically fication of that cause is mandatory.
approved for mood disorders in HD
and other choreic syndromes. There ACKNOWLEDGMENTS
are no cognitive-enhancing medica- The author would like to acknowl-
tions with proven efficacy for HD and edge David Grimes, MD, FRCPC, for
other choreic syndromes. For the con- providing Supplemental Digital Con-
ditions presenting with seizures, gen- tent 8-3 and Supplemental Digital
eral principles for the treatment for Content 8-4, as well insightful com-
epilepsy apply. ments. The author would also like to
In the choreic syndromes that are acknowledge Anthony E. Lang, OC, MD,
part of a multisystem disorder, manage- FRCPC, FAAN, FCAHS, FRSC, for provid-
ment requires the involvement of dif- ing Supplemental Digital Content 8-6
ferent medical specialties. An important and Joaquim J. Ferreira, MD, PhD, for
example is McLeod syndrome, which p r o v i d i n g S u p p l e m e n t a l Di g i -
requires periodic cardiac assessment tal Content 8-5, Supplemental Digital
due to the presence of cardiomyopathy Content 8-7, and Supplemental Digital
and the risk of cardiac sudden death. Content 8-8.
Continuum (Minneap Minn) 2016;22(4):1186–1207 www.ContinuumJournal.com 1203

Copyright © American Academy of Neurology. Unauthorized reproduction of this article is prohibited.


Chorea

VIDEO LEGENDS opening dystonia, chorea of the


Supplemental Digital trunk and neck, and dystonic pro-
Content 8-1 trusion of the tongue when eating
Senile chorea. Video shows a man in or speaking, but not impaired
his seventies with a clinical diagnosis drinking. Mild bradykinesia and
of senile chorea and who was found to sniffing with nose clearing are
have genetically proven Huntington also observed.
disease. Mild chorea is seen in the links.lww.com/CONT/A191
trunk, neck, and orofacial regions, and Courtesy of David Grimes, MD, FRCPC.
an exaggerated pout and grimace is
seen in the orofacial region. Supplemental Digital
Content 8-5
links.lww.com/CONT/A188
B 2016 American Academy of Neurology.
Chorea in Huntington disease. Video
shows florid choreic movements in a
man with genetically proven Hunting-
Supplemental Digital
ton disease. He exhibits brief, irre-
Content 8-2
gular, random, purposeless, involuntary
Levodopa-induced dyskinesia in movements flowing from one muscle
Parkinson disease. Video shows a 62- group to the next, which are super-
year-old woman with a 10-year history imposed on voluntary motor activity,
of Parkinson disease who now has namely walking.
severe, refractory levodopa-induced
dyskinesia. After an acute administra- links.lww.com/CONT/A192
tion of levodopa, she exhibits predom- Courtesy of Joaquim J. Ferreira, MD, PhD.
inantly choreic dyskinesia that involves
the lower limbs, trunk, and, to a lesser Supplemental Digital
extent, the upper limbs. Content 8-6
Chorea of the hands in Huntington
links.lww.com/CONT/A189 disease. Video shows focal chorea
B 2016 American Academy of Neurology. localized to the hands of a man
with Huntington disease. He ex-
Supplemental Digital hibits random piano-playing move-
Content 8-3 ments of the fingers that are brought
Paroxysmal kinesigenic dyskinesia. about by finger tapping and count-
Video shows a man with paroxysmal ing backward.
kinesigenic dyskinesia (PRRT2 gene
links.lww.com/CONT/A193
positive) presenting with chorea and
Courtesy of Anthony E. Lang, OC, MD, FRCPC,
dystonia. Upon standing, a 10-second FAAN, FCAHS, FRSC.
episode occurs involving the left up-
per limb with arm adduction, elbow Supplemental Digital
flexion, hand clenching, and choreic Content 8-7
movements of the fingers. Ballismus. Video shows a man with
links.lww.com/CONT/A190 right upper limb unilateral ballismus. He
Courtesy of David Grimes, MD, FRCPC. exhibits characteristic high-amplitude
involuntary flinging movements involv-
Supplemental Digital ing the most proximal segment of the
Content 8-4 upper limb.
Chorea-acanthocytosis. Video shows links.lww.com/CONT/A194
a 28-year-old man exhibiting jaw- Courtesy of Joaquim J. Ferreira, MD, PhD.

1204 www.ContinuumJournal.com August 2016

Copyright © American Academy of Neurology. Unauthorized reproduction of this article is prohibited.


Supplemental Digital 2013;80(12):1133Y1144. doi:10.1212/
WNL.0b013e3182886991.
Content 8-8
10. Riley D, Lang AE. Hemiballism in multiple
Athetosis. Video shows a woman with sclerosis. Mov Disord 1988;3(1):88Y94.
cerebral palsy and athetosis of the right doi:10.1002/mds.870030111.
hand. She exhibits characteristic, slow 11. Venkatesan EP, Ramadoss K, Balakrishnan R,
involuntary writhing movements of Prakash B. Essential thrombocythemia:
rare cause of chorea. Ann Indian Acad
the distal limb region. Underlying dys- Neurol 2014;17(1):106Y107. doi:10.4103/
tonia is evident with prominent lateral 0972-2327.128569.
deviation and flexion of the wrist and 12. Critchley M. The neurology of old age.
finger extension. Lancet 1931;217(5621):1119Y1127.
doi:10.1016/S0140-6736(00)90705-0.
links.lww.com/CONT/A195
13. Garcia Ruiz PJ, Gómez-Tortosa E, del Barrio A,
Courtesy of Joaquim J. Ferreira, MD, PhD.
et al. Senile chorea: a multicenter
prospective study. Acta Neurol Scand
REFERENCES 1997;95(3):180Y183. doi:10.1111/
1. Pringsheim T, Wiltshire K, Day L, et al. j.1600-0404.1997.tb00092.x.
The incidence and prevalence of 14. Rana AQ, Yousuf MS, Hashmi MZ,
Huntington’s disease: a systematic Kachhvi ZM. Hemichorea and dystonia due
review and meta-analysis. Mov Disord to frontal lobe meningioma. J Neurosci
2012;27(9):1083Y1091. doi:10.1002/ Rural Pract 2014;5(3):290Y292. doi:10.4103/
mds.25075. 0976-3147.133611.
2. Stevanin G, Camuzat A, Holmes SE,
15. Cardoso F, Seppi K, Mair KJ, et al. Seminar
et al. CAG/CTG repeat expansions at the
on choreas. Lancet Neurol 2006;5(7):589Y602.
Huntington’s disease-like 2 locus are rare
doi:10.1016/S1474-4422(06)70494-X.
in Huntington’s disease patients.
Neurology 2002;58(6):965Y967. 16. Erro R, Sheerin UM, Bhatia KP. Paroxysmal
doi:10.1212/WNL.58.6.965. dyskinesias revisited: a review of
3. Wild EJ, Mudanohwo EE, Sweeney MG, 500 genetically proven cases and a new
et al. Huntington’s disease phenocopies are classification. Mov Disord 2014;29(9):
clinically and genetically heterogeneous. 1108Y1116. doi:10.1002/mds.25933.
Mov Disord 2008;23(5):716Y720. doi:10.1002/ 17. Edwards MJ, Schrag A. Hyperkinetic
mds.21915. psychogenic movement disorders. Handb
4. Hensman Moss DJ, Poulter M, Beck J, et al. Clin Neurol 2011;100:719Y729. doi:10.1016/
C9orf72 expansions are the most common B978-0-444-52014-2.00051-3.
genetic cause of Huntington disease 18. Robottom BJ, Weiner WJ. Chorea gravidarum.
phenocopies. Neurology 2014;82(4):292Y299. Handb Clin Neurol 2011;100:231Y235.
doi:10.1212/WNL.0000000000000061. doi:10.1016/B978-0-444-52014-2.00015-X.
5. Paucar M, Xiang F, Moore R, et al. 19. Danek A, Rubio JP, Rampoldi L, et al.
Genotype-phenotype analysis in inherited McLeod neuroacanthocytosis: genotype
prion disease with eight octapeptide and phenotype. Ann Neurol 2001;50(6):
repeat insertional mutation. Prion 755Y764. doi:10.1002/ana.10035.
2013;7(6):501Y510. doi:10.4161/pri.27260.
20. Chinnery PF, Crompton DE, Birchall D, et al.
6. Grimes DA, Lang AE, Bergeron C. Late adult Clinical features and natural history
onset chorea with typical pathology of of neuroferritinopathy caused by the
Hallervorden-Spatz syndrome. J Neurol FTL1 460InsA mutation. Brain 2007;
Neurosurg Psychiatry 2000;69(3):392Y395. 130(pt 1):110Y119. doi:10.1093/
doi:10.1136/jnnp.69.3.392. brain/awl319.
7. Wong JC, Armstrong MJ, Lang AE, Hazrati LN. 21. Schneider SA, Aggarwal A, Bhatt M, et al.
Clinicopathological review of
Severe tongue protrusion dystonia:
pallidonigroluysian atrophy. Mov Disord
clinical syndromes and possible treatment.
2013;28(3):274Y281. doi:10.1002/mds.25232.
Neurology 2006;67(6):940Y943.
8. Piccolo I, Defanti CA, Soliveri P, et al. doi:10.1212/01.wnl.0000237446.06971.72.
Cause and course in a series of patients
22. Schneider SA, Lang AE, Moro E, et al.
with sporadic chorea. J Neurol 2003;250(4):
Characteristic head drops and axial extension
429Y435. doi:10.1007/s00415-003-1010-76.
in advanced chorea-acanthocytosis. Mov
9. O’Toole O, Lennon VA, Ahlskog JE, et al. Disord 2010;25(10):1487Y1491. doi:10.1002/
Autoimmune chorea in adults. Neurology mds.23052.

Continuum (Minneap Minn) 2016;22(4):1186–1207 www.ContinuumJournal.com 1205

Copyright © American Academy of Neurology. Unauthorized reproduction of this article is prohibited.


Chorea

23. Chauveau M, Damon-Perriere N, Latxague C, North American and European pedigrees.


et al. Head drops are also observed in Mov Disord 1997;12(4):519Y530. doi:10.1002/
McLeod syndrome. Mov Disord 2011;26(8): mds.870120408.
1562Y1563. doi:10.1002/mds.23605. 36. Burke JR, Wingfield MS, Lewis KE,
24. Spampinato U, Debruxelles S, Rouanet M, et al. The Haw River syndrome:
et al. Head drops are also observed in dentatorubropallidoluysian atrophy
advanced Huntington disease. Parkinsonism (DRPLA) in an African-American family.
Relat Disord 2013;19(5):569Y570. doi:10.1016/ Nat Genet 1994;7(4):521Y524.
j.parkreldis.2013.01.012. doi:10.1038/ng0894-521.

25. van Duijn E, Craufurd D, Hubers AA, et al. 37. Pedroso JL, de Freitas ME, Albuquerque MV,
Neuropsychiatric symptoms in a European et al. Should spinocerebellar ataxias be
Huntington’s disease cohort (REGISTRY). included in the differential diagnosis for
J Neurol Neurosurg Psychiatry 2014;85(12): Huntington’s diseases-like syndromes? J
1411Y1418. doi:10.1136/jnnp-2013-307343. Neurol Sci 2014;347(1Y2):356Y358.
doi:10.1016/j.jns.2014.09.050.
26. Tabrizi SJ, Langbehn DR, Leavitt BR, et al.
38. Schneider SA, van de Warrenburg BP,
Biological and clinical manifestations of
Hughes TD, et al. Phenotypic homogeneity
Huntington’s disease in the longitudinal
of the Huntington disease-like presentation
TRACK-HD study: cross-sectional analysis of
in a SCA17 family. Neurology
baseline data. Lancet Neurol 2009;8(9):
2006;67(9):1701Y1703. doi:10.1212/
791Y801. doi:10.1016/S1474-4422(09)70170-X.
01.wnl.0000242740.01273.00.
27. Hubers AA, van Duijn E, Roos RA, et al. Suicidal
ideation in a European Huntington’s disease 39. Al-Tahan AY, Divakaran MP, Kambouris M,
population. J Affect Disord 2013;151(1): et al. A novel autosomal recessive ‘‘Huntington’s
248Y258. doi:10.1016/j.jad.2013.06.001. disease-like’’ neurodegenerative disorder in a
Saudi family. Saudi Med J 1999;20(1):85Y89.
28. Fischer CA, Licht EA, Mendez MF.
The neuropsychiatric manifestations of 40. Hardie RJ, Pullon HW, Harding AE, et al.
Huntington’s disease-like 2. J Neuropsychiatry Neuroacanthocytosis. A clinical, haematological
Clin Neurosci 2012;24(4):489Y492. and pathological study of 19 cases. Brain
doi:10.1176/appi.neuropsych.11120358. 1991;114(pt 1A):13Y49. doi:13-49.
29. Walterfang M, Evans A, Looi JC, et al. 41. Bowen J, Mitchell T, Pearce R, Quinn N.
The neuropsychiatry of neuroacanthocytosis Chorea in new variant Creutzfeldt-Jacob
syndromes. Neurosci Biobehav Rev disease. Mov Disord 2000;15(6):1284Y1285.
2011;35(5):1275Y1283. doi:10.1016/ doi:10.1002/1531-8257(200011)
j.neubiorev.2011.01.001. 15:6G1284::AID-MDS104393.0.CO;2-Y.
30. Rajput A. Dentatorubral pallidoluysian 42. Mestre T, Ferreira J, Pimentel J. Putaminal
atrophy. Handb Clin Neurol 2011;100:153Y159. petechial haemorrhage as the cause of
doi:10.1016/B978-0-444-52014-2.00008-2. non-ketotic hyperglycaemic chorea: a
31. Hewer E, Danek A, Schoser BG, et al. McLeod neuropathological case correlated with
myopathy revisited: more neurogenic and MRI findings. BMJ Case Reports 2009;2009.
less benign. Brain 2007;130(pt 12):3285Y3296. pii: bcr08.2008.0785. doi:10.1136/
doi:10.1093/brain/awm269 3285-3296. bcr.08.2008.0785.
32. Jung HH, Danek A, Walker RH. 43. Mestre TA, Ferreira JJ, Pimentel J. Putaminal
Neuroacanthocytosis syndromes. Orphanet J petechial haemorrhage as the cause of
Rare Dis 2011;6:68. doi:10.1186/1750-1172-6-68. non-ketotic hyperglycaemic chorea: a
33. Rodrigues GG, Teive HA, Tumas V. Huntington’s neuropathological case correlated with
disease-like 2 and apparent ancestry. Clin Genet MRI findings. J Neurol Neurosurg Psychiatry
2009;75(2):207; author reply 8. doi:10.1111/ 2007;78(5):549Y550. doi:10.1136/jnnp.
j.1399-0004.2008.01055.x. 2006.105387.

34. Ondo WG, Adam OR, Jankovic J, Chinnery PF. 44. Namekawa M, Takiyama Y, Ando Y, et al.
Dramatic response of facial stereotype/tic to Choreiform movements in spinocerebellar
tetrabenazine in the first reported cases of ataxia type 1. J Neurol Sci 2001;187(1Y2):
neuroferritinopathy in the United States. 103Y106. doi:10.1016/S0022-510X(01)00527-5.
Mov Disord 2010;25(14):2470Y2472. 45. Stevanin G, Hahn V, Lohmann E, et al.
doi:10.1002/mds.23299. Mutation in the catalytic domain of protein
35. Becher MW, Rubinsztein DC, Leggo J, et al. kinase C gamma and extension of the
Dentatorubral and pallidoluysian atrophy phenotype associated with spinocerebellar
(DRPLA). Clinical and neuropathological ataxia type 14. Arch Neurol 2004;61(8):
findings in genetically confirmed 1242Y1248. doi:10.1001/archneur.61.8.1242.

1206 www.ContinuumJournal.com August 2016

Copyright © American Academy of Neurology. Unauthorized reproduction of this article is prohibited.


46. Koutsis G, Karadima G, Pandraud A, et al. 57. Koide R, Onodera O, Ikeuchi T, et al.
Genetic screening of Greek patients with Atrophy of the cerebellum and brainstem
Huntington’s disease phenocopies identifies in dentatorubral pallidoluysian atrophy.
an SCA8 expansion. J Neurol 2012;259(9): Influence of CAG repeat size on MRI findings.
1874Y1878. doi:10.1007/s00415-012-6430-9. Neurology 1997;49(6):1605Y1612.
47. Rampoldi L, Danek A, Monaco AP. doi:10.1212/WNL.49.6.1605.
Clinical features and molecular bases of 58. McNeill A, Birchall D, Hayflick SJ, et al.
neuroacanthocytosis. J Mol Med (Berl) 2002; T2* and FSE MRI distinguishes four
80(8):475Y491. doi:10.1007/s00109-002-0349-z. subtypes of neurodegeneration with
48. Dalmau J, Rosenfeld MR. Paraneoplastic brain iron accumulation. Neurology
syndromes causing movement disorders. 2008;70(18):1614Y1619. doi:10.1212/
Handb Clin Neurol 2011;100:315Y321. 01.wnl.0000310985.40011.d6.
doi:10.1016/B978-0-444-52014-2.00024-0. 59. Ghika-Schmid F, Ghika J, Regli F,
49. Meissner W, Tison F. Acquired Bogousslavsky J. Hyperkinetic movement
hepatocerebral degeneration. disorders during and after acute stroke: the
Handb Clin Neurol 2011;100:193Y197. Lausanne Stroke Registry. J Neurol Sci
doi:10.1016/B978-0-444-52014-2.00011-2. 1997;146(2):109Y116. doi:10.1016/
S0022-510X(96)00290-0.
50. Killoran A, Biglan KM, Jankovic J, et al.
Characterization of the Huntington 60. Collie DA, Summers DM, Sellar RJ, et al.
intermediate CAG repeat expansion Diagnosing variant Creutzfeldt-Jakob
phenotype in PHAROS. Neurology disease with the pulvinar sign: MR imaging
2013;80(22):2022Y2027. doi:10.1212/ findings in 86 neuropathologically
WNL.0b013e318294b304. confirmed cases. AJNR Am J Neuroradiol
2003;24(8):1560Y1569.
51. Semaka A, Hayden MR. Evidence-based
genetic counselling implications for 61. Nicolas G, Pottier C, Charbonnier C, et al.
Huntington disease intermediate allele Phenotypic spectrum of probable and
predictive test results. Clin Genet genetically-confirmed idiopathic basal
2014;85(4):303Y311. doi:10.1111/cge.12324. ganglia calcification. Brain 2013;136(pt 11):
52. Schneider SA, Bhatia KP. Huntington’s disease 3395Y3407. doi:10.1093/brain/awt255.
look-alikes. Handb Clin Neurol 2011;100: 62. Armstrong MJ, Miyasaki JM; American
101Y112. doi:10.1016/B978-0-444-52014-2.00005-7. Academy of Neurology. Evidence-based
53. Castilhos RM, Souza AF, Furtado GV, guideline: pharmacologic treatment of
et al. Huntington disease and Huntington chorea in Huntington disease: report of
disease-like in a case series from Brazil. the guideline development subcommittee
Clin Genet 2014;86(4):373Y377. of the American Academy of Neurology.
doi:10.1111/cge.12283. Neurology 2012;79(6):597Y603. doi:10.1212/
WNL.0b013e318263c443.
54. Scheid R, Bader B, Ott DV, et al. Development
of mesial temporal lobe epilepsy in 63. Miquel M, Spampinato U, Latxague C, et al.
chorea-acanthocytosis. Neurology 2009;73(17): Short and long term outcome of bilateral
1419Y1422. doi:10.1212/WNL.0b013e3181bd80d4. pallidal stimulation in chorea-acanthocytosis.
55. Martino D, Stamelou M, Bhatia KP. PLoS One 2013;8(11):e79241. doi:10.1371/
The differential diagnosis of Huntington’s journal.pone.0079241.
disease-like syndromes: ‘red flags’ for the 64. Fontenelle LF, Leite MA.
clinician. J Neurol Neurosurg Psychiatry Treatment-resistant self-mutilation, tics,
2013;84(6):650Y656. doi:10.1136/ and obsessive-compulsive disorder in
jnnp-2012-302532. neuroacanthocytosis: a mouth guard as a
therapeutic approach. J Clin Psychiatry
56. Simpson M, Smith A, Kent H, Roxburgh R.
2008;69(7):1186Y1187.
Neurological picture. Distinctive MRI
abnormalities in a man with 65. Miyajima H, Takahashi Y, Kamata T, et al.
dentatorubral-pallidoluysian atrophy. Use of desferrioxamine in the treatment of
J Neurol Neurosurg Psychiatry 2012;83(5): aceruloplasminemia. Ann Neurol 1997;41(3):
529Y530. doi:10.1136/jnnp-2011-301612. 404Y407. doi:10.1002/ana.410410318.

Continuum (Minneap Minn) 2016;22(4):1186–1207 www.ContinuumJournal.com 1207

Copyright © American Academy of Neurology. Unauthorized reproduction of this article is prohibited.


Review Article

Ataxia
Address correspondence to
Dr Tetsuo Ashizawa, 6670
Bertner Ave, Houston
Methodist Research Institute,
R11-117, Houston, TX 77030, Tetsuo Ashizawa, MD, FAAN; Guangbin Xia, MD, PhD
tashizawa@houston
methodist.org.
Relationship Disclosure:
Dr Ashizawa receives ABSTRACT
research/grant support as Purpose of Review: This article introduces the background and common etiologies of
principal investigator of studies
for Ionis Pharmaceuticals, Inc ataxia and provides a general approach to assessing and managing the patient with ataxia.
(1515598769-CS2); the Recent Findings: Ataxia is a manifestation of a variety of disease processes, and an
Myotonic Dystrophy Foundation; underlying etiology needs to be investigated. Pure ataxia is rare in acquired ataxia
and the National Institutes of
Health (NS083564). Dr Xia disorders, and associated symptoms and signs almost always exist to suggest an
receives research/grant support underlying cause. While the spectrum of hereditary degenerative ataxias is expanding,
as principal investigator of special attention should be addressed to those treatable and reversible etiologies,
studies for Acorda Therapeutics,
Grand Aerie Fraternal Order of especially potentially life-threatening causes. This article summarizes the diseases that
Eagles, the National Institutes of can present with ataxia, with special attention given to diagnostically useful features.
Health (AR065836-01), While emerging genetic tests are becoming increasingly available for hereditary ataxia,
ReproCell Inc, and
ThermoFisher Scientific. they cannot replace conventional diagnostic procedures in most patients with ataxia.
Unlabeled Use of Special consideration should be focused on clinical features when selecting a cost-
Products/Investigational effective diagnostic test.
Use Disclosure:
Drs Ashizawa and Xia report
Summary: Clinicians who evaluate patients with ataxia should be familiar with the
no disclosures. disease spectrum that can present with ataxia. Following a detailed history and neu-
* 2016 American Academy rologic examination, proper diagnostic tests can be designed to confirm the clinical
of Neurology. working diagnosis.

Continuum (Minneap Minn) 2016;22(4):1208–1226.

INTRODUCTION of therapeutic opportunities. A prompt


Ataxia, defined as impaired coordina- management strategy for treatable
tion of voluntary muscle movement, is causes of ataxia can save the patient’s
a physical finding, not a disease, and life and result in a good long-term
the underlying etiology needs to be outcome. Ataxia can also be benign in
investigated. Ataxia can be the pa- largely symptomatic disorders (eg, ves-
tient’s chief complaint or a component tibular neuritis). With the advance-
among other presenting symptoms. ment of neurogenetics, more inherited
Ataxia is usually caused by cerebellar causes of cerebellar ataxia can be diag-
dysfunction or impaired vestibular or nosed,1,2 but many sporadic ataxias,
proprioceptive afferent input to the including those with a chronic and
cerebellum. Ataxia can have an insidi- progressive course, still remain undiag-
ous onset with a chronic and slowly nosed.3 It cannot be overemphasized
progressive clinical course (eg, spino- that it is easy to give a label of a neuro-
cerebellar ataxias [SCAs] of genetic degenerative cause, but finding a re-
Supplemental digital content: origin) or have an acute onset, espe- versible and treatable etiology should
Videos accompanying this
article are cited in the text as
cially those ataxias resulting from cer- be sought. The evaluation of ataxia has
Supplemental Digital Content. ebellar infarction, hemorrhage, or been reviewed with many different
Videos may be accessed by
clicking on links provided in the
infection, which can have a rapid approaches.4Y7 This article introduces
HTML, PDF, and app versions progression with catastrophic effects. symptomatology, neuroanatomy, clas-
of this article; the URLs are pro- Ataxia can also have a subacute onset, as sification, and common etiologies of
vided in the print version. Video
legends begin on page 1225. from infectious or immunologic disor- ataxia and provides a practical ap-
ders, which may have a limited window proach to the evaluation of ataxia.
1208 www.ContinuumJournal.com August 2016

Copyright © American Academy of Neurology. Unauthorized reproduction of this article is prohibited.


SYMPTOMS AND SIGNS OF Sensory ataxia. Sensory ataxia is
ATAXIA mainly reflected by gait disturbance,
Symptoms and signs are often related as previously described. In addition,
to the location of the lesions in the subjects with sensory ataxia will have
cerebellum. Lateralized cerebellar le- a positive Romberg sign. Subjects
sions cause ipsilateral symptoms and may walk with a high-stepping gait
signs, whereas diffuse cerebellar le- (due to associated motor weakness)
sions give rise to more generalized or feet-slapping gait (to assist with
symmetric symptoms. Lesions in the sound-induced sensory feedback).
cerebellar hemisphere produce limb Pseudoathetosis (random finger move-
(appendicular) ataxia. Lesions of the ments seen on outstretched hands with
vermis cause truncal and gait ataxia eyes closed) may also occur in sensory
with limbs relatively spared. Vestibulo- neuronopathy affecting the upper limbs.
cerebellar lesions cause disequilib- Truncal ataxia. Truncal ataxia may
rium, vertigo, and gait ataxia. Acute result from midline cerebellar lesions.
pathology in the cerebellum may Patients may present with truncal insta-
initially produce severe abnormalities; bility in the form of oscillation of the body
this may recover remarkably with time while sitting (worse with arms stretched
and can become asymptomatic even out in front) or standing (titubation).
when imaging shows persistent dra- Limb ataxia. Limb ataxia is often used
matic structural changes in the cere- to describe ataxia of the upper limbs re-
bellum. Chronic progressive ataxia is sulting from incoordination and tremor
not only due to neurodegenerative or and can be better described by functional
inherited cerebellar diseases but also impairment, such as clumsiness with writ-
neoplasms and chronic infections. ing, buttoning clothes, or picking up small
objects. The patient has to slow down the
Terms Describing Ataxia movement to be accurate in reaching
The following clinical terms are often things. Limb ataxia can be lateralized
used in describing ataxia. with ipsilateral cerebellar lesions.
Stance. A healthy person can stand Dysdiadochokinesia/dysrhythmokinesis.
naturally with feet spread less than Dysdiadochokinesia/dysrhythmokinesis
12 cm apart and is able to stand stable is tested by rapidly alternating hand
with feet together or in tandem for movements or tapping the index finger
more than 30 seconds. An impaired
on the thumb crease. Impairment can
stance in the absence of motor weak-
ness or gross involuntary movements be seen with irregularity of the rhythm
is suggestive of cerebellar ataxia or and amplitude.
sensory ataxia. Intention tremor. Intention tremor
Gait ataxia. Gait ataxia results from results from instability of the proximal
incoordination of the lower extremi- portion of the limb and is manifested
ties due to cerebellar pathology or loss by increasing amplitude of oscillation
of proprioceptive input. Patients often at the end of a voluntary movement. It
feel insecure and have to hold onto the is often tested by finger-to-nose and
wall or furniture and walk with feet
heel-to-shin maneuvers. This is dif-
apart. An increased gait disturbance
when visual cues are removed (walking ferent from essential tremor, which
with eyes closed or in the dark) sug- primarily occurs in the distal portion
gests a sensory or vestibular compo- of the limb.
nent to the ataxia. With cerebellar Dysmetria. Dysmetria is when the
causes, the gait ataxia remains the same patient misses the targeted object
regardless of visual cues. either due to overshoot (hypermetria)
Continuum (Minneap Minn) 2016;22(4):1208–1226 www.ContinuumJournal.com 1209

Copyright © American Academy of Neurology. Unauthorized reproduction of this article is prohibited.


Ataxia

or undershoot (hypometria). Dysmetria cerebellar disease. Square-wave jerks


is often tested by a finger-chasing test appear as two saccades in opposite
and can be quantified by the distance directions separated by a short period
(cm) that is missed. Dysmetria also of no movement. A healthy person may
happens when the eyes switch objects have a 0.1- to 0.3-degree square-wave
(ocular dysmetria); either the eyes need movement but have less than 10 per
a second move to catch the object minute. Large-amplitude square-wave
(hypometric saccades) or have to cor- jerks are more specific of cerebellar
rect the overshoot to focus on the ataxia. Ocular flutter differs from
object (hypermetric saccades). The square-wave jerks in that the repetitive
shin-tap test (accurate tapping of saccades are not separated by short
midshin or knee with a heel of the periods of no movement. Opsoclonus
opposite leg) also tests for dysmetria. is continuous conjugate saccades in all
Dysarthria/scanning speech. Dys- directions in a chaotic fashion. Both
arthria is often described by the ocular flutter and opsoclonus are gen-
patient or relatives as slurred speech. erally indicative of cerebellar disease
The patient’s speech is irregular and from paraneoplastic (neuroblastoma)
slow with unnecessary hesitation. or postinfectious syndromes (as can
Words are often broken into separate be seen in opsoclonus-myoclonus
syllables, and some syllables with a ataxia).
plosive consonant are unusually Neurologists should be familiar
stressed (scanning speech). with the specific ataxia terms and use
Nystagmus. Nystagmus often oc- them appropriately in documentation
curs in cerebellar disease. Lateral and communication with colleagues.
gaze-evoked nystagmus is seen by It is important to understand the
slow drift toward midline followed by nomenclature because it often implies
a fast phase of saccades to the eccen- a certain ataxic disorder.
tric position. Upbeat and downbeat
nystagmus are defined by the rapid NEUROANATOMY OF ATAXIA
phase in the up or down direction. The cerebellum and its afferent and
Upbeat nystagmus is seen in lesions of efferent connections, the vestibular
the anterior vermis. Downbeat nystag- system, and the proprioceptive sen-
mus is typically seen in a lesion in the sory pathway (Figure 9-1) are all
foramen magnum such as an Arnold- involved in ataxia. The cerebellum is
Chiari malformation. usually separated into the midline
Saccades. Saccade speed is typically cerebellum and the cerebellar hemi-
normal in cerebellar disease but often spheres. Lesions in each of these
an overshoot or undershoot (ocular regions can result in a different pre-
dysmetria) is present, and is often fol- sentation of ataxia. For example, dam-
lowed by a corrective saccade in the age to midline cerebellar structures
appropriate direction. However, in SCA usually presents with gait and truncal
type 2 (SCA2) and advanced stages ataxia, while damage to the unilat-
of other SCAs, saccades are slowed eral cerebellar hemisphere usually
(Supplemental Digital Content 9-1, causes ipsilateral cerebellar ataxia.
links.lww.com/CONT/A196). Understanding this neuroanatomy and
Square-wave jerks/ocular flutter/ correlation to coordination can help
opsoclonus. Square-wave jerks, ocular with localization. The correlations are
flutter, and opsoclonus are terms used listed in Table 9-1, although significant
to describe other ocular disturbances in clinical overlap exists among them.
1210 www.ContinuumJournal.com August 2016

Copyright © American Academy of Neurology. Unauthorized reproduction of this article is prohibited.


KEY POINT
h A precise diagnosis of
ataxia cannot be based
on clinical features
alone. Clinical
manifestations can be
variable, and similar
features may not occur
in all individuals with a
particular disease. For
many of the rarer ataxic
disorders, the clinical
features have been
defined on the basis of
limited clinical experience.

FIGURE 9-1 Afferent and efferent connections of the cerebellum. Main cerebellar afferent
connections are by climbing fibers from the inferior olives through the inferior
cerebellar peduncles, and pontine mossy fibers through the middle cerebellar
peduncles. Cerebellar outputs are from the dentate nucleus and other deep cerebellar nuclei
through the superior cerebellar peduncles.

CLASSIFICATION AND ETIOLOGIES and the features indicated may not occur
OF ATAXIA in all individuals with a particular disease.
There are different ways to classify For many of the rarer ataxic disorders,
ataxias: by age of onset, tempo of the clinical features have been defined on
onset, and clinical course; anatomic the basis of limited clinical experience.
involvement; focal or generalized; or
acquired or inherited. Common etiol- Hereditary Ataxias
ogies (mainly acquired ataxias) are Hereditary ataxias are rare disorders, but
listed in Table 9-2, although signifi- are more frequently diagnosed than they
cant clinical overlap exists among were previously as diagnostic technolo-
them. Hereditary ataxias are listed gies are advancing. Hereditary ataxias
separately in Table 9-3 and Table 9-4. are classified as autosomal dominant, au-
Sporadic ataxia remains a temporary tosomal recessive, X-linked, or mitochon-
diagnosis for adults before a definitive drial ataxias (Table 9-3 and Table 9-4).
etiology is found. Cases in which all The characteristic features may help
specific diagnoses listed in Table 9-3 with recognizing a specific diagnosis.9Y11
and Table 9-4 have been ruled out are Inherited ataxias, especially autoso-
classified as sporadic adult-onset ataxia mal dominant cerebellar ataxias, should
of unknown etiology, which still re- be considered when the disease is
mains a diagnostic challenge.8 transmitted vertically from one genera-
The list of clinical features is only a tion to the next within a family. A
rough guide, and a precise diagnosis documentation of a father-to-son trans-
cannot be based on such features alone. mission ascertains the autosomal dom-
Clinical manifestations can be variable, inant inheritance (Case 9-1).

Continuum (Minneap Minn) 2016;22(4):1208–1226 www.ContinuumJournal.com 1211

Copyright © American Academy of Neurology. Unauthorized reproduction of this article is prohibited.


Ataxia

TABLE 9-1 Correlation of Neuroanatomy With Clinical Features of Ataxia

Ataxia or Ataxialike Presentation


Neuroanatomy Function Arising From Damage of the Region
Cerebellar hemisphere, Integration of sensory input and Ipsilateral limb ataxia,
including dentate nuclei motor planning for coordination dysdiadochokinesia, dysmetria,
of complex tasks intention tremor, and scanning speech
Midline cerebellar structures Motor execution, rapid and slow Gait ataxia and imbalance, truncal
(vermis, fastigial and interposed eye movements, balance, lower ataxia, dysmetria, ocular findings,
nuclei, the vestibulocerebellum, extremity coordination, and head bobbing, and vertigo
and the paravermis) vestibular function
Posterior lobe (flocculonodular Integration of information from Nystagmus, postural instability, and
lobe) vestibular nuclei gait ataxia
Cerebral cortex (frontal lobe) Planning and initiating gait Frontal ataxia (Bruns apraxia), magnetic
gait (different from ataxic gait), but
associated pathology in this region can
worsen ataxia
Brainstem (vestibular nuclei, Relay signals in and out of Ataxia associated with cranial nerve
inferior olivary nuclei, pontine cerebellum dysfunction and motor-sensory deficits
nuclei, cerebellar peduncles)
Spinal cord (cuneate fasciculus, Conduction of sensory pathways Sensory ataxia
gracile fasciculus, and
spinocerebellar tracts [mossy fibers])
Musculoskeletal system Stabilizing the weight-bearing hip Waddling gait rather than ataxia, but
(gluteal muscles) associated pathology in this region can
worsen ataxia
Peripheral sensation system and Proprioception, visual cues Sensory ataxia with Romberg sign, can
visual system worsen cerebellar ataxia
Vestibular system (labyrinth of Sense of balance and special Disequilibrium, loss of balance associated
the inner ear, vestibular nerve, orientation, equilibrium with dizziness and vertigo, tinnitus and
vestibular nuclei) hearing impairment, nystagmus

Spinocerebellar Ataxias inherited disorders and up to 5% of


The presence of a family history consis- patients with apparently sporadic degen-
tent with autosomal dominant inheri- erative ataxia may have positive DNA
tance warrants DNA testing for an testing. Because of the cost of DNA test-
SCA, which, in genetic terminology, re- ing, health insurance coverage should
fers to a group of autosomal dominant be taken into consideration. Some clin-
disorders with a known chromosomal ical features described in Table 9-3 and
locus. Primary mitochondrial mutations Table 9-4 may provide clues to stream-
and X-linked mutations may also be line the DNA testing. While no cure for
considered depending on the clinical the SCAs is known, a positive DNA test
manifestations and family history. With result provides important practical in-
the absence of family history and exclu- formation to patients and their families.
sion of secondary ataxias, sporadic cases First, it enables patients to plan for the
may still warrant DNA testing since a future, such as educational or career
negative family history does not exclude decisions, particularly in predictive testing.

1212 www.ContinuumJournal.com August 2016

Copyright © American Academy of Neurology. Unauthorized reproduction of this article is prohibited.


TABLE 9-2 Etiologies of Ataxia According to Different Classifications

Classification Common Etiologies


Time course Acute (hours to days): Ischemic and hemorrhagic strokes,
multiple sclerosis, vestibular neuritis, infections (cerebellitis),
parainfectious syndromes, toxic disorders
Subacute (over weeks): Mass lesions in the posterior fossa;
meningeal infiltrates; infections such as human immunodeficiency
virus (HIV); Creutzfeldt-Jakob disease; deficiency syndromes such
as vitamin B1 and vitamin B12; hypothyroidism; immune
disorders such as paraneoplastic, gluten, and antiYglutamic acid
decarboxylase (GAD) ataxia; alcohol
Chronic (months to years): Mass lesions such as meningiomas,
craniovertebral junction anomalies, alcohol, idiopathic/sporadic
cerebellar ataxias, hereditary ataxias, neurodegenerative ataxias
(Friedreich ataxia, multiple system atrophyYcerebellar type)
Episodic ataxias: Many inborn errors of metabolism, episodic
ataxia syndromes
Distribution Focal ataxias: Posterior circulation strokes (ischemic, hemorrhagic),
primary or metastatic cerebellar tumors, bacterial abscess,
progressive multifocal leukoencephalopathy, multiple sclerosis,
congenital cyst (Dandy-Walker syndrome)
Symmetric ataxias: All other systemic, toxic, genetic and
neurodegenerative causes of ataxia such as intoxication
(alcohol, phenytoin, lithium, barbiturates, toluene, and other
chemicals), postinfectious syndrome, paraneoplastic syndrome,
ataxia associated with antigliadin antibodies, metabolic disorder,
viral cerebellitis, tabes dorsalis, prion disease, hereditary ataxia
(autosomal recessive, autosomal dominant, and mitochondrial
inherited ataxias)

Second, it provides a basis for genetic gists should consider the diagnosis of
counseling for patients and their families. multiple system atrophyYcerebellar type
Finally, it allows patients to engage in (MSA-C). Documentation of other ner-
support group and research activities vous system involvement, particularly
specific for the genetically defined disease basal ganglia and autonomic dysfunc-
entity. For example, the genotype de- tion, is critical for the diagnosis of
fined by DNA testing will be in inclusion MSA-C in patients with sporadic ataxia
criteria of most, if not all, future clinical (Case 9-2). MRI of the brain may
trials of disease-modifying treatments, provide a clue to the diagnosis of MSA
and once a therapy is developed and with brainstem atrophy causing the hot
approved, patients need to know the cross bun sign and posterior putaminal
genotype of their disease to determine hypointensity and a putaminal hyper-
whether the drug is suitable for them. intense rim on T2*-weighted imaging.
The remaining late-onset degenera-
Sporadic and Idiopathic Adult tive ataxias are often referred to as spo-
Ataxias radic adult-onset ataxia in the category
For older patients presenting with known as idiopathic late-onset cerebel-
sporadic degenerative ataxia, neurolo- lar ataxia. While these two terms have
Continuum (Minneap Minn) 2016;22(4):1208–1226 www.ContinuumJournal.com 1213

Copyright © American Academy of Neurology. Unauthorized reproduction of this article is prohibited.


Ataxia

TABLE 9-3 Autosomal Dominant Ataxias

Entity Characteristic Features in Addition to Ataxia Gene/Mutation


Spinocerebellar ataxia Hypermetric saccades, corticospinal ATXN1/CAG
type 1 (SCA1) tract signs expansion
SCA2 Slow saccades ATXN2/CAG
expansion
SCA3 (Machado-Joseph Bulging eyes, fasciculations, parkinsonism ATXN3/CAG
disease) expansion
SCA5 Downbeat nystagmus SPTBN2/deletion,
point mutation
SCA6 Downbeat nystagmus CACNA1A/CAG
expansion
SCA7 Vision loss due to retinal degeneration ATXN7/CAG
expansion
SCA8 Reduced penetrance ATXN8/CTG
expansion
SCA10 Seizures ATXN10/ATTCT
expansion
SCA11 Downbeat nystagmus TTBK2/deletion
SCA12 Action tremor in midlife PPP2R2B/CAG
expansion
SCA13 Developmental disability in childhood onset and a lack KCNC3/point
of eye movement abnormalities in adult onset (R420) mutation
SCA14 Tremor, myoclonus, facial myokymia PRKCG/deletion,
point mutation
SCA15/SCA16 Postural and kinetic tremor, psychiatric symptoms or ITPR1/duplication
dementia
SCA17 Dysarthria before gait ataxia (Huntington diseaseYlike) TBP/CAG
expansion
SCA20 Spasmodic dysphonia, palatal tremor Unknown/point
mutation
SCA27 Developmental disability and tremor FGF14/point
mutation
SCA34 Tongue atrophy and fasciculations AFG3L2/point
mutation
SCA36 Myoclonus, choreoathetosis, dementia (Huntington NOP56/ GGCCTG
diseaseYlike) expansion
Dentatorubral- Hyperkeratosis, multiple system atrophyYcerebellar ATN1/CAG
pallidoluysian atrophy typeYlike (Huntington diseaseYlike) expansion.
Episodic ataxia type 1 Episodic, lasts seconds to minutes, interictal fasciculations KCNA1/point
mutation
Episodic ataxia type 2 Episodic, lasts from hours to days, interictal nystagmus CACNA1A/point
mutation

1214 www.ContinuumJournal.com August 2016

Copyright © American Academy of Neurology. Unauthorized reproduction of this article is prohibited.


TABLE 9-4 Autosomal Recessive Ataxias and Mitochondrial Ataxias

Entity Features in Addition to Ataxia Gene/Mutation


Friedreich ataxia Decreased proprioception, areflexia, FXN/GAA
square-wave jerks, scoliosis, high-arched feet expansion
Abetalipoproteinemia Failure to thrive, steatorrhea TTTP/point
mutation
Cayman ataxia Psychomotor retardation MTTP/point
mutation
Ataxia telangiectasia Telangiectasia in eyes and skin, ATM/point
cancer risk mutation
Ataxia with oculomotor Oculomotor apraxia APTX/deletion
apraxia type 1
Ataxia with oculomotor Oculomotor apraxia SETX/insertion
apraxia type 2
Spinocerebellar ataxia with Axonal neuropathy TDP1/point
axonal neuropathy type 1 mutation
Autosomal recessive spastic ataxia of Spasticity, distal muscle SACS/point
Charlevoix-Saguenay atrophy mutation
Autosomal recessive cerebellar Pure ataxia SYNE1/point
ataxia type 1 mutation
Refsum disease Retinitis pigmentosa, night blindness PAHX, PEX7/
point mutation
Myoclonic epilepsy, myopathy and sensory Axonal neuropathy, epilepsy POLG/point
ataxia/ataxia neuropathy syndrome mutation
Mitochondrial diseases (including myoclonic Maternally inherited, myopathy, external Point mutation
epilepsy with ragged red fibers [MERRF]; ophthalmoplegia, retinal pigmentary
neuropathy, ataxia, retinitis pigmentosa degeneration
[NARP] syndrome; mitochondrial
encephalomyopathy, lactic acidosis, and
strokelike episodes [MELAS]; Kearns-Sayre
syndrome)

been used interchangeably, some ex- and foot deformities, scoliosis, hyper-
perts include MSA-C in the category of trophic cardiomyopathy, and glucose
idiopathic late-onset cerebellar ataxia. intolerance.15 With genetic diagnosis,
the phenotype has expanded to in-
Friedreich Ataxia clude a later presentation (older than
Friedreich ataxia is one of the most 25 years of age up to 60 years of age)
common genetic autosomal recessive and slower progression, which is
ataxia syndromes. Onset is typically in termed late-onset Friedreich ataxia.
childhood and young adulthood with Such individuals have retained re-
progressive ataxia leading to loss of flexes, often referred to as Friedreich
ambulation after 10 to 15 years. Other ataxia with retained reflexes, which
clinical clues include sensory loss due may show spasticity with no cardiac
to dorsal root ganglion and dorsal or skeletal signs.16 The disease arises
column degeneration with areflexia from an abnormal expansion of GAA

Continuum (Minneap Minn) 2016;22(4):1208–1226 www.ContinuumJournal.com 1215

Copyright © American Academy of Neurology. Unauthorized reproduction of this article is prohibited.


Ataxia

Case 9-1
A 44-year-old woman presented with a 6-year history of deteriorating
balance. She had begun to use a cane 1 year prior to presentation. She also
noted neck spasms. Her father and brother had a similar progressive
neurologic disorder. She was of Cuban descent. On examination, she had very
slow saccades, normal pursuit eye movements, and no nystagmus. She had
pancerebellar ataxia with moderate dysmetria, intention tremor and
dysdiadochokinesia in her upper limbs, inability to maintain heel contact to
shin, a wide-based gait, inability to perform tandem standing and walking,
and scanning speech. Reflexes were absent in her arms and trace in her legs.
Cervical dystonia was observed. MRI of the brain showed cerebellar and
brainstem atrophy (Figure 9-212). DNA testing showed a heterozygous CAG
repeat expansion in the ATXN2 gene.

FIGURE 9-2 Sagittal MRI of the patient in Case 9-1


showing atrophy of the cerebellum and brainstem.
Reprinted with permission from Chakor RT, Bharote H, J
Postgrad Med.12 B 2012 Journal of Postgraduate Medicine. www.
jpgmonline.com/article.asp?issn=0022-3859;year=2012;volume=58;
issue=4;spage=318;epage=325;aulast=Chakor.

Comment. The patient’s family history indicates an autosomal dominant


inheritance pattern. The history of adult-onset ataxia, Cuban descent, and
signs characterized by very slow saccades and absent reflexes in addition to
cerebellar ataxia is highly suggestive of spinocerebellar ataxia type 2. In this
situation, a direct genetic test for spinocerebellar ataxia type 2 rather than a
panel is often cost effective.

repeats in the frataxin gene (FXN), of GAA repeats on the smaller allele.
although point mutations may be Loss of frataxin function in mitochondria
present on one of the chromosomes leads to iron-sulfur cluster deficits, im-
in lieu of the GAA expansion in rare paired oxidation, and iron accumulation.
cases. The age of disease onset is Nicotinamide has been shown to in-
inversely correlated with the number crease frataxin but no clinical benefit

1216 www.ContinuumJournal.com August 2016

Copyright © American Academy of Neurology. Unauthorized reproduction of this article is prohibited.


Case 9-2
A 71-year-old woman presented for evaluation
of an 11-year history of slowly progressive
dysarthria and appendicular, gait, and truncal
ataxia. At the time of evaluation, the patient
walked with a walker and noted urinary
urgency and incontinence and frequent
diarrhea with fecal incontinence, which she had
experienced for the past several years. Urinalysis,
urine cultures, and colonoscopy were recently
normal. She reported palpitations, excessive
drooling, and light-headedness. Family history
revealed that her father had been diagnosed
with Parkinson disease. She was on a regimen
of high-dose coenzyme Q10. Previous
neuropsychological testing showed no significant
cognitive impairments. On examination,
orthostatic hypotension was noted, with a
38 mm Hg drop in systolic blood pressure after
standing accompanied by light-headedness.
She had scanning dysarthria, irregular ocular FIGURE 9-3 Axial MRI of the patient in Case 9-2 showing
a hot cross bun sign.
pursuit, and nystagmus. Severe appendicular,
gait, and truncal ataxia was noted, and she also Modified with permission from Srivastava T, et al,
Neurology.13 B 2005 American Academy of Neurology. www.neurology.
demonstrated mild cogwheel rigidity and org/content/64/1/128.full.
bradykinesia. Brain MRI showed cerebellar and
brainstem atrophy with a hot cross bun sign
(Figure 9-313). She had no response to levodopa treatment. She underwent extensive tests for reversible,
treatable etiologies as well as genetic tests for inherited ataxias that were unrevealing.
Comment. This patient has multiple system atrophy (MSA)Ycerebellar type, based on the predominant
symptom of ataxia. If parkinsonism predominates the presentation, it can be classified as
MSAYparkinsonian type.14 The full features of cerebellar, striatonigral, and autonomic degeneration
developed 10 years after the first onset of ataxia. It is often not possible to diagnose MSA at the initial
onset of the disorder. A definitive diagnosis can only be made on pathologic findings of glial cytoplasmic
inclusions in the central nervous system.

has yet been demonstrated, and studies (eg, pancerebellar ataxia, isolated gait
with the iron chelator deferiprone and ataxia), associated manifestations, and
the antioxidant idebenone are unclear the cause of ataxia. Good history
on long-term benefit. Carbamylated taking will allow neurologists to predict
erythropoietin, interferon gamma-1b, the findings of the physical examination
pioglitazone, epoetin alfa, and a few with good accuracy. Detection of any
other clinical trials have been completed. surprising physical findings should
These studies have not shown convinc- warrant revisiting the history.
ing evidence of efficacy.17
History
EVALUATION OF THE PATIENT Ataxia may present with a variety of
WITH ATAXIA associated signs and symptoms that
The history should provide clues to the allow the neurologist to narrow the dif-
type of ataxia (eg, cerebellar ataxia, ferential diagnosis. The standard history
sensory ataxia) or vestibular dysfunc- and physical examination serve as a
tion, the affected parts of the body general framework for the evaluation
Continuum (Minneap Minn) 2016;22(4):1208–1226 www.ContinuumJournal.com 1217

Copyright © American Academy of Neurology. Unauthorized reproduction of this article is prohibited.


Ataxia

of ataxia. As reviewed previously, myelinating disease. Headache with


symptoms include gait disturbance/ nausea and vomiting suggests structural
imbalance, instability with sitting and lesions in the posterior fossa. Cognitive
standing, hand incoordination and impairment and hallucinations suggest
intention tremor, slurred speech, loss Wernicke-Korsakoff syndrome, intoxi-
of sensation, and paresthesia. Symp- cation, or poisoning. If imbalance hap-
toms suggesting serious or potentially pens in the dark, sensory ataxia should
life-threatening causes include head- be considered.
ache, nausea, and vomiting. For func-
The patient’s medication list should
tional disability, it is helpful to ask
about impairment in activities of daily be reviewed to determine a possible
living and instrumental activities of temporal association between suspected
daily living, such as problems with drug use and ataxia onset. Vitamin B6
buttoning clothes or reaching objects (pyridoxine) dosing exceeding 50 mg/d
and difficulty getting to the bathroom to 100 mg/d may induce neuropathy/
at night. neuronopathy, leading to sensory ataxia.
Age and gender. Cerebellitis is Prescription use of anticonvulsants (eg,
more common in children, and mi- phenytoin, which may cause ataxia
graine is more common in younger due to either acute toxicity with high
patients, particularly women. doses or chronic use), lithium, or
Acute, subacute, or chronic onset. chemotherapeutic agents may indicate
The rate and pattern of the develop- drug reaction.
ment of ataxia help to narrow the Social history can include questions
differential diagnosis. Acute-onset regarding occupation, possible toxic
ataxia needs special attention as some exposure to chemicals, sexual history
of the etiologies (eg, stroke) could be (eg, human immunodeficiency virus
life threatening. Gradual onset of [HIV], syphilis), drug abuse, and ex-
ataxia with or without persistent head- cessive alcohol intake. History of drug
ache or change in mentation may or alcohol abuse may indicate solvent
indicate a primary tumor. These symp- or alcohol poisoning or Wernicke
toms in association with unintentional encephalopathy. Occupational expo-
weight loss may indicate primary or sure to heavy metals or solvents may
metastatic malignancy. An intermittent indicate poisoning. Gait disorders that
pattern to the symptoms may indicate appear to be ataxia with exaggerated
entities such as transient ischemic and bizarre movements that are in-
attack, multiple sclerosis, and inher- consistent and incongruous with
ited episodic ataxia. known ataxia may indicate a nonor-
Associated symptoms and histori- ganic or functional etiology. However,
cal features. Associated symptoms can clinicians should remember that the
be clues to localize the lesion and existence of inconsistency and incon-
determine the underlying etiology. gruousness does not necessarily ex-
Symptoms include impairment of con- clude a coexisting organic ataxia. When
sciousness, visual changes, trouble underlying neurologic causes of ataxia
speaking and swallowing, focal sensory are distorted by psychogenic features,
loss or weakness, vertigo, slow and evaluation becomes challenging. Func-
abnormal movements, cognitive im- tional ataxia should be a diagnosis of
pairment, and behavior changes. Fa- exclusion.
cial droop, diplopia, pupillary defects, A review of systems should include
tongue deviation, dysarthria, and dys- constitutional symptoms (eg, weight
phonia suggest stroke, tumors, or de- loss, fever, night sweats) and history
1218 www.ContinuumJournal.com August 2016

Copyright © American Academy of Neurology. Unauthorized reproduction of this article is prohibited.


KEY POINT
of hypertension, diabetes mellitus, ing loss should be further evaluated h Physical examination
dyslipidemia, and atrial fibrillation. to rule out inner ear issues. should give attention
History of chronic diarrhea may be Cerebellar signs. The gait is often not only to appendicular
associated with bismuth abuse and affected first. Patients cannot stand ataxia by examination of
toxicity. Previous history of migraine, with feet together. A more sensitive limb movements and
visual aura, and triggers for onset of test is tandem stance or walking. upright ataxia by
ataxia may indicate migraine as cause. Patients tend to veer to the same side. examination of stance,
Recent infectious illness may indicate Gait is wide-based with titubation. A gait, and truncal ataxia,
cerebellitis or Miller Fisher syndrome. focal cerebellar lesion often leads to but also to ocular
ipsilateral impaired cerebellar func- dyskinesia, speech
Special Considerations on abnormalities,
tions, including limb dysmetria, inten-
Neurologic Examination of the proprioceptive loss, and
tion tremor, loss of check, hypotonia,
vestibular dysfunction.
Patient With Ataxia and dorsal spooning (hyperextension
Mental status examination. The role of interphalangeal joints) of the hand,
of the cerebellum in cognition is in- as well as dysarthria and nystagmus
creasingly recognized. The cerebellum (refer to the previous section on
is not only the site of motor coordina- symptoms and signs of ataxia).
tion but intimately communicates with Physical examination should give
the cerebrum for higher cortical func- attention not only to appendicular
tions, including frontal executive func- ataxia by examination of limb move-
tions, spatial orientation, motor ments and upright ataxia by examina-
memory, language functions, and emo- tion of stance, gait, and truncal ataxia,
tional recognition and production. Pa- but also to ocular dyskinesias, speech
tients with cerebellar disorders may abnormalities, proprioceptive loss,
exhibit disorders of these cognitive and vestibular dysfunction.
functions, which should be assessed Extrapyramidal signs. It is not
for on examination. uncommon for chronic progressive
Cranial nerve examination. In var- ataxia to be associated with extrapyra-
ious cerebellar disorders, examination midal signs. In hereditary ataxias,
of extraocular movements often re- extrapyramidal signs are often the
veals abnormal pursuit and saccade, indication of spreading of an underly-
ocular dyskinesia such as square-wave ing neurodegenerative process be-
jerks, ocular flutter, and opsoclonus. yond the cerebellum and brainstem.
Papilledema may develop with a cere- For example, while most SCAs com-
bellar mass lesion, especially with monly affect gait first, MSA and some
hydrocephalus. Ipsilateral loss of cor- SCAs may also have associated parkin-
neal reflex and eighth cranial nerve sonism. Parkinsonism in SCA2, SCA3,
dysfunction may suggest a cerebellopon- and SCA17 often responds to levo-
tine angle tumor. Facial and tongue dopa; however, when the striatum is
fasciculations may be a prominent sign involved, patients tend to have parkin-
of SCA3, and severe tongue atrophy and sonism and are not responsive to
fasciculations are signs of SCA36. levodopa. While levodopa therapy
Vestibular signs. Ataxia from the can induce dyskinesia in these pa-
vestibular system is almost always tients treated with levodopa, involun-
associated with vertigo and slow nys- tary movements, including dystonia,
tagmus with or without change of may be part of the disease process
position. Affected patients also tend of SCAs. Careful observation during
to veer to the ipsilateral side when the examination may be needed to
they try to walk in a straight line. Hear- detect them.
Continuum (Minneap Minn) 2016;22(4):1208–1226 www.ContinuumJournal.com 1219

Copyright © American Academy of Neurology. Unauthorized reproduction of this article is prohibited.


Ataxia

Strength. It is important to assess ataxias to monitor the natural history


whether the degree of ataxia can be and therapeutic response. However,
explained by muscle weakness. To they can also be used in the evaluation
offset ataxia, the examiner can hold of acquired ataxias to help gauge a
the patient’s hands and ask him or her patient’s functional disability and ac-
to stand from a sitting position and to tivities of daily living. The Brief Ataxia
stand on toes and heels to examine Rating Scale (BARS)21 is less time
the functional proximal and distal consuming than these rating scales
muscle strength. Symmetric proximal yet captures key ataxia measures and
muscle weakness suggests myopathy. is useful in clinical practice.
Distal muscle weakness suggests gen-
eralized neuropathy. Gait dysfunction DIAGNOSTIC TESTING
may also suggest muscle weakness The selection of diagnostic tests
rather than ataxia. For example, when should be tailored according to the
the hip girdle is weak due to myopa- clinical presentation and signs on
thy, the pelvis tends to shift toward physical examination.
the side, causing what is called wad-
dling gait, which should not be con- Laboratory Testing
fused with ataxic gait. If drug-induced ataxia is suspected,
Proprioceptive sensory system. therapeutic drug levels should be ex-
Loss of sensory input from spinocer- amined. Phenobarbital and butabarbital
ebellar tracts to the cerebellum may overdose may occur accidentally
cause sensory ataxia. Any impairment or intentionally. The therapeutic range
along the proprioceptive pathway may of lithium is narrow, and regular mon-
cause sensory loss (for example, itoring of blood levels is required.
Friedreich ataxia, ataxia with vitamin E Phenytoin does not follow linear phar-
deficiency, acquired sensory ataxias macokinetics and can easily attain
related to ataxic polyneuropathies supratherapeutic drug concentrations.
[eg, paraneoplastic sensory neurono- In the evaluation of acute ataxia in the
pathy], Sjögren syndrome, diabetes emergency department, blood alcohol
mellitus, vitamin B6 toxicity, Miller levels should always be tested. A level of
Fisher syndrome). This can be tested 150 mg/dL to 300 mg/dL will almost
by examining vibration and proprio- always affect coordination and balance.
ception at the great toe. The ataxia is Other laboratory tests include vitamin
usually worsened when taking out the B6 and vitamin B12 levels, thyroid-
visual cues, in contrast to cerebellar stimulating hormone (TSH), heavy
ataxia, where there is no difference in metal levels (bismuth, lead, mercury),
the degree of ataxia with and without HIV, syphilis, Lyme and toxoplasmosis
eyes closed. Such patients also have serology, paraneoplastic panel (includ-
trouble standing with feet together ing anti-Yo for breast and ovarian
and eyes closed (Romberg sign). cancers, anti-P/Q type voltage-gated
calcium channel for lung cancer, anti-Tr
Ataxia Evaluation Scales for Hodgkin disease, anti-Ri for breast
The International Cooperative Ataxia cancer, anti-Hu for lung cancer), and
Rating Scale (ICARS),18 the Scale for other autoantibodies (antiYglutamic
the Assessment and Rating of Ataxia acid decarboxylase [GAD], anti-GQ1b
(SARA),19 and the Friedreich’s Ataxia for Miller Fisher syndrome, and anti-
Rating Scale (FARS)20 were developed gliadin antibody). If hereditary ataxia is
for research purposes in inherited suspected, certain laboratory tests can
1220 www.ContinuumJournal.com August 2016

Copyright © American Academy of Neurology. Unauthorized reproduction of this article is prohibited.


help narrow down the specific genetic useful for ischemic stroke and infraten-
test needed, such as high or low torial structural lesion evaluation. Non-
vitamin E and abnormal lipoprotein invasive vascular imaging may also be
in abetalipoproteinemia, high "- diagnostically useful, as indicated.
fetoprotein and low immunoglobulin Some MRI signs are diagnostically
in ataxia telangiectasia, high serum useful, such as the hot cross bun sign
cholesterol and low albumin in ataxia (MSA), middle cerebellar peduncle high
with oculomotor apraxia type 1, high signal on T2-weighted images (fragile X
"-fetoprotein in ataxia with oculomo- tremor-ataxia syndrome [FXTAS]), and
tor apraxia type 2, and high plasma molar tooth sign (Joubert syndrome),
phytanic acid in Refsum disease. among others (Figure 9-2, Figure 9-3,
Figure 9-4,22 Figure 9-5,23 Figure 9-6,24
Imaging Figure 9-7,25 Figure 9-826).
Head CT may detect a mass in the
posterior cranial fossa and is exten- Genetic Testing
sively used in the clinical evaluation of For inherited ataxias, genetic tests
acute stroke, especially for the rapid are available for the CAG expan-
exclusion of intracerebral hemorrhage. sions involved in SCA1, SCA2, SCA3
However, head CT may not be ideal (Machado-Joseph disease), SCA6, SCA7,
for structural lesions in the cerebellum SCA12, SCA17, and dentatorubral-
or brainstem because of bone artifacts. pallidoluysian atrophy; CTG expansion
For that purpose, brain MRI is more associated with SCA8; GAA expansion
appropriate. Brain MRI is especially in Friedreich ataxia; pentanucleotide

FIGURE 9-4 Axial MRI of a 17-year-old girl with autosomal recessive spastic ataxia of
Charlevoix-Saguenay showing degeneration of the corticospinal tract in the
brainstem. The patient has had spasticity, ataxia, reduced fine motor function,
and abnormal plantar responses since she was 5 years old. The patient’s family history was
negative except for a cousin with spasticity. Consecutive axial T2-weighted MRI slices (A, B)
demonstrate linear hypointensity in the pons.
22
Reprinted with permission from Martin MH, et al, AJNR Am J Neuroradiol. B 2007 American Society of
Neuroradiology. www.ajnr.org/content/28/8/1606.short.

Continuum (Minneap Minn) 2016;22(4):1208–1226 www.ContinuumJournal.com 1221

Copyright © American Academy of Neurology. Unauthorized reproduction of this article is prohibited.


Ataxia

drial encephalomyopathy, lactic acido-


sis, and strokelike episodes (MELAS);
and neuropathy, ataxia, retinitis
pigmentosa (NARP) syndrome. Many
other recently defined gene mutations
can be tested for by selected lab-
oratories or research facilities.
Genetic tests are very accurate and
becoming less expensive. The choice of
tests can be made based on the
inheritance pattern, laboratory abnor-
malities, clinical characteristics, and
epidemiologic data. In autosomal dom-
inant and a handful of autosomal reces-
sive ataxias, genetic testing should be
done first, and if DNA testing is pos-
FIGURE 9-5 Joubert syndrome. Axial itive, conventional MRI and blood tests
MRI of a 5-year-old boy
with delayed milestones, become unnecessary for establishing
a prominent forehead and low-set ears, the diagnosis. For example, if siblings
and cerebellar ataxia, hypotonia, and
hyperreflexia, showing dysgenesis of the are affected with the typical clinical
isthmus (the part of the brainstem presentation of Friedreich ataxia (Sup-
between the pons and inferior colliculus),
thick superior cerebellar peduncles, and plemental Digital Content 9-2,
hypoplasia of the vermis. These MRI links.lww.com/CONT/A197), a DNA
findings produce the molar tooth sign. test should be ordered first. If two
Reprinted with permission from Koshy B, et al, J GAA expansion alleles are identified,
Trop Pediatr.23 B 2009 The Authors. Published by
Oxford University Press. MRI, nerve conduction studies and
EMG, and many other tests add only
ATTCT repeat expansion in SCA10 and academic footnotes to the diagnosis.
TGGAA repeat expansion in SCA31; These tests would change neither diag-
hexanucleotide GGCCTG repeat expan- nosis nor management unless atypical
sion in SCA36; conventional mutations features suggest concomitant pathol-
in SCA5, SCA13, SCA14, and episodic ogy. Similarly, in autosomal dominant
ataxia type 2; and mutations in POLG disorders with an established geno-
(mitochondrial recessive ataxia syn- type in a relative, direct DNA testing
drome), APTX (ataxia with oculomotor should be ordered rather than order-
apraxia type 1), and SETX (ataxia with ing other laboratory tests or MRI. The
oculomotor apraxia type 2). Other website www.genetests.org is an ex-
point mutations can also be commer- cellent resource for information re-
cially tested by specific mutation garding the evolving facts about such
analyses or whole exome sequencing, tests. A test panel without consider-
although interpretation of results be- ation of clinical features is not encour-
yond the well-characterized disease- aged because it is not cost effective.
causing mutations is challenging (eg, However, if the clinical features are
variants of unknown significance). Tests more typical for an inherited ataxia with
are also commercially available for a negative family history or if the family
the common mutations in mitochon- history is incomplete, the entire dom-
drial DNA that may be associated with inant ataxia panel should be per-
ataxias such as myoclonic epilepsy with formed. Whole exome sequencing,
ragged red fibers (MERRF); mitochon- whole genome sequencing, and whole
1222 www.ContinuumJournal.com August 2016

Copyright © American Academy of Neurology. Unauthorized reproduction of this article is prohibited.


KEY POINT
h When sequence analysis
is done in a patient with
ataxia, polymorphisms
of unknown significance
may be reported. Such
alterations have not been
previously reported to be
pathogenic. Whether
such polymorphisms are
truly pathogenic will
have to be determined by
additional research.

FIGURE 9-6 Fragile X tremor-ataxia syndrome. Axial


T2-weighted MRI of a 76-year-old man showing
increased signal intensity in the middle cerebellar
peduncles. The patient presented to a movement disorders
clinic with progressive gait ataxia since the age of 68, with later
development of memory problems, occasional confusion, and
tremor in his left hand during walking. His family history revealed
developmental disability in his grandson through his daughter
and premature ovarian failure in one of his granddaughters.
Reprinted with permission from Hagerman PJ, Hagerman RJ, Nat Clin Pract
Neurol.24 B 2007 Nature Publishing Group. www.nature.com/nrneurol/
journal/v3/n2/full/ncpneuro0373.html.

mitochondrial sequencing are becom- reported to be pathogenic. Whether


ing more and more available for idio- such polymorphisms are truly patho-
pathic late-onset cerebellar ataxia and genic will have to be determined by
sporadic adult-onset ataxia.2,27,28 Whole additional research.
exome sequencing and whole genome
sequencing are especially useful in CONCLUSION
detecting genetic mutations known to Ataxia is a condition that can be easily
cause specific diseases. However, these recognized by the patient’s coordina-
technologies have technical limitations tion problems, gait disturbance, and
in diagnosing a repeat expansion dis- trouble speaking. However, the inves-
order. Whole exome sequencing and tigation of underlying causes requires
whole genome sequencing usually gen- systematic evaluation. Other neuro-
erate sequence variants of unknown logic disorders that can give rise to
significance. Advanced bioinformatics, similar problems with gait and dexter-
which may be unavailable or costly, ity (eg, nerve and muscle disorders,
would be necessary to analyze the spinal cord diseases, and basal ganglia
pathogenic significance of these vari- diseases) can usually be distinguished
ants, although some research protocols on the basis of physical signs alone. In
offer thorough whole exome sequenc- acute settings, the clinician’s main mis-
ing and whole genome sequencing sion is to recognize life-threatening
applications. When sequence analysis events. One common cause of acute
is done, polymorphisms of unknown ataxia in adults is stroke, which typically
significance may be reported. Such happens suddenly with headache, nau-
alterations have not been previously sea, and vomiting. In nonacute settings,

Continuum (Minneap Minn) 2016;22(4):1208–1226 www.ContinuumJournal.com 1223

Copyright © American Academy of Neurology. Unauthorized reproduction of this article is prohibited.


Ataxia

FIGURE 9-7 Sensory ataxic neuropathy with dysarthria and


ophthalmoparesis (with POLG mutation). Axial
T2-weighted MRI of a 50-year-old man with
a 4-year history of cerebellar ataxia, numb feet, diplopia, and
slurred speech, showing well-defined symmetric signal
abnormality in the cerebellar white matter, lateral to and sparing
the dentate nuclei.
Reprinted with permission from van Gaalen J, van de Warrenburg BP, Pract
25
Neurol. B 2012, BMJ Publishing Group. pn.bmj.com/content/12/1/14.short.

FIGURE 9-8 Atrophy of the cervical spinal cord in a patient with Friedreich ataxia shown on
T1-weighted sagittal section (A) and T2-weighted horizontal section (B) showing
the atrophic cervical spinal cord (arrow) on MRI.
26
Reprinted with permission from Mascalchi M, et al, AJR. B 1994 American Roentgen Ray
Society. www.ajronline.org/doi/abs/10.2214/ajr.163.1.8010211.

1224 www.ContinuumJournal.com August 2016

Copyright © American Academy of Neurology. Unauthorized reproduction of this article is prohibited.


no simple algorithm or guideline to over 15,000 genes and resources. A
follow exists. However, being familiar search for ataxia links to a large array
with ataxia disorders will help physi- of genetic resources and research
cians solicit related symptoms and related to various ataxias.
diagnostic signs on physical examina- www.omim.org
tion, which can often prompt appropri-
ate laboratory and diagnostic tests to VIDEO LEGENDS
confirm a clinical working diagnosis. In Supplemental Digital Content 9-1
certain situations, finding an etiology can Early-onset spinocerebellar ataxia
become a daunting task. For acquired type 2. This video shows a girl with
ataxias, after the offending factor is slow eye movements and ataxia due to
removed, the disease progression nor- early-onset spinocerebellar ataxia type 2.
mally stops and the patient will recover She also has brisk tendon reflexes (not
in 6 months to a year. With early shown), which is not usual in adult-
intervention, especially at a young age, onset spinocerebellar ataxia type 2.
most patients compensate well and will links.lww.com/CONT/A196
only have very mild incoordination or B 2016 American Academy of Neurology.
none at all.
Supplemental Digital Content 9-2
USEFUL WEBSITES Friedreich ataxia. This video shows a
Ataxia Study Group. The Ataxia Study boy with Friedreich ataxia exhibiting gait
Group carries out clinical trials and other ataxia, proprioceptive loss, and dysarthria.
research to identify the causes of ataxia. links.lww.com/CONT/A197
www.ataxia-study-group.net B 2016 American Academy of Neurology.
Friedreich’s Ataxia Research Alliance.
Friedreich’s Ataxia Research Alliance is a ACKNOWLEDGMENTS
nonprofit organization engaged in sci- This work was supported by a grant
entific research dedicated to treating awarded to Dr Tetsuo Ashizawa from the
and curing Friedreich ataxia. National Institutes of Health (NS083564).
www.curefa.org
GeneTests. GeneTests provides informa- REFERENCES
1. Németh AH, Kwasniewska AC, Lise S,
tion on the various disorders and available et al. Next generation sequencing for
tests, laboratories, and clinics available for molecular diagnosis of neurological
the testing of inherited conditions. disorders using ataxias as a model. Brain
2013;136(pt 10):3106Y3118. doi:10.1093/
www.genetests.org brain/awt236.
National Ataxia Foundation. The Na- 2. Fogel BL, Lee H, Deignan JL, et al. Exome
tional Ataxia Foundation provides in- sequencing in the clinical diagnosis of
sporadic or familial cerebellar ataxia.
formation on the diagnosis and causes JAMA Neurol 2014;71(10):1237Y1246.
of ataxia, along with information about doi:10.1001/jamaneurol.2014.1944.
support groups, research, and care- 3. Barsottini OG, Albuquerque MV,
giver resources. Braga-Neto P, Pedroso JL. Adult
onset sporadic ataxias: a diagnostic
www.ataxia.org challenge. Arq Neuropsiquiatr
Online Mendelian Inheritance in 2014;72(3):232Y240. doi:10.1590/
0004-282X20130242.
Man (OMIM). OMIM is an online
catalog of human genes and genetic 4. Zeigelboim BS, Teive HA, Santos RS, et al.
Audiological evaluation in spinocerebellar
disorders containing information on ataxia. Codas 2013;25(4):351Y357.
all known mendelian disorders and doi:10.1590/S2317-17822013005000001.

Continuum (Minneap Minn) 2016;22(4):1208–1226 www.ContinuumJournal.com 1225

Copyright © American Academy of Neurology. Unauthorized reproduction of this article is prohibited.


Ataxia

5. Whelan HT, Verma S, Guo Y, et al. Evaluation Scale for pharmacological assessment of
of the child with acute ataxia: a systematic the cerebellar syndrome. The Ataxia
review. Pediatr Neurol 2013;49(1):15Y24. Neuropharmacology Committee of the
doi:10.1016/j.pediatrneurol.2012.12.005. World Federation of Neurology. J Neurol
6. Koontz DW, Maddux B, Katirji B. Sci 1997;145(2):205Y211. doi:10.1016/
Evaluation of a patient presenting with S0022-510X(96)00231-6.
rapidly progressive sensory ataxia. J Clin 19. Schmitz-Hübsch T, du Montcel ST, Baliko L,
Neuromuscul Dis 2004;6(1):40Y47. et al. Scale for the assessment and rating
doi:10.1097/01.cnd.0000133065.28161.00. of ataxia: development of a new clinical
7. Dinolfo EA. Evaluation of ataxia. Pediatr Rev scale. Neurology 2006;66(11):1717Y1720.
2001;22(5):177Y178. doi:10.1542/pir.22-5-177. doi:10.1212/01.wnl.0000219042.60538.92.
8. Klockgether T. Sporadic ataxia with adult 20. Subramony SH, May W, Lynch D, et al.
onset: classification and diagnostic criteria. Measuring Friedreich ataxia: interrater
Lancet Neurol 2010;9(1):94Y104. doi:10.1016/ reliability of a neurologic rating scale.
S1474-4422(09)70305-9. Neurology 2005;64(7):1261Y1262.
9. Jayadev S, Bird TD. Hereditary ataxias: doi:10.1212/01.WNL.0000156802.15466.79.
overview. Genet Med 2013;15(9):673Y683. 21. Schmahmann JD, Gardner R, MacMore J,
doi:10.1038/gim.2013.28. Vangel MG. Development of a brief ataxia
10. van de Warrenburg BP, van Gaalen J, Boesch S, rating scale (BARS) based on a modified
et al. EFNS/ENS consensus on the diagnosis form of the ICARS. Mov Disord 2009;24(12):
and management of chronic ataxias in 1820Y1828. doi:10.1002/mds.22681.
adulthood. Eur J Neurol 2014;21(4):552Y562. 22. Martin MH, Bouchard JP, Sylvain M, et al.
doi:10.1111/ene.12341. Autosomal recessive spastic ataxia of
11. Subramony SH, Xia G. Ataxic and cerebellar Charlevoix-Saguenay: a report of MR imaging
disorders. In: Daroff RB, Jankovic J, in 5 patients. AJNR Am J Neuroradiol 2007;
Mazziotta JC, Pomeroy SL, editors. Bradley’s 28(8):1606Y1608. doi:10.3174/ajnr.A0603.
neurology in clinical practice. 7th ed. 23. Koshy B, Oommen SP, Jasper S, et al.
New York, NY: Elsevier, 2015:217Y222. Development and dysmorphism in Joubert
12. Chakor RT, Bharote H. Inherited ataxia with syndromeVshort case series from India.
slow saccades. J Postgrad Med 2012;58(4): J Trop Pediatr 2010;56(3):209Y212.
318Y325. doi:10.4103/0022-3859.105471. doi:10.1093/tropej/fmp084.
13. Srivastava T, Singh S, Goyal V, et al. ‘‘Hot 24. Hagerman PJ, Hagerman RJ. Fragile
cross bun’’ sign in two patients with multiple X-associated tremor/ataxia syndromeVand
system atrophy-cerebellar. Neurology older face of the fragile X gene. Nat Clin
2005;64(1):128. doi:10.1212/ Pract Neurol 2007;3(2):107Y112. doi:10.1038/
01.WNL.0000141862.49535.E1. ncpneuro0373.

14. Gilman S, Wenning GK, Low PA, et al. 25. van Gaalen J, van de Warrenburg BP. A
Second consensus statement on the diagnosis practical approach to late-onset cerebellar
of multiple system atrophy. Neurology ataxia: putting the disorder with lack of
2008;71(9):670Y676. doi:10.1212/ order into order. Pract Neurol 2012;12(1):
01.wnl.0000324625.00404.15. 14Y24. doi:10.1136/practneurol-2011-000108.

15. Reetz K, Dogan I, Costa AS, et al. Biological 26. Mascalchi M, Salvi F, Piacentini S, Bartolozzi
C. Friedreich’s ataxia: MR findings involving
and clinical characteristics of the European
the cervical portion of the spinal cord. AJR
Friedreich’s Ataxia Consortium for Translational
Am J Roentgenol 1994;163(1):187Y191.
Studies (EFACTS) cohort: a cross-sectional
doi:10.2214/ajr.163.1.8010211.
analysis of baseline data. Lancet Neurol
2015;14(2):174Y182. doi:10.1016/ 27. Protasova MS, Grigorenko AP, Tyazhelova TV,
S1474-4422(14)70321-7. et al. Whole-genome sequencing identifies
a novel ABCB7 gene mutation for X-linked
16. Lecocq C, Charles P, Azulay JP, et al.
congenital cerebellar ataxia in a large family
Delayed-onset Friedreich’s ataxia revisited.
of Mongolian ancestry. Eur J Hum Genet
Mov Disord 2016;31(1):62Y69.
2016;24(4):550j555. doi:10.1038/
doi:10.1002/mds.26382.
ejhg.2015.139.
17. Abrahão A, Pedroso JL, Braga-Neto P, et al.
28. Pierson TM, Adams D, Bonn F, et al. Whole-exome
Milestones in Friedreich ataxia: more than a
sequencing identifies homozygous AFG3L2
century and still learning. Neurogenetics 2015;
mutations in a spastic ataxia-neuropathy
16(3):151Y160. doi:10.1007/s10048-015-0439-z.
syndrome linked to mitochondrial m-AAA
18. Trouillas P, Takayanagi T, Hallett M, et al. proteases. PLoS Genet 2011;7(10):e1002325.
International Cooperative Ataxia Rating doi:10.1371/journal.pgen.1002325.

1226 www.ContinuumJournal.com August 2016

Copyright © American Academy of Neurology. Unauthorized reproduction of this article is prohibited.


Review Article

Diagnosis and
Address correspondence to
Dr Vicki Shanker, Mount Sinai
Beth Israel, Phillips Ambulatory
Care Center, 10 Union Square

Management of East, Suite 5H, New York, NY


10003, vshanker@chpnet.org.
Relationship Disclosure:

Dystonia Dr Shanker reports no


disclosure. Dr Bressman
receives personal compensation
for serving on the scientific
Vicki Shanker, MD; Susan B. Bressman, MD, FAAN advisory boards of the
Bachmann-Strauss Dystonia &
Parkinson Foundation, Inc and
the Michael J. Fox Foundation
ABSTRACT and for serving on the editorial
Purpose of Review: This article highlights the clinical and diagnostic tools used to board of Movement Disorders.
assess and classify dystonia and provides an overview of the treatment approach. Dr Bressman receives
research/grant support from
Recent Findings: In the past 4 years, the definition and classification of dystonia the Michael J. Fox Foundation.
have been revised, and new genes have been identified in patients with isolated Unlabeled Use of
hereditary dystonia (DYT23, DYT24, and DYT25). Expanded phenotypes were reported Products/Investigational
Use Disclosure:
in patients with combined dystonia, such as those with mutations in ATP1A3. Treatment Drs Shanker and Bressman
offerings have expanded as there are more neurotoxins, and deep brain stimulation discuss the unlabeled/
has been employed successfully in diverse populations of patients with dystonia. investigational use of oral
medications for the treatment
Summary: Diagnosis of dystonia rests upon a clinical assessment that requires of dystonia, none of which are
the examiner to understand the characteristic disease features that are elicited approved by the US Food and
through a careful history and physical examination. The revised classification system Drug Administration.
uses two distinct nonoverlapping axes: clinical features and etiology. A growing * 2016 American Academy
of Neurology.
understanding exists of both isolated and combined dystonia as new genes are
identified and our knowledge of the phenotypic presentation of previously reported
genes has expanded. Genetic testing is commercially available for some of these
conditions. Treatment options for dystonia include pharmacologic therapy, chemo-
denervation, and surgical intervention. Deep brain stimulation benefits many patients
with various types of dystonia.

Continuum (Minneap Minn) 2016;22(4):1227–1245.

INTRODUCTION dated classification system character-


Dystonia is characterized by sustained izes dystonia on two axes, the first of
or intermittent muscle contraction caus- which is dependent on the clinical
ing abnormal, often repetitive move- presentation, and the second of which
ments, postures, or both. Recognition is dependent on etiology as defined by
of the patient’s historical and clinical supportive testing. Both the phenom-
features is essential to achieving the enology and classification system are
diagnosis. Once identified, classification reviewed in the following sections.
is necessary to guide further evaluation
and treatment. This article reviews the Phenomenology
key clinical features of dystonia, the cur- Dystonia is a hyperkinetic movement
disorder characterized by involuntary Supplemental digital content:
rent system of classification of dystonia, Videos accompanying this ar-
and the current approach to treatment. muscle contractions often initiated or ticle are cited in the text as
worsened by voluntary action. The mus- Supplemental Digital Content.
DIAGNOSIS OF DYSTONIA Videos may be accessed by
cles involved in the action (eg, writing, clicking on links provided in the
An understanding of dystonia phenom- foot tapping) are typically the same HTML, PDF, and app versions
of this article; the URLs are pro-
enology is vital to an accurate and muscles affected by dystonia, but dys- vided in the print version. Video
thorough patient assessment. An up- tonic contractions can occur distant legends begin on page 1241.

Continuum (Minneap Minn) 2016;22(4):1227–1245 www.ContinuumJournal.com 1227

Copyright © American Academy of Neurology. Unauthorized reproduction of this article is prohibited.


Dystonia

KEY POINTS
h Dystonia is a from the action as overflow (eg, foot group characterized as idiopathic is
hyperkinetic movement tapping on the right produces dysto- fluid, as causative genes may be dis-
disorder characterized nia in the resting left foot). When con- covered, and patients will subsequently
by involuntary muscle tractions are sustained, they produce be recategorized.
contractions often twisted or abnormal postures. These Diagnostic evaluation, including ge-
initiated or worsened by postures have a characteristic direction- netic testing, requires an understanding
voluntary action. ality, referred to as patterning. When of the clinical phenotypes associated
h In 2013 a consensus contractions are intermittent, they fre- with specific etiologies; this presents a
document presented a quently produce tremorlike activity or daunting challenge as we discover new
revised classification jerky movements.1 The amplitude of genetic etiologies and as we learn about
that categorized dystonic tremors is enhanced when the the variation in clinical expression of a
dystonia into two affected area is positioned against the genotype. DYT-TOR1A and DYT-THAP1
nonoverlapping axes. direction of the pull (ie, turning the neck are the most common genetic causes of
Axis I categorizes to the right in a patient with left cervical isolated dystonia, and commercial test-
dystonia by the clinical
dystonia). Likewise, dystonic movements ing is available for both. DYT-TOR1A
features and axis II
may subside when the affected area is has autosomal dominant inheritance
by etiology.
placed in the maximum direction of the with reduced penetrance estimated at
h DYT-TOR1A and pull. This placement is identified as the 30%. One recurring TOR1A mutation,
DYT-THAP1 are the
null point. Another characteristic, but a deletion of one of a pair of GAG triplets
most common genetic
not universal, feature in dystonia is the coding for glutamic acid, is responsible
causes of isolated
dystonia, and commercial
sensory trick, or geste antagoniste, which for almost all dystonia associated with
testing is available describes a tactile or proprioceptive this gene. TOR1A encodes the protein
for both. maneuver performed to minimize the torsinA that resides in the endoplasmic
dystonic movement.2 For example, plac- reticulum system. Its normal biological
ing a scarf around the neck may de- function is still under study and thought
crease cervical dystonia. Even the to involve protein trafficking; further-
thought of performing the trick can more, recent work using TOR1A mutant
reduce the dystonic movement.3 Pa- models demonstrated a loss of normal
tients may have several tricks, and tricks function with activation of endoplasmic
may vary among individuals. reticulum stress and discrete neurode-
generation.4 TOR1A dystonia is more
Classification common in the Ashkenazi Jewish pop-
Classification schemas for dystonia have ulation, which has a carrier mutation
evolved since first formulated in the frequency of 1:1000 to 1:3000, approx-
1980s. In 2013 a consensus document imately sixfold higher than the general
presented a revised classification that population. The mean age of onset is
categorized dystonia into two nonover- 13 years of age with an initial presen-
lapping axes.1 Axis I categorizes dysto- tation typically in an arm or a leg; leg
nia by the clinical features and axis II onset is often younger than arm onset
by etiology. Clinical features in axis I (mean of 9 years versus 15 years of age).
include the age of onset, affected body Spread to other limbs often occurs, al-
region, temporal pattern, and associ- though almost one-fourth of affected
ated features, as defined in Table 10-1. patients remain as focal dystonia; fur-
Axis II delineates the etiology of the thermore, the cranial muscles are usu-
dystonia and contains two approaches ally spared. DYT-THAP1 is also autosomal
to etiology: (1) Is there neuropathology dominant with reduced penetrance ap-
(degeneration, static/structural, or nei- proximated at 60% and is caused by
ther)? (2) Is there a cause (genetic, ac- mutations in THAP1 (Thanatos-associated
quired, or neither [idiopathic])? The protein domain containing apoptosis
1228 www.ContinuumJournal.com August 2016

Copyright © American Academy of Neurology. Unauthorized reproduction of this article is prohibited.


KEY POINT

TABLE 10-1 New Axis I Dystonia Classifications h Since the advent of


next-generation
b Age of Onset sequencing, three
additional genes have
Infancy (birthY2 years) been reported to cause
Childhood (3Y12 years) isolated adult-onset
dystonia: DYT-CIZ1,
Adolescence (13Y20 years)
DYT-ANO3, and
Early adulthood (21Y40 years) DYT-GNAL.
Late adulthood (940 years)
b Affected Body Region
Focal: One body region
Segmental: Two or more contiguous regions
Multifocal: Two or more noncontiguous regions
Hemidystonia: Multiple regions on one body side
Generalized: Trunk and two or more body regions (with or without leg)
b Temporal Pattern
Static
Progressive
Variability
Persistent dystonia
Action specific
Diurnal: Symptom occurrence and severity fluctuates with circadian variation
Paroxysmal: Self-limited episodes
b Associated Features
Isolated: Dystonia with or without tremor
Combineda:
Dystonia with other movement disorders (ie, myoclonus, parkinsonism)
Dystonia with other neurologic manifestations (ie, ataxia, dementia)
Dystonia with systemic manifestations (ie, organomegaly)
a
These groups are not mutually exclusive.

associated protein 1). The mean age of been reported to cause isolated adult-
onset is 16 years, and the usual site of onset dystonia: DYT-CIZ1, DYT-ANO3,
onset involves brachial, cervical, or cra- and DYT-GNAL. Prior to these discov-
nial muscles, and unlike DYT-TOR1A, eries, little was known about the cause
speech is often affected. DYT-THAP1 of isolated adult-onset dystonia. All three
dystonia may remain as a focal dystonia; conditions have autosomal dominant
however, more commonly, it becomes inheritance. Commercial testing is not
segmental or generalized. currently available for these mutations.
Since the advent of next-generation CIZ1 (cyclin-dependent kinase inhib-
sequencing, three additional genes have itor 1AYinteracting zinc finger protein 1)

Continuum (Minneap Minn) 2016;22(4):1227–1245 www.ContinuumJournal.com 1229

Copyright © American Academy of Neurology. Unauthorized reproduction of this article is prohibited.


Dystonia

KEY POINT
h The most common cause was identified in a large white kindred population screening, the mutation
of dopa-responsive with adult-onset cervical dystonia.4 accounts for only a very small propor-
dystonia is mutations in Subsequent screens have identified rare tion (1% to 2%) of cervical dystonia in
GCH1, which encodes or no pathogenic variants, suggesting adults. The full phenotypic spectrum and
the rate-limiting enzyme this gene very rarely underlies dysto- penetrance requires additional study.8,9
in the biosynthesis of nia.5,6 ANO3 (anoctamin 3) mutations Many of the combined dystonias have
tetrahydrobiopterin. manifest as tremor-predominant cervi- identified genes as well. Dopa-responsive
cal dystonia. Dystonia may also begin dystonia is due to deficiencies in en-
or spread to involve brachial, laryngeal, zymes involved in the biosynthesis of
or facial muscles. Some patients have dopamine. The most common cause
myocloniclike jerks in the head or arms of dopa-responsive dystonia is muta-
and the age of onset is broad, ranging tions in GCH1, which encodes the rate-
from childhood (4 to 5 years of age) to limiting enzyme in the biosynthesis of
adulthood (40 years of age).7 GNAL mu- tetrahydrobiopterin. Commercially
tations were first identified in two fam- available DNA tests include gene se-
ilies and then confirmed in about 15 quencing and also testing for deletions
additional families; this disorder is in- and duplications. CSF analysis and
herited in an autosomal dominant fash- phenylalanine loading testing are sup-
ion and produces a phenotype usually plementary tests that can assist in
marked by adult-onset cervical and cra- making the diagnosis. Most often,
nial dystonia (either focal or segmental, GCH1mutations cause a childhood
with rare early onset or generalization). onset disorder that is inherited domi-
The GNAL (G protein subunit alpha L) nantly with reduced penetrance.
encoded protein (G"olf) couples the Symptoms are predominantly dystonic
dopamine D1 receptor of the direct and often diurnal. Mean age of onset
pathway and adenosine A2A receptor ranges between 6 and 12 years of age,
of the indirect pathway to the activation with larger studies favoring an older age
of adenylate cyclase type 5 (ADCY5), re- of onset.10 Girls are more likely to be
sulting in adenosine 3¶,5¶-cyclic mon- affected. Case 10-1 illustrates a case of
ophosphate (cAMP) production.8 With dopa-responsive dystonia. Younger

Case 10-1
An 11-year-old girl presented for gait evaluation. Her mother had had a
normal pregnancy, and there were no complications at birth. The patient
walked at 9.5 months but her parents noted that her gait was clumsy and her
toes pointed inward. She walked better in the morning than in the evening.
Prior assessments included a diagnosis of femoral anteversion from a pediatric
orthopedist and cerebral palsy from a neurologist. When she was 6 years old,
she developed clenching of her left hand. The patient had a normal MRI of
her brain and spine. At the time of presentation, her parents noted new right
hand clenching and retrocollis when walking. Family history was significant
for a paternal grandfather with Parkinson disease. Neurologic examination
was pertinent for increased patellar reflexes. While sitting, there was mild
retrocollis and intoeing of both feet. There was dystonic posturing in both
arms (left greater than right) and legs when performing activities in the
limbs. Dystonic posturing was noted in all limbs when walking. She was
started on one-half of a 25 mg/100 mg tablet of carbidopa/levodopa per
day, and she was slowly increased to a whole 25 mg/100 mg tablet 2 times per
Continued on page 1231

1230 www.ContinuumJournal.com August 2016

Copyright © American Academy of Neurology. Unauthorized reproduction of this article is prohibited.


KEY POINTS
Continued from page 1230 h Rapid-onset
day and became asymptomatic. A diagnosis of dopa-responsive dystonia was
dystonia-parkinsonism is
made and was ultimately confirmed with genetic testing. Her examination
another combined
pre- and postlevodopa is shown in Supplemental Digital Content 10-1,
dystonia associated with
links.lww.com/CONT/A198.
parkinsonian features
Comment. Dopa-responsive dystonia should be considered in the evaluation with an identified
of all children presenting with dystonic symptoms. Clinical clues of genetic origin: mutations
dopa-responsive dystonia in this patient included diurnal variation and a in ATP1A3.
family history of parkinsonism. Levodopa therapy can completely eliminate
symptoms in the majority of cases. Early intervention can make a significant h Myoclonus-dystonia is
impact in the quality of the patient’s life. a syndrome most
commonly associated
patients often develop segmental or tions in sepiapterin reductase (SPR) are with mutations in the
generalized dystonia with greatest se- another cause of dopa-responsive dys- sarcoglycan epsilon
verity in the lower extremities; tonia. Onset is usually in the first year gene (SGCE).
hyperreflexia may be present. Chil- of life. Up to 75% of patients have an
dren may manifest parkinsonian fea- oculogyric crisis in the first 10 months
tures such as bradykinesia and rigidity, of life. Dystonia is not universally pres-
and presentation in adults may mimic ent in patients with SPR mutations.
Parkinson disease. Patients are also Parkinsonism may be seen as well.
more likely to have comorbid psychiat- Rapid-onset dystonia-parkinsonism
ric conditions such as obsessive-com- is another combined dystonia associ-
pulsive disorder, depression, or anxiety. ated with parkinsonian features with
Atypical presentations, which include an identified genetic origin: mutations
oculogyric crisis, tics, and myoclonus, in ATP1A3. The phenotype is marked
have been reported. Autosomal reces- by generalized or segmental dystonia
sive GCH1 deficiency was reported; it and parkinsonism; usually the bulbar
causes more severe symptoms, such as muscles are most severely affected. A
truncal hypotonia, diffuse spasticity, or hallmark feature present in most but
oculogyric episodes. Tyrosine hydroxy- not all is rapid onset or worsening of
lase deficiency is a rare, autosomal symptoms over hours to days, often
recessive cause of dopa-responsive dys- after an emotional or physical trigger.
tonia due to mutations to the TH gene Patients are generally poorly responsive
on chromosome 11. Patients have an to oral medications, including levo-
age of onset ranging from weeks after dopa. Alternating hemiplegia of child-
birth until 6 years of age. A range of hood and cerebellar ataxia, areflexia,
phenotypes exists, from a dopa- pes cavus, optic atrophy, and sensori-
responsive dystoniaYlike phenotype neural hearing loss (CAPOS) syndrome
with dystonia and mild parkinsonism are also caused by mutations in ATP1A3.
responsive to levodopa, to severe dis- Alternating hemiplegia of childhood,
ease seen in those with onset in infancy CAPOS syndrome, and rapid-onset
where there may be a progressive dystonia-parkinsonism differ in certain
encephalopathy and a variable re- clinical features, including age of onset,
sponse to levodopa. Clinical features in but also share features consistent with
severe cases include truncal hypotonia, their shared genetic etiology.11
severe hypokinesia, oculogyric crises, Myoclonus-dystonia is a syndrome
ptosis, and paroxysmal periods of leth- most commonly associated with mu-
argy associated with increased sweating tations in the sarcoglycan epsilon gene
and drooling as well as a marked delay (SGCE). Patients typically present in
in motor development. Biallelic muta- childhood and inheritance is autosomal

Continuum (Minneap Minn) 2016;22(4):1227–1245 www.ContinuumJournal.com 1231

Copyright © American Academy of Neurology. Unauthorized reproduction of this article is prohibited.


Dystonia

KEY POINTS
h The movement disorder dominant with imprinting (the epige- PxMD-PNKD.17 Patients develop acute
examination is crucial to netic process that leads to inactivation dystonia, chorea, or ballistic movements
identifying dystonia of a paternal or maternal allele) with lasting minutes to hours. Attacks are
phenomenology and preferential expression of the paternal infrequent and are commonly triggered
includes several allele. Myoclonus (which is the promi- by alcohol, caffeine, or stress.
elements not routinely nent feature) and dystonia are most All of the axis I features of dystonic
included in the frequently present in the cervical area syndromes are obtained clinically. His-
neurologic examination. and the upper extremities. Symptoms tory should include a careful screen
h Additional studies such often improve with alcohol. However, for exposure to causative agents and a
as genetic testing and patients are at risk for developing detailed family history for movement
neuroimaging may be alcohol dependence, which is seen in disorders. Patients should be asked
required to characterize approximately one-fourth of manifest- about the presence of any ameliorat-
a patient with dystonia ing carriers. Comorbid psychiatric diag- ing or exacerbating features including
along axis II of the noses are common with social and sensory tricks. The presence of depres-
classification system.
specific phobias present in one-third sion and anxiety, which may be comor-
of manifesting carriers. An excess of bid with many forms of dystonia, should
major affective disorder and obsessive- be assessed as well. The movement
compulsive disorder have been re- disorder examination is crucial to iden-
ported.12 A missense mutation in tifying dystonia phenomenology and
CACNA1B was recently identified in includes several elements not routinely
one family with myoclonus-dystonia included in the neurologic examina-
and may also be associated with car- tion. Table 10-2 supplies features of
diac arrhythmias. Painful limb cramps patient history and examination find-
and continuous high-frequency leg my- ings that are helpful to the diagnosis.
oclonus were also present in some of Case 10-2 illustrates the importance
this population.13 of clinical assessment in achieving the
Paroxysmal kinesigenic dyskinesia correct diagnosis and treatment plan.
is a condition where frequent brief Additional studies may be required
(seconds to minutes) hyperkinetic to characterize the etiology of a pa-
movements, including dystonia, are tient’s dystonia, as listed in axis II of
triggered with activity. Attacks are often the revised classification system. Aside
preceded with a sensory warning. Mu- from neurologic signs, the clinical eval-
tations in PRRT2 (proline-rich trans- uation should include an ophthalmic
membrane protein 2) on chromosome and general medical examination that
16 can cause the disease state, which is may point to a specific etiology (eg,
usually autosomal dominant.14,15 This Kayser-Fleischer rings and Wilson dis-
mutation is also associated with infan- ease). When concern exists for vocal
tile convulsions with choreoathetosis cord involvement, ear, nose, and throat
with or without paroxysmal kinesigenic referral is recommended for fiberoptic
dyskinesia, benign familial infantile laryngoscopy. MRI of the brain or spinal
seizures, episodic ataxia, hemiplegic cord should be ordered if the history or
migraine, and benign paroxysmal tor- examination suggests an underlying
ticollis of infancy. Paroxysmal exertionY etiology associated with structural pa-
induced dyskinesia and epilepsy can be thology. In cases where the diagnosis of
caused by mutations in SLC2A1, the dystonia due to a parkinsonian condition
gene that encodes the glucose trans- is considered, a dopamine transporter
porter 1 (GLUT1) of the blood-brain scan may be helpful. If the dopamine
barrier.16 Paroxysmal nonkinesigenic dys- transporter scan shows reduced striatal
kinesia is associated with mutations in dopamine terminals, Parkinson disease
1232 www.ContinuumJournal.com August 2016

Copyright © American Academy of Neurology. Unauthorized reproduction of this article is prohibited.


TABLE 10-2 Associated Clinical Features of Dystonia

Site of Assessment Historical Features Clinical Examination


18
Eyes (blepharospasm) Photophobia (common) ; other eye Stereotyped bilateral and synchronous
problems (ie, blepharitis, iritis, corneal orbicularis oculi spasms inducing
disease, or conjunctivitis); eyelid narrowing/closure of the eyelids19;
opening apraxia (rare) increased blinking
Vocal cords (laryngeal dystonia/ Change in voice quality; vocal tremor; Have the patient sustain ‘‘ahh’’ or ‘‘e’’
spasmodic dysphonia) complaints of cough, paroxysmal sound for 7 seconds, listening for
sneezing, hiccups, or dyscoordinate phonatory breaks; Adductor
breathing (typically seen when involvement: Constant harsh or tight
vocal cords are one of many areas voice, intermittent pitch and voice
of involvement)20 breaks in the middle of a word, or
glottal stops with tremor in the
middle of the word21; Abductor
involvement: Constant whispering,
intermittent breathiness with
consonants at the beginnings of
words, or voice tremor with breathy
breaks at 4 Hz to 5 Hz in the middle
of words
Neck (cervical dystonia) Neck pain22,23; isolated neck tremor24 Look for muscle hypertrophy; examine
the neck in multiple positions (ie,
head to each shoulder, complete head
turn, chin to chest, head tilted
upward); ask the patient to close eyes
and allow the neck to move in the
direction it feels most comfortable
Arms/hands (limb dystonia) Cramping or tightness in the hand Writing tasks (ie, drawing spirals,
while writing or during prolonged drawing loops across a page, writing
activity of the limb; involuntary a sentence in cursive), may show a
hand movements tight grip of the pen, unusual
posturing of the hand that may
progress as the patient continues to
write, breaks in the writing, dystonic
mirroring/overflow in another limb,
and involvement of more proximal
muscles; with tremor, observe for
directionality and examine the arms
in positions to look for null point
Legs/feet (limb dystonia) Cramping of feet; toe curling; Observe leg movements including
involuntary foot/leg/hip movements stomping movements, toe tapping,
and rapid alternating movements
(heel/toe); look for torsion in limb as
well as overflow; movements may
be slow but will not diminish in
amplitude; symptoms improve when
walking backward

Continuum (Minneap Minn) 2016;22(4):1227–1245 www.ContinuumJournal.com 1233

Copyright © American Academy of Neurology. Unauthorized reproduction of this article is prohibited.


Dystonia

Case 10-2
A 65-year-old ambidextrous woman presented with a chief complaint of
head tremor, which developed when she was 30 years old. The tremor was
associated with an uncomfortable tightness in her neck muscles. She was
never exposed to dopamine-blocking medications prior to symptom onset.
The patient’s symptoms improved if she lightly touched her cheek and
when she turned her head to the left. She never had tremor in her voice or
in her limbs, and she denied symptoms of writer’s cramp. Her mother had a
neck tremor as an adult. The patient had been previously diagnosed with
essential tremor, and propranolol had been prescribed but did not improve
symptoms. On examination, there was hypertrophy in both sternocleidomastoid
muscles. She had a jerky, multidirectional neck tremor, notable for a null
point with head turn to the left and neck flexed. There was no tremor in the
arms (Supplemental Digital Content 10-2, links.lww.com/CONT/A199). She
was diagnosed with cervical dystonia with dystonic tremor. Symptoms improved
with botulinum toxin injections that were initially placed in the right
sternocleidomastoid muscle and left splenius cervicis muscle with EMG guidance.
Later injections sets were modified to include bilateral splenii muscles as
well as the right semispinalis capitis and right longissimus muscles.
Comment. The presence of a tremor in the neck is often associated with
essential tremor. However, essential tremor never presents with isolated
neck tremor. The presence of isolated neck tremor should raise suspicion
for an alternate diagnosis. In this case, the patient had a sensory trick, muscle
pain, muscle hypertrophy, and a null point, all suggestive of a diagnosis of
cervical dystonia. Early misdiagnosis led to an ineffective treatment plan.
Careful attention to history and the physical examination will allow the
clinician to offer effective treatments early in the disease.

or an atypical parkinsonian condition is fending agents that may cause or ex-


the likely diagnosis. However, a dopa- acerbate symptoms. Dopamine-blocking
mine transporter scan does not have medications are common offenders and
100% specificity or sensitivity, and include some antiemetics (ie, prochlor-
further follow-up of the patient will perazine, metoclopramide) and antipsy-
be required.25 chotics including newer generation and
atypical agents (ie, risperidone, olanza-
TREATMENT OF DYSTONIA pine, ziprasidone, and aripiprazole)
The approach to dystonia management as well as older, typical, agents. Rarely,
is often multidisciplinary and may in- dystonia may be due to selective se-
clude physical and psychological therapy. rotonin reuptake inhibitors (SSRIs) such
Consideration of the patient’s age, symp- as fluoxetine and serotonin norepi-
tom location, symptom severity, and the nephrine reuptake inhibitors (SNRIs),
side effect profile of the proposed inter- including venlafaxine and duloxetine.26,27
vention is vital when developing a man- Tapering and discontinuation of these
agement plan. Treatment can be classified drugs often require coordinated care
into oral medications, chemodenervation, with a psychiatrist or primary care phy-
and surgical intervention. sician as alternate medications may be
needed for underlying conditions.
Oral Medications Aside from drug-induced dystonia,
It is helpful to first review the patient’s targeted, disease-specific treatments are
current medications to assess for of- increasing as our understanding of
1234 www.ContinuumJournal.com August 2016

Copyright © American Academy of Neurology. Unauthorized reproduction of this article is prohibited.


the etiology for the combined/complex doubt evolve with the increased ac-
dystonia syndromes expands (eg, cessibility to exome and genome se-
GLUT1 deficiency, cerebrotendinous quencing). Table 10-3 reviews these
xanthomatosis, manganese transport conditions and their treatments. One
disorder, Wilson disease).27,28 These targeted therapy that has been incor-
etiologies are rare, and a comprehen- porated into clinical diagnostic practice
sive screening approach has yet to be is treating children and adolescents,
established (although this will no especially those with diurnal variation,

TABLE 10-3 Treatment of Dystonia in Conditions With Disease-Modifying Therapies

Common Age of Onset Disease State Treatment


Infancy Aromatic L-amino acid Dopamine agonists, monoamine oxidase
decarboxylase deficiency inhibitors
Biotinidase deficiency Biotin
Cobalamin deficiencies Cobalamin derivatives, protein restriction,
(inherited subtypes AYG) or both
Cerebral creatine deficiency Creatine
type 3
Guanidinoacetate Arginine restriction, creatine, and ornithine
methyltransferase deficiency
Pyruvate dehydrogenase Thiamine, ketogenic diet, dichloroacetate
deficiency
Early childhood to adolescence Cerebral folate deficiency Folinic acid
Propionic aciduria Protein restriction, avoid or treat aggressively
any intercurrent illness
Early childhood to adulthood Glutaricaciduria type 1 Lysine restriction, avoid or treat aggressively
any intercurrent illness
Niemann-Pick C disease N-Butyldeoxynojirimycin (miglustat)
Wilson disease Zinc, penicillamine, trientine
Childhood Dystonia with brain Chelation therapy
manganese accumulation
Homocystinuria Methionine restriction
Maple syrup urine disease Leucine restriction, thiamine
Methylmalonic aciduria Protein restriction, avoid or treat
aggressively any intercurrent illness
Childhood to adulthood Abetalipoproteinemia Vitamin E, reduced-fat diet
Ataxia with vitamin E deficiency Vitamin E
Cerebrotendinous Chenodeoxycholic acid
xanthomatosis
Galactosemia Lactose restriction
Any age Coenzyme Q10 deficiency Coenzyme Q10

Continuum (Minneap Minn) 2016;22(4):1227–1245 www.ContinuumJournal.com 1235

Copyright © American Academy of Neurology. Unauthorized reproduction of this article is prohibited.


Dystonia

KEY POINTS
h Targeted, disease-specific with low-dose levodopa for possible have a significant and sustained improve-
treatments are increasing dopa-responsive dystonia (see the ment to low-dose therapy. A gradual
as our understanding of following section on dopaminergic titration starting at 25 mg/d to 50 mg/d
the etiology for the medications). of levodopa for children and 50 mg/d
combined/complex For most patients with multifocal and to 100 mg/d of levodopa for adults is
dystonia syndromes generalized dystonia, oral medical ther- recommended. Slow titration reduces
expands (eg, apy is the first-line approach. Medica- the risk of side effects such as nausea
GLUT1 deficiency, tion options include carbidopa/levodopa, and transient dyskinesias. Usually, a ro-
cerebrotendinous anticholinergics (ie, trihexyphenidyl), bust response occurs with 200 mg/d
xanthomatosis, benzodiazepines, baclofen, and to 300 mg/d but, in rare cases, higher
manganese transport
dopamine-depleting agents. A sum- doses up to 1000 mg/d are needed.29
disorder, Wilson disease).
mary of these medications, along with Wearing-off symptoms and motor fluc-
h For most patients with common doses and side effects, are tuations are not commonly seen in re-
multifocal and listed in Table 10-4. With the exception sponse to long-term use as they are
generalized dystonia,
of a few special circumstances, no in patients with Parkinson disease. Pa-
oral medical therapy is
controlled or randomized studies are tients with nonYdopa-responsive dys-
the first-line approach.
available to guide clinicians to choose tonia, whether idiopathic, genetic, or
one drug over another. acquired, may also have a partial clin-
Dopaminergic medications. ical response to levodopa. Some cases
Carbidopa/levodopa is the standard of dystonic tremor worsen with levo-
of care treatment for dopa-responsive dopa.30 Patients with cerebral palsy may
dystonia as patients characteristically not benefit from levodopa.31

TABLE 10-4 Common Oral Medications Used in Treatment of Dystonia

Therapeutic Daily Dose


Therapeutic Class and (Typically Divided Into
Medication 2Y4 Times a Day Dosing) Common Side Effect Profile
Anticholinergic
Trihexyphenidyl 6Y40 mg Blurry vision, confusion, constipation,
urinary retention, xerostomia
Benzodiazepine
Clonazepam 1Y4 mg Drowsiness, fatigue, aspartate transaminase,
and alanine transaminase elevation
Diazepam 10Y40 mg Drowsiness, fatigue
Dopamine precursor
Carbidopa/levodopa 75 mg/300 mgY500 mg/2000 mg Nausea
Dopamine-depleting agents
Tetrabenazine 12.5Y100 mg Akathisia, anxiety/nervousness, depression/
suicidality, neuroleptic malignant syndrome,
parkinsonism, sleepiness
+-Aminobutyric acid B
(GABA-B) agonist
Baclofen 40Y120 mg Drowsiness, fatigue, nausea, muscle weakness

1236 www.ContinuumJournal.com August 2016

Copyright © American Academy of Neurology. Unauthorized reproduction of this article is prohibited.


KEY POINTS
Antidopaminergic medications. Tetra- preferred anticholinergic agent and the h Carbidopa/levodopa is
benazine is an inhibitor of the vesicular only oral medication studied in a pro- the standard of care
monoamine transporter type 2 in spective double-blinded fashion for treatment for
the central nervous system, depleting the treatment of dystonia.36 Trihexy- dopa-responsive
dopamine in addition to norepineph- phenidyl is considered a first-line dystonia as patients
rine and serotonin. Tetrabenazine is therapy for dystonic tremor.37 The characteristically
used off-label for the treatment of proposed active mechanism is block- have a significant and
dystonia. It may be most helpful in age of muscarinic acetylcholine recep- sustained improvement
patients with tardive dystonia, with tors in the basal ganglia and is usually to low-dose therapy.
one retrospective chart review started at a 1 mg/d dose (one-half of a h Tetrabenazine may be
reporting that 70% of patients with 2 mg tablet) and slowly titrated to most helpful in patients
tardive dystonia had moderate to excel- efficacy or side effects. The medication with tardive dystonia,
lent response to the drug.32 A case is divided into doses given 2 to 4 times a with one retrospective
report described two siblings with day. Typical therapeutic dosing ranges chart review reporting
that 70% of patients
myoclonus-dystonia and SGCE muta- from 4 mg/d to 40 mg/d, with adults
with tardive dystonia
tions who improved while taking usually tolerating only the lower end of
had moderate to
tetrabenazine 25 mg 3 times per that range, whereas generally children excellent response
day.33 Dosing typically begins with tolerate higher dosages, up to 100 mg/d. to the drug.
one-half of a 25 mg tablet and is slowly Patients should be counseled on the
h Trihexyphenidyl is the
titrated by 12.5 mg doses every 3 to potential side effects of this medication,
preferred anticholinergic
5 days to efficacy, with most patients which include dry mouth, constipation, agent and the only oral
taking 50 mg to 100 mg divided into urinary retention, blurry vision, and medication studied
2 or 3 daily doses. Reserpine is another gastrointestinal upset. Potential central in a prospective
dopamine-depleting agent that has an effects include confusion, memory im- double-blinded fashion
additional mechanism of action causing pairment, hallucinations, and fatigue. for the treatment
irreversible inhibition of the vesicular Patients should not receive anticholin- of dystonia.
monoamine transporter type 1 in the ergics if they have a history of closed-
peripheral nervous system, which can angle glaucoma. Pyridostigmine (30 mg/d
lead to peripheral side effects of diz- to 120 mg/d) is often prescribed con-
ziness and gastrointestinal upset. For currently when patients reach higher
this reason, tetrabenazine is the pre- doses of trihexyphenidyl to counteract
ferred dopamine-depleting agent. Pa- the peripheral anticholinergic effects.
tients taking dopamine-depleting agents +-Aminobutyric acidYrelated drugs.
should be monitored for akathisia, par- Despite the lack of double-blind and
kinsonism, and depression. controlled studies, benzodiazepines
In addition to the dopamine-depleting such as clonazepam and diazepam
agents, trials of clozapine, a D4 receptor are commonly used in the treatment
blocker, have shown moderate bene- of dystonia. These drugs are pos-
fits in various types of dystonia.34 Pa- itive allosteric modulators of the
tients taking clozapine must enter a +-aminobutyric acid A (GABA-A) recep-
registry and are monitored closely for tor and are thought to amplify trans-
signs of developing agranulocytosis. mission through the GABA receptors.
Patients with tardive dystonia may also Retrospective studies have reported
benefit when quetiapine replaces the benefit in tremor-predominant forms
formerly used antipsychotic.35 of dystonia, myoclonus-dystonia, ble-
Anticholinergic medications. Anti- pharospasm, dystonia associated with
cholinergic medications are considered spasticity, and paroxysmal dyskinesias
first-line treatment in patients with gen- with prominent dystonia.38Y40 Clo-
eralized dystonia. Trihexyphenidyl is the nazepam is commonly used for the
Continuum (Minneap Minn) 2016;22(4):1227–1245 www.ContinuumJournal.com 1237

Copyright © American Academy of Neurology. Unauthorized reproduction of this article is prohibited.


Dystonia

KEY POINTS
h Baclofen is a treatment of paroxysmal nonkinesigenic range from 1 g/d to 7.6 g/d. Sodium oxy-
+-aminobutyric acid B dyskinesia. Dosing is usually twice daily, bate is a Schedule III controlled substance,
receptor agonist although, when initiated, often begins which can cause significant central ner-
reported in several with evening doses to offset possible vous system and respiratory suppression.
retrospective studies to sedation. Titration should be slow. Im- Pharmacologic treatment in spe-
demonstrate benefit in paired mentation and nausea are com- cial circumstances. Antiepileptic drugs
dystonia management, mon side effects. Patients are instructed have variable benefit in dystonia. Car-
especially in children to avoid sudden discontinuation of the bamazepine is the exception, shown to
with comorbid dystonia medication as cessation of large doses be beneficial in cases of paroxysmal
and spasticity. can cause seizures and delirium. kinesigenic dyskinesia. In Wilson dis-
h Chemodenervation with Baclofen is a GABA-B receptor ago- ease, a disorder of copper metabolism,
botulinum neurotoxin is nist reported in several retrospective copper chelators such as peni-
first-line therapy for studies to demonstrate benefit in dys- cillamine and trientine are typically
most patients with focal tonia management, especially in chil- first-line therapy. For more information,
and segmental dystonia.
dren with comorbid dystonia and refer to the article ‘‘Wilson Disease’’ by
spasticity. Because of its mild side effect Ronald F. Pfeiffer, MD, FAAN,44 in this
profile, oral baclofen is frequently tried issue of Continuum. Acute medication-
in adults as well. Patients are often induced dystonic reactions are dystonic
started on 10 mg/d to 15 mg/d given in movements that can occur within mi-
2 to 3 equally divided doses per day and nutes to hours of receiving a medi-
effective doses range from 30 mg/d to cation that causes a nigrostriatal D2
120 mg/d. Common side effects include dopamine blockade. These episodes
sedation, nausea, dizziness, hypotonia, are terminated commonly with the anti-
and impaired cognition. Continuous in- histamine diphenhydramine (25 mg to
trathecal baclofen is an alternative de- 50 mg IV or IM) or the anticholinergic
livery method considered for children benztropine (1 mg to 2 mg IV or IM).
with dystonia and leg spasticity or
cerebral palsy.41 Prior to surgical im- Chemodenervation
plementation, a trial is conducted where Chemodenervation with botulinum neu-
patient response to baclofen delivered rotoxin is first-line therapy for most
through a temporary catheter is as- patients with focal and segmental dys-
sessed. Intraventricular baclofen has tonia. Patients with generalized dysto-
shown benefit in patients with symp- nia can also receive therapy to targeted
tomatic generalized dystonia who are areas. Botulinum neurotoxin is a neu-
refractory to oral medications. Patients rotoxic protein that is produced by the
taking baclofen can also develop sei- bacterium Clostridium botulinum. The
zures, delirium, rebound muscle rigid- toxin is injected into the affected
ity, rhabdomyolysis, hyperpyrexia, and muscle(s) and taken up into associated
organ failure with abrupt discontinua- motor neurons. The toxin then blocks
tion of the baclofen; a slow taper off the vesicular release of acetylcholine
the medication is recommended. into the neuromuscular junction, ulti-
Sodium oxybate is the sodium salt mately reducing the involuntary activity
of +-hydroxybutyrate, a metabolite of of the affected muscles. Although
the neurotransmitter GABA. It is ap- seven distinct serotypes (A to G) exist,
proved for use in cataplexy and exces- only types A and B are used commer-
sive daytime sleepiness in narcolepsy. cially and have US Food and Drug Ad-
However there are reports that it is ministration (FDA) approval for clinical
helpful in myoclonus-dystonia and spas- use. Serotype A is available as onabotu-
modic dysphonia.42,43 Effective doses linumtoxinA, abobotulinumtoxinA, and
1238 www.ContinuumJournal.com August 2016

Copyright © American Academy of Neurology. Unauthorized reproduction of this article is prohibited.


KEY POINT
incobotulinumtoxinA. Another type A for the use of onabotulinumtoxinA for h Selection of the specific
toxin has completed phase 3 trials and laryngeal dystonia. botulinum toxin may
is similar to incobotulinumtoxinA in Patients may begin to have symp- have to do with outside
that it lacks complexing proteins.45 tomatic benefit as early as 2 days after factors including
All three available serotype A toxins treatment, and maximum benefit is medication cost,
are FDA approved for use in cervical typically reported 2 to 6 weeks after physician experience
dystonia. OnabotulinumtoxinA and injection, after which a tapering off with toxin, and office
incobotulinumtoxinA are FDA approved of efficacy occurs. Treatment benefit access to refrigeration.
for blepharospasm. OnabotulinumtoxinA, usually persists for 2 and one-half to
abobotulinumtoxinA, and incobotuli- 4 months, such that most patients re-
numtoxinA are approved by the FDA quire treatments three to four times
for upper limb spasticity and onabotu- per year. Injections should start at the
linumtoxinA is approved for lower limb lowest doses in the recommended
spasticity, but not specifically for limb range to decrease the risk of deleteri-
dystonia. Serotype B is available as ous side effects. Side effects are typi-
rimabotulinumtoxinB, which is ap- cally local and occasionally are due to
proved for the use in patients with diffusion of the toxin to unintended
cervical dystonia. locations adjacent to the intended
In 2016, the American Academy of target. Patients injected for blepharo-
Neurology (AAN) released updated spasm may report dry eyes, focal
guidelines for the use of botulinum hematoma at the injection site, ptosis,
toxin in dystonia. The committee re- and rare diplopia. Patients with cervi-
ported that abobotulinumtoxinA and cal dystonia may report dry mouth, ex-
rimabotulinumtoxinB are established cessive neck weakness, and, occasionally,
as effective and should be offered for the dysphagia. Patients with anterocollis
treatment of cervical dystonia (Level A). are more susceptible to developing
In addition, onabotulinumtoxinA and dysphagia due to the location of the
incobotulinumtoxinA are probably ef- injections. Patients injected for limb
fective and should be considered dystonia may develop weakness in the
(Level B) for use in cervical dystonia. injected limbs. Injections for vocal cord
Level B evidence exists that onabotu- adduction can cause hypophonia or
linumtoxinA and incobotulinumtoxinA hoarseness. On rare occasions, patients
are probably effective and should report dysphagia.
be considered for blepharospasm. Typically, treatment for dystonia be-
AbobotulinumtoxinA is possibly effec- gins with botulinum neurotoxin A;
tive and may be considered (Level C) however, comparison between the two
for blepharospasm.46 There were no strains has not shown a marked differ-
recommendations made for limb, ence in efficacy or side effects.49 Thus,
oromandibular, or laryngeal dystonia. selection of toxin may have to do with
In 2013, an expert panel performed outside factors including medication
a literature review and made recom- cost, physician experience with toxin,
mendations for the use of botulinum and office access to refrigeration. EMG
toxin in dystonia.47,48 The committee or ultrasound are often used to assist
gave a Level B recommendation for injections, especially in lower face,
the use of abobotulinumtoxinA and neck, and limbs. A systematic review of
onabotulinumtoxinA for limb dystonia the literature, performed to assess the
and Level C recommendations for the use impact of EMG and ultrasound on the
of these toxins in oromandibular dystonia. effectiveness of botulinum toxin type A,
They reported a Level C recommendation reported Level I evidence that either
Continuum (Minneap Minn) 2016;22(4):1227–1245 www.ContinuumJournal.com 1239

Copyright © American Academy of Neurology. Unauthorized reproduction of this article is prohibited.


Dystonia

KEY POINT
h Substantial data instrument significantly improved injec- pallidus internus (GPi) is the target for
suggest that patients tion outcomes.50 most surgeries. In 2006, the first sham
with DYT1 and isolated In the future, mapping of muscle study, including a group of 40 patients
non-DYT1 generalized endplates may improve efficacy and re- with isolated segmental or generalized
dystonia are most duce the side effects of toxin injection. A dystonia, was published.52 Patients
responsive to deep brain study using high-density surface EMG receiving true stimulation had signifi-
stimulation intervention. found that patients with cervical dysto- cant improvement when compared to
nia had equal clinical benefit when the sham. Substantial data suggest that
toxin was injected at half dose in the patients with DYT1 and isolated non-
motor unit endplates of the splenius DYT1 generalized dystonia are most
capitis or sternocleidomastoid mus- responsive to DBS intervention.53 How-
cles compared to normal doses during ever, patients with DYT6 dystonia may
the patients’ traditional injections.51 not have as robust of a response.54
More recent studies have shown im-
Surgical Intervention provement in patients with isolated
In 2003, deep brain stimulation (DBS) cervical dystonia resistant to chemode-
received a humanitarian exemption by nervation, combined dystonia such as
the FDA for use in dystonia. Since this myoclonus-dystonia, and dystonia in
time, it has emerged as first-line sur- other identified genetic and central
gical therapy for dystonia. The globus nervous system diseases.55,56 Case 10-3

Case 10-3
A 51-year-old man presented with a chief complaint of involuntary tongue
protrusion, dysphagia, increased salivation, and impaired speech progressing
over a 1-year period. He had not been exposed to dopamine-blocking
medication prior to symptom onset. On physical examination he had severe
dysarthria and hypometric saccades on vertical gaze. Motor examination
was pertinent for involuntary tongue protrusion with improvement when a
tongue depressor rested on the tongue, diffuse rigidity, dystonic posturing
of all four limbs, and truncal tilt to the right with retropulsion when walking.
Imaging was normal, as was video laryngoscopy. There was no significant
clinical response to trihexyphenidyl or clonazepam. Genetic testing revealed
a mutated allele (T) at DSC3 and a mutated allele (G) at DSC12 on his X
chromosome. This confirmed a diagnosis of X-linked dystonia-parkinsonism
(DYT3). The patient received bilateral globus pallidus internus (GPi) deep
brain stimulation (DBS). Early improvements were seen in the limbs. Two
years after surgery the patient had no involuntary tongue protrusion,
improved dysarthria and dysphagia, no limb involvement, and improved
truncal movements. Supplemental Digital Content 10-3, links.lww.com/
CONT/A200 illustrates this case pre- and post-DBS.
Comment. X-linked dystonia-parkinsonism is a condition that typically
presents in adulthood. The disease is also called Lubag disease from first
descriptions of the disease in subjects from the island of Panay in the
Philippines. Patients may have a focal onset, but dystonia typically generalizes.
This case demonstrates clinical improvement in a patient with DYT3 dystonia.
A recent case report and review of the literature found significant clinical
improvement in other patients with DYT3 dystonia.57 Benefits of DBS for
DYT1 are well reported. However, growing evidence suggests that other
genetic forms of dystonia may improve with surgical intervention.

1240 www.ContinuumJournal.com August 2016

Copyright © American Academy of Neurology. Unauthorized reproduction of this article is prohibited.


KEY POINT
illustrates the benefits of DBS in a pa- by the patient on testing. Patients with h Children with dystonia
tient with X-linked dystonia-parkinsonism. dystonia often have a delayed response who receive globus
MRI and intraoperative single-unit mi- to stimulation with improvement be- pallidus internus
croelectrode as well as local field po- ginning in weeks to months. Initial stimulation may respond
tential recordings are used to ensure programming is often modified every to low frequencies
proper placement. Placement of neu- few weeks during initial sessions, and (60 Hz to 80 Hz) while
rostimulators in the chest and connec- intervals between later appointments higher frequencies
tion of the extension cables to the are lengthened to every few months. (130 Hz to 180 Hz) are
neurostimulators are usually done in a Mood and cognition do not appear to often used in adults.
separate surgery under sedation. Patients worsen postsurgically.61,62 Speech does
may have two single lead channels placed not predictably worsen either.63
bilaterally or one neurostimulator with
two channels leading to either side. CONCLUSION
The surgical risks for all patients Dystonia is a hyperkinetic movement
who undergo DBS, including those with disorder characterized by patterned sus-
Parkinson disease and essential tremor, tained or intermittent muscle contrac-
are reported as a 0.4% risk of death tions that can cause abnormal postures
in 30 days, a 1% risk of morbidity, and a or tremorlike movements. It is diag-
3% risk of intracranial bleed (many of nosed clinically and characterized under
which are asymptomatic). However, it is a recently revised classification system
hypothesized that those with dystonia designed to reflect the growing knowl-
have fewer complications as they typi- edge of genetic etiologies. The approach
cally undergo surgery at a younger age.53 to treatment depends on the extent of
It is estimated that hardware malfunc- muscle involvement. In addition to phar-
tion or infections may occur in as many macologic therapy, surgical intervention
as 10% of patients over the course of a is a treatment option for an increasing
lifetime.58 A 2014 randomized, sham- group of affected patients. Growing un-
controlled trial of pallidal stimulation in derstanding of the etiology and patho-
patients with medication refractory physiology of dystonia will allow more
cervical dystonia reported adverse events targeted therapies in upcoming years.
in 34% (n = 11) of those receiving
neurostimulation. Most events resolved VIDEO LEGENDS
without sequelae, and five events were Supplemental Digital
serious and related to the placement of Content 10-1
the device or the device itself.59 Dopa-responsive dystonia pre-
Patients’ stimulators are programmed treatment and posttreatment with
in the office approximately 2 weeks levodopa. Video shows an 11-year-old
after the final stage of surgery. Children girl who presented with generalized
with dystonia who receive GPi stimu- dystonia that started with clumsy gait
lation may respond to low frequencies and toe walking, which worsened in the
(60 Hz to 80 Hz) while higher frequen- evening. On examination, dystonic pos-
cies (130 Hz to 180 Hz) are often used turing in both arms, left greater than
in adults. Low frequency has an addi- right, and legs was present. When
tional benefit of preserving battery life. walking, dystonic posturing was noted
The pulse width used in dystonia ranges in all limbs. She was prescribed
between 60 2s to 210 2s.58,60 The initial carbidopa/levodopa 25 mg/100 mg and
voltage is determined in the office and slowly increased to 500 mg/d of levodopa
is chosen based on physician prefer- in divided doses and became asymptom-
ence and the side effects experienced atic after treatment.
Continuum (Minneap Minn) 2016;22(4):1227–1245 www.ContinuumJournal.com 1241

Copyright © American Academy of Neurology. Unauthorized reproduction of this article is prohibited.


Dystonia

links.lww.com/CONT/A198 bilateral globus pallidus internus deep


B 2016 American Academy of Neurology. brain stimulation. Two years after
surgery the patient had no involuntary
Supplemental Digital tongue protrusion, improved dysar-
Content 10-2 thria and dysphagia, no limb involve-
Cervical dystonia misdiagnosed as ment, and improved truncal
essential tremor. This video shows a movements as demonstrated in this
65-year-old woman who presented with video showing the examination
a 35-year history of head tremor that pretreatment and posttreatment.
was previously diagnosed as essential
tremor but was unresponsive to links.lww.com/CONT/A200
medication for essential tremor. She B 2016 American Academy of Neurology.
did not experience hand tremor, but
experienced neck tightness and en- USEFUL WEBSITES
dorsed a ‘‘sensory trick’’ where tremor
American Dystonia Society. The
improved when she lightly touched her
American Dystonia society advocates
cheek. On examination, hypertrophy was
for veterans, Hispanics, and young
present in both sternocleidomastoid
adults with dystonia and promotes gen-
muscles. She had a jerky, multidirec-
eral public and governmental aware-
tional neck tremor, notable for a null
ness of the condition.
point with head turn to the left. Symp-
toms improved with botulinum toxin www.dystoniasociety.org
injections, performed with EMG guid- Bachmann-Strauss Dystonia &
ance, in the bilateral sternocleidomastoid Parkinson Foundation, Inc. The
muscles and bilateral splenius capitis Bachmann-Strauss Dystonia & Par-
muscles. Further improvement was kinson Foundation, Inc is a nonprofit
later seen adding injections into the organization dedicated to finding bet-
right semispinalis capitis and right ter treatments and cures for dystonia
longissimus muscles. and Parkinson disease. The organiza-
tion has a collaborative research alliance
links.lww.com/CONT/A199
with the Michael J. Fox Foundation for
B 2016 American Academy of Neurology. Parkinson’s Research.
Supplemental Digital www.dystonia-parkinsons.org
Content 10-3 Benign Essential Blepharospasm
X-linked dystonia-parkinsonism Research Foundation. The Benign
(Lubag disease) improved after Essential Blepharospasm Research
globus pallidus internus deep brain Foundation searches for the cause and
stimulation. This video shows a 51- cure of benign essential blepharo-
year-old man with X-linked dystonia- spasm and other related disorders.
parkinsonism who presented with 1 www.blepharospasm.org
year of progressive involuntary tongue
Dystonia Medical Research Foun-
protrusion, dysphagia, increased saliva-
dation. The Dystonia Medical Research
tion, and impaired speech. Motor exam-
Foundation is dedicated to finding
ination was pertinent for involuntary
a cure for dystonia while providing
tongue protrusion, diffuse rigidity, dys-
support for individuals affected by
tonic posturing of all four limbs, and
the condition.
truncal tilt to the right with retropulsion
when walking. The patient received www.dystonia-foundation.org

1242 www.ContinuumJournal.com August 2016

Copyright © American Academy of Neurology. Unauthorized reproduction of this article is prohibited.


National Center for Biotechnology 5. Dufke C, Hauser AK, Sturm M, et al.
Mutations in CIZ1 are not a major cause for
Information GeneReviews: Dystonia. dystonia in Germany. Mov Disord 2015;30(5):
GeneReviews are expertly written dis- 740Y743. doi:10.1002/mds.26198.
ease descriptions and information 6. Ma L, Chen R, Wang L, et al. No mutations in
related to the diagnosis, management, CIZ1 in twelve adult-onset primary cervical
and genetic counseling of patients with dystonia families. Mov Disord 2013;28(13):
1899Y1901. doi:10.1002/mds.25542.
these conditions. This listing specifically
refers to dystonia. 7. Charlesworth G, Plagnol V, Holmström KM,
et al. Mutations in ANO3 cause dominant
www.ncbi.nlm.nih.gov/books/ craniocervical dystonia: ion channel implicated
NBK1155 in pathogenesis. Am J Hum Genet 2012;91(6):
1041Y1050. doi:10.1016/j.ajhg.2012.10.024.
National Institute of Neurological 8. Fuchs T, Saunders-Pullman R, Masuho I, et al.
Disorders and Stroke: Dystonia Fact Mutations in GNAL cause primary torsion
Sheet. The National Institute of Neuro- dystonia. Nat Genet 2013;45(1):88Y92.
doi:10.1038/ng.2496.
logical Disorders and Stroke provides
9. Balint B, Bhatia KP. Isolated and combined
information on the symptoms, classifi- dystonia syndromesVan update on new
cation, and treatment of dystonia. genes and their phenotypes. Eur J Neurol
2015;22(4):610Y617. doi:10.1111/ene.12650.
www.ninds.nih.gov/disorders/
dystonias/detail_dystonias.htm 10. Wijemanne S, Jankovic J. Dopa-responsive
dystoniaVclinical and genetic heterogeneity.
National Spasmodic Torticollis Asso- Nat Rev Neurol 2015;11(7):414Y424.
ciation. The National Spasmodic Torti- doi:10.1038/nrneurol.2015.86.

collis Association supports individuals 11. Sweney MT, Newcomb TM, Swoboda KJ. The
expanding spectrum of neurological
and families affected by the condition, phenotypes in children with ATP1A3 mutations,
promotes awareness, and advances Alternating Hemiplegia of Childhood,
research for treatment. Rapid-onset Dystonia-Parkinsonism, CAPOS
and beyond. Pediatr Neurol 2015;52(1):56Y64.
www.torticollis.org doi:10.1016/j.pediatrneurol.2014.09.015.
Spasmodic Torticollis Dystonia. The 12. Peall KJ, Dijk JM, Saunders-Pullman R, et al.
Psychiatric disorders, myoclonus dystonia
Spasmodic Torticollis Dystonia organiza-
and SGCE: an international study. Ann Clin
tion provides information and research Transl Neuro 2016;3(1):4Y11. doi:10.1002/
on the condition as well as doctor and acn3.263.
patient opinions on various treatments. 13. Groen JL, Andrade A, Ritz K, et al. CACNA1B
mutation is linked to unique myoclonus-dystonia
www.spasmodictorticollis.org syndrome. Hum Mol Genet 2015;24(4):
987Y993. doi:10.1093/hmg/ddu513.
REFERENCES 14. Wang JL, Cao L, Li XH, et al. Identification of
1. Albanese A, Bhatia K, Bressman SB, et al. PRRT2 as the causative gene of paroxysmal
Phenomenology and classification of dystonia: kinesigenic dyskinesias. Brain 2011;134(pt 12):
a consensus update. Mov Disord 2013;28(7): 3493Y3501. doi:10.1093/brain/awr289.
863Y873. doi:10.1002/mds.25475.
15. Chen GH. Five cases of paroxysmal kinesigenic
2. Deuschl G. Dystonic tremor. Rev Neurol dyskinesia by genetic diagnosis. Exp Ther Med
(Paris) 2003;159(10 pt 1):900Y905. 2015;9(3):909Y912. doi:10.3892/etm.2014.2155.
3. Greene PE, Bressman S. Exteroceptive and 16. Suls A, Dedeken P, Goffin K, et al. Paroxysmal
interoceptive stimuli in dystonia. Mov Disord exercise-induced dyskinesia and epilepsy is
1998;13(3):549Y551. doi:10.1002/mds. due to mutations in SLC2A1, encoding the
870130329. glucose transporter GLUT1. Brain 2008;131(pt 7):
1831Y1844. doi:10.1093/brain/awn113.
4. Liang CC, Tanabe LM, Jou S, et al. TorsinA
hypofunction causes abnormal twisting 17. Charlesworth G, Bhatia KP, Wood NW. The
movements and sensorimotor circuit genetics of dystonia: new twists in an old
neurodegeneration. J Clin Invest 2014;124(7): tale. Brain 2013;136(pt 7):2017Y2037.
3080Y3092. doi:10.1172/JCI72830. doi:10.1093/brain/awt138.

Continuum (Minneap Minn) 2016;22(4):1227–1245 www.ContinuumJournal.com 1243

Copyright © American Academy of Neurology. Unauthorized reproduction of this article is prohibited.


Dystonia

18. Peckham EL, Lopez G, Shamim EA, et al. 31. Pozin I, Bdolah-Abram T, Ben-Pazi H. Levodopa
Clinical features of patients with blepharospasm: does not improve function in individuals with
a report of 240 patients. Eur J Neurol dystonic cerebral palsy. J Child Neurol 2014;
2011;18(3):382Y386. doi:10.1111/j.1468-1331. 29(4):534Y537. doi:10.1177/0883073812473645.
2010.03161.x.
32. Kenney C, Hunter C, Jankovic J. Long-term
19. Defazio G, Hallett M, Jinnah HA, Berardelli A. tolerability of tetrabenazine in the treatment
Development and validation of a clinical of hyperkinetic movement disorders. Mov
guideline for diagnosing blepharospasm. Disord 2007;22(2):193Y197. doi:10.1002/
Neurology 2013;81(3):236Y240. doi:10.1212/ mds.21222.
WNL.0b013e31829bfdf6.
33. Luciano AY, Jinnah HA, Pfeiffer RF, et al.
20. Payne S, Tisch S, Cole I, et al. The clinical Treatment of myoclonus-dystonia syndrome
spectrum of laryngeal dystonia includes with tetrabenazine. Parkinsonism Relat
dystonic cough: observations of a large Disord 2014;20(12):1423Y1426. doi:10.1016/
series. Mov Disord 2014;29(6):729Y735. j.parkreldis.2014.09.029.
doi:10.1002/mds.25865.
34. Illowsky Karp BI, Goldstein SR, Chen R, et al.
21. Grillone GA, Chan T. Laryngeal dystonia. An open trial of clozapine for dystonia. Mov
Otolaryngol Clin North Am 2006;39(1): Disord 1999;14(4):652Y657. doi:10.1002/
87Y100. doi:10.1007/s00415-014-7586-2. 1531-8257(199907)14:4G652::AID-
22. Comella C, Bhatia K. An international survey MDS101593.0.CO;2-G.
of patients with cervical dystonia. J Neurol 35. Gourzis P, Skokou M, Soubasi E, et al.
2015;262(4):837Y848. doi:10.1007/ Treatment of tardive dystonia induced by
s00415-014-7586-2. antipsychotics, old and new. Clin
23. Charles PD, Adler CH, Stacy M, et al. Cervical Neuropharmacol 2015;38(4):121Y126.
doi:10.1097/WNF.0000000000000086.
dystonia and pain: characteristics and
treatment patterns from CD PROBE (Cervical 36. Burke RE, Fahn S, Marsden CD. Torsion dystonia:
Dystonia Patient Registry for Observation a double-blind, prospective trial of
of OnabotulinumtoxinA Efficacy). J Neurol high-dosage trihexyphenidyl. Neurology 1986;
2014;261(7):1309Y1319. doi:10.1007/ 36(2):160Y164. doi:10. 1212/WNL.36.2.160.
s00415-014-7343-6.
37. Fasano A, Bove F, Lang AE. The treatment of
24. Louis ED, Dogu O. Isolated head tremor: part dystonic tremor: a systematic review.
of the clinical spectrum of essential tremor? J Neurol Neurosurg Psychiatry 2014;85(7):
Data from population-based and clinic-based 759Y769. doi:10.1136/jnnp-2013-305532.
case samples. Mov Disord 2009;24(15):
38. Kinugawa K, Vidailhet M, Clot F, et al.
2281Y2285. doi:10.1002/mds.22777.
Myoclonus-dystonia: an update. Mov Disord
25. Schneider SA, Edwards MJ, Mir P, et al. 2009;24(4):479Y489. doi:10.1002/mds.22425.
Patients with adult-onset dystonic tremor 39. Strzelczyk A, Bürk K, Oertel WH. Treatment
resembling parkinsonian tremor have scans of paroxysmal dyskinesias. Expert Opin
without evidence of dopaminergic deficit Pharmacother 2011;12(1):63Y72.
(SWEDDs). Mov Disord 2007;22(15): doi:10.1517/14656566.2010.513971.
2210Y2215. doi:10.1002/mds.21685.
40. Greene P, Shale H, Fahn S. Experience with
26. Hawthorne JM, Caley CF. Extrapyramidal high dosages of anticholinergic and other
reactions associated with serotonergic drugs in the treatment of torsion dystonia.
antidepressants. Ann Pharmacother Adv Neurol 1988;50:547Y556.
2015;49(10):1136Y1152. doi:10.1177/
1060028015594812. 41. Albright AL, Ferson SS. Intraventricular baclofen
for dystonia: techniques and outcomes.
27. Mehta SH, Morgan JC, Sethi KD. Drug-induced Clinical article. J Neurosurg Pediatr 2009;3(1):
movement disorders. Neurol Clin 2015;33(1): 11Y14. doi:10.3171/2008.10.PEDS0847.
153Y174. doi:10.1016/j.ncl.2014.09.011.
42. Frucht SJ, Bordelon Y, Houghton WH,
28. Jinnah HA, Factor SA. Diagnosis and treatment Reardan D. A pilot tolerability and efficacy
of dystonia. Neurol Clin 2015;33(1):77Y100. trial of sodium oxybate in ethanol-responsive
doi:10.1016/j.ncl.2014.09.002. movement disorders. Mov Disord 2005;20(10):
29. Thenganatt MA, Jankovic J. Treatment of 1330Y1337. doi:10.1002/mds.20605.
dystonia. Neurotherapeutics 2014;11(1):
43. Simonyan K, Frucht SJ. Long-term effect of
139Y152. doi:10.1007/s13311-013-0231-4.
sodium oxybate (XyremA) in spasmodic
30. Hensman DJ, Bain PG. Levodopa can worsen dysphonia with vocal tremor. Tremor Other
tremor associated with dystonia. Mov Disord Hyperkinet Mov (N Y) 2013;3. doi:10.7916/
2006;21(10):1778Y1780. doi:10.1002/mds.21043. D8CJ8C5S

1244 www.ContinuumJournal.com August 2016

Copyright © American Academy of Neurology. Unauthorized reproduction of this article is prohibited.


44. Pfeiffer RF. Wilson disease. Continuum 54. Brüggemann N, Kühn A, Schneider SA, et al.
(Minneap Minn) 2016;22(4 Movement Short- and long-term outcome of chronic
Disorders):1246Y1261. pallidal neurostimulation in monogenic
isolated dystonia. Neurology 2015;84(9):
45. Walker TJ, Dayan SH. Comparison and
895Y903. doi:10.1212/WNL.0000000000001312.
overview of currently available neurotoxins.
J Clin Aesthet Dermatol 2014;7(2):31Y39. 55. Roze E, Vidailhet M, Hubsch C, et al. Pallidal
stimulation for myoclonus-dystonia: ten
46. Simpson DM, Hallet M, Ashman EJ, et al.
years’ outcome in two patients. Mov Disord
Practice guideline update summary:
2015;30(6):871Y872. doi:10.1002/mds.26215.
botulinum neurotoxin for the treatment of
blepharospasm, cervical dystonia, adult 56. Nam TM, Cho KR, Youn J, et al. Deep brain
spasticity, and headache. Neurology stimulation in a dentatorubral-pallidoluyisian
2016;86(10):1818Y1826. doi:10.1212/wnl. atrophy patient with myoclonic dystonia.
0000000000002560. J Clin Neurosci 2015;22(12):1976Y1978.
doi:10.1016/j.jocn.2015.04.010.
47. Simpson DM, Blitzer A, Brashear A, et al;
Assessment: botulinum neurotoxin for the 57. Patel AJ, Sarwar AI, Jankovic J, Viswanathan A.
treatment of movement disorders (an Bilateral pallidal deep brain stimulation for
evidence-based review): report of the X-linked dystonia-parkinsonism. World
Therapeutics and Technology Assessment Neurosurg 2014;82(1-2):241.e1Y241.e4.
Subcommittee of the American Academy of doi:10.1016/j.wneu.2013.09.039.
Neurology. Neurology 2008;70(19):
58. Kupsch A, Tagliati M, Vidailhet M, et al.
1699Y1706. doi:10.12/01.wnl.0000311389.
Early postoperative management of DBS in
26145.95.48.
dystonia: programming, response to
48. Hallett M, et al. Evidence-based review and stimulation, adverse events, medication
assessment of botulinum neurotoxin for the changes, evaluations, and troubleshooting.
treatment of movement disorders. Toxicon Mov Disord 2011;26(suppl 1):S37YS53.
2013;67:94Y114. doi:10.1016/j.toxicon. doi:10.1002/mds.23624.
2012.12.004.
59. Volkmann J, Mueller J, Deuschl G, et al.
49. Ramirez-Castaneda J, Jankovic J. Long-term Pallidal neurostimulation in patients with
efficacy and safety of botulinum toxin medication-refractory cervical dystonia: a
injections in dystonia. Toxins (Basel) 2013;5(2): randomised, sham-controlled trial. Lancet
249Y266. doi:10.3390/toxins5020249. Neurol 2014;13(9):875Y884. doi:10.1016/
S1474-4422(14)70143-7.
50. Grigoriu AI, Dinomais M, Rémy-Néris O,
Brochard S. Impact of injection-guiding 60. Kim JP, Chang WS, Park YS, Chang JW.
techniques on the effectiveness of botulinum Effects of relative low-frequency bilateral
toxin for the treatment of focal spasticity and globus pallidus internus stimulation for
dystonia: a systematic review. Arch Phys treatment of cervical dystonia. Stereotact
Med Rehabil 2015;96(11):2067Y2078.e1. Funct Neurosurg 2012;90(1):30Y36.
doi:10.1016/j.apmr.2015.05.002. doi:10.1159/000333839.
51. Delnooz CC, Veugen LC, Pasman JW, et al. 61. Owen T, Gimeno H, Selway R, Lin JP. Cognitive
The clinical utility of botulinum toxin function in children with primary dystonia
injections targeted at the motor endplate before and after deep brain stimulation.
zone in cervical dystonia. Eur J Neurol Eur J Paediatr Neurol 2015;19(1):48Y55.
2014;21(12):1486Ye98. doi:10.1111/ene.12517. doi:10.1016/j.ejpn.2014.09.004.
52. Kupsch A, Benecke R, Müller J, et al. 62. Meoni S, Zurowski M, Lozano AM, et al.
Pallidal deep-brain stimulation in primary Long-term neuropsychiatric outcomes after
generalized or segmental dystonia. N Engl J pallidal stimulation in primary and secondary
Med 2006;355(19):1978Y1990. doi:10.1056/ dystonia. Neurology 2015;85(5):433Y440.
NEJMoa063618. doi:10.1212/WNL.0000000000001811.
53. Fox MD, Alterman RL. Brain stimulation for 63. Bressman SB, de Leon D, Brin MF, et al.
torsion dystonia. JAMA Neurol 2015;72(6): Inheritance of idiopathic torsion dystonia among
713Y719. doi:10.1001/jamaneurol.2015.51. Ashkenazi Jews. Adv Neurol 1988;50:45Y56.

Continuum (Minneap Minn) 2016;22(4):1227–1245 www.ContinuumJournal.com 1245

Copyright © American Academy of Neurology. Unauthorized reproduction of this article is prohibited.


Review Article

Wilson Disease
Address correspondence to
Dr Ronald F. Pfeiffer,
Department of Neurology,
Oregon Health & Science
University, 3181 SW Sam Ronald F. Pfeiffer, MD, FAAN
Jackson Park Rd, Portland,
OR 97239-3098,
pfeiffro@ohsu.edu.
Relationship Disclosure: ABSTRACT
Dr Pfeiffer received personal Purpose of Review: This article reviews the clinical features of Wilson disease,
compensation for serving on
the advisory boards of focusing on the neurologic and psychiatric abnormalities, and addresses the diagnostic
Acadia Pharmaceuticals Inc, workup and treatment approaches to managing the disease.
Lundbeck, and Pfizer Inc, Recent Findings: The list of known mutations causing Wilson disease continues to
as a consultant for Clintara
LLC, for serving as co-editor- grow, but advances in genetic testing may soon make it feasible to routinely perform
in-chief of Parkinsonism & genetic testing on individuals suspected of having Wilson disease.
Related Disorders, and for Summary: Wilson disease is a rare genetic disorder with protean manifestations that
serving on the speakers
bureau of Teva should be considered in the differential diagnosis of any individual presenting with
Pharmaceutical Industries Ltd. unexplained neurologic, psychiatric, or hepatic dysfunction. Appropriate diagnostic
Dr Pfeiffer served on the testing should be expeditiously performed and treatment promptly initiated and
editorial boards of the
International Journal of maintained since failure to diagnose and treat Wilson disease will result in progressive
Brain Science and the Journal and ultimately irreversible damage to the neurologic and other systems.
of Parkinson’s Disease and
receives publishing royalties
from CRC Press, Taylor & Continuum (Minneap Minn) 2016;22(4):1246–1261.
Francis Group, and Humana
Press/Springer. Dr Pfeiffer
served as principal
investigator of a study for the INTRODUCTION the incorporation of copper into
Michael J. Fox Foundation, the
Parkinson Study Group, and In 1912 S. A. Kinnear Wilson published apoceruloplasmin to form ceruloplas-
Rhythm Pharmaceuticals, Inc. a detailed description of the illness that min. In Wilson disease, the defective
Unlabeled Use of
Products/Investigational now bears his name.1 Over the years, ATP7B protein cannot perform this
Use Disclosure: many others have contributed to the function, resulting in abnormally low
Dr Pfeiffer discusses the levels of ceruloplasmin. When hepatic
unlabeled/investigational use
recognition and knowledge of Wilson
of zinc for the treatment of disease, leading to our current under- copper levels are elevated, ATP7B is
Wilson disease. standing that the fundamental abnor- redistributed to cytoplasmic vesicles
* 2016 American Academy where it facilitates copper transport into
of Neurology.
mality of Wilson disease is the impaired
biliary excretion of copper. This results the bile canaliculi.2 This function also
in a gradual accumulation of copper is impaired in Wilson disease, which
and consequent tissue damage, initially leads to a gradual increase in copper
within the liver but eventually also in levels within the liver, and when the
other organs, including the brain. Al- capacity of the liver to store the
though a genetic basis for Wilson relentlessly accumulating copper is
disease was documented within 10 exceeded, the copper spills out of the
years of Wilson’s initial publication and liver to inflict damage in other organs
subsequently identified as an autosomal as well. ATP7B dysfunction and hepatic
recessive disorder, it was not until 1993 dysfunction may also lead to accumu-
that the responsible gene, now known lation of other metals, such as iron and
as ATP7B, was identified on chromo- manganese, in the brains of individuals
Supplemental digital content:
Videos accompanying this ar- some 13. with Wilson disease, perhaps contrib-
ticle are cited in the text as The protein (also named ATP7B) uting to symptoms.3
Supplemental Digital Content.
Videos may be accessed by encoded by the gene typically resides Over 520 Wilson disease mutations
clicking on links provided in the within the trans-Golgi network in have been identified, and most in-
HTML, PDF, and app versions
of this article; the URLs are
hepatocytes and transports copper dividuals with Wilson disease are com-
provided in the print version. Video across organelle membranes, enabling pound heterozygotes. 4 Missense
legends begin on page 1259.

1246 www.ContinuumJournal.com August 2016

Copyright © American Academy of Neurology. Unauthorized reproduction of this article is prohibited.


KEY POINTS
mutations are the most frequently presentations (14.4 months versus h Wilson disease is caused
identified mutation type; deletions, 44.4 months).13 The reported onset by a mutation in the
insertions, nonsense, and splice site of hepatic symptoms ranges from the ATP7B gene on
mutations also occur. 5 Genotype- asymptomatic elevation of liver en- chromosome 13.
phenotype correlation is not clearly zymes in a child 9 months old to the The ATP7B protein is
evident in Wilson disease, and in- appearance of Wilson disease symp- involved in the
dividuals with identical mutations, toms in the form of hepatic dysfunc- incorporation of copper
even monozygotic twins, may differ tion as late as age 74.14,15 into ceruloplasmin and
widely in both age of symptom onset The most frequent mode of hepatic the transport of copper
and clinical manifestations.5,6 presentation is slowly progressive he- into the bile canaliculi.
Recent information suggests a prev- patic failure with cirrhosis, ascites, h Hepatic symptoms are
alence rate for Wilson disease in the esophageal varices, and splenomegaly. the initial clinical
United Kingdom of approximately 1 in Acute hepatitis, similar in many as- manifestation of Wilson
7000 individuals, which is much higher pects to viral hepatitis, also may be disease in 40% to 50%
of patients, with an
than the usually quoted figure of 1 in the presenting condition; the con-
appearance between
30,000 individuals.7 An incidence rate comitant presence of hemolytic ane-
11.4 to 15.5 years of
of 1 per 55,000 live births in the United mia may be a clue to the diagnosis of age, on average.
States has been calculated.8 Wilson disease in this setting.12 A
more chronic presentation similar to
CLINICAL FEATURES autoimmune hepatitis is yet another
Several scoring systems and scales mode of presentation of hepatic Wilson
have been devised and tested to assist disease and may even show an initial
in the evaluation of persons with response to steroids. Acute fulminant
suspected Wilson disease, including hepatic failure with coagulopathy, en-
one devised and published by a group cephalopathy, cerebral edema, renal
of experts following the 8th Interna- failure, and metabolic derangements,
tional Meeting on Wilson Disease and is the least frequent mode of hepatic
Menkes Disease.9 A Global Assess- Wilson disease presentation, occur-
ment Scale for Wilson Disease specif- ring in approximately 5% of individ-
ically intended for use in routine uals, predominantly young females
clinical practice to allow more objec- (Case 11-1).16 The pattern of liver
tive assessment of patients with Wilson enzyme abnormalities in the setting
disease has also been devised, as has of fulminant hepatic failure may pro-
the Unified Wilson’s Disease Rating vide clues to the diagnosis of Wilson
Scale.10,11 disease. The combination of an alka-
line phosphatase to total bilirubin
Hepatic Manifestations ratio of less than 4 and an aspartate
Hepatic symptoms (Table 11-1) are the aminotransferase to alanine amino-
initial clinical manifestation of Wilson transferase ratio of greater than 2.2
disease in 40% to 50% of patients and in this setting is highly suggestive of
tend to appear at an earlier age Wilson disease, especially if hemoglo-
(average of 11.4 to 15.5 years of age) bin is also reduced.16
than neurologic or psychiatric presen-
tations.12,13 Individuals presenting with Neurologic Manifestations
hepatic symptoms also tend to be Neurologic dysfunction (Table 11-1)
diagnosed more quickly; in one study, is the initial clinical manifestation of
persons with hepatic presentations Wilson disease in 40% to 60% of
were diagnosed, on average, 30 months individuals, with a typical symptom
earlier than those with neuropsychiatric onset at approximately 20 years of
Continuum (Minneap Minn) 2016;22(4):1246–1261 www.ContinuumJournal.com 1247

Copyright © American Academy of Neurology. Unauthorized reproduction of this article is prohibited.


Wilson Disease

KEY POINTS
h Neurologic dysfunction TABLE 11-1 Multisystem TABLE 11-1 Continued
is the initial clinical Involvement in
manifestation of Wilson Wilson Disease b Musculoskeletal
disease in 40% to 60%
Osteoporosis
of patients, with a b Hepatic
typical appearance at Slowly progressive hepatic Osteomalacia
approximately 20 years failure with cirrhosis Osteoarthritis
of age.
Mimicking acute viral hepatitis Spontaneous fractures
h Basal gangliaYbased
abnormalities, especially Mimicking autoimmune b Myocardial
chronic active hepatitis
tremor and dystonia, Cardiac hypertrophy
are the most common Acute fulminant hepatic failure
neurologic features of Cardiomyopathy
b Neurologic
Wilson disease. Electrocardiographic
Basal ganglia dysfunction abnormalities
Tremor b Renal
Dystonia Renal impairment
Chorea Nephrocalcinosis
Athetosis Hematuria
Parkinsonism Proteinuria
Myoclonus b Dermatologic
Cerebellar dysfunction Hyperpigmentation of
Combined dysfunction the legs

Dysarthria Acanthosis nigricans

Dysphagia Bluish discoloration of the


lunulae of the nails
Other features
Autonomic dysfunction
Seizures
age but with a broad reported range
that extends from as early as 6 to as
Headaches late as 72 years of age.12,13,17
b Psychiatric Basal gangliaYbased movement ab-
Personality changes normalities are hallmarks of Wilson
disease. Tremor is the initial neuro-
Depression
logic feature in approximately 50% of
Acute psychosis individuals and may be proximal or
Cognitive impairment distal and evident at rest or with
movement (Supplemental Digital
b Ophthalmic
Content 11-1, links.lww.com/CONT/
Kayser-Fleischer rings A201). The classic presentation is a
Sunflower cataracts proximal, coarse tremor involving the
arms that has been likened to a bird
b Hematologic
beating its wings. Dystonia also is very
Coombs-negative hemolytic
anemia common in neurologic Wilson disease
and may be present in up to 69% of pa-
Thrombocytopenia
tients (Supplemental Digital Content
11-2, links.lww.com/CONT/A202). 18

1248 www.ContinuumJournal.com August 2016

Copyright © American Academy of Neurology. Unauthorized reproduction of this article is prohibited.


KEY POINT

Case 11-1 h Psychiatric features are


evident at or prior to
A 17-year-old girl presented to the emergency department with a
diagnosis in 30% to
1-week history of nausea and vomiting. On examination she was
40% of persons with
jaundiced, had a tender abdomen, and was disoriented and obtunded.
Wilson disease.
On funduscopic examination her disc margins were indistinct. She
was hypotensive. Laboratory studies demonstrated hyponatremia,
hypokalemia, hypophosphatemia, and an international normalized ratio
(INR) of 3.0. Liver biopsy could not be performed safely because of the
coagulopathy. The alkaline phosphatase to total bilirubin ratio was
3.3 and the aspartate aminotransferase to alanine aminotransferase ratio
was 2.9. She was severely anemic, but was Coombs negative. A serum
copper level was markedly elevated at 250 mcg/L.
Comment. The diagnosis of Wilson disease in the setting of acute
fulminant hepatic failure can be very difficult. Liver biopsy may not be
feasible because of coagulopathy; ceruloplasmin levels become unreliable,
and renal failure may hamper 24-hour urine collection. In this setting,
the presence of a Coombs-negative hemolytic anemia, an alkaline
phosphatase to total bilirubin ratio of less than 4, and an aspartate
aminotransferase to alanine aminotransferase ratio of greater than
2.2 may suggest the diagnosis of Wilson disease. In this situation, serum
copper will also be markedly elevated because of massive release of
copper from the liver.

Risus sardonicus, the fixed facial gri- Wilson disease, but upper motor neu-
mace or smile, is produced by dysto- ron signs, lower motor neuron signs,
nia of the facial muscles. Parkinsonism sensory loss, and sphincter dysfunction
is evident in approximately 40% of all are unusual.19,21
persons with Wilson disease.19 Chorea,
athetosis, and myoclonus are less com- Psychiatric Manifestations
mon, but do occur.18,20 Most reports indicate that psychiatric
Cerebellar dysfunction develops in symptoms are evident at or prior to
approximately 30% of individuals with diagnosis in approximately 30% to
Wilson disease.20 Dysarthria may be 40% of individuals with Wilson
cerebellar or hypokinetic in character disease.22 Psychiatric dysfunction in
and eventually develops in the vast Wilson disease may range from subtle
majority of individuals with Wilson personality changes to frank psychosis
disease. Dysarthria may progress to the (Table 11-1). Depression is common,
point of complete anarthria. Dysphagia and suicidal behavior has been reported.
also may develop during the course of Acute psychosis may be the present-
Wilson disease and may be present in ing feature of Wilson disease and is
up to 50% of individuals at the time of characterized by paranoid ideation, de-
diagnosis.18 Some disturbance of gait is lusional thinking, hallucinations, and
evident in 45% to 75% of patients with even catatonia.23 Psychosis may first
Wilson disease at the time of diagnosis appear during recovery of motor func-
and may range in character from par- tion following initiation of copper
kinsonian to cerebellar.12,18 chelation therapy and in that setting
Other neurologic abnormalities, is attributed to improved motor func-
such as autonomic dysfunction, sei- tion unveiling previously masked psy-
zures, and headaches, may occur in chosis.10 Dementia is uncommon in

Continuum (Minneap Minn) 2016;22(4):1246–1261 www.ContinuumJournal.com 1249

Copyright © American Academy of Neurology. Unauthorized reproduction of this article is prohibited.


Wilson Disease

KEY POINTS
h Kayser-Fleischer rings Wilson disease but may develop in ent in only 2% to 17% of individuals
may not be present in individuals with advanced disease. Mild with untreated Wilson disease. They
persons with Wilson cognitive impairment is present more are produced by deposition of copper
disease who are frequently.24 in the anterior capsule and have a
asymptomatic or sunflower or sunburstlike appearance.
who have only Ophthalmic Manifestations Sunflower cataracts do not interfere
hepatic dysfunction; Although various ophthalmic abnor- with vision and typically require slit-
furthermore, they may malities have been described in pa- lamp examination to be seen.
occur in other tients with Wilson disease, two classic
conditions characterized findings are important to remember Other Manifestations
by hepatic dysfunction.
(Table 11-1). Formed by the deposi- Coombs-negative hemolytic anemia
h Coombs-negative tion of copper within Descemet mem- may be the initial manifestation of
hemolytic anemia in brane in the cornea, Kayser-Fleischer Wilson disease in 10% to 15% of cases.26
combination with liver rings are the best known ophthalmic The combination of unexplained
disease should always
abnormality in Wilson disease. They Coombs-negative hemolytic anemia
prompt evaluation for
usually are gold, brown, or green in and liver disease should always prompt
Wilson disease.
color and almost always are bilateral. evaluation for possible Wilson dis-
Slit-lamp examination often is necessary ease.12 Osteoporosis and bone fractures
to visualize Kayser-Fleischer rings, es- are quite common in Wilson disease.27
pecially in brown-eyed individuals. Be- Joint involvement also is frequent,
cause they first appear in the superior possibly the result of copper and iron
and then the inferior aspect of the deposition within joints producing free
cornea, lifting the eyelid during exami- radicalYinduced synovial and cartilage
nation is important to make sure early damage. Myocardial involvement in
Kayser-Fleischer rings are not over- Wilson disease is not widely recognized,
looked. Kayser-Fleischer rings may not but also is surprisingly common and
yet have formed in persons who are includes cardiac hypertrophy and elec-
presymptomatic or have only hepatic trocardiographic abnormalities. Renal
involvement. In one study, Kayser- involvement may also occur in Wilson
Fleischer rings were evident in 100% disease and is the presenting feature in
of patients with neurologic dysfunction, rare instances. Hyperpigmentation of
but in 86% of persons with just hepatic the legs may occur and be mistaken
involvement, and in only 59% of for Addison disease.
presymptomatic individuals.20 Others
have reported that Kayser-Fleischer DIAGNOSTIC EVALUATION OF
rings are evident at the time of diag- WILSON DISEASE
nosis in only 44% to 62% of patients Accurate and affordable genetic test-
presenting with hepatic dysfunction.25 ing, which would permit diagnosis of
Kayser-Fleischer rings are not patho- Wilson disease by a single simple test,
gnomonic for Wilson disease, and may soon become routinely available,
pigmented corneal rings indistinguish- but until then diagnosis is still depen-
able from Kayser-Fleischer rings have dent upon the judicious and expedi-
been described in a number of condi- tious employment of a constellation of
tions characterized by hepatic dysfunc- diagnostic studies (Table 11-2).6 A
tion, including primary biliary cirrhosis scoring system has been devised to
and others. assist in the diagnosis of Wilson dis-
The sunflower cataract is another ease.9 Wilson disease should be con-
classic, but much less frequent, ocular sidered in the differential diagnosis of
manifestation of Wilson disease, pres- any individual, particularly any young
1250 www.ContinuumJournal.com August 2016

Copyright © American Academy of Neurology. Unauthorized reproduction of this article is prohibited.


KEY POINT
h Wilson disease should
TABLE 11-2 Diagnostic Evaluation of Wilson Disease be considered in the
differential diagnosis of
Presentation and Diagnostic Evaluation Classic Abnormal Findings
any young person with
Hepatic presentation unexplained hepatic,
Slit-lamp examination Kayser-Fleischer ring may be present neurologic, or
psychiatric dysfunction.
Ceruloplasmin Reduced
24-hour urine copper Elevated
Liver biopsy Increased copper
a
Genetic testing ATP7B mutation
Neuropsychiatric presentation
Slit-lamp examination Kayser-Fleischer ring will be present
Ceruloplasmin Reduced
24-hour urine copper Elevated
a
Genetic testing ATP7B mutation
a
Genetic testing will supplant other tests for diagnosis when it becomes routinely available.

person, with unexplained hepatic, are clinically asymptomatic. However,


neurologic (especially with basal in one study, 3.5% of patients diag-
ganglia or cerebellar involvement), or nosed with Wilson disease had hepatic
psychiatric dysfunction. copper levels below 50 mcg/g and in
13% the hepatic copper level was be-
Genetic Testing tween 50 mcg/g and 250 mcg/g.28 Con-
Although the landscape is rapidly versely, elevation of hepatic copper
evolving, genetic testing still remains content may be present in other condi-
impractical in most clinical settings as tions characterized by hepatic dysfunc-
a screening tool for Wilson disease. tion, particularly obstructive liver
However, genetic screening of rela- diseases. Because liver biopsy is not
tives of patients with Wilson disease in without risk, it should not be used as a
whom the responsible mutations have first-line screening procedure in every
been identified should be performed individual suspected of having Wilson
if at all feasible (Case 11-2). disease. In patients with neurologic or
psychiatric dysfunction, liver biopsy is
Liver Biopsy generally unnecessary since other tests
Liver biopsy is currently the single most will provide the diagnosis. It is, however,
sensitive and accurate test for the usually required in individuals presen-
diagnosis of Wilson disease. Hepatic ting with hepatic dysfunction.
copper content will be significantly
elevated (typically greater than Slit-lamp Examination
250 mcg/g dry tissue, compared with Slit-lamp examination by a neuro-
normal values of 15 mcg/g to 55 mcg/g) ophthalmologist or experienced oph-
in the vast majority of individuals thalmologist is a vital part of the
with Wilson disease, even those who diagnostic evaluation for suspected

Continuum (Minneap Minn) 2016;22(4):1246–1261 www.ContinuumJournal.com 1251

Copyright © American Academy of Neurology. Unauthorized reproduction of this article is prohibited.


Wilson Disease

KEY POINTS
h Ceruloplasmin is not an
adequate sole screening
Case 11-2
An 11-year-old girl was brought in for evaluation by her parents following
test for Wilson disease
her 15-year-old brother’s recent diagnosis of Wilson disease. The patient
because it be may
felt completely normal, had not experienced any symptoms of hepatic,
normal or only slightly
neurologic, or psychiatric dysfunction, and her general physical and
low in 5% to 15%
neurologic examinations were completely normal. She had no
of persons with
Kayser-Fleischer rings on ophthalmologic examination. Hepatic function
Wilson disease.
studies, ceruloplasmin, and 24-hour urinary copper levels were all within
h Elevated 24-hour urine normal limits. Formal genetic testing was performed, however, and
copper is not demonstrated the same ATP7B mutation as her brother. She was placed on
pathognomonic for treatment with zinc.
Wilson disease because Comment. Screening laboratory studies may be entirely normal in
it may be elevated in individuals with Wilson disease who have not yet developed symptoms.
obstructive liver diseases Therefore, all asymptomatic siblings of individuals with Wilson disease
and in carriers of should undergo genetic testing and, if test results are positive, should be
Wilson disease. placed on therapy with zinc.

Wilson disease, particularly in persons levels in a person with Wilson disease.


displaying neurologic or psychiatric There also are situations other than
dysfunction, since Kayser-Fleischer Wilson disease in which ceruloplasmin
rings often are not readily visible on levels may be low. For example, up to
routine examination. Individuals with 20% of heterozygotes for Wilson dis-
only hepatic dysfunction may not ease may have ceruloplasmin levels
display Kayser-Fleischer rings because below the normal range.12,25
copper accumulation may not yet
have exceeded the liver’s capacity to Twenty-FourYHour Urinary
store the excess. Presymptomatic pa- Copper Excretion
tients with Wilson disease often do Urinary copper levels in patients with
not have Kayser-Fleischer rings for the symptomatic Wilson disease typically
same reason. are greater than 100 mcg/d. How-
ever, just as with ceruloplasmin levels,
Ceruloplasmin an elevated 24-hour urinary copper
Serum ceruloplasmin determination is level is not necessarily present in
a readily available screening tool for all patients with Wilson disease, nor
suspected Wilson disease and typically is an elevation pathognomonic for
is reduced in individuals with Wilson Wilson disease. In patients with
disease, but it cannot be relied upon as asymptomatic Wilson disease, the
the sole screening study. Ceruloplasmin 24-hour urinary copper excretion may
may be normal or only slightly lower still be normal if the ability of the liver
than normal in 5% to 15% of persons to store the accumulating copper has
with Wilson disease.12 As an acute phase not yet been exceeded. In contrast,
reactant, ceruloplasmin may increase heterozygous Wilson disease carriers
in a variety of circumstances, such as may have modestly elevated urinary
during pregnancy or while receiving copper levels, although typically not
birth control pills, estrogen, or steroids greater than 100 mcg/d.12 Urinary cop-
and also may become elevated during per levels may become elevated in
infections or in various inflammatory primary biliary cirrhosis and other ob-
processes,12 resulting in transiently structive liver diseases. Despite these
normal or near-normal ceruloplasmin limitations, a 24-hour urinary copper

1252 www.ContinuumJournal.com August 2016

Copyright © American Academy of Neurology. Unauthorized reproduction of this article is prohibited.


determination is probably the single often in the basal ganglia, but fre-
best screening test for Wilson disease quently also in the brainstem and
in symptomatic patients. thalamus.30 Several distinctive neuro-
imaging abnormalities have been de-
Serum Copper scribed in Wilson disease. The face of
Routine serum copper level determi- the giant panda sign in the midbrain,
nation is not particularly helpful in the the face of the miniature panda sign
evaluation of suspected Wilson dis- in the pons, and the bright claustrum
ease because it measures total serum sign have all been described, but are
copper, the vast majority of which is present in a small minority of patients
bound to ceruloplasmin and is low and, thus, of limited value.
simply because of the marked reduc-
tion in ceruloplasmin. Other Testing Procedures
Measurement of the incorporation of
Free (Nonceruloplasmin-Bound) radioactive copper (64Cu) into cerulo-
Copper plasmin has been employed in the diag-
Unlike routine serum copper determi- nostic evaluation of Wilson disease, but
nation, free copper measures just the is rarely used because of difficulty in
portion of copper that is not bound to obtaining the radioactive isotope and
ceruloplasmin and is available for depo- overlap of values between individuals
sition in tissues. This portion of copper with Wilson disease and heterozygous
typically is elevated in Wilson disease. carriers.5,25 CSF copper levels are ele-
However, serum free copper levels vated in persons with Wilson disease
generally are calculated rather than and neurologic dysfunction and may
directly measured, and simultaneous actually be the most accurate reflection
total serum copper and ceruloplasmin of brain copper load. However, mea-
levels are needed to make this calcula- surement of CSF copper is not
tion. To perform the calculation, the performed in routine clinical practice
ceruloplasmin level (reported in mcg/dL) and its use is limited to research.
is multiplied by 3 and then subtracted
from the total serum copper level Diagnostic Testing Guidelines
(reported in mcg/dL), producing the The diagnostic testing approach will
calculated free copper level, which is differ between individuals who pres-
10 mcg/dL to 15 mcg/dL in normal indi- ent with hepatic dysfunction and
viduals and higher than 25 mcg/dL in those who present with neurologic or
most persons with untreated Wilson psychiatric dysfunction.
disease.12,25 A method for direct mea- Hepatic presentation. Kayser-
surement of free copper in serum or Fleischer rings are not consistently pre-
plasma by means of inductively coupled sent in individuals evaluated for Wilson
mass spectrometry has been reported.29 disease who have only hepatic dysfunc-
tion, so their absence does not exclude
Neuroimaging Studies the diagnosis of Wilson disease. Al-
MRI frequently is abnormal in individ- though urinary copper content is usu-
uals with Wilson disease who have ally elevated and serum ceruloplasmin
developed neurologic dysfunction. is typically reduced, liver biopsy still is
The most characteristic abnormalities generally warranted to confirm the
are increased signal intensity on T2- diagnosis. Genetic testing will become
weighted images and reduced signal the diagnostic test of choice when it is
intensity on T1-weighted images, most widely available and affordable.
Continuum (Minneap Minn) 2016;22(4):1246–1261 www.ContinuumJournal.com 1253

Copyright © American Academy of Neurology. Unauthorized reproduction of this article is prohibited.


Wilson Disease

KEY POINTS
h In individuals with Neuropsychiatric presentation. In ping results in a small, but significant,
neurologic or psychiatric individuals presenting with either neu- negative copper balance.
dysfunction, a rologic or psychiatric dysfunction, a Initially, its use was limited to pre-
combination of combination of Kayser-Fleischer rings, symptomatic individuals, but with fur-
Kayser-Fleischer rings, elevated 24-hour urinary copper, and ther experience, the use of zinc as
elevated 24-hour urine reduced ceruloplasmin essentially maintenance therapy following initial
copper, and reduced confirms the diagnosis of Wilson dis- treatment of neurologically symptom-
ceruloplasmin confirms ease, and liver biopsy is unnecessary. atic individuals with other more potent
the diagnosis of Wilson However, in the occasional situa- decoppering agents also has become
disease, and liver biopsy tion where either the 24-hour urine accepted.12,25,31 Some, although cer-
is not necessary.
or the ceruloplasmin level is inconclu- tainly not all, investigators now even
h Zinc, penicillamine, and sive, liver biopsy may still be neces- consider zinc to be first-line therapy for
trientine for treatment sary. Once again, genetic testing will Wilson disease.4,31,32
of Wilson disease eventually become the diagnostic test Zinc may be administered as zinc
should be taken on an
of choice. acetate, sulfate, or gluconate and
empty stomach.
should always be taken on an empty
TREATMENT stomach. Adverse effects are rare, but
A variety of treatment approaches gastric irritation may occur; zinc acetate
have been utilized in the treatment is less likely to cause this than zinc
of Wilson disease. Treatment also will sulfate. The typical dose of zinc used
vary depending on the clinical presen- for both presymptomatic and symp-
tation of the patient. tomatic patients with Wilson disease
is 50 mg of elemental zinc 3 times daily
Dietary Therapy (zinc sulfate is often marketed as
Dietary restriction of copper intake containing 220 mg of zinc sulfate salt,
has not been a successful treatment but this translates to 50 mg of elemen-
approach in Wilson disease, although tal zinc).
some investigators recommend avoid-
ance of foods such as shellfish, liver, Copper Chelation Therapy
nuts, chocolate, and mushrooms, In contrast to decreasing absorption,
which have particularly high cop- the other treatment approach for
per content.12,25 Wilson disease entails increasing the
elimination of copper from the body
Inhibition of Intestinal Copper by means that circumvent the dam-
Absorption aged and inactive hepatic route. Che-
Limiting the accumulation of copper lation of copper with subsequent renal
by inhibiting its absorption from the excretion of the chelated compound
gastrointestinal tract is one approach has been the standard treatment of
that may be taken in the treatment of Wilson disease. Two agents currently
Wilson disease. The agent currently are used.
used for this is zinc. Penicillamine. Penicillamine is a
Zinc. Zinc is absorbed by intestinal metabolic byproduct of penicillin that
enterocytes and induces metallothionein avidly chelates copper. Penicillamine
formation within them. The induced should always be given on an empty
metallothionein binds dietary copper stomach because its bioavailability is
and traps it within the enterocytes until reduced by approximately 50% if it
the cells are eventually sloughed and is ingested with food. Pyridoxine sup-
excreted in the feces. This zinc-induced, plementation sometimes is recom-
metallothionein-mediated copper trap- mended because penicillamine is a
1254 www.ContinuumJournal.com August 2016

Copyright © American Academy of Neurology. Unauthorized reproduction of this article is prohibited.


KEY POINT
pyridoxine antagonist. A penicillamine Penicillamine may produce para- h Penicillamine may
dose of 250 mg to 500 mg 4 times daily doxical deterioration in neurologic produce paradoxical
frequently is employed; however, function following treatment initia- deterioration in
some authorities recommend lower tion. How frequently this occurs is neurologic function in
initial doses (125 mg/d to 250 mg/d) controversial, but may be in the range approximately 11% of
with upward titration by 250 mg at of 11% and may not be fully reversible patients with Wilson
2 to 3 week intervals to a goal of in close to 50% of individuals affected.36 disease following
1000 mg/d to 2500 mg/d.33 Why this deterioration in neurologic initiation of treatment.
It often takes several months be- function develops is not certain. Mobi-
fore improvement becomes evident lization of copper from the liver with
following penicillamine initiation; im- subsequent redistribution to the brain
provement may gradually continue is one possibility.
over 1 to 2 years. Although most fea- Because of the risk that penicillamine
tures of Wilson disease improve over might induce irreversible neurologic
time with penicillamine treatment, not deterioration in individuals following
all improve equally; dysarthria and the its initiation, some investigators recom-
fixed smile (risus sardonicus) may not mend initiating treatment with other
improve at all. Psychiatric symptoms medications, such as trientine or even
typically do not improve as rapidly or as zinc, instead of penicillamine, while
completely as neurologic symptoms others suggest using penicillamine with
and signs. The presence of extensive lower initiating doses. Complete avoid-
white matter changes on MRI may be a ance of penicillamine, except possibly
poor prognostic sign. Recent MRI data in the patient with fulminant hepatic
indicate that if correct diagnosis and failure while awaiting liver transplanta-
adequate treatment are not initiated tion, also has been advocated.12
within 24 months after onset of symp- Trientine. Trientine has a mecha-
toms, irreversible brain lesions are nism of action similar to that of
likely,34 although others report that penicillamine but it is a less vigorous
the diagnostic delay between the onset decoppering agent than penicilla-
of symptoms and the initiation of mine, which may make it less prone
medical therapy is not a statistically to trigger deterioration in neurologic
significant predictor of clinical nonre- function. This perceived higher safety
sponse.35 Although penicillamine is level of trientine has led to its increas-
very effective in improving symptoms, ing use as a first-line treatment of pa-
it also is capable of producing poten- tients with Wilson disease who present
tially disastrous adverse effects. Acute with neurologic dysfunction.
sensitivity reactions consisting of skin As is true with zinc and penicilla-
rash, fever, eosinophilia, thrombocy- mine, trientine should be taken on an
topenia, leukopenia, and lymphade- empty stomach. The typical daily dose
nopathy develop in 20% to 30% of is 750 mg/d to 2000 mg/d, divided into
individuals receiving penicillamine, 3 doses. Adverse effects from trientine
typically within 2 weeks of initiating are similar to but less frequent than
therapy. Penicillamine should be with penicillamine.
promptly discontinued if an acute sen-
sitivity reaction is recognized. A variety Combination Agents
of other potential adverse affects may Agents that both inhibit copper ab-
occur in the setting of chronic penicil- sorption and increase the elimination
lamine therapy, including an array of of copper are being developed and
dermatologic problems. may combine the benefits of currently
Continuum (Minneap Minn) 2016;22(4):1246–1261 www.ContinuumJournal.com 1255

Copyright © American Academy of Neurology. Unauthorized reproduction of this article is prohibited.


Wilson Disease

KEY POINT
h Liver transplantation available medications should they be- topic liver transplantation has been
typically is necessary come available. reported, presumably due to massive
in individuals with Tetrathiomolybdate. Ammonium release of copper from the liver dam-
Wilson disease who tetrathiomolybdate was first tested aged by Wilson disease just prior to
develop acute for the treatment of Wilson disease in and during its removal.
fulminant hepatic 1984, but it remains an experimental The potential utility of orthotopic
failure. Following treatment modality, unavailable for liver transplantation in treating the
successful transplantation, general use. patient with Wilson disease with stable
ongoing pharmacologic liver function but severe neurologic
treatment is not Liver Transplantation dysfunction who is advancing despite
necessary.
Historically, the mortality rate of ful- optimal medical management has
minant hepatic failure in the setting of been proposed, but is still considered
Wilson disease has been virtually 100% to be experimental and not routine
if treatment is confined to medical standard of care.
management, although in a recent Living related liver transplantation,
report, six out of nine patients treated in which the donor is a living relative
with a combination of a corticosteroid, of the affected patient and donates
a copper chelating agent, and thera- part of his or her liver, has been suc-
peutic plasma exchange recovered.37 cessfully employed in Wilson disease.
Orthotopic liver transplantation still More temporary or bridging methods
remains the definitive treatment for this also are being investigated to support
complication of Wilson disease and individuals until a donor liver becomes
also may be appropriately employed in available, such as exchange transfusion,
patients with chronic severe hepatic plasma exchange, hemofiltration, albu-
insufficiency who are unresponsive to min dialysis, and the molecular absor-
medical management. With growing bent recirculating system.16
experience and surgical refinements,
the 5-year survival rate following ortho- Deep Brain Stimulation
topic liver transplantation for Wilson For patients with Wilson disease with
disease is now 85% to 90%.16 Since the neurologic dysfunction who do not
transplanted liver is free of the genetic achieve a satisfactory outcome from
defect that characterizes Wilson dis- medical treatment, evidence is emerg-
ease and contains normally functioning ing that deep brain stimulation (DBS)
ATP7B protein, copper metabolism may be useful in some instances, such
normalizes following orthotopic liver as refractory tremor or dystonia, but
transplantation, and continued chela- experience is still limited.38
tion or other medical therapy generally
is no longer necessary. However, the Treatment Guidelines
genetic abnormality remains in all Recommended treatment approaches
other body tissues and, thus, will be to Wilson disease differ, depending on
passed on to all children, and if the the clinical presentation of the patient.
transplanted liver was received from a These are summarized in the follow-
parent as a living related donor, the ing paragraphs and in Table 11-3.
transplanted liver will bear the genetic Asymptomatic patients. In the indi-
makeup of a heterozygote for Wilson vidual who is still asymptomatic, most
disease and may not have entirely investigators now recommend that
normal liver function. therapy be initiated with zinc alone.
The appearance of neurologic Hepatic presentation. In the indi-
symptoms shortly following ortho- vidual who has hepatic but not
1256 www.ContinuumJournal.com August 2016

Copyright © American Academy of Neurology. Unauthorized reproduction of this article is prohibited.


KEY POINTS
TABLE 11-3 Treatment of Wilson Disease h In persons with Wilson
disease who are still
b Asymptomatic Patients asymptomatic, zinc
monotherapy is the
Zinc monotherapy
recommended treatment.
b Hepatic Presentation
h In individuals with
Chelating agent initially Wilson disease who
have developed
Trientine may be preferable to penicillamine where available
neurologic or psychiatric
Some physicians use zinc for initial therapy dysfunction, copper
Zinc for maintenance therapy chelation therapy with
either trientine or
b Neuropsychiatric Presentation penicillamine initially
Chelating agent initially is employed.

Trientine may be preferable to penicillamine where available


Some physicians use zinc for initial therapy
Zinc for maintenance therapy
b Fulminant Hepatic Failure
Liver transplantation
Bridging measures may be used if needed

neurologic or psychiatric dysfunction, chiatric dysfunction. The primary


introduction of both a chelating agent choice to be made is whether to initiate
and zinc simultaneously may be ideal, therapy with penicillamine or trientine.
although compliance issues because Both have their advocates, but a grow-
of the complicated dosing schedule ing preference for trientine seems evi-
necessary if both drugs are used dent, at least in locations where trientine
simultaneously makes this approach is available. Zinc is recommended by
unwieldy and ill-advised in the eyes of some investigators for initiation of ther-
some authorities.39 Penicillamine has apy, but most physicians reserve its use
been the standard chelating agent in the neurologically affected patient for
used in this situation in the past, but maintenance therapy following initial
trientine has gained favor because of employment of a chelating agent.
the perception that it is less likely than Fulminant hepatic failure. For the
penicillamine to induce neurologic individual with either fulminant hepatic
deterioration upon initiation of treat- failure or severe, chronic liver failure,
ment. In a recent retrospective review, orthotopic liver transplantation may be
zinc monotherapy was found to be the only viable treatment option. Tempo-
comparably effective to penicillamine rizing bridging measures to be employed
as first-line therapy in all patients with while awaiting donor liver availability
Wilson disease, not just asymptomatic are being evaluated, but should still be
or mildly affected ones.40 considered experimental.
Neuropsychiatric presentation. No
unequivocally clear consensus has yet Treatment Monitoring
developed for treating the individual Adequate monitoring of patients follow-
who has developed neurologic or psy- ing initiation of treatment is an extremely

Continuum (Minneap Minn) 2016;22(4):1246–1261 www.ContinuumJournal.com 1257

Copyright © American Academy of Neurology. Unauthorized reproduction of this article is prohibited.


Wilson Disease

KEY POINT
h Copper deficiency may important, but often neglected, aspect stable penicillamine therapy, the
develop during of Wilson disease management. Even 24-hour urinary copper level should
long-term therapy with with close follow-up, compliance be in the range of 200 mcg/d to
chelating agents in problems become evident in a signif- 500 mcg/d; levels below 200 mcg/d
individuals with Wilson icant portion of patients with Wilson may indicate either noncompliance
disease, and anemia disease, which makes it vital that or overtreatment.25 The serum free
may be the initial sign of compliance be monitored regularly. (nonceruloplasmin-bound) copper level
copper deficiency. Strict compliance is so vital that some can also be a useful monitoring tool;
authorities recommend supervised elevation above 15 mcg/dL suggests
medication administration and weekly inadequate compliance.12
or biweekly clinic visits during the It is important to remember that
initial treatment stages.33 Compliance prolonged treatment with zinc and
with zinc therapy can be assessed by chelating agents poses the risk of
measurement of 24-hour urinary zinc actually inducing a copper deficiency
and copper levels. A 24-hour urinary state in the patient with Wilson dis-
zinc level of less than 2 mg (normal is ease (Case 11-3). Anemia, sometimes
0.1 mg to 0.4 mg) indicates inadequate with associated leukopenia, may be
compliance. Monitoring compliance the initial sign of copper deficiency.
with trientine or penicillamine therapy In patients on zinc maintenance ther-
is somewhat more difficult, but a spike apy, a 24-hour urinary copper level
in a previously gradually decreasing below 35 mcg/d is suggestive of
24-hour urinary copper level may copper deficiency due to over-
indicate inadequate compliance. In treatment.12 For individuals on trien-
patients with Wilson disease on chronic, tine or penicillamine, a serum-free

Case 11-3
A 27-year-old man presented in routine follow-up to the neurology clinic.
He had been diagnosed with Wilson disease 6 years previously. After his
initial diagnosis, the patient was placed on treatment with penicillamine
and both neurologic and hepatic function improved markedly. This
improvement had been maintained. Two years ago he was switched to
zinc maintenance therapy. Periodic monitoring with 24-hour urine copper
determinations demonstrated a level of 200 mcg/d to 300 mcg/d when he
was on penicillamine, but his most recent level was only 20 mcg/d. A recent
blood test documented the presence of both anemia and leukopenia.
Comment. The presence of anemia and leukopenia in an individual with
Wilson disease on chronic treatment with either a chelating agent or zinc
should raise the question of acquired copper deficiency. In addition to
hematologic dysfunction, prolonged copper deficiency can result in
progressive neuropathy and myelopathy that may be irreversible. There
are no firm guidelines on how to treat acquired copper deficiency in the
setting of Wilson disease. The zinc or chelating agent should be
temporarily stopped. Copper supplementation, as would be employed in
other copper deficiency syndromes, would not be prudent in most
instances in this setting, but if myelopathy has developed subacutely,
copper should be administered to more rapidly replenish copper to normal
levels. Monitoring copper status and resuming chelation or zinc therapy
at a lower dose once copper status has reached normal levels is a
reasonable treatment approach.

1258 www.ContinuumJournal.com August 2016

Copyright © American Academy of Neurology. Unauthorized reproduction of this article is prohibited.


(nonceruloplasmin-bound) copper Supplemental Digital
level below 5 mcg/dL suggests Content 11-2
overtreatment.12,25 Dystonia in Wilson disease. Video
A guideline for the diagnosis shows a man with dystonia, which is a
and treatment of Wilson disease, common manifestation of Wilson dis-
approved by the American Associa- ease. Prominent dystonia is evident in
tion for the Study of Liver Diseases the hands, arms, and facial musculature.
(AASLD), has been published and pro- Dysarthria may progress to the point of
vides an excellent in-depth review with anarthria, as is evident in this patient.
23 specific diagnostic and treatment Cognitive function may be intact despite
recommendations.25 devastating neurologic dysfunction.
CONCLUSION links.lww.com/CONT/A202
B 2016 American Academy of Neurology.
Wilson disease is a rare genetic disor-
der with protean manifestations that REFERENCES
should be considered in the differen- 1. Wilson SAK. Progressive lenticular
tial diagnosis of any individual degeneration: a familial nervous system
disease. Br Med J 1912;2(2710):1645.
presenting with unexplained neuro-
logic, psychiatric, or hepatic dysfunc- 2. Lutsenko S. Modifying factors and
phenotypic diversity in Wilson’s disease.
tion. Thorough diagnostic testing, Ann N Y Acad Sci 2014;1315:56Y63.
which will depend upon the clinical doi:10.1111/nyas.12420.
presentation, should be expeditiously 3. Dusek P, Roos PM, Litwin T, et al.
performed. Appropriate treatment, The neurotoxicity of iron, copper and
which may include copper chelating manganese in Parkinson’s and Wilson’s
diseases. J Trace Elem Med Biol 2015;31:
agents, zinc, or liver transplantation, 193Y203. doi:10.1016/j.jtemb.2014.05.007.
depending on the circumstances,
4. Ranucci G, Socha P, Iorio R. Wilson disease:
should be promptly initiated and what is still unclear in pediatric patients?
maintained, since failure to diagnose Clin Res Hepatol Gastroenterol 2014;38(3):
and treat Wilson disease will result in 268Y272. doi:10.1016/j.clinre.2014.03.002.

progressive and ultimately irreversible 5. Bandmann O, Weiss KH, Kaler SG.


Wilson’s disease and other neurological
damage to the neurologic and other
copper disorders. Lancet Neurol
systems. Ongoing monitoring for treat- 2015;14(1):103Y113. doi:10.1016/
ment compliance also is an extremely S1474-4422(14)70190-5.
important and often neglected facet of 6. Ferenci P. Phenotype-genotype correlations
disease management. in patients with Wilson’s disease. Ann N Y
Acad Sci 2014;1315:1Y5. doi:10.1111/
nyas.12340.
VIDEO LEGENDS
Supplemental Digital 7. Coffey AJ, Durkie M, Hague S, et al.
A genetic study of Wilson’s disease in
Content 11-1 the United Kingdom. Brain 2013;136(pt 5):
Tremor in Wilson disease. Video 1476Y1487. doi:10.1093/brain/awt035.
shows a woman demonstrating resting 8. Olivarez L, Caggana M, Pass KA, et al.
tremor in Wilson disease. Although Estimate of the frequency of Wilson’s
disease in the US Caucasian population: a
the classic tremor in Wilson disease is
mutation analysis approach. Ann Hum
a coarse, proximal, upper extremity Genet 2001;65(pt 5):459Y463.
“wing-beating” tremor, resting tremor doi:10.1017/S0003480001008764.
also may occur. Very mild risus sardo- 9. Ferenci P, Caca K, Loudianos G, et al.
nicus is also evident. Diagnosis and phenotypic classification
of Wilson disease. Liver Int
links.lww.com/CONT/A201 2003;23(3):139Y142. doi:10.1034/
B 2016 American Academy of Neurology. j.1600-0676.2003.00824.x.

Continuum (Minneap Minn) 2016;22(4):1246–1261 www.ContinuumJournal.com 1259

Copyright © American Academy of Neurology. Unauthorized reproduction of this article is prohibited.


Wilson Disease

10. Aggarwal A, Aggarwal N, Nagral A, et al. Psychiatry 2014;36(1):53Y62. doi:10.1016/


A novel global assessment scale for j.genhosppsych.2013.08.007.
Wilson’s disease (GAS for WD). Mov Disord
23. Basu A, Thanapal S, Sood M, Khandelwal SK.
2009;24(4):509Y518. doi:10.1002/mds.22231.
Catatonia: an unusual manifestation of
11. Członkowska A, Tarnacka B, Möller JC, et al. Wilson’s disease. J Neuropsychiatry Clin
Unified Wilson’s Disease Rating ScaleVa Neurosci 2015;27(1):72Y73. doi:10.1176/
proposal for the neurological scoring of appi.neuropsych.13120362.
Wilson’s disease patients. Neurol Neurochir 24. Frota NA, Barbosa ER, Porto CS, et al.
Pol 2007;41(1):1Y12. Cognitive impairment and magnetic
12. Brewer GJ. Wilson’s disease: a clinician’s resonance imaging correlations in
guide to recognition, diagnosis, and Wilson’s disease. Acta Neurol Scand
management. Boston, MA: Kluwer, 2001. 2013;127(6):391Y398. doi:10.1111/ane.12037.

13. Merle U, Schaefer M, Ferenci P, Stremmel W. 25. Roberts EA, Schilsky ML; American Association
Clinical presentation, diagnosis, and for Study of Liver Diseases (AASLD). Diagnosis
long-term outcome of Wilson’s disease: a and treatment of Wilson disease: an
cohort study. Gut 2007;56(1):115Y120. update. Hepatology 2008;47(6):2089Y2111.
doi:10.1136/gut.2005.087262. doi:10.1002/hep.22261.

14. Kim JW, Kim JH, Seo JK, et al. Genetically 26. Roberts EA, Schilsky ML; Division of
confirmed Wilson disease in a 9-month old Gastroenterology and Nutrition, Hospital
boy with elevations of aminotransferases. for Sick Children, Toronto, Ontario, Canada.
World J Hepatol 2013;5(3):156Y159. A practice guideline on Wilson disease.
doi:10.4254/wjh.v5.i3.156. Hepatology 2003;37(6):1475Y1492.
doi:10.1053/jhep.2003.50252.
15. Członkowska A, Rodo M, Gromadzka G.
27. Quemeneur AS, Trocello JM, Ea HK, et al.
Late onset Wilson’s disease: therapeutic
Bone status and fractures in 85 adults with
implications. Mov Disord 2008;23(6):
Wilson’s disease. Osteoporos Int
896Y898. doi:10.1002/mds.21985.
2014;25(11):2573Y2580. doi:10.1007/
16. Schilsky ML. Liver transplantation for s00198-014-2806-2.
Wilson’s disease. Ann N Y Acad Sci
28. Ferenci P, Steindl-Munda P, Vogel W, et al.
2014;1315:45Y49. doi:10.1111/nyas.12454.
Diagnostic value of quantitative hepatic
17. Ala A, Borjigin J, Rochwarger A, Schilsky M. copper determination in patients with
Wilson disease in septuagenarian siblings: Wilson’s disease. Clin Gastroenterol Hepatol
raising the bar for diagnosis. Hepatology 2005;3(8):811Y818. doi:10.1016/S1542-
2005;41(3):668Y670. doi:10.1002/ 3565(05)00181-3.
hep.20601.
29. McMillin GA, Travis JJ, Hunt JW. Direct
18. Machado A, Chien HF, Deguti MM, et al. measurement of free copper in serum or
Neurological manifestations in Wilson’s plasma ultrafiltrate. Am J Clin Pathol
disease: report of 119 cases. Mov Disord 2009;131(2):160Y165. doi:10.1309/
2006;21(12):2192Y2196. doi:10.1002/ AJCP7Z9KBFINVGYF.
mds.21170.
30. Sinha S, Taly AB, Ravishankar S, et al.
19. Dusek P, Litwin T, Czlonkowska A. Wilson Wilson’s disease: cranial MRI observations
disease and other neurodegenerations and clinical correlation. Neuroradiology
with metal accumulations. Neurol Clin 2006;48(9):613Y621. doi:10.1007/
2015;33(1):175Y204. doi:10.1016/ s00234-006-0101-4.
j.ncl.2014.09.006.
31. Hoogenraad TU. Paradigm shift in treatment
20. Taly AB, Meenakshi-Sundaram S, Sinha S, of Wilson’s disease: zinc therapy now
et al. Wilson disease: description of 282 treatment of choice. Brain Dev 2006;28(3):
patients evaluated over 3 decades. Medicine 141Y146. doi:10.1016/j.braindev.2005.08.008.
(Baltimore) 2007;86(2):112Y121. doi:10.1097/
32. Weiss KH, Gotthardt DN, Klemm D, et al.
MD.0b013e318045a00e.
Zinc monotherapy is not as effective as
21. Berger B, Mader I, Damjanovic K, et al. chelating agents in treatment of Wilson
Epileptic status immediately after initiation disease. Gastroenterology
of D-penicillamine therapy in a patient with 2011;140(4):1189Y1198. e1. doi:10.1053/
Wilson’s disease. Clin Neurol Neurosurg j.gastro.2010.12.034.
2014;127:122Y124. doi:10.1016/
33. Aggarwal A, Bhatt M. The pragmatic
j.clineuro.2014.09.030.
treatment of Wilson’s disease. Mov Disord
22. Zimbrean PC, Schilsky ML. Psychiatric aspects Clin Pract 2014;1:14Y23. doi:10.1002/
of Wilson’s disease: a review. Gen Hosp mdc3.12003.

1260 www.ContinuumJournal.com August 2016

Copyright © American Academy of Neurology. Unauthorized reproduction of this article is prohibited.


34. Kozić DB, Petrović I, Svetel M, et al. 37. Tian Y, Gong GZ, Yang X, Peng F. Diagnosis and
Reversible lesions in the brain parenchyma management of fulminant Wilson’s disease: a
in Wilson’s disease confirmed by magnetic single center’s experience. World J Pediatr 2016;
resonance imaging: earlier administration 12(2):209Y214. doi:10.1007/s12519-015-0026-2
of chelating therapy can reduce the
38. Hedera P. Treatment of Wilson’s disease motor
damage to the brain. Neural Regen Res
complications with deep brain stimulation.
2014;9(21):1912Y1916. doi:10.4103/
1673-5374.145360. Ann N Y Acad Sci 2014;1315:16Y23.
doi:10.1111/nyas.12372.
35. Weiss KH, Stremmel W. Clinical considerations
39. Annu A, Mohit B. Importance of adequate
for an effective medical therapy in Wilson’s
decoppering in Wilson’s disease. Mov Disord
disease. Ann N Y Acad Sci 2014;1315:81Y85.
2014;29(8):1089. doi:10.1002/mds.25900.
doi:10.1111/nyas.12437.
36. Litwin T, Dzieżyc K, Karliński M, et al. Early 40. Członkowska A, Litwin T, Karliński M, et al.
neurological worsening in patients with D-penicillamine versus zinc sulfate as first-line
Wilson’s disease. J Neurol Sci 2015;355(1Y2): therapy for Wilson’s disease. Ear J Neurol
162Y167. doi:10.1016/j.jns.2015.06.010. 2014;21(4):599Y606. doi:10.1111/ene.12348.

Continuum (Minneap Minn) 2016;22(4):1246–1261 www.ContinuumJournal.com 1261

Copyright © American Academy of Neurology. Unauthorized reproduction of this article is prohibited.


LIFELONG LEARNING IN NEUROLOGY ®
\

Diagnosing Parkinson Disease


Christopher W. Hess MD; Michael S. Okun, MD, FAAN. Continuum (Minneap Minn).
August 2016; 22 (4 Movement Disorders):1047Y1063.

Abstract
Purpose of Review:
While establishing the diagnosis of Parkinson disease (PD) can be straightforward, it can be
challenging in some patients, even for the experienced neurologist. The misdiagnosis rate ranges
from 10% to 20% or greater depending on clinician experience.
Recent Findings:
Despite promise in the search for a biomarker that can establish the presence of PD and
act as a marker of its progression, the diagnosis of PD continues to be based on clinical
examination. Core criteria, exclusion criteria, and supportive criteria have been developed to
aid the clinician in establishing the diagnosis. Nonmotor symptoms of PD are usually present
at the time of diagnosis, may precede motor symptoms, and should be specifically sought during
evaluation. Ancillary testing can be appropriate, but its indications and utility must be
clearly understood.
Summary:
The diagnosis of PD requires the recognition of the core features of PD and the differentiation
of its clinical presentation from other entities with similar and potentially overlapping
symptoms. A careful history and examination guided by clinical diagnostic criteria will usually
establishthe diagnosis of PD or uncover red flags for the possibilities of other diagnoses.
Appropriate selection and interpretation of ancillary testing is critical to avoid misdiagnosis and
unnecessary tests.

Key Points
& Parkinsonism is a clinical syndrome that can have a variety of possible causes, only one of
which is Parkinson disease.
& There are no clinically available biomarkers to indicate the presence of Parkinson disease
or track disease progression.
& The diagnosis of Parkinson disease during the patient’s lifetime is based on the
clinical examination.

* 2016 American Academy of Neurology.

Copyright © American Academy of Neurology. Unauthorized reproduction of this article is prohibited.


& Core criteria, exclusion criteria, and supportive criteria have been established that can
assist in the diagnosis of Parkinson disease.
& Rigidity giving rise to decreased range of motion and shoulder pain (often misdiagnosed
as orthopedic or arthritic) and difficulty turning over in bed at night are common
presenting symptoms of Parkinson disease.
& Parkinson tremor can fluctuate significantly in amplitude with mental activity or
voluntary movements of other limbs, and, thus, it is not unusual to be initially described as
intermittent. It often has a reemergent quality such that it can reappear with postural
sustention after a variable delay, and patients may report noticing it for the first time with
sustained postures, such as holding a telephone or newspaper.
& The lack of resting tremor does not exclude the diagnosis of Parkinson disease.
& The cardinal symptoms of Parkinson disease are bradykinesia, rigidity, resting tremor,
and postural instability.
& Actual falls occurring early in the disease course and especially in the first year of presenting
symptoms should elicit suspicion of the possibility of an atypical form of parkinsonism.
& Nonmotor symptoms such as hyposmia, erectile dysfunction, constipation, and rapid eye
movement sleep behavior disorder can precede the onset of motor symptoms of Parkinson
disease by years.
& Parkinson disease results in a variety of nonmotor symptoms that can sometimes be more
disabling than the motor symptoms, and studies suggest that the nonmotor symptoms
have a greater effect on quality of life.
& At least two subtypes (tremor-predominant Parkinson disease and postural instability
and gait difficulty Parkinson disease) have been identified, and both of these entities
manifest different rates of progression, disability, nonmotor symptoms, and complications
of therapy.
& Atypical parkinsonism can sometimes initially be mistaken for idiopathic Parkinson
disease. Falls and significant autonomic dysfunction early within the disease course, a
poor response to levodopa, symmetry of symptoms at onset, and rapid symptom
progression are useful red flags indicating the possibility of atypical parkinsonism.
& Drug-induced parkinsonism can be very difficult to distinguish from Parkinson disease;
the most important factor in its consideration is a careful and complete medication history.
& Laboratory testing can sometimes be useful in ruling out metabolic abnormalities that
may present as parkinsonism.
& Genetic testing in Parkinson disease is generally not recommended except for very
specific patient populations.
& While the research utility of dopamine transporter single-photon emission computed
tomography is universally agreed upon, its use in the clinical setting for differential
diagnosis in the individual remains controversial.
& If dopamine transporter single-photon emission computed tomography imaging is
pursued, it is important to note that a number of commonly prescribed medications can
interfere with dopamine transporter binding and produce erroneous results.

Treating the Motor Symptoms of


Parkinson Disease
John C. Morgan, MD, PhD; Susan H. Fox, MBChB, MRCP(UK), PhD. Continuum (Minneap Minn).
August 2016; 22 (4 Movement Disorders):1064Y1085.

* 2016 American Academy of Neurology.

Copyright © American Academy of Neurology. Unauthorized reproduction of this article is prohibited.


Abstract
Purpose of Review:
After a patient is diagnosed with Parkinson disease (PD), there are many therapeutic options
available. This article provides examples of prototypical patients encountered in clinical practice
and illustrates the various pharmacologic and nonpharmacologic treatment options for the motor
symptoms of PD.

Recent Findings:
Levodopa became available in the late 1960s and remains the gold standard for the treatment of
PD even today. Since that time, amantadine, monoamine oxidase type B inhibitors, dopamine
agonists, and catechol-O-methyltransferase inhibitors have emerged as monotherapy, add-on
therapies, or both, in the armamentarium against PD. The most appropriate time to start such
drugs remains a clinical decision according to patient symptoms. However, earlier use of
levodopa is the more common practice due to its superior benefit and the side effects of dopamine
agonists. Deep brain stimulation continues to be the most effective treatment for motor symptoms
in appropriate patients, and advances in technology may improve efficacy. New ways to deliver
levodopa have emerged (effective extended-release oral preparations and levodopa/carbidopa
intestinal gel), and these medications provide additional options for certain patients. Exercise and
neurorehabilitation are increasingly recognized as important tools to combat the motor symptoms
of PD. Nondopaminergic drugs may help nonYlevodopa-responsive motor symptoms.

Summary:
Treatment of PD is multifaceted and requires a tailored pharmacotherapeutic and nonpharmacologic
approach for a given patient. Patients should be at the center of care, and clinicians should try
to provide optimum benefit through the many treatment options available.

Key Points
& A younger patient with early Parkinson disease is very different from an older, retired
patient. Young, thin patients appear to have higher risk of developing dyskinesias with
levodopa therapy.
& While more recently neurologists have used levodopa-sparing therapies or
levodopa-delaying therapies in younger patients with Parkinson disease, physicians
should not hesitate to start levodopa if motor disability warrants it.
& Exercise, if safe for the patient, should be a part of every patient’s regimen to battle
against Parkinson disease. This not only includes physical exercise, but cognitive exercise
such as reading, crossword puzzles, and current events.
& Many younger patients with Parkinson disease embrace alternative therapies, and it is
important, as a clinician, to work within the patient’s framework as much as possible.
At the same time, discouraging use of unproven, expensive therapies that are highly
unlikely to provide any subjective or objective benefit is key.
& Levodopa is the drug of choice for patients with significant comorbidities, more advanced
age, postural instability, or advancing disability in activities of daily living or occupation.
& Physicians should exercise caution when prescribing dopamine agonists, amantadine,
monoamine oxidase type B inhibitors, or anticholinergic agents in patients presenting
with significant cognitive impairment.

* 2016 American Academy of Neurology.

Copyright © American Academy of Neurology. Unauthorized reproduction of this article is prohibited.


& When possible, the dose of levodopa should be kept lower to reduce the risk of wearing
off and dyskinesia.
& It is important to consider all pharmacotherapies in patients with early Parkinson disease
and to consider the patient’s goals as primary in helping them select what medication
they begin.
& Clinicians should not withhold levodopa from patients due to fear of motor complications.
& Always screen for impulse control disorders in patients with Parkinson disease and
counsel patients and families about the potential for these issues to arise during therapy
with any dopaminergic drug, but especially with dopamine agonists.
& Wearing off is generally more bothersome to patients than on-period dyskinesia, so
clinicians should focus on strategies to improve off symptoms.
& If a patient has levodopa-induced peak-dose dyskinesia already, then adding
in a monoamine oxidase type B inhibitor (selegiline or rasagiline), or a
catechol-Omethyltransferase inhibitor (entacapone) will likely increase dyskinesia.
& When prescribing dopamine agonists for patients with Parkinson disease, impulse
control disorders and milder variations are far more common than clinical studies
suggest. Physicians should always have a very high index of suspicion and should
withdraw the offending drug slowly to reduce the risk of dopamine agonist
withdrawal syndrome.
& Coprescribing certain antidepressants with rasagiline/selegiline as add-on therapy has been
shown to be safe from use in phase 3 randomized controlled trials. These include sertraline
100 mg/d, citalopram 20 mg/d, paroxetine 30 mg/d, fluvoxamine 150 mg/d, trazodone
100 mg/d, and amitriptyline 50 mg/d. Clinicians should assess the timing ofmovements in
relation to the dose of levodopa as high dose (peak-dose of levodopa action) or low dose
(onset and end of levodopa dose, or in the off periods) to help classify and then treat the
type of dyskinesia.
& Often, patients with Parkinson disease are unaware of the movements, especially chorea,
and as such do not report any disability. Painful dystonia is more likely to be reported.
& Not all dyskinesia needs to be treated. Rather, focus on reducing only disabling,
bothersome dyskinesia with medical/surgical strategies.
& Referral to a specialist center and careful selection criteria are key to successful surgery
for patients with Parkinson disease.
& Early referral for deep brain stimulation in subjects not yet optimized on medical therapies
remains controversial.
& Infusional therapies require nursing and care support for patients with advanced
Parkinson disease and their families, and device-complications are common.
& Counseling patients that some symptoms (eg, falling, dysphagia) will not respond to a higher
dose of levodopa is important, and nondrug management strategies are far more effective.
& Less is more in the treatment of patients with Parkinson disease, and simplifying
drug regimens allows for less adverse effects and better patient compliance in
advanced disease.

Neuropsychiatric Issues in
Parkinson Disease
Jennifer G. Goldman, MD, MS, FAAN. Continuum (Minneap Minn). August 2016;
22 (4 Movement Disorders):1086Y1103.

* 2016 American Academy of Neurology.

Copyright © American Academy of Neurology. Unauthorized reproduction of this article is prohibited.


Abstract
Purpose of Review:
This article reviews the recognition and management of neuropsychiatric issues in
Parkinson disease (PD), including mood disorders, cognitive impairment, and
behavioral disturbances.

Recent Findings:
Patients with PD frequently develop neuropsychiatric issues, and these issues can greatly
affect their quality of life. In recent years, mood, cognitive, and behavioral issues in PD
have received greater recognition, with increasing attention directed toward improved screening
and therapeutic interventions for symptomatic treatment. Taken together as a group,
neuropsychiatric issues can be found throughout the whole course of PD, from early in the
disease, potentially even in a premotor stage, to the time of diagnosis and later in the course with
more advanced disease.

Summary:
In the comprehensive care of patients with PD, recognition of neuropsychiatric issues is critical.
Advances in therapeutics for the different neuropsychiatric symptoms are still needed, although
several pharmacologic and nonpharmacologic options are available. Patient management
frequently requires a multidisciplinary approach, with collaboration of neurologists with
neuropsychologists, psychologists, psychiatrists, and other health professionals.

Key Points
& Neuropsychiatric issues are common in Parkinson disease and include mood, cognitive,
and behavioral symptoms that can occur at all stages of the disease.
& Depression in Parkinson disease is likely underdiagnosed and may manifest as milder
forms of depression rather than meeting Diagnostic and Statistical Manual of Mental
Disorders criteria definitions for major depression.
& Some features of Parkinson disease depression may overlap with other symptoms of
Parkinson disease, including motor features (slowness or psychomotor retardation,
blunted affect), somatic symptoms (muscle tension, gastrointestinal problems),
vegetative symptoms (changes in weight or appetite, changes in sleep [insomnia or
hypersomnia], fatigue, or loss of energy), and cognitive symptoms (impaired attention
and concentration, indecisiveness).
& It is important to ask patients with Parkinson disease about depressive symptoms, even
when mild in severity, as well as about symptoms of hopelessness and suicidality.
& Depression in Parkinson disease may also occur as a nonmotor fluctuation or off period
phenomenon, which necessitates management strategies that target cyclical fluctuations.
& Treatment of Parkinson disease depression, as well as several other neuropsychiatric issues
in Parkinson disease, involves both pharmacologic and nonpharmacologic strategies.
& Anxiety in Parkinson disease can occur both by itself and with depression.
& Patients with Parkinson disease who have extreme anxiety when wearing off or in off
periods can have panic attacks with marked anxiety, shortness of breath, diaphoresis,
and autonomic and gastrointestinal symptoms, which can even lead to emergency
department visits and evaluations for cardiopulmonary reasons.

* 2016 American Academy of Neurology.

Copyright © American Academy of Neurology. Unauthorized reproduction of this article is prohibited.


& Specific anxiety states in Parkinson disease can include generalized anxiety disorder,
panic disorder, phobias (eg, social, agoraphobia), obsessive-compulsive disorder, and
situational or stress-related anxiety.
& Some symptoms of anxiety can overlap Parkinson disease symptoms, such as muscle
tension, poor concentration, fatigue, sleep problems (insomnia), and autonomic
dysfunction (cardiovascular, respiratory, gastrointestinal).
& To date, randomized controlled trials of anxiety medications and therapies are lacking in
Parkinson disease.
& For anxiety symptoms related to off periods, modification of the dopaminergic regimen
may be effective, such as with use of longer-acting dopaminergics, higher doses or
adjunctive dopaminergic treatment, or more frequent and smaller doses.
& Mild cognitive impairment in Parkinson disease is recognized as a potential prodromal
stage to Parkinson disease dementia.
& Parkinson disease dementia has been defined as a syndrome with an insidious onset
and impairment in more than one cognitive domain that has a significant effect on
activities of daily living.
& In contrast to the definition for Parkinson disease dementia in the Diagnostic and
Statistical Manual of Mental Disorders, Fourth Edition, the International Parkinson and
Movement Disorder Society criteria for Parkinson disease dementia propose that memory
does not have to be impaired in Parkinson disease and that dementia can be present if
other cognitive domains (eg, executive function, visuospatial function) are affected and
include behavioral problems (eg, psychosis, sleep disturbances, mood disturbances),
which are common in Parkinson disease dementia.
& Patients with parkinsonism who develop an early dementia (within 1 year of the motor
symptoms) are diagnosed as having dementia with Lewy bodies, although it has overlap
of neuropathology with Parkinson disease dementia.
& Parkinson disease cognitive impairment is associated with poor outcomes, negatively
impacting patients’ quality of life, contributing to morbidity and mortality, and increasing
nursing home placement.
& In the evaluation for acute or new-onset cognitive issues, infections (eg, urinary tract
infection, pneumonia), metabolic derangements, dehydration, new neurologic problems
(eg, stroke, subdural hematoma), new medical problems (eg, vitamin B12 deficiency,
thyroid disorders), and medication effects (eg, pain, bladder, sedating medications)
should be excluded.
& It is important to inquire about other comorbid neuropsychiatric problems, including poor
nighttime sleep, excessive daytime sleepiness, psychosis, depression, anxiety, and
apathy, which may influence cognitive function.
& Medications, including Parkinson disease medications such as anticholinergics and
dopamine agonists, should be carefully reviewed so that agents that may contribute to
impaired cognition, confusion, or sleepiness can be reduced or discontinued.
& It is important to discuss driving and home safety issues with patients who have Parkinson
disease with cognitive impairment as well as with their caregivers.
& Apathy in Parkinson disease may manifest as decreased goal-directed behaviors, loss of
motivation, decreased interest, or blunted emotional responses.
& Proposed subdomains or types of apathy in Parkinson disease include: (1) a reward
deficiency syndrome in which patients lack emotional resonance, (2) depression, (3)
dysexecutive syndrome of cognitive deficits, and (4) auto-activation deficit in which
patients lack spontaneous activation of mental processes without external stimulation.

* 2016 American Academy of Neurology.

Copyright © American Academy of Neurology. Unauthorized reproduction of this article is prohibited.


& Apathy is also associated with poor outcomes, decreased quality of life, and increased
caregiver burden, similar to many of the other neuropsychiatric symptoms encountered in
Parkinson disease.
& Punding is a stereotyped, repetitive, purposeless behavior performed by the patient.
In Parkinson disease, patients with punding will exhibit an excessive and intense
preoccupation with items or activities (eg, disassembling and reassembling objects or
rearranging papers).
& Dopamine dysregulation syndrome typically manifests in younger male patients with
Parkinson disease who develop an addictive like regimen of self-medicating with
high doses of dopaminergic medications, especially short-acting medications and
levodopa. Many of these patients have very minimal motor symptoms on examination,
even in off periods, but may have dyskinesia.
& Punding and dopamine dysregulation syndrome share repetitive excessive compulsive
features and a relationship with dopaminergic stimulation.
& Impulse control disorders in Parkinson disease frequently include pathologic gambling,
compulsive shopping, compulsive eating, and hypersexuality.
& Treatment of impulse control disorders in patients with Parkinson disease typically
includes a decrease or withdrawal of the dopamine agonist, which may cause worsening
of motor symptoms.
& According to the criteria for Parkinson disease psychosis from the National Institute of
Neurological Disorders and Stroke/National= Institute of Mental Health, the diagnosis
of Parkinson disease psychosis requires at least one of the following: illusions, false
sense of presence, hallucinations, or delusions. These features should occur after the
onset of Parkinson disease and be present for at least 1 month, either as a recurrent or
continuous symptom; other medical, neurologic, or psychiatric causes and acute delirium
should be excluded.
& Minor psychotic phenomena and nonvisual hallucinations, features not often included in
older epidemiologic studies, represent an important part of the Parkinson disease
psychosis spectrum.
& Illusions or misperceptions of real stimuli include the interpretation of inanimate objects
as living beings (eg, a chair mistaken for a dog, a lamppost mistaken for a tree).
& Many, but not all, visual hallucinations in Parkinson disease are nonthreatening, brief
(lasting seconds to minutes), and worse at night or in instances of compromised or
low vision.
& Auditory hallucinations in Parkinson disease are often vague and, in contrast to
schizophrenia, are less common and less likely to be threatening or directly interact with
the patient.
& Acute Parkinson disease psychosis should prompt an evaluation for medical and neurologic
conditions other than Parkinson disease and a review of the patient’s medication regimen.
& Causes of Parkinson disease psychosis may include dopaminergic medication effects,
but other causes, such as toxic-metabolic encephalopathy, urinary or pulmonary
infections, stroke, subdural hematoma, or medication interactions, should be excluded.

Treatment of Advanced Parkinson


Disease and Related Disorders
Janis M. Miyasaki, MD, MEd, FRCPC, FAAN. Continuum (Minneap Minn). August 2016;
22 (4 Movement Disorders):1104Y1116.

* 2016 American Academy of Neurology.

Copyright © American Academy of Neurology. Unauthorized reproduction of this article is prohibited.


Abstract
Purpose of Review:
Parkinson disease often spans decades of a patient’s lifetime. Over time, nonmotor symptoms
predominate and may limit dopaminergic therapy. Neurologists continue to play a vital role in
treatment. In addition to balancing neurobehavioral complications of Parkinson disease with
motor benefit, addressing nonmotor symptoms common in the advanced stage may improve
quality of life and reduce symptom burden. Symptoms such as dysphagia, constipation,
urinary dysfunction, orthostatic hypotension, and pain respond to nonpharmacologic and
pharmacologic therapies.
Recent Findings:
Evidence for treatment of many nonmotor symptoms is weak or lacking. The evidence for
treatment of the atypical parkinsonian syndromes (progressive supranuclear palsy, multiple
system atrophy, and corticobasal degeneration) in advanced stages is even more scant.
Summary:
Engaging palliative care physicians in the joint care of patients can provide patients with access to
expertise in end-of-life issues. Neurologic illnesses have specific hospice criteria to guide
clinicians for referrals. Evidence supports that assisting patients with advance directives can
result in improved satisfaction with care and improved quality of life in the last weeks of life.
Neurologists can remain engaged in their patients’ care throughout the course of illness.

Key Points
& Although individual progression is hard to predict in patients with Parkinson disease,
overall there is predictable progression over time to more nonmotor symptoms such as
dysphagia, cognitive decline, and often diminished dopaminergic responsiveness.
& Questions about function at work and the ability to participate in activities with the family
can help patients gauge whether work is becoming too difficult to manage. Similarly,
questions about managing stairs, the home setup, or organizing a home safety visit can
nudge patients and caregivers to consider moving to a more accessible home.
& Assessing caregiver burden with the Zarit Burden Interview will help identify those
caregivers requiring assistance. Small changes may enable patients to remain at home
longer by providing respite to the caregiver.
& Advance care planning or advance directives are often completed by the patient without a
physician providing context for interventions or the likelihood that an intervention will
help the patient meet their care goals. Neurologists have the information that can help
patients make informed choices to complete their advance directives.
& Advance directives should be reviewed periodically. They are a living document that
requires revision according to the patient’s condition and experiences with hospitalizations.
& Pain is often not recognized in Parkinson disease. Use of a pain scale and physical measures
(range-of-motion exercises, heat or cold compresses, and topical analgesics) can be effective.
& Constipation reduces the effectiveness of dopaminergic medications. Ensure patients
have a bowel movement daily or, at least, every 2 days.
& Orthostatic hypotension can limit treatment in up to 30% of patients with Parkinson
disease and significantly higher in multiple system atrophy where dysautonomia is a
diagnostic criterion.

* 2016 American Academy of Neurology.

Copyright © American Academy of Neurology. Unauthorized reproduction of this article is prohibited.


& Orthostatic hypotension can lead to falls, confusion, fatigue, and shoulder pain in addition
to syncope.
& Appropriate goals for blood pressure are at least 90 mm Hg systolic while standing and
less than 180 mm Hg systolic supine.
& Hospice care is available to patients with Parkinson disease and related disorders. Use the
hospice guidelines to refer patients for in-home care, inpatient care, or respite care.
& Aspiration is underestimated by bedside testing. The gold standard remains
videofluoroscopic swallow study. Changes to diet and modifying the meal routine can
improves swallowing safety.

Diagnostic Approach to Atypical


Parkinsonian Syndromes
Nikolaus R. McFarland, MD, PhD. Continuum (Minneap Minn). August 2016;
22 (4 Movement Disorders):1117Y1142.

Abstract
Purpose of Review:
Although increasingly recognized, atypical parkinsonian syndromes remain challenging to
diagnose and are underrecognized due to overlap with other parkinsonisms. This article
provides a diagnostic approach to atypical parkinsonian syndromes, including progressive
supranuclear palsy (PSP), multiple system atrophy (MSA), corticobasal degeneration (CBD), and
dementia with Lewy bodies. The goal of this review is to aid the clinician in recognizing
key clinical and pathologic features and to raise awareness of recent advances in diagnostics
and treatment.
Recent Findings:
Diagnostic criteria for atypical parkinsonian syndromes are evolving to encompass increasingly
recognized heterogeneity in the presentation of these disorders and information gleamed from
clinicopathologic correlations. PSP and CBD in particular now share similar pathologic clinical
features and include a number of phenotypic variants. Pathologic diagnoses are increasingly
used in clinical practice, and there is frequent reference now by clinicians to tauopathies,
including PSP and CBD, and the synucleinopathies, which include MSA and dementia with
Lewy bodies (aswell as Parkinson disease). Research into biomarkers, including both tissue and
imaging modalities and genetics, has the potential to increase disease recognition and make
earlier diagnosis and treatment possible. Although novel therapeutics are being studied for
atypical parkinsonian syndromes such as PSP, no new breakthrough interventions have
emerged for the treatment of PSP, CBD, and MSA. Current therapeutic management for these
disorders frequently uses a multidisciplinary team approach.
Summary:
The approach to atypical parkinsonian syndromes requires recognition of a constellation of
overlapping but distinct clinical features that help with identifying and distinguishing them from
Parkinson disease and other similar disorders.

* 2016 American Academy of Neurology.

Copyright © American Academy of Neurology. Unauthorized reproduction of this article is prohibited.


Key Points
& Key features of progressive supranuclear palsy include early gait instability, unexplained
falls, supranuclear gaze palsy, axial rigidity, dysarthric speech, and dementia.
& The rocket sign occurs in patients with progressive supranuclear palsy who have lost
insight into their postural instability and ‘‘rocket’’ out of their chair without assistance,
resulting in a high risk for falling.
& Early signs of supranuclear gaze palsy in patients with progressive supranuclear palsy
include slowed vertical saccades and reduced optokinetic nystagmus. Square-wave jerks,
or minute saccadic eye movements, may also be present, representing fixation instability.
& To assess for the applause sign, a clinician can demonstrate three claps to the patient
and ask him or her to copy. The applause sign is present if the patient claps more than
three times and continues (perseverates).
& Atrophy of the midbrain and superior cerebellar peduncles correlates with disease
progression in progressive supranuclear palsy.
& The approach to treatment of patients with progressive supranuclear palsy should include
a multidisciplinary team and involve physical, occupational, and speech therapy;
psychiatry; neuropsychology; social work; and palliative care.
& Levodopa therapy should be tried in most progressive supranuclear palsy cases, with a
levodopa dose of up to 1200 mg/d in divided doses as tolerated. Partial response is
possible in early progressive supranuclear palsy, particularly in progressive supranuclear
palsyYparkinsonism.
& The most common presenting features for corticobasal degeneration are asymmetric
hand clumsiness or apraxia followed by early bradykinesia, frontal syndrome, tremor,
and rigidity.
& Ideomotor apraxia is defined by an inability to perform a skilled motor task despite having
intact language, motor, and sensory function. Examples include inability to imitate
gestures or mime a certain task (eg, use a screwdriver or cut with a pair of scissors). This
type of apraxia can be difficult to distinguish from limb-kinetic apraxia, which is
frequently seen in parkinsonisms, but is independent of modality (imitation versus miming).
& Multiple system atrophyYparkinsonian type may be differentiated from Parkinson disease
by its more symmetrical appearance, atypical tremor, dystonia (antecollis), early
dysarthria/dysphonia, gait and postural instability, dysautonomia, and rapid progression.
& Multiple system atrophyYcerebellar type is one of the most common causes of
sporadic, adult-onset ataxia and is distinguished by parkinsonism, dysautonomia, and
rapid progression.
& Glial cytoplasmic inclusions that are enriched with >-synuclein are a pathologic hallmark
of multiple system atrophy.
& Pharmacologic treatment of orthostatic hypotension may include enhancing blood
volume with fludrocortisone or desmopressin or adding drugs that increase vascular
resistance such as midodrine, droxidopa, or pyridostigmine.
& Dementia with Lewy bodies is characterized by rapid-onset dementia, parkinsonism
(coincident or following cognitive decline), mental status fluctuations, and hallucinations.

Diagnosis and Management of Tremor


Elan D. Louis, MD, MS, FAAN. Continuum (Minneap Minn). August 2016;
22 (4 Movement Disorders):1143Y1158.

* 2016 American Academy of Neurology.

Copyright © American Academy of Neurology. Unauthorized reproduction of this article is prohibited.


Abstract
Purpose of Review:
Tremor, which is a rhythmic oscillation of a body part, is among the most common involuntary
movements. Rhythmic oscillations may manifest in a variety of ways; as a result, a rich
clinical phenomenology surrounds tremor. For this reason, diagnosing tremor disorders can be
particularly challenging. The aim of this article is to provide the reader with a straightforward
approach to the diagnosis and management of patients with tremor.
Recent Findings:
Scientific understanding of the pathophysiologic basis of tremor disorders has grown
considerably in recent years with the use of a broad range of neuroimaging approaches and
rigorous, controlled postmortem studies. The basal ganglia and cerebellum are structures that
seem to play a prominent role.
Summary:
The diagnosis of tremor disorders is challenging. The approach to tremor involves a history and a
neurologic examination that is focused on the nuances of tremor phenomenology, of which
there are many. The evaluation should begin with a tremor history and a focused neurologic
examination. The examination should attend to themany subtleties of tremor phenomenology.
Among other things, the history and examination are used to establish whether the main type of
tremor is an action tremor (ie, postural, kinetic, or intention tremor) or a resting tremor. The
clinician should then formulate two sets of differential diagnoses: disorders in which action
tremor is the predominant tremor versus those in which resting tremor is the main tremor. Among
the most common of the former type are essential tremor, enhanced physiologic tremor,
drug-induced tremor, dystonic tremor, orthostatic tremor, and cerebellar tremor. Parkinson
disease is the most common form of resting tremor, along with drug-induced resting tremor. This
article details the clinical features of each of these as well as other tremor disorders.

Key Points
& Tremors are rhythmic oscillations of a body part.
& The cardinal feature of essential tremor is kinetic tremor.
& In essential tremor, the amplitude of kinetic tremor is generally greater than that of
postural tremor.
& Resting tremor may occur in advanced essential tremor but is restricted to the arms rather
than the legs.
& Given its progressive nature and propensity to result in functional disabilities, the term
benign essential tremor is no longer favored.
& The jaw tremor of essential tremor typically occurs when the mouth is open rather
than closed.
& Flexed posturing of the hand during arm extension is an early sign of Parkinson disease.
& Neck tremor should not occur in patients with enhanced physiologic tremor.
& With enhanced physiologic tremor, no intentional component occurs on the
finger-nose-finger maneuver.
& While limb tremor may be present, head tremor should not be a feature of drug-induced
action tremor.
& The tremor in dystonia is often neither rhythmic nor oscillatory.

* 2016 American Academy of Neurology.

Copyright © American Academy of Neurology. Unauthorized reproduction of this article is prohibited.


& Voice tremor in patients with dystonia is often associated with voice breaks.
& Orthostatic tremor may be more easily felt or auscultated than seen.
& In patients with diseases of the cerebellum, it is important to distinguish tremor
from dysmetria.
& The presence of entrainment, distractibility, and suggestibility all point to a diagnosis of
psychogenic tremor.
& Wing-beat tremor is considered a classic tremor in Wilson disease, but it is not the most
common type of tremor in that disease.
& Patients with fragile X tremor-ataxia syndrome may have a mixture of different tremor
phenomenologies.
& Midbrain (rubral) tremor is generally a unilateral tremor accompanied by rest, postural,
and kinetic/intentional components.
& In addition to the classic resting tremor, a large proportion of patients with Parkinson
disease also have postural or kinetic tremors of the arms.
& Drug-induced resting tremor may be either unilateral or bilateral.

Movement Disorders Presenting


in Childhood
Manju A. Kurian, MBBChir, MRCPCH, PhD; Russell C. Dale, MBChB, MRCP, PhD.
Continuum (Minneap Minn). August 2016; 22 (4 Movement Disorders):1159Y1185.

Abstract
Purpose of Review:
This article provides an overview of movement disorders that present in childhood. Key clinical
features are discussed, and a brief guide to management strategies is provided. Recent advances in
the field of pediatric movement disorders are also a focus of the article.
Recent Findings:
Advances in genetic technologies and cell biology have contributed greatly to the elucidation of
underlying disease mechanisms in childhood movement disorders. This article discusses the
expanding spectrum of both genetic and acquired movement disorders that present in childhood,
including benign, acquired, genetic, and psychogenic movement disorders.
Summary:
Movement disorders in childhood comprise a wide spectrum of both genetic and acquired
diseases, ranging from benign self-limiting conditions to more progressive phenotypes associated
with significant morbidity and mortality. Elucidation of the underlying cause is achieved through
accurate history, detailed clinical examination, review of video footage (including home videos),
and, where appropriate, neuroimaging and laboratory investigations. Early accurate diagnosis
will facilitate the instigation of appropriate management strategies.
Key Points
& Movement disorders in childhood are heterogeneous and range from mild self-limiting
phenotypes to more severe progressive neurodegenerative disorders.

* 2016 American Academy of Neurology.

Copyright © American Academy of Neurology. Unauthorized reproduction of this article is prohibited.


& A number of benign movement disorders of childhood have been identified, which are
transient in nature and usually have a good long-term prognosis.
& Tics and stereotypy are the most common movement disorders of childhood. Diagnosis
is dependent on careful history, observation, and parental videos. Education of the
family and searching for neurodevelopmental and neuropsychiatric comorbidity are
paramount, as associated comorbidities such as anxiety and obsessive-compulsive
disorder can exacerbate the movements and are arguably more amenable to psychological
or medical treatments.
& Cerebral palsy is one of the most common causes of movement disorders in children.
Detailed clinical history, examination, and neuroimaging may aid diagnosis, although
alternative diagnoses should be sought in cases that present atypically.
& Encephalitis may be associated with a wide spectrum of causative organisms and can
manifest with both hyperkinetic and hypokinetic phenotypes.
& N-Methyl-D-aspartate encephalitis is an emerging important cause of cerebral
autoimmune disease, and both hyperkinetic and hypokinetic movement disorders are
reported as part of the clinical disease spectrum.
& Sydenham chorea continues to be an important cause of immune-mediated chorea
worldwide and can be associated with significant long-term morbidity.
& Features of opsoclonus myoclonus ataxia syndrome should prompt screening for neural
crest tumors and initiation of immunotherapy.
& Acute presentation of a new movement disorder with rapid evolution should prompt
neuroimaging to exclude a structural or vascular cause.
& Iatrogenic etiologies should be considered as a differential diagnosis for movement disorders,
and a detailed drug history and toxicology screen may aid in the diagnosis of such disorders.
& Acquired movement disorders have multiple potential causes. Although investigations
such as MRI, CSF analysis, and autoantibody testing are important, it cannot be
overemphasized that a careful clinical history and examination for specific movement
disorder phenomenology is essential. The focus should be on defining the etiology and
providing specific targeted intervention. Early recognition and intervention are
increasingly emphasized.
& Primary genetic dystonias are defined as those generally associated with normal brain
imaging. In childhood, reported forms include DYT1, DYT5, DYT11, DYT12, DYT23,
and DYT6.
& The spectrum of genetic choreas is expanding and now includes disorders due to
NKX2-1-AS1and ADCY5 mutations as well as those associated with more complex
neurologic disorders such as FOXG1-related disease.
& Genetic paroxysmal movement disorders of childhood have been increasingly
recognized, and some have important treatment implications. Paroxysmal movement
disorders may often coexist with other episodic conditions, including epilepsy
and migraine.
& Nonepileptic myoclonus should prompt testing for genetic and neurometabolic
syndromes.
& A number of recessive genetic ataxias have been identified, including Friedreich ataxia
and ataxia telangiectasia.
& Clinical parkinsonism in childhood should prompt diagnostic testing for underlying
genetic and neurometabolic etiologies.
& Genetic movement disorders are increasingly being recognized and diagnosed in
children. Often, chromosomal microarray is an early investigation in such children to
identify copy number variants accounting for disease phenotypes. Further genetic testing
is guided by clinical history, features on examination, and radiologic or metabolic

* 2016 American Academy of Neurology.

Copyright © American Academy of Neurology. Unauthorized reproduction of this article is prohibited.


biomarkers. The evolution of multiple gene panels and whole exome sequencing
technologies has further improved diagnostic approaches for this group of patients.
& Early recognition and intervention as well as involving a multidisciplinary team to
rehabilitate the patient both physically and psychologically are essential in management
of psychogenic movement disorders. Treating psychogenic movement disorders can be
rewarding to the child, family, and health practitioners.

Chorea
Tiago A. Mestre, MSc, MD. Continuum (Minneap Minn). August 2016;
22 (4 Movement Disorders):1186Y1207.

Abstract
Purpose of Review:
This article reviews the clinical approach to the diagnosis of adult patients presenting with chorea,
using Huntington disease (HD) as a point of reference, and presents the clinical elements that help
in the diagnostic workup. Principles of management for chorea and some of the associated
features of other choreic syndromes are also described.
Recent Findings:
Mutations in the C9orf72 gene, previously identified in families with a history of frontotemporal
dementia, amyotrophic lateral sclerosis, or both, have been recognized as one of the most
prevalent causes of HD phenocopies in the white population.
Summary:
The diagnosis of chorea in adult patients is challenging. A varied number of associated causes
require a physician to prioritize the investigations, and a detailed history of chorea and associated
findings will help. For chorea presenting as part of a neurodegenerative syndrome, the consideration
of a mutation in the C9orf72 gene is a new recommendation after excluding HD. There are no new
treatment options for chorea, aside from dopamine blockers and tetrabenazine. There are no
disease-modifying treatments for HD or other neurodegenerative choreic syndromes.

Key Points
& Chorea may present as an isolated symptom or as a syndrome with a variable
combination of a mixed movement disorder, behavioral and cognitive symptoms,
seizures, or polyneuropathy.
& Most of the cases of senile chorea correspond to cases of late-onset Huntington disease.
& Chorea secondary to stroke has an acute onset and will, most often, improve over time.
& Recurrent episodes of chorea, frequently in association with dystonia, suggest a form of
paroxysmal dyskinesia.
& Orofacial chorea is suggestive of a classic neuroacanthocytosis syndrome.
& In chorea-acanthocytosis, seizures can occur in about 50% of cases.
& Huntington diseaseYlike syndrome type 2 is most prevalent in patients with a sub-Saharan
African ancestry.

* 2016 American Academy of Neurology.

Copyright © American Academy of Neurology. Unauthorized reproduction of this article is prohibited.


& In the Western world, chorea gravidarum has become a rare cause of chorea.
& In C9orf72-related Huntington disease, phenocopies present with significant phenotypic
heterogeneity and have, in most of the cases, a positive family history.
& Hemichorea or hemiballismus suggest the presence of a focal brain lesion.
& The combination of other movement disorders can help with the diagnosis of a particular
chorea syndrome.
& In neuroacanthocytosis syndromes, the clinical multisystemic involvement is
characteristic and has diagnostic and management implications.
& Huntington disease is the most frequent genetic cause of chorea.
& The vast majority of Huntington disease phenocopies remain undiagnosed.
& Spinocerebellar ataxia type 17 and C9orf7-related Huntington disease phenocopy are the
most frequent causes for a Huntington disease phenocopy.
& Caudate atrophy can be found in Huntington disease phenocopies, namely in Huntington
diseaseYlike syndrome type 2, chorea-acanthocytosis, and McLeod syndrome.
& The treatment of chorea should aim at reducing related disability and improving
overall function.
& In Huntington disease and Huntington disease phenocopies, a multidisciplinary
management plan is required.

Ataxia
Tetsuo Ashizawa, MD, FAAN; Guangbin Xia, MD, PhD. Continuum (Minneap Minn).
August 2016; 22 (4 Movement Disorders):1208Y1226.

Abstract
Purpose of Review:
This article introduces the background and common etiologies of ataxia and provides a general
approach to assessing and managing the patient with ataxia.

Recent Findings:
Ataxia is a manifestation of a variety of disease processes, and an underlying etiology needs to be
investigated. Pure ataxia is rare in acquired ataxia disorders, and associated symptoms and
signs almost always exist to suggest an underlying cause. While the spectrum of hereditary
degenerative ataxias is expanding, special attention should be addressed to those treatable and
reversible etiologies, especially potentially life-threatening causes. This article summarizes
the diseases that can present with ataxia, with special attention given to diagnostically useful
features. While emerging genetic tests are becoming increasingly available for hereditary ataxia,
they cannot replace conventional diagnostic procedures in most patients with ataxia. Special
consideration should be focused on clinical features when selecting a cost-effective
diagnostic test.

Summary:
Clinicians who evaluate patients with ataxia should be familiar with the disease spectrum that
can present with ataxia. Following a detailed history and neurologic examination, proper
diagnostic tests can be designed to confirm the clinical working diagnosis.

* 2016 American Academy of Neurology.

Copyright © American Academy of Neurology. Unauthorized reproduction of this article is prohibited.


Key Points
& A precise diagnosis of ataxia cannot be based on clinical features alone. Clinical
manifestations can be variable, and similar features may not occur in all individuals with a
particular disease. For many of the rarer ataxic disorders, the clinical features have been
defined on the basis of limited clinical experience.
& Physical examination should give attention not only to appendicular ataxia by
examination of limb movements and upright ataxia by examination of stance, gait, and
truncal ataxia, but also to ocular dyskinesia, speech abnormalities, proprioceptive loss,
and vestibular dysfunction.
& When sequence analysis is done in a patient with ataxia, polymorphisms of unknown
significance may be reported. Such alterations have not been previously reported to be
pathogenic. Whether such polymorphisms are truly pathogenic will have to be
determined by additional research.

Diagnosis and Management of Dystonia


Vicki Shanker, MD; Susan B. Bressman, MD, FAAN. Continuum (Minneap Minn).
August 2016; 22 (4 Movement Disorders):1227Y1245.

Abstract
Purpose of Review:
This article highlights the clinical and diagnostic tools used to assess and classify dystonia and
provides an overview of the treatment approach.
Recent Findings:
In the past 4 years, the definition and classification of dystonia have been revised, and new genes
have been identified in patients with isolated hereditary dystonia (DYT23, DYT24, and DYT25).
Expanded phenotypes were reported in patients with combined dystonia, such as those with
mutations in ATP1A3. Treatment offerings have expanded as there are more neurotoxins, and deep
brain stimulation has been employed successfully in diverse populations of patients with dystonia.
Summary:
Diagnosis of dystonia rests upon a clinical assessment that requires the examiner to understand
the characteristic disease features that are elicited through a careful history and physical
examination. The revised classification system uses two distinct nonoverlapping axes: clinical
features and etiology. A growing understanding exists of both isolated and combined dystonia as
new genes are identified and our knowledge of the phenotypic presentation of previously reported
genes has expanded. Genetic testing is commercially available for some of these conditions.
Treatment options for dystonia include pharmacologic therapy, chemodenervation, and surgical
intervention. Deep brain stimulation benefits many patients with various types of dystonia.

Key Points
& Dystonia is a hyperkinetic movement disorder characterized by involuntary muscle
contractions often initiated or worsened by voluntary action.

* 2016 American Academy of Neurology.

Copyright © American Academy of Neurology. Unauthorized reproduction of this article is prohibited.


& In 2013 a consensus document presented a revised classification that categorized dystonia
into two nonoverlapping axes. Axis I categorizes dystonia by the clinical features and axis
II by etiology.
& DYT-TOR1A and DYT-THAP1 are the most common genetic causes of isolated dystonia,
and commercial testing is available for both.
& Since the advent of next-generation sequencing, three additional genes have been
reported to cause isolated adult-onset dystonia: DYT-CIZ1, DYT-ANO3,
and DYT-GNAL.
& The most common cause of dopa-responsive dystonia is mutations in GCH1, which
encodes the rate-limiting enzyme in the biosynthesis of tetrahydrobiopterin.
& Rapid-onset dystonia-parkinsonism is another combined dystonia associated with
parkinsonian features with an identified genetic origin: mutations in ATP1A3.
& Myoclonus-dystonia is a syndrome most commonly associated with mutations in the
sarcoglycan epsilon gene (SGCE).
& The movement disorder examination is crucial to identifying dystonia phenomenology
and includes several elements not routinely included in the neurologic examination.
& Additional studies such as genetic testing and neuroimaging may be required to
characterize a patient with dystonia along axis II of the classification system.
& Targeted, disease-specific treatments are increasing as our understanding of the etiology
for the combined/complex dystonia syndromes expands (eg, GLUT1 deficiency,
cerebrotendinous xanthomatosis, manganese transport disorder,Wilson disease).
& For most patients with multifocal and generalized dystonia, oral medical therapy is the
first-line approach.
& Carbidopa/levodopa is the standard of care treatment for dopa-responsive dystonia
as patients characteristically have a significant and sustained improvement to
low-dose therapy.
& Tetrabenazine may be most helpful in patients with tardive dystonia, with one
retrospective chart review reporting that 70% of patients with tardive dystonia had
moderate to excellent response to the drug.
& Trihexyphenidyl is the preferred anticholinergic agent and the only oral medication
studied in a prospective double-blinded fashion for the treatment of dystonia.
& Baclofen is a F-aminobutyric acid B receptor agonist reported in several retrospective
studies to demonstrate benefit in dystonia management, especially in children with
comorbid dystonia and spasticity.
& Chemodenervation with botulinum neurotoxin is first-line therapy for most patients with
focal and segmental dystonia.
& Selection of the specific botulinum toxin may have to do with outside factors including
medication cost, physician experience with toxin, and office access to refrigeration.
& Substantial data suggest that patients with DYT1 and isolated non-DYT1 generalized
dystonia are most responsive to deep brain stimulation intervention.
& Children with dystonia who receive globus pallidus internus stimulation may respond to
low frequencies (60 Hz to 80 Hz) while higher frequencies (130 Hz to 180 Hz) are often
used in adults.

Wilson Disease
Ronald F. Pfeiffer, MD, FAAN. Continuum (Minneap Minn). August 2016;
22(4 Movement Disorders):1246Y1261.

* 2016 American Academy of Neurology.

Copyright © American Academy of Neurology. Unauthorized reproduction of this article is prohibited.


Abstract
Purpose of Review:
This article reviews the clinical features of Wilson disease, focusing on the neurologic and
psychiatric abnormalities, and addresses the diagnostic workup and treatment approaches to
managing the disease.

Recent Findings:
The list of known mutations causing Wilson disease continues to grow, but advances in genetic
testing may soon make it feasible to routinely perform genetic testing on individuals suspected of
having Wilson disease.

Summary:
Wilson disease is a rare genetic disorder with protean manifestations that should be considered
in the differential diagnosis of any individual presenting with unexplained neurologic,
psychiatric, or hepatic dysfunction. Appropriate diagnostic testing should be expeditiously
performed and treatment promptly initiated and maintained since failure to diagnose and treat
Wilson disease will result in progressive and ultimately irreversible damage to the neurologic
and other systems.

Key Points
& Wilson disease is caused by a mutation in the ATP7B gene on chromosome 13. The
ATP7B protein is involved in the incorporation of copper into ceruloplasmin and the
transport of copper into the bile canaliculi.
& Hepatic symptoms are the initial clinical manifestation of Wilson disease in 40% to 50%
of patients, with an appearance between 11.4 to 15.5 years of age, on average.
& Neurologic dysfunction is the initial clinical manifestation of Wilson disease in 40% to
60% of patients, with a typical appearance at approximately 20 years of age.
& Basal gangliaYbased abnormalities, especially tremor and dystonia, are the most common
neurologic features of Wilson disease.
& Psychiatric features are evident at or prior to diagnosis in 30% to 40% of persons with
Wilson disease.
& Kayser-Fleischer rings may not be present in persons with Wilson disease who are
asymptomatic or who have only hepatic dysfunction; furthermore, they may occur in
other conditions characterized by hepatic dysfunction.
& Coombs-negative hemolytic anemia in combination with liver disease should always
prompt evaluation for Wilson disease.
& Wilson disease should be considered in the differential diagnosis of any young person
with unexplained hepatic, neurologic, or psychiatric dysfunction.
& Ceruloplasmin is not an adequate sole screening test for Wilson disease because it be may
normal or only slightly low in 5% to 15% of persons with Wilson disease.
& Elevated 24-hour urine copper is not pathognomonic for Wilson disease because it may
be elevated in obstructive liver diseases and in carriers of Wilson disease.
& In individuals with neurologic or psychiatric dysfunction, a combination of
Kayser-Fleischer rings, elevated 24-hour urine copper, and reduced ceruloplasmin
confirms the diagnosis of Wilson disease, and liver biopsy is not necessary.

* 2016 American Academy of Neurology.

Copyright © American Academy of Neurology. Unauthorized reproduction of this article is prohibited.


& Zinc, penicillamine, and trientine for treatment of Wilson disease should be taken on an
empty stomach.
& Penicillamine may produce paradoxical deterioration in neurologic function in
approximately 11% of patients with Wilson disease following initiation of treatment.
& Liver transplantation typically is necessary in individuals with Wilson disease who
develop acute fulminant hepatic failure. Following successful transplantation, ongoing
pharmacologic treatment is not necessary.
& In persons with Wilson disease who are still asymptomatic, zinc monotherapy is the
recommended treatment.
& In individuals with Wilson disease who have developed neurologic or psychiatric
dysfunction, copper chelation therapy with either trientine or penicillamine initially
is employed.
& Copper deficiency may develop during long-term therapy with chelating agents in
individuals with Wilson disease, and anemia may be the initial sign of copper deficiency.

* 2016 American Academy of Neurology.

Copyright © American Academy of Neurology. Unauthorized reproduction of this article is prohibited.


AUTHOR QUERY

No query.

Copyright © American Academy of Neurology. Unauthorized reproduction of this article is prohibited.


Ethical Issues

Ethics of Preclinical
Address correspondence to
Dr Thomas I. Cochrane,
Brigham and Women’s Hospital,
75 Francis St, Tower 5D EMG,
Boston, MA 02115-6110,
tcochrane@partners.org.
Relationship Disclosure:
Dopamine Transporter
Dr Cochrane serves as a
consultant for Merck & Co, Inc.
Unlabeled Use of
Imaging
Products/Investigational
Use Disclosure:
Thomas I. Cochrane, MD, MBA
Dr Cochrane reports
no disclosure.
* 2016 American Academy ABSTRACT
of Neurology.
While dopamine transporter single-photon emission computed tomography (DAT-
SPECT) imaging is sensitive and specific when performed in patients with signs or
symptoms of parkinsonism, its predictive value is uncertain in healthy subjects, even
with patients who have first-degree relatives affected by Parkinson disease. In
deciding whether to honor a patient’s request for a DAT-SPECT, neurologists must
balance a patient’s autonomy rights with beneficence and nonmaleficence and also
consider the distributive justice implications of ordering the test. Generally speaking,
the benefits of a DAT-SPECT will be too small to justify its use in an asymptomatic
patient concerned about developing Parkinson disease.

Continuum (Minneap Minn) 2016;22(4):1262–1265.

Case
A 52-year-old man presented to a neurologist because he was worried that
he might have Parkinson disease (PD). His mother had been diagnosed
with classic idiopathic PD at age 48 and developed progressively worsening
disability and experienced significant side effects of treatment. She spent
the last 5 years of her life in a nursing home as her physical and cognitive
abilities declined, and she died at age 65. She and her family considered the
last 2 years of her life of very poor quality; she had experienced pain due
to immobility and pressure sores and was frequently confused or delirious.
The patient was worried that he would also develop idiopathic PD. He
reported that his hand occasionally shook slightly when he drank caffeine,
but his neurologic examination was normal, and no tremor was noted.
The patient was only slightly reassured by hearing that his examination
was normal, and he wanted a dopamine transporter single-photon emission
computed tomography (DAT-SPECT) ‘‘just to be sure.’’ He had read that
DAT-SPECT could diagnose presymptomatic PD, and he insistently asked the
neurologist to order the test.

DISCUSSION
Dopamine transporter (DAT) imaging is a sensitive method for detecting pre-
synaptic dopamine neuronal dysfunction and can be abnormal when performed
in patients with the earliest symptoms of nigrostriatal degeneration.1 DAT
imaging cannot reliably distinguish between idiopathic Parkinson disease (PD)
and other causes of parkinsonism (eg, multiple system atrophy, corticobasal
degeneration, progressive supranuclear palsy). A normal DATYsingle-photon
emission computed tomography (SPECT) in a symptomatic patient suggests

1262 www.ContinuumJournal.com August 2016

Copyright © American Academy of Neurology. Unauthorized reproduction of this article is prohibited.


alternative diagnoses, such as essential tremor, dystonic tremor, drug-induced
parkinsonism, or a psychogenic movement disorder.2 In an asymptomatic
patient, the predictive value (negative or positive) of the test is uncertain.
The decision to order a DAT-SPECT for this patient requires the neurologist
to balance the patient’s autonomy rights with considerations of beneficence and
nonmaleficence. Although patients with decision-making capacity have the right
to make decisions that affect their health, this right to self-determination is not
an absolute right that trumps sound medical judgment. The neurologist may
use his or her best clinical judgment and refuse to offer unwarranted, unnecessary,
or potentially harmful interventions. In situations such as the present case, when
the utility of an intervention is uncertain, the neurologist should work to optimize
potential benefits (beneficence) and minimize harm (nonmaleficence) by
undertaking a careful risk-benefit analysis. Additionally, the neurologist must
consider who will pay the cost of this expensive test with small or uncertain
benefits, which is a question of the distributive justice of ordering the test. What
follows is a consideration of the risks, benefits, and distributive justice con-
siderations of ordering DAT imaging in this asymptomatic patient concerned about
preclinical PD. The results of this analysis will allow the neurologist to consider
how much weight to give to the patient’s autonomy rights.

Beneficence and Nonmaleficence


The physical risks of a DAT-SPECT are quite low. The radiopharmaceutical used
in the scan is ioflupane I-123; the radiation exposure associated with the agent
is minor (approximately 2 to 3 mSv)3 and, even with an accompanying diagnostic
CT scan (approximately 1.4 mSv), represents only a fraction of an individual’s
yearly radiation exposure from the environment.4 The benefits, therefore, would
not have to be of great magnitude to outweigh the risks. But more important than
the physical risks of the scan itself are the potential risks and benefits of the results.
A normal result is likely to reassure the patient, which is the very benefit the
patient is hoping to achieve. However, while a DAT-SPECT is highly sensitive in
patients with early symptomatic PD, the negative posttest probability of DAT-
SPECT for preclinical PD is unknown.5,6 A normal DAT-SPECT, therefore, would
not constitute strong evidence that the patient is not going to develop PD, so
any reassurance would be false reassurance with great potential for the patient
to misapprehend.
The psychological impact will depend very much on the individual patient’s
psychologic makeup, beliefs, and values. Depending on the patient, a positive
(abnormal) result could be viewed as either a harm or a benefit. A positive
result, while not absolutely diagnostic or prognostic, might reasonably be in-
terpreted as suggesting an increased likelihood that the patient will develop PD.
In turn, this news, however uncertain, might be a source of great distress for the
patient, given that neither preventative nor neuroprotective therapy is currently
available. This knowledge burden would serve as an argument against performing
the test. On the other hand, the patient might be grateful for any additional
knowledge about his medical future, providing an increased impetus to prepare
financially and socially. A physician would have to know more about the individual
patient before deciding whether a positive result is a harm to be avoided or a
benefit to be pursued. The neurologist’s skill in managing prognostic uncertainty
will also affect how the test results are conveyed to the patient.
Continuum (Minneap Minn) 2016;22(4):1262–1265 www.ContinuumJournal.com 1263

Copyright © American Academy of Neurology. Unauthorized reproduction of this article is prohibited.


Dopamine Transporter Imaging

Perhaps more concretely, a positive result might tempt the patient to begin
therapies for PD before they are medically indicated, thus incurring the burden
of side effects without any compensating benefit. To the extent that these
therapies are medications requiring prescription, this risk is avoidable as long as
the patient’s clinicians understand the proper indications and advise the patient
appropriately. Some patients, however, might be tempted to seek out risky
unproven or alternative therapies. Naturally, this would be inadvisable even
after the patient developed clinically evident PD, and so it is not a particular risk
of the DAT-SPECT itself. However, a positive scan might prompt such a patient
to incur the risks of unproven or dangerous therapies earlier than he or she
otherwise would have.
In order to properly weigh these risks and benefits, the neurologist would
have to engage in thoughtful deliberation with the patient and consider the
following: Why exactly does the patient want the test? What will the patient do
with a positive result? How will the patient react emotionally? What if the result
is negative? Does the patient understand the uncertainty of interpreting the
results in an asymptomatic patient?
Even if the patient and the neurologist both agree that the minimal risks of
the test do not outweigh the minimal benefits, the physician must still consider
the cost of the test not only to the patient, but to society as a whole.

Justice Considerations
Spending resources on one patient that impacts other patients or society in
general implicates distributive justice. In this case, would it be just to spend
$2500 to $5000 on a DAT-SPECT for this patient, assuming scanner time is not
scarce and the time this patient must spend in the scanner will not come at the
expense of another patient who must access the machine? The answer to this
question will depend entirely on where the money comes from and whether
the money spent on this test will limit the availability of testing or therapies for
other patients.
At one extreme, under a libertarian conception of distributive justice, the
analysis is straightforward: If the patient were willing and able to pay the entire
expense, then there would not be a compelling distributive justice argument
against ordering this specific test. At another extreme, the analysis might be
made by a third party payer such as a private insurer or a government insurer
(eg, Medicare or Medicaid) who is concerned with either an egalitarian (everyone
gets the same benefits) approach to distributive justice and is willing to pay for
tests only where good evidence supports the test’s utility.
It is quite possible, however, that the question of distributive justice will not
be settled for either the neurologist or the patient, and thus some analysis will
be required. It is possible that an insurer would be willing to pay, especially if
the neurologist were willing to appeal an initial denial since the neurologist can
argue that the patient has a first-degree relative with PD as well as a tremor. The
insurance reviewer may not have the expertise to realize that the reported tremor
is most probably a completely normal physiologic tremor.
If the insurer will pay for the test under these conditions, the neurologist
risks violating an ethical and legal obligation to be honest even when advocating
for a patient. From a distributive justice perspective, the benefits of the test for
this patient are very small, so obligating a governmental insurer to pay for this
1264 www.ContinuumJournal.com August 2016

Copyright © American Academy of Neurology. Unauthorized reproduction of this article is prohibited.


test is not supported by either egalitarian or ‘‘basic decent minimum’’ (everyone
gets a certain level of health care and anything above this requires the patient to
pay out of pocket) conceptions of distributive justice. On the other hand, in the
United States, where the total pool of resources to be spent on health care is
not fixed but instead open, it is difficult for an individual physician to deny his
or her patient a beneficial test (even an only slightly beneficial test) on the
grounds that the expense is unjustified, especially if the patient is willing to pay
beyond the ‘‘basic decent minimum’’ covered by insurance.

CONCLUSION
To summarize, the neurologist must weigh a number of ethical appeals and must
weigh autonomy against beneficence, nonmaleficence, and distributive justice in
deciding whether to accede to the patient’s request and order the DAT-SPECT in
this asymptomatic patient with a family history of PD. This process of weighing
ethical appeals requires the physician to consider the following questions:
1. Does this patient understand the difficulty in interpreting the results of the
scan in an asymptomatic patient?
2. How will the patient react psychologically to either a positive or a negative
result?
3. What will the patient do with a positive result?
4. How likely is the patient to seek out inappropriate, risky, or unproven
treatments if the result is positive?
5. Based on the answers to questions 1 through 4, is the test likely to be more
beneficial than harmful?
6. Should distributive justice play a role in the neurologist’s decision to order
the test? Does it matter if the patient can afford to pay for the test out of
pocket rather than have insurance pay for all or part of the test?
Generally speaking, the benefits of DAT-SPECT in this scenario will be too
small to justify ordering the test in the first place. However, a scenario is possible in
which the patient and the neurologist have carefully weighed benefits, risks, and
costs, and decide that the scan is justified.

REFERENCES
1. Kägi G, Bhatia KP, Tolosa E. The role of DAT-SPECT in movement disorders. J Neurol Neurosurg
Psychiatry 2010;81(1):5Y12. doi:10.1136/jnnp.2008.157370.
2. Sadasivan S, Friedman JH. Experience with DaTscan at a tertiary referral center. Parkinsonism
Relat Disord 2015;21(1):42Y45. doi:10.1016/j.parkreldis.2014.10.022.
3. DaTscan Ioflupane I123 Injection. GE Healthcare web site. us.datscan.com/wp-content/uploads/
2014/06/prescribing-information.pdf. Updated April 2011. Accessed June 14, 2016.
4. Radiation Dose Calculator. American Nuclear Society web site. www.ans.org/pi/resources/
dosechart. Updated February 8, 2016. Accessed June 14, 2016.
5. Hauser RA, et al. Sensitivity, specificity, positive and negative predictive values and diagnostic
accuracy of DaTscan (ioflupane I123 injection): Predicting clinical diagnosis in early clinically
uncertain parkinsonian syndrome. J Neurol Stroke 2014;1(1):1Y13. doi:10.15406/
jnsk.2014.01.00003.
6. Thiriez C, et al. Clinical routine use of dopamine transporter imaging in 516 consecutive patients.
J Neurol 2015;262(4):909Y915. doi:10.1007/s00415-014-7634-y.

Continuum (Minneap Minn) 2016;22(4):1262–1265 www.ContinuumJournal.com 1265

Copyright © American Academy of Neurology. Unauthorized reproduction of this article is prohibited.


Practice Issues

Chemodenervation
Address correspondence to
Dr Allan D. Wu, 710
Westwood Plaza, Reed
Neurological Research Center
A-153, University of California,
Los Angeles, Los Angeles,
CA 90095,
Coding for Neurologists
allanwu@mednet.ucla.edu.
Allan D. Wu, MD; Dawn Eliashiv, MD, FAAN;
Relationship Disclosure:
Dr Wu has received personal Marc Nuwer, MD, PhD, FAAN
compensation for serving on
the advisory board of Merz
Pharma Group and received
research/grant support as
co-investigator of a study for
the Dr Miriam & Sheldon G.
Accurate coding is an important function of neurologic practice. This contribution to
Adelson Medical Research Continuum is part of an ongoing series that presents helpful coding information
Foundation. Dr Eliashiv along with examples related to the issue topic. Tips for diagnosis coding, Evaluation
receives personal
compensation for serving on
and Management coding, procedure coding, or a combination are presented,
the speaker’s bureau of depending on which is most applicable to the subject area of the issue.
Cyberonics, Inc; Sunovion
Pharmaceuticals, Inc; and
UCB, Inc and receives
research/grant support as
co-investigator of a study by INTRODUCTION
the Defense Advanced
Research Projects Agency. Significant changes in Current Procedural Terminology (CPT) codes were in-
Dr Nuwer receives
research/grant support as
troduced on January 1, 2014, in which procedure codes for chemodenervation
investigator of studies for the with botulinum neurotoxins were revised. The uses for botulinum neurotoxins
Congressionally Directed has continued to expand, raising risks that patients may be offered or be ad-
Medical Research Programs
(SC130209), the National ministered botulinum neurotoxins from multiple providers for multiple diag-
Institutes of Health/National noses. This article provides information regarding these CPT changes up to the
Institute of Neurological
Disorders and Stroke most recent 2016 CPT edition.1,2
(1R01NS078494-01A1), and
the United States Army EVALUATION AND MANAGEMENT OF MULTIPLE INDICATIONS FOR
Medical Research Acquisition
Activity (11501944). Dr Nuwer CHEMODENERVATION
has received personal
compensation for book
Inquiring carefully about a patient’s prior experience with botulinum neurotoxins
royalties from Cambridge is becoming more important as the use of botulinum neurotoxins continues to
University Press and has expand. Coding Table 1 shows the US Food and Drug Administration (FDA)
provided expert medical
testimony for trials and indications for botulinum neurotoxins as of June 2016. Neurologists frequently
depositions as a medical treat both on- and off-label diagnoses, including hemifacial spasm, focal dystonia,
expert and treating physician.
Unlabeled Use of
spasticity, and sialorrhea. In addition to well-known cosmetic purposes, additional
Products/Investigational off-label uses of botulinum neurotoxins include gastrointestinal indications (eg,
Use Disclosure: achalasia or gastroparesis), urologic indications (eg, spastic bladder or neurogenic
Drs Wu, Eliashiv, and Nuwer
discuss the unlabeled/ bladder), and pain indications (eg, neurogenic pain, back pain, or temporoman-
investigational use of dibular joint syndrome).3 Patients may not recognize that the botulinum neuro-
botulinum neurotoxins as a
generic biologic agent that toxins used for wrinkles or offered for bladder incontinence may be similar (or
can be used with appropriate identical) to the botulinum neurotoxins used by neurologists. Furthermore, be-
medication by physicians
across all of these indications cause these toxins may be provided by the clinic and may not be prescribed
based on labeling approved through standard outpatient pharmacies, documentation in some electronic med-
by the US Food and
Drug Administration.
ical records may not list botulinum neurotoxins in standard ambulatory patient
* 2016 American Academy medication lists. All of this underscores the need to carefully ask the patient not
of Neurology. only about past but current use of botulinum neurotoxins. Even within a neu-
rologic practice, patients may start to receive botulinum neurotoxins for multiple
indications, examples of which include patients with focal dystonia who also have
migraine, patients with dystonia syndromes and sialorrhea, or patients with cra-
niocervical dystonia who may receive botulinum neurotoxin from the neurologist
1266 www.ContinuumJournal.com August 2016

Copyright © American Academy of Neurology. Unauthorized reproduction of this article is prohibited.


CODING TABLE 1 Toxins Approved by the US Food and Drug Administrationa

OnabotulinumtoxinA IncobotulinumtoxinA AbobotulinumtoxinA RimabotulinumtoxinB


Condition (J0585) (J0588) (J0586) (J0587)
Cervical dystonia X X X X
Blepharospasm X X
Hemifacial X
spasm
Upper limb X X X
spasticity
Lower limb X
spasticity
Chronic X
migraineb
Primary axillary X
hyperhidrosis
Glabellar lines X X
Overactive X
bladder
Urinary X
incontinence
due to detrusor
overactivity with
neurologic
condition
Strabismus X
Sialorrheac
Focal dystonia
(other than
cervical
dystonia and
blepharospasm)c
a
Table lists toxins approved by the US Food and Drug Administration (FDA) as of June 2016.
b
Chronic migraine occurs Q than 15 days per month with headache lasting 4 hours a day or longer.
c
No toxins are FDA-approved for sialorrhea and focal dystonia (other than cervical dystonia and blepharospasm).

for cervical dystonia and also from a head and neck consultant for spas-
modic dysphonia.
To date, no guidelines exist regarding best practice on how to handle bot-
ulinum neurotoxin injections that are used for multiple indications for the same
patient, especially when given by more than one provider. Obtaining records of
previous botulinum neurotoxin procedures with brand used and dose given is
always good practice. In addition, to reduce risk of overlapping systematic side ef-
fects of additive doses at given times, efforts can be made to assure that botulinum
neurotoxin injections by different providers will be given around the same time
and to ensure that the overall dose given is appropriate. Doses used for migraine

Continuum (Minneap Minn) 2016;22(4):1266–1275 www.ContinuumJournal.com 1267

Copyright © American Academy of Neurology. Unauthorized reproduction of this article is prohibited.


Chemodenervation Coding

(onabotulinumtoxinA 155 units or higher), overactive bladder (onabotulinumtoxinA


200 units to 300 units), and achalasia (usually onabotulinumtoxinA 100 units) can
be rapidly additive for a given patient.4 Furthermore, since usual effects of neu-
rotoxin commonly last up to 3 months, repeat injections for different indications
at different time points within a given 3 month period can confound inter-
pretation of effects and put patients at potentially greater risk for side effects. The
authors recommend educating patients during the consent process about the
possibility of multiple botulinum neurotoxin exposures that overlap in time; if
multiple injections of botulinum neurotoxins are unavoidable (ie, administered
by different providers), encouraging scheduling of the multiple injections to occur
all within a week of each other can help track the total dose given per injection
cycle and help determine dose-related beneficial effects and possible side effects.
Currently, there are four botulinum neurotoxins in use (Coding Table 2),
each with its own set of indications and dosage recommendations. The units
dosage of one toxin type are not equivalent and not interchangeable with the units
dosage of another toxin type. As such, when patients receive botulinum neuro-
toxin for more than one indication, keeping to one type of toxin is advisable.
Documentation of the total estimated doses that the patient has been given for
each cycle of injection should be done. Some insurance providers will provide
authorization for only one botulinum neurotoxin injection provider at a time.
Double checking insurance coverage for a given injection when a patient has
received botulinum neurotoxin for another indication is often advisable.
Communication with all toxin providers is important so all remain on the same
page. Additionally, providing a written note for the patient informing him or her of
the date and amount of the toxin injection is often helpful.

CODING TABLE 2 Botulinum Neurotoxins and Dosage Information

Selected Dosage Informationa


Toxin Name HCPCS code Billing Units (Quantity) (From Prescribing Information)
OnabotulinumtoxinA J0585 1 unit toxin = 1 billing unit Do not exceed a total dose of
400 units administered in a
3-month interval5
AbobotulinumtoxinA J0588 5 units toxin = 1 billing unit Doses above 1000 units have not
been systematically evaluated6
IncobotulinumtoxinA J0586 1 unit toxin = 1 billing unit Recommended maximum cumulative
dose for any indication should not
exceed 400 units in a treatment
session7
RimabotulinumtoxinB J0587 100 units toxin = 1 billing unit For cervical dystonia, duration of
effect has been observed in studies to
be between 12 and 16 weeks at doses
of 5000 units or 10,000 units8

HCPCS = Healthcare Common Procedure Coding System.


a
Maximum doses noted here are based on specific indications and do not apply to most indications, which may have lower recom-
mended doses. This information is provided to guide monitoring of maximum doses of a given toxin that a patient may receive across all
indications for a given treatment cycle.

1268 www.ContinuumJournal.com August 2016

Copyright © American Academy of Neurology. Unauthorized reproduction of this article is prohibited.


DIAGNOSES FOR CHEMODENERVATION PROCEDURES
All diagnosis coding was converted from the International Classification of
Diseases, Ninth Revision, Clinical Modification (ICD-9-CM) to the International
Classification of Diseases, Tenth Revision, Clinical Modification (ICD-10-CM)
on October 1, 2015. Coding Table 3 lists ICD-10-CM codes for diagnoses
commonly associated with chemodenervation procedures.9,10 Individual insur-
ance companies and contractors from the Centers for Medicare & Medicaid
Services will have their own lists of botulinum neurotoxin procedureYapproved
diagnoses codes and criteria for use. ICD-10-CM codes carry a much greater
specificity than ICD-9-CM, with the fourth through seventh characters specify-
ing etiology, anatomic site, and specificity of the base category code (first
through third characters). In August 2015, Medicare announced a 12-month
grace period during which no claims will be denied ‘‘based solely on the
specificity of the ICD-10-CM diagnosis code as long as the physician/
practitioner used a valid code from the right family.’’11 The grace period
will end October 1, 2016.

DISCUSSION OF CURRENT PROCEDURAL TERMINOLOGY CODES


SINCE JANUARY 2014
CPT codes related to chemodenervation procedures underwent major revision
on January 1, 2014. The key relevant changes are summarized in Coding Table 4.
Two CPT codes for chemodenervation of neck muscles (64613) and for truncal/
extremity muscles (64614) were deleted.
The cervical chemodenervation code is now replaced with two separate codes:
chemodenervation of cervical muscles, excluding laryngeal muscles (64616), and
chemodenervation of laryngeal muscles (64617). Both new codes are unilateral,
and modifier 50 may be appended if the injection involves muscles on both
sides. If EMG or electrical stimulation guidance is used for cervical muscle
chemodenervation, add-on codes for EMG guidance (95874) or electrical stimula-
tion guidance (95873) may be added to CPT code 64616. The language of CPT
code 64617 includes EMG guidance, so if only 64617 is being done, no added
EMG guidance code is allowed.
The limb chemodenervation code has been replaced by a set of four limb
codes and two truncal codes (Coding Table 4). For limb injections, the first
code billed must be a base code: either 64642 (if the first limb had one to four
muscles injected) or 64644 (if the limb had five or more muscles injected). The
base code is always billed as one unit, and only one base code is billed for a
given session. Subsequent limbs are billed with an add-on code, either 64643
(if the subsequent limb had one to four muscles injected) or 64645 (if the
subsequent limb had more than five muscles injected). If there is more than
one subsequent limb injected (not counting the base limb), the units of the
add-on limb code are incremented. Code 64643 can be used if either base code
64642 or 64644 is billed first; however, code 64645 only can be used if base
code 64644 is billed first. Truncal codes are now either 64646 (for one to five
truncal muscles) or 64647 (if more than five truncal muscles are injected).
Truncal muscles are defined as erector spinae and paraspinal muscles, rectus
abdominis, and obliques. Either 64646 or 64647, but not both, may be billed for
a given patient. No modifier 50 (bilateral) is allowed for any of these limb or
extremity codes.
Continuum (Minneap Minn) 2016;22(4):1266–1275 www.ContinuumJournal.com 1269

Copyright © American Academy of Neurology. Unauthorized reproduction of this article is prohibited.


Chemodenervation Coding

CODING TABLE 3 Common ICD-10-CM Diagnosis Codes Used for


Botulinum Neurotoxina

Common Movement Disorders Diagnoses


ICD-10-CM Description
G24.01 Drug-induced subacute dyskinesia
G24.02 Drug-induced acute dystonia
G24.1 Genetic torsion dystonia
G24.3 Spasmodic torticollis (used for cervical dystonia)
G24.4 Idiopathic orofacial dystonia (often used for cranial dystonia
such as oromandibular dystonia)
G24.5 Blepharospasm
G24.8 Other dystonia
G24.9 Dystonia, unspecified
G51.3 Clonic hemifacial spasm
G51.8 Other disorders of facial nerve
J38.5 Laryngeal spasm
R49.0 Dysphonia
Other Diagnoses in Neurology
G43.909 Migraine, unspecified, not intractable, without status migrainosus
G43.919 Migraine, unspecified, intractable, without status migrainosus
G80.1 Spastic diplegic cerebral palsy
G80.3 Athetoid cerebral palsy
G81.1* Spastic hemiplegia affecting * (where * is 1=right dominant
side, 2=left dominant, 3=right nondominant, 4=left
nondominant, nondominant, 0=unspecified)
G82.2* Paraplegia (paraparesis), * (where * is 1=complete, 2=incomplete,
0=unspecified)
G82.5* Quadriplegia, * (where * is 1=C1-C4 complete, 2=C1-C4 incomplete,
3=C5-C7 complete, 4=C5-C7 incomplete, 0=unspecified)
I69.*5† Hemiplegia and hemiparesis following * (where * is
0=nontraumatic subarachnoid hemorrhage,
1=intraparenchymal hemorrhage, 3=cerebral infarction,
8=other cerebrovascular disease, 9=unspecified; where † are
laterality codes 1Y4 same as G81.1† or 9=unspecified)
K11.7 Sialorrhea
L74.51* Primary focal hyperhidrosis, * (where * is 0=axilla, 1=face,
2=palms, 3=soles, 9=unspecified)
a
This table lists some common International Classification of Diseases, Tenth Revision, Clinical Modification
(ICD-10-CM)10 codes that are associated with diagnoses associated with botulinum neurotoxin
injections. These codes do not represent approved billable codes for botulinum neurotoxin. Please
check with specific payers for details on which codes are considered approved for that payer.

1270 www.ContinuumJournal.com August 2016

Copyright © American Academy of Neurology. Unauthorized reproduction of this article is prohibited.


CODING TABLE 4 Current Procedural Terminology (CPT) Codesa

CPT Code Comment Description


Retired CPT code (prior to January 2014)
64613 Used for cervical dystonia (spasmodic Chemodenervation of muscle(s); neck
torticollis) muscle(s), excluding muscles of the
larynx, unilateral
64614 Used for trunk and limb injections Chemodenervation of muscle, extremity
and/or trunk muscle
Current CPT code (as of January 2014)
64616 Use for cervical dystonia (spasmodic Chemodenervation of neck muscles(s),
torticollis) excluding muscles of the larynx, unilateral
(eg, for cervical dystonia, spasmodic torticollis)
64612 Use for blepharospasm, hemifacial Chemodenervation of muscle(s); muscle(s)
spasm innervated by facial nerve, unilateral (eg,
for blepharospasm, hemifacial spasm)
64615 Use for migraine treatments Chemodenervation of muscle(s) innervated
by facial, trigeminal, cervical spinal,
and accessory nerves, bilateral (eg, for
chronic migraine)
64642 Base code for extremity injections (use Chemodenervation of one extremity; one
either 64642 or 64644 per session) to four muscle(s)
64643 Use in conjunction with 64642 or 64644 Each additional extremity, one to four
muscle(s) (List separately in addition to
code for primary procedure)
64644 Base code for extremity injections Chemodenervation of one extremity; five
(use either 64642 or 64644 per or more muscles
session)
64645 Only used in conjunction with 64644 Each additional extremity, five or more
muscles (List separately in addition to code
for primary procedure)
64646 Use either 64646 or 64647 per session Chemodenervation of trunk muscles(s); one
to five muscle(s)
64647 Use either 64646 or 64647 per session Chemodenervation of the trunk muscle(s);
six or more muscles
a
CPT B 2016 American Medical Association. All rights reserved. CPT is a registered trademark of the American Medical Association.

The other two codes worth noting are 64612 for unilateral facial nerve inner-
vated muscle chemodenervation and 64611 for bilateral salivary gland (parotid and
submandibular glands) chemodenervation. Code 64612 is usually used for ble-
pharospasm, hemifacial spasm, oromandibular dystonia, or facial dystonia. Modi-
fier 50 (bilateral procedure) may be applied to code 64612 if injections are done in
muscles on both sides of facial midline. On the other hand, code 64611 specifies
bilateral injections and no modifier 50 is allowed.
Finally, although beyond movement disorders, it is worth noting there is a new
dedicated code for the migraine chemodenervation protocol (64615). This code

Continuum (Minneap Minn) 2016;22(4):1266–1275 www.ContinuumJournal.com 1271

Copyright © American Academy of Neurology. Unauthorized reproduction of this article is prohibited.


Chemodenervation Coding

covers bilateral injections of muscles innervated by facial, trigeminal, cervical, and


spinal accessory nerves. Only one unit of the code may be billed per session, and
it is not expected to be used with any other chemodenervation CPT code. No
modifier 50 is allowed.
In most cases, chemodenervation CPT codes are not expected to be billed
with an Evaluation and Management (E/M) code on the same day by the same
provider. If the procedure is performed on the same day as E/M work, then
modifier 25 must be added to the E/M code to indicate that a procedure was
performed separate from that E/M activity. The separate clinical activity must be
clearly documented.

Case 1
A 60-year-old woman presented with a 1-year history of head tremor along
with symptoms of head pulling toward the right. The tremor improved with
a sensory trick, and tremor was reduced with the head turned toward the
right. At the initial evaluation and management visit, a right rotational cervical
dystonia was diagnosed, and botulinum neurotoxin chemodenervation
therapy was recommended. For the subsequent procedure visits,
onabotulinumtoxinA was injected (150 units total), distributed into the
left sternocleidomastoid, right splenius capitis, and left splenius capitis
muscles. Because two 100-unit vials were used (or one 200-unit vial), 50 units
remain within the vials, which were discarded by the end of the day. To guide
injections, EMG guidance was used.

DISCUSSION
Coding for the procedure discussed in Case 1 would include a single unit of 64616
(chemodenervation of cervical muscles, excluding larynx, unilateral). Modifier 50
(bilateral) can be applied because muscles injected were on both sides of mid-
line. For EMG guidance, one unit of 95874 is billed, regardless of the number of
muscles EMG was used for. To bill for the botulinum neurotoxin medication,
Healthcare Common Procedure Coding System (HCPCS) code J0585 (for onabo-
tulinumtoxinA) is billed with 150 units used and 50 units discarded to account for
all 200 units used. If the patient has Medicare as the primary insurer, modifier GA
should be applied to J0585 to indicate that the patient was informed of the fact
that Medicare will cover 80% of the amount specified in the Medicare fee schedule
(as documented by a signed advance beneficiary notice form).

Case 1 Continued
The patient continued to receive 150 units of onabotulinumtoxinA every
3 months for cervical dystonia. The patient also had a history of chronic
migraine headaches, which became refractory to medications over time.
A second opinion by a neurologist outside the practice recommended
botulinum neurotoxin therapy for chronic migraine headaches.

DISCUSSION
If the patient wished to proceed with both chemodenervation procedures (cervical
dystonia procedure and the proposed migraine procedure), full informed consent

1272 www.ContinuumJournal.com August 2016

Copyright © American Academy of Neurology. Unauthorized reproduction of this article is prohibited.


and revised risk benefits would need to be discussed with the patient and with all
providers, given the increase in dose of toxin needed to adequately treat both
diagnoses. Communication between the two neurologists is essential to ensure
that the total doses used in the one patient are appropriate and worth potential
risks and costs versus the potential benefits.
There are two options to consider if the patient wishes to receive both chem-
odenervation procedures: either both procedures are done by each respective
neurologist, or the two procedures are performed by a single neurologist ex-
perienced in both procedures. For the former option, each neurologist would
provide his or her own procedure and each office can work with the other to
ensure that (1) the procedures occur within the same time frame (so the botu-
linum neurotoxin cycle is in sync) and (2) appropriate preauthorization is obtained
for both toxin and procedure for each clinician providing the respective service.
Billing is straightforward for each provider. The latter option is more convenient
for the patient and consolidates both injections in one visit with one provider. In
this case, billing is more complex, and reimbursement details will usually be de-
pendent on the local payers involved. Assuming the latter option, the neurologist
might first adjust doses to account for both procedures.

Case 1 Continued
Respecting typical maximum doses of onabotulinumtoxinA approved for
cervical dystonia, 300 units of onabotulinumtoxinA were provided for the
consolidated procedures (from one vial of 200 units plus one vial of 100 units
or three vials of 100 units). Dilution was performed carefully to ensure that
concentration and doses given were correct. For the chronic migraine
indication, 155 units total were given to frontalis, temporalis, occipitalis,
trapezius, and upper cervical paraspinals. For cervical dystonia,
120 units were given to the bilateral splenius and left sternocleidomastoid
muscles. For onabotulinumtoxinA, 275 units of J code J0585 were billed as
used and 25 units were billed as discarded for a total of 300 units within vials
available for the procedure.
For cervical dystonia, CPT code 64616 was coded once (with modifier
50 for bilateral) as before. For refractory episodic migraine, CPT code
64615 was coded once (and automatically accounts for the bilateral nature
of the injections). Because 64616 and 64615 are usually not expected to
be billed at the same time, modifier 51 was added to describe an
additional procedure being done by the same provider at the same session.

DISCUSSION
The first CPT procedure listed will typically be the more resource-intensive pro-
cedure and the subsequent, separate CPT procedure will have modifier 51 attached
to it. Because modifier 51 generally indicates greater than usual resource utiliza-
tion, careful documentation and preauthorization with payers is recommended.

Continuum (Minneap Minn) 2016;22(4):1266–1275 www.ContinuumJournal.com 1273

Copyright © American Academy of Neurology. Unauthorized reproduction of this article is prohibited.


Chemodenervation Coding

Case 2
A 47-year-old man with cerebral palsy presented with progressive spasticity
and dystonic posturing of the lower and upper extremities in spite of
medication treatment. Botulinum neurotoxin chemodenervation therapy
was recommended and repeated every 3 months while doses were adjusted
based on response to each therapy session. For the first procedure visit, the
patient received onabotulinumtoxinA into three muscles in the right leg
(tibialis posterior, gastrocnemius, and soleus) and into the same three muscles
of the left leg.
Three months later, assessment indicated that the leg injections were
helpful in temporarily reducing painful spasms and inversion-plantar flexion
dystonic posturing. Right arm flexion posture of wrist and pronation of
the forearm was becoming more functionally symptomatic. For this second
procedure visit, the patient received the same injections in the legs, but
additional botulinum neurotoxin was injected into muscles in the right arm
(flexor digitorum superficialis, pronator teres, flexor carpi radialis, and flexor
carpi ulnaris for a total of four muscles).
Three months after the previous visit involving injections to three limbs,
responses to injections in the lower extremities remained stable and helpful.
The right arm procedure done at the last visit was moderately helpful, and
right elbow flexion, which had not yet been a target for chemodenervation,
was felt to be an additional symptomatic contributor to uncomfortable right
arm spasticity. As such, in this third procedure visit, both lower extremities
were injected as in the previous visit, but the right arm received injections
into six total muscles (flexor digitorum superficialis, pronator teres, flexor
carpi radialis, flexor carpi ulnaris, plus the biceps brachii and brachioradialis).

DISCUSSION
For the first procedure visit, in which the patient received onabotulinumtoxinA
into three muscles on each leg, the base code would be 64642 (one extremity,
one to four muscles) and a second additional code would be added with 64643
(additional extremity, one to four muscles). Each code would be billed with
quantity one for a total of two limbs.
For the second procedure visit, all three limbs were injected in one to four
muscles so the two CPT codes billed would be the same as in the previous visit.
To account for the additional injection in the right arm, the physician would
increment the quantity of the add-on code (64643) and would bill one unit for
the base code (64642) and two units of the additional extremity code (64643) to
account for all three limbs. If the left arm had also been injected with four or fewer
muscles (for four total limbs), then 64643 would be billed with quantity three.
In the third patient visit, the base code billed would be 64644 (one extremity,
five or more muscles) using the more resource-intensive base code (right arm with
six muscles). Note that the two base codes 64642 and 64644 cannot be used
together on the same day. To code for the two legs with three muscles each,
code two units of the add-on code 64643. This accounts for all three limbs, one
at the higher muscle count and two at the lower count.
All of these limb codes (and trunk code 64646) are considered valid when
used within one procedural session, and there is no need to consider any

1274 www.ContinuumJournal.com August 2016

Copyright © American Academy of Neurology. Unauthorized reproduction of this article is prohibited.


modifier 51 to indicate separately performed procedures. If EMG or electrical
stimulation is used for any of the procedures above, it is always billed with one
unit of 95874 (EMG) or 95873 (electrical stimulation) regardless of number of
muscles or limbs injected.

CONCLUSION
This article highlighted three contemporary issues regarding proper chemo-
denervation coding for movement disorder indications and other indications of
interest to neurologists such as spasticity and chronic migraine. First, the article
discussed and provided advice for dealing with the possibility of patients re-
ceiving botulinum neurotoxin for multiple indications at the same time. Some-
times, these procedures are done by separate providers and sometimes they can
be consolidated in one office. Careful documentation and coding in these sit-
uations should be done carefully. Second, common ICD-10-CM diagnoses as-
sociated with chemodenervation were discussed. All submitted diagnoses to
Medicare now require ICD-10-CM codes since October 2015 with virtually all
payers following suit. Finally, the article discussed CPT chemodenervation
code revisions since January 2014 and provided examples of their use.

REFERENCES
1. American Medical Association.Current procedural terminology (CPT) 2014. Chicago: American
Medical Association Press, 2014.
2. American Medical Association.Current procedural terminology (CPT) 2016. Chicago: American
Medical Association Press, 2016.
3. Chen JJ,Dashtipour K. Abo-, inco-, ona-, and rima-botulinum toxins in clinical therapy: a primer.
Pharmacotherapy 2013;33(3):304Y318. doi:10.1002/phar.1196.
4. Stavropoulos SN, Friedel D, Modayil R, et al. Endoscopic approaches to treatment of
achalasia.Therap Adv Gastroenterol 2013;6(2);115Y135. doi:10.1177/1756283X12468039.
5. Allergan. Highlights of prescribing information for onabotulinumtoxinA. Allergan website.
www.allergan.com/assets/pdf/botox_pi.pdf. Updated January 2016. Accessed June 17, 2016.
6. Allergan. Highlights of prescribing information for abobotulinumtoxinA. Allergan website.
www.dysport.com/pdf/Dysport%20Full%20Prescribing%20Information.pdf. Updated July 2015.
Accessed June 17, 2016.
7. Allergan. Highlights of prescribing information for incobotulinumtoxinA. Allergan website.
www.xeomin.com/wp-content/uploads/xeomin-full-prescribing-information.pdf. Updated December
2015. Accessed June 17, 2016.
8. Myobloc. Highlights of prescribing information for rimabotulinumtoxinB. Solstic Neuroscience, Inc
website. www.myobloc.com/hp_about/PI_5-19-10.pdf. Updated May 2010. Accessed June 17, 2016.
9. Centers for Medicare & Medicaid Services, National Center for Health Statistics.International
classification of diseases, ninth revision, clinical modification (ICD-9-CM). www.cdc.gov/nchs/icd/
icd9cm.htm. Updated June 18, 2013. Accessed June 17, 2016.
10. Centers for Medicare & Medicaid Services, National Center for Health Statistics.International
classification of diseases, tenth revision, clinical modification (ICD-10-CM). www.cdc.gov/nchs/icd/
icd10cm.htm. Updated October 29, 2015. Accessed June 17, 2016.
11. CMS and AMA Announce Efforts to Help Providers Get Ready For ICD-10 Frequently Asked
Questions. www.cms.gov/Medicare/Coding/ICD10/Downloads/ICD-10-guidance.pdf. Accessed
June 17, 2016.

Continuum (Minneap Minn) 2016;22(4):1266–1275 www.ContinuumJournal.com 1275

Copyright © American Academy of Neurology. Unauthorized reproduction of this article is prohibited.


Self-Assessment and CME

Postreading
Self-Assessment and
CME Test—Preferred
Responses
James W. M. Owens Jr, MD, PhD; Joseph E. Safdieh, MD, FAAN

Following are the preferred responses to the questions in the Postreading


Self-Assessment and CME Test in this Continuum issue. The questions and
answer options are repeated, and the preferred response is given, followed
by an explanation and a reference with which you may seek more specific
information. You are encouraged to review the responses and explanations
carefully to evaluate your general understanding of the course material.
The comments and references included with each question are intended
to encourage independent study.
Participants who complete the Postreading Self-Assessment and CME Test
and issue evaluation online at www.aan.com/continuum/cme may earn up
to 12 AMA PRA Category 1 Creditsi toward SA-CME. Participants have up
to 3 years from the date of publication to earn CME credits. No SA-CME
will be awarded for this issue after August 31, 2019.

b 1. A 9-year-old boy presents for evaluation of abnormal involuntary movements.


His mother reports that he started developing odd movements in his arms
and face 1 week ago and also became irritable and anxious. He had a sore throat
and fever 4 weeks ago and was treated with over-the-counter supplements.
No family history exists of any neurologic disorder. On examination, he is afebrile
with frequent choreiform movements in his limbs and face. The reminder of
the neurologic examination is normal. What is the most likely diagnosis?
A. diphtheria
B. Huntington disease
C. N-methyl-D-aspartate (NMDA) receptor encephalitis
D. subcortical cerebral infarct
E. Sydenham chorea

Continuum (Minneap Minn) 2016;22(4):1295–1320 www.ContinuumJournal.com 1295

Copyright © American Academy of Neurology. Unauthorized reproduction of this article is prohibited.


Postreading Test—Preferred Responses

The preferred response is E (Sydenham chorea). The patient is a child


who presents a few weeks after a pharyngeal infection with chorea and
neurobehavioral changes. This is most consistent with Sydenham chorea,
part of the clinical syndrome of rheumatic fever. Diphtheria can also cause
pharyngitis but would not be expected to cause chorea, although it can cause
a neuropathy. Huntington disease is unlikely to be this acute. An infarct
can cause acute abnormal movements, but is unlikely to be bilateral and
associated with behavioral changes. NMDA receptor encephalitis can cause
any abnormal movement but would be expected to have more associated
encephalitis signs and symptoms. For more information, refer to page 1165
of the Continuum article ‘‘Movement Disorders Presenting in Childhood.’’

b 2. A 55-year-old right-handed man presents with a 1-year history of tremor


and increasingly frequent falls. The tremor has always been more noted in
his right hand, is approximately 5 Hz, and is present at rest and decreases
with activity. The falls, which began around the same time that he noted the
tremor, are not preceded by any presyncopal symptoms, occur ‘‘randomly,’’
and are not caused by tripping over objects or other obvious explanations. The
only other symptoms he endorses are some decrease in his sense of smell
and chronic constipation, both of which began a few years before the tremor.
He has otherwise been healthy with no chronic medical problems. No one in his
immediate family has a history of tremor. His examination is significant for
the right hand tremor, which is present at rest and which reemerges with
sustention. Mild right wrist rigidity is revealed by augmentation. Which clinical
feature of this patient’s history raises a red flag suggesting an atypical
parkinsonian syndrome?
A. age of onset
B . asymmetric onset of tremor
C. decrease in tremor with activity
D. frequent falls early in the clinical course
E . olfactory impairment before the onset of motor symptoms

The preferred response is D (frequent falls early in the clinical course).


This patient presents with a tremor typical of Parkinson disease, including
asymmetric onset, frequency of 5 Hz, as well as presence at rest and reemergence
with sustention. However, his frequent falls raise concern that this may
be an atypical parkinsonian syndrome. The age of onset is typical of
Parkinson disease as is the presence of prodromal nonmotor symptoms such
as constipation and olfactory dysfunction. For more information, refer to
Table 5-3 on page 1121 of the Continuum article ‘‘Diagnostic Approach to
Atypical Parkinsonian Syndromes.’’

1296 www.ContinuumJournal.com August 2016

Copyright © American Academy of Neurology. Unauthorized reproduction of this article is prohibited.


b 3. When recommending the addition of a dopamine agonist, patients should
be counseled about the possible onset or exacerbation of which of the
following neuropsychiatric issues?
A. anxiety
B . apathy
C. dementia
D. depression
E . impulse control disorders

The preferred response is E (impulse control disorders). As dopaminergic


therapy is intensified with the use of dopamine agonists, patients may
experience the onset of impulse control disorders. Psychosis, which was not
listed, could also be produced or worsened. Anxiety, apathy, and depression
might improve somewhat with increased dopaminergic therapy, particularly
if anxiety or depression are being experienced as nonmotor off-period
phenomena. Development of dementia from these medications would not
be expected. For more information, refer to page 1093 of the Continuum
article ‘‘Neuropsychiatric Issues in Parkinson Disease.’’

b 4. A 56-year-old man who was diagnosed with Parkinson disease 5 years ago
presents with increased stiffness. He is maintained on carbidopa/levodopa
25 mg/100 mg 4 times a day and pramipexole 1.5 mg 3 times daily. Initially,
he found the treatment to be extremely effective, but the patient notes
that over the past year he can feel when he needs the next dose. The patient
feels very stiff and slow with worsening tremor about an hour before his
next dose of carbidopa/levodopa. What is the most appropriate next step?
A. deep brain stimulation
B . entacapone
C. infusional levodopa/carbidopa intestinal gel
D. ropinirole
E . trihexyphenidyl

The preferred response is B (entacapone). The patient has predictable


wearing off of levodopa with off time before his next scheduled dose.
Strategies to improve this problem include decreasing the interval between
doses, adding a catechol-O-methyltransferase inhibitor like entacapone,
adding a monoamine oxidase type B inhibitor, or adding a dopamine agonist.
The patient is already on a maximal dose of pramipexole (a dopamine agonist).
Deep brain stimulation may be considered after medical therapies are tried.
Infusional levodopa/carbidopa intestinal gel is considered in patients who are
referred for deep brain stimulation but are poor surgical candidates.
Trihexyphenidyl is an anticholinergic that may help somewhat in Parkinson
disease but causes sedation and is unlikely to meaningfully improve this
patient’s off time. For more information, refer to page 1077 of the Continuum
article ‘‘Treating the Motor Symptoms of Parkinson Disease.’’

Continuum (Minneap Minn) 2016;22(4):1295–1320 www.ContinuumJournal.com 1297

Copyright © American Academy of Neurology. Unauthorized reproduction of this article is prohibited.


Postreading Test—Preferred Responses

b 5. A 45-year-old right-handed man presents for evaluation of right hand


tremor and stiffness. The tremor started 6 months ago and has been worsening
over time. The patient only notes the tremor when his arm is at rest, and he is
bothered by stiffness in his right arm and hand when he is working. He works
as a chef and is experiencing significant trouble whisking eggs and working
with knives. He is worried he will lose his job. The patient has a history of a
gambling problem. On examination he has a slightly masked face, rigidity, and
bradykinesia in the right upper extremity, a resting tremor in the right hand,
with reduced right arm swing when walking. Which of the following therapies
is most appropriate for this patient?
A. amantadine
B . carbidopa/levodopa
C. entacapone
D. pramipexole
E . rasagiline

The preferred response is B (carbidopa/levodopa). This patient has


early Parkinson disease, and his work as a chef is affected by the rigidity
and bradykinesia in the affected dominant arm. Although he is young,
the most appropriate choice in this patient is carbidopa/levodopa as he
is manifesting functional disability from his Parkinson disease and would have
the greatest motor benefit from this option. Due to his gambling history,
a dopamine agonist like pramipexole would not be advised. Entacapone is not
used in monotherapy. Rasagiline is unlikely to have a symptomatic benefit
that will be robust enough to help him work. Amantadine may be somewhat
effective but is unlikely to restore function to the extent of levodopa. For more
information, refer to page 1072 of the Continuum article ‘‘Treating the
Motor Symptoms of Parkinson Disease.’’

b 6. Which of the following clinical features is most likely to be associated with


Friedreich ataxia?
A. adrenal insufficiency
B . diabetes mellitus
C. hepatopathy
D. renal failure
E . retinitis pigmentosa

The preferred response is B (diabetes mellitus). Friedreich ataxia is the


most common cause of genetic ataxia and is associated with a number of
non-neurologic manifestations, including diabetes mellitus and hypertrophic
cardiomyopathy. It is not usually associated with adrenal insufficiency,
liver disease, or retinitis pigmentosa. For more information, refer to
page 1172 of the Continuum article ‘‘Movement Disorders Presenting
in Childhood.’’

1298 www.ContinuumJournal.com August 2016

Copyright © American Academy of Neurology. Unauthorized reproduction of this article is prohibited.


b 7. A 56-year-old man presents for tremor that he has experienced for the past
3 years. The patient works as a dentist, and while the tremor initially did not
interfere with his work, over the past 6 months the tremor has manifested
while he performs procedures on patients. He notes the tremor is worsened
by caffeine and improves with a glass of wine. He has a history of asthma
but takes no medications. Family history includes tremor in his father and
paternal aunt. Neurologic examination is significant for mild head tremor and
bilateral postural tremor of the outstretched hands. The tremor is exacerbated
by drawing a spiral and worsens as the patient stretches his arms to approach
a target. What is the most appropriate initial therapy?
A. deep brain stimulation
B . primidone
C. propranolol
D. topiramate
E . verapamil

The preferred response is B (primidone). The most likely diagnosis is


essential tremor due to the bilateral postural and kinetic tremor with
intention component as well as family history. Primidone and propranolol
are first-line therapies for essential tremor and should be tried before any
alternative agents. Because the patient has asthma, propranolol should be
avoided as a first-line agent. Topiramate is a second-line agent. Deep brain
stimulation may be quite effective but would be reserved for refractory severe
cases. For more information, refer to page 1148 of the Continuum article
‘‘Diagnosis and Management of Tremor.’’

b 8. A 67-year-old woman with advanced Parkinson disease reports mild chronic


constipation. Her family is concerned because the patient often reports
symptoms of abdominal discomfort and poor appetite because she feels
bloated. Which of the following is the most appropriate next step?
A. abdominal massage
B . anticholinergic therapy
C. apomorphine
D. hydration and exercise
E . manual disimpaction

The preferred response is D (hydration and exercise). Mild chronic constipation


is a common symptom in Parkinson disease and may be difficult to manage. The
first steps in management include changes in diet, appropriate hydration, and
physical activity. Abdominal massage and polyethylene glycol (PEG) 3350 1 to
3 times per day may be used as later steps if diet, hydration, and exercise
are not fully effective. Manual disimpaction may become necessary for severe
constipation but the other options would more likely be effective on an ongoing
basis. Aggressively managing mild constipation will avoid the complications of
bowel obstruction and emergency department visits for constipation. Apomorphine
does not improve constipation. Constipation is a known side effect of anticholinergic
agents. For more information, refer to pages 1113Y1114 of the Continuum
article ‘‘Treatment of Advanced Parkinson Disease and Related Disorders.’’

Continuum (Minneap Minn) 2016;22(4):1295–1320 www.ContinuumJournal.com 1299

Copyright © American Academy of Neurology. Unauthorized reproduction of this article is prohibited.


Postreading Test—Preferred Responses

b 9. A 26-year-old woman is hospitalized for a rapidly progressive movement


disorder. She initially noted difficulty initiating voluntary facial movements
and a harsher tone to her voice. Within 2 days she also had significantly
slowed voluntary arm movements as well as unsteadiness with falls. Her arms
and legs involuntarily adopt abnormal postures that are sustained for up to
several seconds. Her neurologic examination reveals slow but otherwise
grossly normal cognition, a harsh and hypophonic voice, hypomimia, and
waxing and waning dystonic posturing of her arms and legs. MRI of the
brain is unremarkable. Mutation of which of the following genes is most
likely responsible for this woman’s condition?
A. ATP1A3
B . GCH1
C. GNAL
D. THAP1
E . TOR1A

The preferred response is A (ATP1A3). This patient presents with a history


and examination highly suggestive of rapid dystonia parkinsonism
(a condition associated with mutation in ATP1A3, which encodes the
!3 subunit of the sodium-potassium transporting adenosine triphosphatase
[ATPase]). Dopa-responsive dystonia is caused by deficiencies in enzymes
producing dopamine with the most common cause being mutations in the
GCH1 gene. GNAL mutations produce adult-onset cervical and cranial dystonia.
TOR1A and THAP1 are also associated with isolated dystonia rather than
dystonia combined with parkinsonism. For more information, refer to
page 1231 of the Continuum article ‘‘Diagnosis and Management of Dystonia.’’

b 10. A 22-year-old woman in her third trimester of pregnancy develops


repetitive, fairly stereotypical but nonrhythmic involuntary facial grimacing.
In the subsequent days she is noted to have choreiform movements of her
arms. She has also become somewhat disinhibited during this time period.
Her pregnancy has otherwise been uneventful. She has had two prior
miscarriages. She has no family history of neurologic disease. She denies
any other neurologic or systemic symptoms. Her neurologic examination is
remarkable only for the choreiform movements of her arms as well as
facial grimacing. Which of the following is the most likely diagnosis?
A. benign hereditary chorea
B . chorea gravidarum
C. Huntington disease
D. hypothyroidism
E . McLeod syndrome

1300 www.ContinuumJournal.com August 2016

Copyright © American Academy of Neurology. Unauthorized reproduction of this article is prohibited.


The preferred response is B (chorea gravidarum). Chorea arising during
pregnancy (chorea gravidarum) can be caused by a variety of underlying
conditions, the most common of which are antiphospholipid syndrome
and systemic lupus erythematosus. Benign hereditary chorea would be
expected to develop in childhood and to not be associated with cognitive
or psychiatric features. The age of onset and the rapid development of
symptoms would not be typical of Huntington disease. Hyperthyroidism
(not hypothyroidism) can be associated with chorea, and the patient
does not have clinical evidence of thyroid disease. Although orofacial
chorea may be seen in McLeod syndrome, it would tend to present later
and to be seen in men. For more information, refer to page 1195 of the
Continuum article ‘‘Chorea.’’

b 11. A 24-year-old woman is diagnosed with Wilson disease after presenting


with progressive tremor, dystonia, and psychosis. She is started on therapy
and experiences rapid neurologic deterioration within weeks. Which of the
following therapies most likely caused this presentation?
A. deferoxamine
B . penicillamine
C. tetrabenazine
D. trientine
E . zinc acetate

The preferred response is B (penicillamine). Penicillamine is a potent


copper chelator commonly used in the management of Wilson disease.
Although it is generally effective, it may induce a paradoxical deterioration
in neurologic function following initiation of therapy. Trientine is a
weaker copper chelator that is also used for treating Wilson disease but is
less frequently associated with early deterioration. Zinc interferes with
copper absorption and is commonly used as maintenance therapy in
Wilson disease, although some physicians advocate using it for initial
therapy. Deferoxamine is an iron chelator. Tetrabenazine is used to reduce
chorea in Huntington disease. For more information, refer to page 1255 of
the Continuum article ‘‘Wilson Disease.’’

Continuum (Minneap Minn) 2016;22(4):1295–1320 www.ContinuumJournal.com 1301

Copyright © American Academy of Neurology. Unauthorized reproduction of this article is prohibited.


Postreading Test—Preferred Responses

b 12. A 70-year-old man with a 15-year history of Parkinson disease is brought


to clinic by his family with concerns for dementia. Two years ago, his
family noted that he was beginning to have increasing difficulty with memory
for recent events as well as a tendency to use shorter words and sentences
and to have difficulty understanding others. Neuropsychiatric testing confirms
significant deficits in memory and semantic fluency as well as visuospatial
function, without evidence of significant depression. No contributory conditions
are detected upon examination, MRI of the brain, or screening laboratory
studies. Which of the following medications has been approved by the
US Food and Drug Administration (FDA) for the treatment of this man’s
cognitive disorder?
A. atomoxetine
B . memantine
C. quetiapine
D. rivastigmine
E . selegiline

The preferred response is D (rivastigmine). This patient developed


symptoms of dementia 13 years after being diagnosed with Parkinson
disease and appears most likely to have Parkinson disease dementia. Of
the medications listed, only rivastigmine has been FDA approved for
treating this cognitive disorder. Memantine has not been associated with
clear improvement in patients with Parkinson disease dementia. Atomoxetine
has been investigated in the treatment of apathy, and quetiapine can be
used for treating psychosis. Selegiline, a monoamine oxidase B inhibitor,
could be helpful for his motor symptoms but not for cognitive impairment
in Parkinson disease. For more information, refer to pages 1092Y1093 of
the Continuum article ‘‘Neuropsychiatric Issues in Parkinson Disease.’’

b 13. A 69-year-old man presents with a tremor in his right hand that developed
1 week ago and has become quite disabling. On examination, he has a
mild resting tremor in the right hand. The tremor worsens when he holds
his arms outstretched and is especially exacerbated by performing a task
with his right hand, such as drawing a spiral or pouring water into a cup.
What is the most likely cause of the tremor?
A. dystonic tremor
B . essential tremor
C. fragile X tremor-ataxia syndrome
D. midbrain infarct
E . Parkinson disease

1302 www.ContinuumJournal.com August 2016

Copyright © American Academy of Neurology. Unauthorized reproduction of this article is prohibited.


The preferred response is D (midbrain infarct). The patient has a disabling
unilateral tremor that has resting, postural, and kinetic components. This
is consistent with midbrain (rubral/Holmes) tremor and generally occurs
with lesions in the midbrain involving the cerebellar outflow tracts and
the substantia nigra. All of the other choices can cause tremor, but would
not be expected to be acute in onset and have early disability with significant
rest, postural, and kinetic components. Dystonic tremor may be present
at rest and with posture but is associated with dystonic posturing, which
is not noted in this patient. Essential tremor may be asymmetric but is
most often bilateral, and no resting component occurs early in the disease.
Fragile X tremor-ataxia syndrome is slowly progressive and is associated
with tremor as well as ataxia and cognitive deficits. For more information,
refer to page 1154 of the Continuum article ‘‘Diagnosis and Management
of Tremor.’’

b 14. A 10-year-old boy is evaluated for repetitive episodes of facial movements


and throat clearing, which started 1 year ago. His parents report that he
makes a grunting, throat-clearing sound multiple times per hour and has
experienced bullying by peers in school for ‘‘making weird sounds.’’ He
also often twitches his eyebrows. Evaluation by his pediatrician and two
otolaryngologists did not reveal an underlying cause. On examination, he
makes frequent grunting sounds and raises his eyebrows, both of which he
can suppress upon request and when distracted. The remainder of the
neurologic examination is normal. Which of the following is the most
appropriate initial therapy?
A. clomipramine
B. clonazepam
C. cognitive-behavioral therapy
D. guanfacine
E. levodopa

The preferred response is C (cognitive-behavioral therapy). This child is


experiencing multiple chronic tics, which can develop into Tourette
syndrome. The tics are clearly leading to social impairment as they are
causing him to be bullied at school. Cognitive-behavioral therapy is typically
the first-line therapy before consideration of drug therapy. However,
when needed, initial pharmacologic therapies for tics include clonidine and
guanfacine. If these are not effective, or in cases of severe disabling tics,
neuroleptics may be prescribed. Clomipramine may be prescribed for
obsessive-compulsive disorder, which often is comorbid with Tourette
syndrome. There is no established role for benzodiazepines or levodopa in
the management of Tourette syndrome. For more information, refer to
page 1162 of the Continuum article ‘‘Movement Disorders Presenting
in Childhood.’’

Continuum (Minneap Minn) 2016;22(4):1295–1320 www.ContinuumJournal.com 1303

Copyright © American Academy of Neurology. Unauthorized reproduction of this article is prohibited.


Postreading Test—Preferred Responses

b 15. Wilson disease is caused by a mutation of which of the following genes?


A. ATM
B . ATP7B
C. FXN
D. HTT
E . TOR1A

The preferred response is B (ATP7B). The genetic mutation in Wilson


disease is on the ATP7B gene. The other genes listed are associated
with other genetic movement disorders. ATM mutations cause
ataxia telangiectasia. TOR1A mutations cause idiopathic torsion dystonia.
HTT mutations cause Huntington disease. FXN mutations cause Friedreich
ataxia. For more information, refer to page 1246 of the Continuum
article ‘‘Wilson Disease.’’

b 16. A 65-year-old man with recently diagnosed cognitive impairment presents


with increasing falls and difficulty walking. On neuropsychological testing
he is found to have difficulties with attention, working memory,
visuospatial construction, and executive function. His family reports that he
has ‘‘good days,’’ in which he seems ‘‘almost like his old self,’’ and ‘‘bad
days,’’ in which he will sit in his chair interacting minimally. He has also
recently reported seeing ‘‘little children playing’’ in the house, although he
knows that they aren’t there. His sleep is restless as well, and his wife no
longer sleeps in the same bed due to his active dreams. More recently he
has begun to walk ‘‘more stiffly’’ and to fall without clear explanation. On
examination he has moderate appendicular rigidity and significant postural
instability as well as hypomimia. His gait is stiff without ataxia. Which of the
following clinical conditions would best explain this patient’s presentation?
A. corticobasal syndromeYfrontal behavioral variant
B . dementia with Lewy bodies
C. multiple system atrophyYparkinsonian type
D. Parkinson disease dementia
E . progressive supranuclear palsyYfrontotemporal dementia type

The preferred response is B (dementia with Lewy bodies). The constellation


of multifocal cognitive impairment preceding onset of parkinsonian
symptoms together with significant fluctuations in mental status and
nonthreatening visual hallucinations would be most consistent with dementia
with Lewy bodies. For Parkinson disease dementia and for the parkinsonism
variant of multiple system atrophy, cognitive impairment would be expected to
occur after the onset of motor symptoms. Early progressive frontal-subcortical
predominant dementia, fluctuations (in mental status and motor function),
rapid eye movement (REM) sleep behavior disorder, and hallucinations
all weigh against a corticobasal syndrome and progressive supranuclear palsy.
For more information, refer to page 1137 of the Continuum article
‘‘Diagnostic Approach to Atypical Parkinsonian Syndromes.’’

1304 www.ContinuumJournal.com August 2016

Copyright © American Academy of Neurology. Unauthorized reproduction of this article is prohibited.


b 17. A 52-year-old woman who was diagnosed with Parkinson disease
5 years ago is brought to clinic by her husband with concerns for changes
in thinking and mood. Since diagnosis, her motor symptoms have been
well controlled with carbidopa/levodopa. Over the past year, however, her
husband has noted that her mood has become more bland and that she has
more difficulty getting herself to get up and do anything. As a result, she
has become less socially engaged. Her husband also reports that her thinking
has become much slower. These symptoms do not seem to fluctuate
significantly day to day or over the course of the day. She does not endorse
feeling particularly hopeless and finds her emotional changes more curious than
disturbing. She does not feel guilty and denies any suicidal ideation. The
patient has not been taking extra doses of her dopaminergic medication.
Cognitive screening does not reveal any significant deficits, although her
processing speed is slow. Which of the following neuropsychiatric issues is most
likely emerging in this patient?
A. anxiety
B . apathy
C. depression
D. dopamine dysregulation syndrome
E . mild cognitive impairment in Parkinson disease

The preferred response is B (apathy). While some features of this patient’s


symptoms suggest depression, the overall picture is more consistent
with apathy, dissociated from any depressive disorder. She is experiencing
emotional blunting as well as difficulty with motivation. However, she
does not experience these changes as distressing nor is she experiencing
guilty or anxious ruminations. Her cognitive slowing does not appear to
be associated with decline in any particular cognitive domains, making
mild cognitive impairment in Parkinson disease unlikely. She has not
been taking additional doses of her dopaminergic medication, and no
impulse control difficulties have been noted, both of which weigh against
a dopamine dysregulation syndrome. For more information, refer to page 1093
of the Continuum article ‘‘Neuropsychiatric Issues in Parkinson Disease.’’

b 18. What is the mechanism of action of droxidopa?


A. !1 receptor agonist
B . aldosterone receptor agonist
C. dopamine receptor agonist
D. norepinephrine prodrug
E . serotonin 5-HT2A receptor agonist

Continuum (Minneap Minn) 2016;22(4):1295–1320 www.ContinuumJournal.com 1305

Copyright © American Academy of Neurology. Unauthorized reproduction of this article is prohibited.


Postreading Test—Preferred Responses

The preferred response is D (norepinephrine prodrug). Droxidopa is


a newly approved drug indicated for the treatment of orthostatic hypotension
in patients with Parkinson disease. Droxidopa is a norepinephrine precursor
and is able to cross the blood-brain barrier and is then decarboxylated to
form norepinephrine. For more information, refer to page 1112 of the
Continuum article ‘‘Treatment of Advanced Parkinson Disease and
Related Disorders.’’

b 19. A 28-year-old right-handed man who is a general surgery resident presents


for evaluation of increasingly bothersome head tremor while assisting in
procedures. The symptoms began more than 1 year ago and initially were
mild. However, more recently, he has noted pain and tightness in his neck
after a few minutes of standing in one position, after which his neck develops
a tremor. His medical history is otherwise unremarkable. His neurologic
examination demonstrates a slight head turn to the right and associated
head tremor. Which of the following treatments would be best to treat this
patient’s condition?
A. botulinum toxin
B . carbamazepine
C. carbidopa/levodopa
D. tetrabenazine
E . trihexyphenidyl

The preferred response is A (botulinum toxin). This patient has a focal neck
dystonia that is interfering with his work. Botulinum toxin injection would
be considered first-line therapy for this condition. Carbamazepine
is beneficial for paroxysmal kinesigenic dyskinesia, among other conditions.
Carbidopa/levodopa is optimal for patients with dopa-responsive dystonia.
Trihexyphenidyl may be considered for dystonic tremor if botulinum toxin
fails to adequately control the symptoms. Tetrabenazine, which inhibits
presynaptic vesicular uptake of dopamine, would not be considered first-line
therapy in a patient with a focal neck dystonia, but would be considered in
a patient with a more persistent, and particularly tardive, dystonia. For more
information, refer to page 1238 of the Continuum article ‘‘Diagnosis and
Management of Dystonia.’’

1306 www.ContinuumJournal.com August 2016

Copyright © American Academy of Neurology. Unauthorized reproduction of this article is prohibited.


b 20. Which of the following ophthalmic findings is associated with Wilson disease?
A. central retinal artery occlusion
B . keratoconus
C. macular degeneration
D. retinal detachment
E . sunflower cataract

The preferred response is E (sunflower cataract). The most well-known


ophthalmic manifestation of Wilson disease is Kayser-Fleischer rings. However,
sunflower cataracts are also associated with Wilson disease and are caused
by copper deposition in the anterior capsule. They do not cause visual loss
and are only detectable by slit-lamp examination. The other listed ophthalmic
syndromes are not specifically associated with Wilson disease. For more
information, refer to page 1250 of the Continuum article ‘‘Wilson Disease.’’

b 21. A 65-year-old man is brought to clinic by his family due to difficulty


walking and increasingly frequent falls that began about 2 years ago and
recently resulted in a fracture of his right wrist. They report that his speech
has also become increasingly slow, slurred, and monotonous, and he has
begun to cough and choke at times while eating or drinking. His family is
concerned that he may have significant anxiety or depression as he always
has a ‘‘worried expression.’’ Also, they report that he will suddenly start
laughing or crying for no reason. On examination, he has axial rigidity,
symmetric rigidity of his arms and legs, and a stiff gait with pivot turning.
He is unable to look up or down but this can be overcome by cervical-ocular
reflex maneuvers. Given the most likely diagnosis for this patient, which of
the following MRI findings would be expected?
A. asymmetric frontoparietal atrophy
B . bilateral T2 hyperintensities in the posterolateral putamen
C. diffuse cerebral atrophy with relative preservation of occipital volume
D. midbrain and superior cerebellar peduncle atrophy
E . olivopontocerebellar atrophy

The preferred response is D (midbrain and superior cerebellar peduncle


atrophy). This patient presents with symptoms and signs most consistent
with progressive supranuclear palsy: frequent falls, dysarthria, dysphagia,
pseudobulbar affect, axial rigidity, pivot turning, and supranuclear vertical
gaze palsy. Atrophy of the midbrain and superior cerebellar peduncle would
be expected. The brainstem might also take on a ‘‘beaked’’ appearance.
Asymmetric frontoparietal atrophy would suggest corticobasal syndrome
while bilateral T2 hyperintensities in the posterolateral putamen would
support the diagnosis of multiple system atrophy. Diffuse cerebral atrophy
with relative preservation of occipital and mesial temporal volume is seen in
dementia with Lewy bodies, while olivopontocerebellar atrophy is seen in
multiple system atrophy with predominant cerebellar ataxia. For more
information, refer to page 1126 of the Continuum article ‘‘Diagnostic Approach
to Atypical Parkinsonian Syndromes.’’

Continuum (Minneap Minn) 2016;22(4):1295–1320 www.ContinuumJournal.com 1307

Copyright © American Academy of Neurology. Unauthorized reproduction of this article is prohibited.


Postreading Test—Preferred Responses

b 22. Which of the following features most reliably distinguishes between


essential tremor and enhanced physiologic tremor?
A. action tremor
B . age of onset
C. intention tremor
D. response to beta-blockers
E . voice tremor

The preferred response is C (intention tremor). Enhanced physiologic


tremor can occur to some degree in all people. The enhanced physiologic
tremor may have both postural and kinetic components, but does not have
an associated intention tremor (ie, a tremor that increases in amplitude as
the patient approaches a target). Intention tremor is present in up to one-half
of patients with essential tremor. Both enhanced physiologic tremor
and essential tremor can manifest at any age, respond to propranolol,
and can have a vocal component. For more information, refer to
page 1148 of the Continuum article ‘‘Diagnosis and Management
of Tremor.’’

b 23. A 10-year-old girl presents for evaluation of muscle stiffness and abnormal
gait. Her symptoms began at age 8 and have been progressing. Initially,
her feet would turn inward when walking, and more recently she reports that
her thighs feel very tight. A family history of dystonia exists in her mother
and a maternal aunt. On examination, she demonstrates dystonic posturing
of her left foot, and when walking, she develops dystonic posturing in
both lower extremities. What is the most appropriate initial therapy?
A. benztropine
B . levodopa
C. onabotulinumtoxinA
D. tetrabenazine
E . trihexyphenidyl

The preferred response is B (levodopa). The patient presents with a


syndrome of an autosomal dominant progressive dystonia that is worse
in the lower extremities. This presentation could be consistent with
dopa-responsive dystonia, and levodopa therapy should be tried as the first
agent, as it may be very beneficial. Although it is difficult to make a specific
diagnosis of a dystonia syndrome on the basis of clinical features alone,
levodopa should be tried when any suspicion exists for dopa-responsive
dystonia. For more information, refer to pages 1167Y1168 of the Continuum
article ‘‘Movement Disorders Presenting in Childhood.’’

1308 www.ContinuumJournal.com August 2016

Copyright © American Academy of Neurology. Unauthorized reproduction of this article is prohibited.


b 24. A 55-year-old right-handed man reports right hand clumsiness, which has
progressed over the past 2 years. He reports no weakness or clear sensory
disturbance of the hand but reports increasing difficulty holding objects or
using utensils. He now uses his left hand more than his right. Over the last
6 months he has begun to notice a right hand tremor that has been slowly
worsening in intensity. A trial of levodopa up to a dose of 1200 mg/d provided no
noticeable improvement. On examination he has a coarse 6 Hz tremor at the
right wrist at rest and with action. His motor examination reveals normal
strength throughout, with rigidity of the right arm at the wrist and elbow and
dystonic posturing of the hand. Primary sensory modalities are intact but he
exhibits agraphesthesia and astereognosis of the right hand. He also is unable
to mime tasks or imitate gestures with his right hand, although he has no
difficulty doing so with his left. His Montreal Cognitive Assessment (MoCA)
score is 25 out of 30; he is unable to complete the Trails test, draw a cube,
and has difficulty drawing a clock face. What is the most likely diagnosis
for this patient?
A. corticobasal degeneration
B . dementia with Lewy bodies
C. idiopathic Parkinson disease
D. multiple system atrophy
E . progressive supranuclear palsy

The preferred response is A (corticobasal degeneration). While some


features of this patient’s presentation suggest idiopathic Parkinson disease
(eg, asymmetry, tremor, rigidity), the lack of response to levodopa,
the presence of cortical sensory dysfunction and ideomotor apraxia, as
well as the presenting concern of hand clumsiness strongly suggest
corticobasal degeneration. Dementia with Lewy bodies would be expected
to present with more significant cognitive dysfunction and more diffuse
parkinsonism. Progressive supranuclear palsy would typically present
with falls, gait instability, and supranuclear vertical gaze palsies. The
absence of autonomic symptoms and ataxia argue against multiple system
atrophyYparkinsonian type or multiple system atrophyYcerebellar type.
For more information, refer to page 1128 of the Continuum article ‘‘Diagnostic
Approach to Atypical Parkinsonian Syndromes.’’

Continuum (Minneap Minn) 2016;22(4):1295–1320 www.ContinuumJournal.com 1309

Copyright © American Academy of Neurology. Unauthorized reproduction of this article is prohibited.


Postreading Test—Preferred Responses

b 25. A 42-year-old man presents with changes to his personality and


demeanor as well as involuntary movements of his arms and legs.
Approximately 4 years ago his family began to notice that he appeared
increasingly depressed with a generally sad mood and exhibited social
disengagement, apparent apathy, irritability, as well as motor restlessness.
Over the past year he began to have involuntary, nonrhythmic movements
of his fingers, at times with a twisting movement of his wrists, and his gait
has become unsteady with involuntary movements of his legs. The patient’s
family is originally from Mexico, and the patient’s father had experienced
‘‘depression and dementia’’ and committed suicide in his late fifties. On
examination, the patient’s affect is restricted. Bedside cognitive testing
reveals problems with abstract thinking and working memory. Choreiform
movements of his hands are noted and his gait appears unsteady and erratic,
which is associated with chorea and dystonia of the lower limbs. MRI of the
brain demonstrates mild caudate atrophy, and laboratory studies, including
a complete blood count, liver function tests, creatine kinase, and thyroid
studies are normal. Which of the following diagnoses is most likely
in this patient?
A. dentatorubral-pallidoluysian atrophy
B . Huntington disease
C. Lesch-Nyhan syndrome
D. McLeod syndrome
E . Wilson disease

The preferred response is B (Huntington disease). This patient’s history


is typical of Huntington disease. Patients with dentatorubral-pallidoluysian
atrophy tend to be Japanese and to have myoclonus, dystonia, ataxia, and
psychosis, in addition to chorea. Lesch-Nyhan syndrome typically presents
early in life with hypotonia and developmental impairment followed by
spasticity, choreoathetosis, and self-injurious behavior. Patients with McLeod
syndrome tend to present later in life, may have seizures, and may show
evidence of hemolytic anemia. Wilson disease usually presents earlier
with dystonia, parkinsonism, and tremors. For more information, refer to
page 1186 of the Continuum article ‘‘Chorea.’’

1310 www.ContinuumJournal.com August 2016

Copyright © American Academy of Neurology. Unauthorized reproduction of this article is prohibited.


b 26. A 77-year-old man with long-standing Parkinson disease presents for
evaluation of abnormal movements. He takes carbidopa/levodopa 25 mg/100 mg
5 times per day, which helps him maintain mobility. However, over the past
few months, he notes abnormal jerking movements in his limbs and
his head that are distracting to others. Usually these occur 2 hours after his
levodopa dose. When the patient tried to lower his levodopa dose at the
suggestion of his primary care doctor, he felt much slower and stiffer. What
is the most appropriate therapy?
A. amantadine
B . deep brain stimulation
C. entacapone
D. pramipexole
E . trihexyphenidyl

The preferred response is A (amantadine). The patient is experiencing


levodopa-induced dyskinesia, and amantadine is the most effective
therapeutic option for reducing the symptoms. Entacapone may worsen
the symptoms as it bolsters the effect of levodopa. Pramipexole is unlikely
to work for this issue. Trihexyphenidyl can worsen choreiform dyskinesia.
Deep brain stimulation may help reduce the overall dose of levodopa
and ultimately reduce dyskinesia but is not the first step. For more
information, refer to page 1079 of the Continuum article ‘‘Treating the
Motor Symptoms of Parkinson Disease.’’

b 27. Which of the following features on neurologic examination is most likely


to be associated with a lesion of the cerebellar vermis?
A. downbeat nystagmus
B . intention tremor
C. limb ataxia
D. Romberg sign
E . truncal ataxia

The preferred response is E (truncal ataxia). Lesions of the cerebellar


vermis generally cause gait instability with truncal instability manifested
by oscillations of the body while sitting or standing as well as upbeat
nystagmus. Downbeat nystagmus is usually caused by lesions at the
foramen magnum. Intention tremor and limb ataxia are generally caused
by ipsilateral cerebellar lesions. Romberg sign is associated with sensory
ataxia or vestibulopathy. For more information, refer to page 1209 of the
Continuum article ‘‘Ataxia.’’

Continuum (Minneap Minn) 2016;22(4):1295–1320 www.ContinuumJournal.com 1311

Copyright © American Academy of Neurology. Unauthorized reproduction of this article is prohibited.


Postreading Test—Preferred Responses

b 28. A 25-year-old woman presents for a second opinion regarding Tourette


syndrome. The patient had been neurologically normal until approximately
2 years ago when she developed dystonic tongue thrusting, particularly when
speaking or eating. She also began to make grunting noises and has been
unable to suppress these movements and noises. Over time she developed
other involuntary movements including intermittent flinging movements of
her arms and quick, nonrhythmic movements of her fingers. Her gait has
also become unsteady, and she has fallen a few times. She has had three
apparently generalized convulsive seizures with the first occurring approximately
1 year ago. Having done well academically in college and law school, she is
now struggling in her first job as a corporate lawyer due to irritability, slowed
thinking, and emotional instability. She is the youngest of four children, and one
of her brothers has epilepsy and a mild movement disorder described as tics.
Her examination is significant for dystonic tongue protrusion, mild generalized
chorea, absent ankle and biceps deep tendon reflexes, and a hesitating
unsteady gait. Her primary physician had noted moderate hepatomegaly.
Which of the following conditions is most likely in this patient?
A. chorea-acanthocytosis
B . dentatorubral-pallidoluysian atrophy
C. Huntington disease
D. Lesch-Nyhan syndrome
E . Wilson disease

The preferred response is A (chorea-acanthocytosis). This patient presents


with a history typical of chorea-acanthocytosis with young-adult onset,
dystonic tongue thrusting, and with other choreiform and ballistic movements,
vocalizations, and behavior changes. The presence of seizures is also common
in this condition. Patients with dentatorubral-pallidoluysian atrophy tend
to have myoclonus, dystonia, ataxia, and psychosis in addition to chorea.
Lesch-Nyhan syndrome tends to present early in life with hypotonia and
developmental impairment followed by spasticity, chorea, and self-injurious
behavior. The age of onset and regional distribution of her chorea is atypical
for Huntington disease. Wilson disease tends to have more prominent
parkinsonism, with orofacial involvement more in the form of dystonia with a
risus sardonicus. For more information, refer to page 1193 of the Continuum
article ‘‘Chorea.’’

1312 www.ContinuumJournal.com August 2016

Copyright © American Academy of Neurology. Unauthorized reproduction of this article is prohibited.


b 29. A 56-year-old man presents with progressive gait instability. His symptoms
started 8 years ago and have been gradually worsening, and he now requires
a walker to ambulate. His father and paternal uncle both experienced similar
syndromes. On examination the patient has bulging eyes, tongue fasciculations,
generalized rigidity and bradykinesia, bilateral dysmetria and dysdiadochokinesia,
as well as a very wide-based ataxic gait. What is the most likely diagnosis?
A. spinocerebellar ataxia type 1
B . spinocerebellar ataxia type 2
C. spinocerebellar ataxia type 3
D. spinocerebellar ataxia type 5
E . spinocerebellar ataxia type 6

The preferred response is C (spinocerebellar ataxia type 3). Spinocerebellar


ataxia type 3 (also known as Machado-Joseph disease) is an autosomal
dominant hereditary ataxia. Clues to the specific diagnosis of spinocerebellar
ataxia type 3, in this case, are the presence of parkinsonism, bulging eyes,
and fasciculations. Spinocerebellar ataxia type 1 is associated with hypometric
saccades and corticospinal tract signs. Spinocerebellar ataxia type 2 is
associated with slow saccades and areflexia. Spinocerebellar ataxia types 5
and 6 are associated with downbeat nystagmus. For more information, refer
to Table 9-3 on page 1214 and page 1219 of the Continuum article ‘‘Ataxia.’’

b 30. A 57-year-old man is referred for evaluation of right hand tremor that he
has experienced for the past year. The tremor is most prominent at rest
and has gradually worsened since onset. He also is experiencing increasingly
impaired dexterity in the right hand, such as when typing. He and his wife
agree that his sense of smell has decreased. He reports no balance changes or
cognitive symptoms. On examination, he has a 4 Hz to 6 Hz resting tremor
in the right hand. He has a decrease in blink rate and facial expression, and
his finger taps on the right side are hypokinetic with decrement. He has mild
rigidity of his right wrist when tested with augmentation, but otherwise his
tone, bulk, and strength are normal. Right arm swing is reduced, but he has
otherwise normal gait. What is the most likely diagnosis?
A. corticobasal syndrome
B. dementia with Lewy bodies
C. multiple system atrophy
D. Parkinson disease
E. progressive supranuclear palsy

Continuum (Minneap Minn) 2016;22(4):1295–1320 www.ContinuumJournal.com 1313

Copyright © American Academy of Neurology. Unauthorized reproduction of this article is prohibited.


Postreading Test—Preferred Responses

The preferred response is D (Parkinson disease). This patient has a typical


history for Parkinson disease with onset of asymmetric resting tremor and
bradykinesia in the sixth decade associated with olfactory dysfunction and
decreased finger dexterity. He has no evidence of cortical findings, which
would suggest dementia with Lewy bodies or corticobasal syndrome. No
evidence exists of early autonomic dysfunction as might be seen in
multiple system atrophy and no difficulty with eye movements or early
gait dysfunction to suggest progressive supranuclear palsy. For more
information, refer to Table 1-2 on page 1049 of the Continuum article
‘‘Diagnosing Parkinson Disease.’’

b 31. A 20-year-old man presents for evaluation of involuntary abnormal


movements while walking. He first noticed the movements 5 years ago but
they have recently become more prominent and bothersome. Upon standing,
particularly after sitting for a period of time, he develops involuntary
nonrhythmic twisting and flinging movements of his right leg superimposed
upon his otherwise normal gait. The attack will last for 5 to 20 seconds and
then resolve completely. He notices a ‘‘funny feeling’’ in his right leg
immediately prior to the onset of the movements. The attacks are easily
reproducible, with nothing else remarkable on his neurologic examination.
Which of the following medications would be most likely to significantly
decrease this patient’s involuntary movements?
A. botulinum toxin
B . carbamazepine
C. carbidopa/levodopa
D. clonazepam
E . valproic acid

The preferred response is B (carbamazepine). This patient’s events are


most consistent with paroxysmal kinesigenic dyskinesia, which is a condition
that often responds readily to carbamazepine. For more information, refer to
page 1238 of the Continuum article ‘‘Diagnosis and Management of Dystonia.’’

b 32. A 44-year-old woman presents for evaluation of head tremor that began
3 years ago. The tremor is present throughout the day and also occurs
when she lies down to go to sleep. She notes associated neck pain and a
pulling sensation in her neck. Examination demonstrates an irregular
tremor of the head with a slight head tilt to the left. The patient can
improve the tremor when she touches her chin with her left hand. What is
the most likely diagnosis?
A. dystonic tremor
B . essential tremor
C. orthostatic tremor
D. Parkinson disease
E . psychogenic tremor

1314 www.ContinuumJournal.com August 2016

Copyright © American Academy of Neurology. Unauthorized reproduction of this article is prohibited.


The preferred response is A (dystonic tremor). Head tremor can be a
feature of dystonia or essential tremor. This patient likely has dystonic tremor
due to the irregular nature of the tremor as well as the presence of dystonic
neck posture, associated neck pain, improvement with touching the chin (geste
antagoniste), and persistence of the tremor upon lying down. With essential
tremor, the head tremor is rhythmic and oscillatory with improvement or
cessation upon lying down. Dystonia may occur in essential tremor but is
typically mild and a later feature of the disease. Orthostatic tremor occurs in
the legs. Head tremor is atypical in Parkinson disease, although jaw tremor is
characteristic. Psychogenic tremor should be considered if the tremor has
atypical features such as entrainment or suggestibility. For more information,
refer to page 1150 of the Continuum article ‘‘Diagnosis and Management
of Tremor.’’

b 33. Which of the following patients with Parkinson disease is most likely to
develop an impulse control disorder from dopamine agonist therapy?
A. a 42-year-old man
B. a 42-year-old woman
C. a 71-year-old man
D. a 71-year-old woman
E. an 87-year-old man

The preferred response is A (a 42-year-old man). Impulse control disorders


are important potential side effects of dopamine agonists and may lead
to significant social, financial, or occupational harm. Patients and families
should be counseled about the risk for impulse control disorders and should
be monitored at interval visits for the development of impulse control
disorders. Patients with the highest risk of impulse control disorders are
younger males and those with a prior history of addiction in other settings.
For more information, refer to pages 1070 and 1073 of the Continuum
article ‘‘Treating the Motor Symptoms of Parkinson Disease.’’

b 34. A 17-year-old boy presents for evaluation of tremor. His symptoms


started 6 months ago and have been progressively worsening. He reports a
tremor in his arms and a sense of tightness in his limbs and face. He has
no family history of neurologic disease. Neurologic examination is notable
for facial dystonia, proximal, bilateral upper extremity high-amplitude
tremor, and mild symmetric rigidity. Which of the following additional
neurologic signs is most likely to be present in this patient?
A. Babinski sign
B . distal symmetric sensory loss
C. fasciculations
D. Kayser-Fleischer rings
E . reduced rectal tone

Continuum (Minneap Minn) 2016;22(4):1295–1320 www.ContinuumJournal.com 1315

Copyright © American Academy of Neurology. Unauthorized reproduction of this article is prohibited.


Postreading Test—Preferred Responses

The preferred response is D (Kayser-Fleischer rings). The patient presents


with a constellation of neurologic signs and symptoms most consistent with
Wilson disease. He has a proximal, high-amplitude upper extremity tremor
(wing beating) as well as dystonia. Kayser-Fleischer rings are a feature of
Wilson disease with neurologic manifestations. Wilson disease can also be
associated with parkinsonism. For more information, refer to pages 1248Y1250
of the Continuum article ‘‘Wilson Disease.’’

b 35. A 64-year-old man with early Parkinson disease has been maintained on
rasagiline for 10 months. He presents because his symptoms, which had initially
stabilized, seem to be worsening over the past 2 months. He describes a
sense of stiffness in his left arm as well as difficulty keeping up with his
spouse when they are walking outside. On examination, he is noted to have
decreased blink rate, bilateral upper extremity rigidity and bradykinesia
(left greater than right), mild resting tremor in the left hand, and mildly
stooped posture with no retropulsion. Which of the following is the most
appropriate next step?
A. add amantadine
B . add benztropine
C. add droxidopa
D. add rotigotine
E . change rasagiline to selegiline

The preferred response is D (add rotigotine). Patients with mild Parkinson


disease may initially respond to rasagiline, which is a monoamine oxidase
type B inhibitor. However, after 6 to 24 months, the disease will usually
progress and another medication will need to be added. In this case, the
next step would likely be to add either levodopa or a dopamine agonist.
Rotigotine is a transdermal dopamine agonist. Benztropine is likely to cause
significant sedation and is rarely used in Parkinson disease. Amantadine
may help, but is less likely to help than a dopamine agonist. For more
information, refer to page 1069 of the Continuum article ‘‘Treating
the Motor Symptoms of Parkinson Disease.’’

1316 www.ContinuumJournal.com August 2016

Copyright © American Academy of Neurology. Unauthorized reproduction of this article is prohibited.


b 36. An 80-year-old man presents to clinic reporting a several-year history of
involuntary movements, which have been getting worse and have recently
been causing imbalance while walking. He first noted the movements,
characterized as ‘‘little random finger jerks,’’ in his hands, which slowly
spread to involve his arms and legs. He has not noted a change in his
speech, nor has any change in cognition or personality been noticed by the
patient or been evident to his family. Overall, the patient has been quite
healthy with well-controlled hypertension. The patient has no history of
similar problems in his relatives. His examination shows generalized chorea
with an unsteady gait but is otherwise unremarkable. Which of the following
investigations is most likely to result in a diagnosis for this patient’s
movement disorder?
A. analysis of the HTT gene
B . ceruloplasmin level
C. quantification of antiphospholipid antibodies
D. review of a fresh peripheral blood smear
E . thyroid function tests

The preferred response is A (analysis of the HTT gene). This patient has
late-onset chorea with no other significant neurologic findings or issues,
a presentation that has been called senile chorea. The majority of these
cases appear to be late-onset Huntington disease and, therefore, analysis
of the HTT gene is likely to be diagnostic. Ceruloplasmin levels will screen
for aceruloplasminemia and Wilson disease, neither of which would be
likely in this patient given the age of onset of symptoms and the isolated
choreiform nature of his movement disorder. Review of a fresh peripheral
blood smear would be helpful in conditions such as chorea-acanthocytosis,
McLeod syndrome, or Huntington diseaseYlike 2. While hyperthyroidism
can be associated with chorea, this patient has no other constitutional or
neurologic signs or symptoms pointing to this condition. For more information,
refer to page 1192 of the Continuum article ‘‘Chorea.’’

Continuum (Minneap Minn) 2016;22(4):1295–1320 www.ContinuumJournal.com 1317

Copyright © American Academy of Neurology. Unauthorized reproduction of this article is prohibited.


Postreading Test—Preferred Responses

b 37. A 46-year-old man presents for evaluation of tremor that began 2 years
ago. The tremor initially did not bother him, but has worsened to the point
where it interferes with his handwriting. He takes no medications and has a
family history of Parkinson disease in his grandfather. Neurologic examination is
significant for asymmetric, bilateral postural tremor of the outstretched hands.
The tremor is also noted while the patient draws a spiral and holds a cup. No
rigidity or bradykinesia is seen. What is the most likely diagnosis?
A. dystonic tremor
B . essential tremor
C. Parkinson disease
D. psychogenic tremor
E . Wilson disease

The preferred response is B (essential tremor). The patient presents with


bilateral tremor in the hands with both postural and kinetic components. The
most common cause of this presentation is essential tremor, which is often
asymmetric. On examination, no dystonia, rigidity, bradykinesia, or resting
components are noted, making dystonic tremor and Parkinson disease unlikely.
Wilson disease is unlikely to present with isolated tremor and is also unlikely to
present at age 46. Psychogenic tremor should be considered if the tremor has
atypical features such as entrainment or suggestibility. For more information, refer
to page 1145 of the Continuum article ‘‘Diagnosis and Management of Tremor.’’

b 38. A 56-year-old man presents for evaluation of gait instability and frequent
falls. His symptoms started 4 years ago and have been progressively worsening.
The patient also reports several episodes of syncope with standing, erectile
dysfunction, and reduced voice volume. He has no family history of neurologic
disease. Neurologic examination demonstrates masked face, reduced blink
rate, scanning dysarthria, nystagmus, symmetric appendicular rigidity, titubation,
and wide-based unsteady gait. On orthostatic blood pressure testing, his systolic
blood pressure drops 30 points when going from supine to standing, without
any change in pulse. Brain MRI demonstrates brainstem and cerebellar atrophy.
What is the most likely diagnosis?
A. Friedreich ataxia
B . multiple system atrophy
C. paraneoplastic cerebellar degeneration
D. Refsum disease
E . spinocerebellar ataxia type 6

1318 www.ContinuumJournal.com August 2016

Copyright © American Academy of Neurology. Unauthorized reproduction of this article is prohibited.


The preferred response is B (multiple system atrophy). The patient manifests
with slowly progressive cerebellar ataxia with associated parkinsonism and
autonomic dysfunction. These symptoms are most consistent with multiple
system atrophy, which involves cerebellar, extrapyramidal, and autonomic
systems to varying degrees. This patient’s history and examination are most
consistent with multiple system atrophyYparkinsonian type. The absence of a
family history and the noncerebellar symptoms are atypical for spinocerebellar
ataxia type 6. Development of Friedreich ataxia and Refsum disease would
be highly unusual in the sixth decade. Paraneoplastic disease would typically be
more rapidly progressive. For more information, refer to page 1213 of
the Continuum article ‘‘Ataxia.’’

b 39. Which of the following is the most sensitive diagnostic test for
Wilson disease?
A. 24-hour urinary copper excretion
B . liver biopsy
C. serum ceruloplasmin
D. serum copper
E . slit-lamp examination

The preferred response is B (liver biopsy). Wilson disease causes copper


accumulation in various organs, including the liver, brain, and eyes. Liver
biopsy is the most sensitive test for a Wilson disease diagnosis and may
be abnormal even in asymptomatic individuals. However, because of the
potential risk associated with liver biopsy, other diagnostic tests may
preclude the need for biopsy. For example, in patients with neuropsychiatric
symptoms, the presence of Kayser-Fleischer rings associated with low
serum ceruloplasmin and elevated urinary copper excretion essentially
confirms the diagnosis of Wilson disease. Total serum copper levels generally
do not add useful information in the diagnostic evaluation of Wilson
disease because they primarily are a reflection of the reduced ceruloplasmin
level. For more information, refer to page 1251 of the Continuum article
‘‘Wilson Disease.’’

b 40. Two days after returning from a family vacation to Korea, a 6-year-old girl
develops a high fever, headache, and significant dystonic movements.
Which of the following infectious agents is most likely responsible for
her presentation?
A. Cryptococcus neoformans
B . herpes simplex virus type 1
C. Japanese B encephalitis virus
D. Mycobacterium tuberculosis
E . poliomyelitis virus

Continuum (Minneap Minn) 2016;22(4):1295–1320 www.ContinuumJournal.com 1319

Copyright © American Academy of Neurology. Unauthorized reproduction of this article is prohibited.


Postreading Test—Preferred Responses

The preferred response is C (Japanese B encephalitis virus). A number of


direct central nervous system infections can lead to acute encephalitic
syndromes with associated abnormal movements. Japanese B encephalitis
virus is an important cause of this syndrome and is endemic in much of Asia.
Children are particularly susceptible to this infection. Herpes simplex virus
type 1 encephalitis causes a limbic encephalitis syndrome, although can be
associated with a postinfectious autoimmune N-methyl-D-aspartate (NMDA)
encephalitis with abnormal movements. Poliomyelitis virus causes a
meningoencephalitis associated with lower motor neuron paresis. C. neoformans
is a cause of fungal meningitis, and M. tuberculosis causes acute to subacute
meningitis. Neither of these are associated with prominent movement disorders.
For more information, refer to page 1163 of the Continuum article
‘‘Movement Disorders Presenting in Childhood.’’

1320 www.ContinuumJournal.com August 2016

Copyright © American Academy of Neurology. Unauthorized reproduction of this article is prohibited.


Self-Assessment and CME

Patient Management
Address correspondence to
Dr Susan H. Fox, Toronto
Western Hospital, 399 Bathurst
St, Movement Disorders Clinic,

Problem—Preferred Toronto, ON M5T 2S8, Canada,


sfox@uhnresearch.ca.
Relationship Disclosure:

Responses Dr Fox has received salary


support for serving as the
co-editor of the International
Parkinson and Movement
Susan H. Fox, MBChB, MRCP(UK), PhD Disorder Society website,
as a speaker for Ipsen
Pharmaceuticals, Inc; Teva
Pharmaceutical Industries Ltd;
Following are the preferred responses for the Patient Management Problem and Zambon Company SpA;
and for serving on the
in this Continuum issue. The case, questions, and answer options are re- advisory boards of Lundbeck,
peated, and the preferred response is given, followed by an explanation and Orion Pharma Ltd, and
a reference with which you may seek more specific information. You are Novartis International AG.
Dr Fox has received research/
encouraged to review the responses and explanations carefully to evaluate grant support from Avanir
your general understanding of the material. The comment and references Pharmaceuticals, the Michael
J. Fox Foundation, the
included with each question are intended to encourage independent study. National Institutes of Health,
To obtain CME credits for this activity, subscribers must complete this and Parkinson Society of
Canada and has received
Patient Management Problem online at www.aan.com/continuum/cme. research/grant support as site
Upon completion of the Patient Management Problem, participants may principal investigator for
clinical studies from Adamas
earn up to 2 AMA PRA Category 1 CreditsTM. Participants have up to 3 years Pharmaceuticals Inc,
from the date of publication to earn CME credits. No CME will be awarded Cynapsus Pharmaceuticals
for this issue after August 31, 2019. Inc, and Kyowa Hakko USA
Inc. Dr Fox receives
royalties from Oxford
University Press.
Unlabeled Use of
Learning Objectives Products/Investigational
Use Disclosure:
Upon completion of this activity, the participants will be able to: Dr Fox reports no disclosure.
* 2016 American Academy
& Distinguish the clinical presentation of tardive dyskinesia of Neurology.
& Recognize the risk factors for developing drug-induced movement
disorders

A 70-year-old woman presents with abnormal mouth and truncal


movements. She also has vocalizations characterized by intermittent
grunting. The symptoms started about 6 months ago and are bothersome
to her and her family. Her family reports that she has a long-standing
history of depression with psychotic features for which she has been
treated with a number of drugs, but they cannot recall all the different
medications. The patient is currently on quetiapine 75 mg at bedtime
to control her psychiatric symptoms. The patient also has a history of
hypertension, dyslipidemia, and a prior myocardial infarction for which
her current medications include aspirin, simvastatin, and amlodipine.
There is a family history of depression but no known family history of
neurologic disease.

Continuum (Minneap Minn) 2016;22(4):1327–1336 www.ContinuumJournal.com 1327

Copyright © American Academy of Neurology. Unauthorized reproduction of this article is prohibited.


PMP—Preferred Responses

b 1. Which of the following additional historical features is most likely to be


beneficial in diagnosing this patient?
A. obtain a complete prior medication history
B. obtain additional family history
C. obtain further history about exposure to illicit substances
D. obtain further history of occupational toxic exposure
E . obtain history related to her cardiovascular disease

The preferred response is A (obtain a complete prior medication history).


Obtaining a full and detailed history of current medications as well as those
taken in the past is vital. The patient’s primary care provider as well as
her pharmacist should be contacted. The patient’s history is very suggestive
of a drug-induced movement disorder, most likely classic tardive dyskinesia.1
Exposure to a dopamine D2 receptor antagonist within the past 3 to 6 months
is likely. She is on the atypical antipsychotic quetiapine, which has dopamine D2
antagonist properties but generally has a low propensity to cause tardive
dyskinesia. The patient may have had exposure to other more potent (typical)
dopamine D2 antagonists in the past to treat her mood disorder. In the
elderly, a relatively shorter exposure time (eg, 1 month) can induce tardive
movements.2 Genetic disorders may cause orofacial chorea including
Huntington disease (HD), Huntington diseaseYlike (HDL) syndrome types 1
through 4, and neuroacanthocytosis, but she is unlikely to have such
disorders due to her age and lack of associated clinical features. She has
vascular risk factors, and cerebrovascular disease can be a cause of orofacial
chorea in the elderly. Involuntary movements may not necessarily be
noticed by the subject or family if they are acute in onset or in the context
of a stroke. However, in this case, the associated truncal movements
and vocalization would not be typical of poststroke chorea.

1. Jankovic J. Tardive syndromes and other drug-induced movement disorders. Clin Neuropharmacol
1995;18:(3):197Y214.
2. Jeste DV. Tardive dyskinesia rates with atypical antipsychotics in older adults. Clin Psychiatry
2004;65(suppl 9):21Y24.

Neurologic examination reveals a stereotypic, rhythmic chewing movement


of the patient’s mouth. She also has repetitive choreiform movements of
her tongue. She has intermittent grunting vocalizations that seem to be
associated with some mild truncal rocking. There are no other involuntary
movements, and her gait and balance are normal.

b 2. Which of the following bedside techniques is the most useful for


determining the cause of the patient’s abnormal oral movements?
A. have the patient copy movements made by the examiner
B. have the patient perform distraction maneuvers
C. have the patient perform tasks of mental calculation
D. have the patient remove her dentures
E . watch the patient eat

1328 www.ContinuumJournal.com August 2016

Copyright © American Academy of Neurology. Unauthorized reproduction of this article is prohibited.


The preferred response is E (watch the patient eat). Tardive dyskinesia will
typically suppress when eating or drinking,1 which is in contrast to chorea
due to HD or neuroacanthocytosis, where food is often forced out of the
mouth while eating.2 Having the patient perform tasks of mental calculation
or distracting movements (eg, asking the patient to open and close his or
her hand) can increase and initiate any involuntary movements in other body
parts. However, this may not help distinguish the cause of the chorea.
Examining the patient with and without her dentures is helpful to ensure the
movements are not simply due to how the dentures fit and may be part
of the etiology, but is not as useful as watching her eat.

1. Cardoso F. How to examine a patient with chorea. Movement Disord Clin Practice 2015;1(4):397.
doi:10.1002/mdc3.12107.
2. Gooneratne IK, Weeratunga PN, Gamage R. Teaching video neuroimages: orofacial dyskinesia
and oral ulceration due to involuntary biting in neuroacanthocytosis. Neurology 2014;82(8):e70.
doi:10.1212/WNL. 0000000000000144.

The patient’s primary care provider is contacted and reports that the
patient had been taking risperidone for nighttime agitation for the past
year, but she had stopped taking the medication 1 month ago and then
switched to quetiapine.

b 3. What is the most likely cause of the patient’s vocalizations?


A. agitation
B. akathisia
C. chronic cough due to dry mouth
D. tardive dyskinesia
E. vocal tics

The preferred response is D (tardive dyskinesia). Vocalizations and grunting


are often part of the spectrum of tardive dyskinesia movement disorders.1,2
Associated diaphragmatic dyskinesia may occur, and the patient also exhibits
truncal rocking movements that are very typical of tardive dyskinesia. Some
patients may experience associated drug-induced akathisia, where the patient
finds sitting still difficult. Vocalizations can be part of akathisia, but the
underlying cause in these cases would be drug induced and part of a tardive
syndrome. Tics have been reported as part of the spectrum of tardive
movement disorders, although are more commonly motor than vocal.

1. Soares-Weiser K, Fernandez HH. Tardive dyskinesia. Semin Neurol 2007;27(2):159Y169.


2. Fernandez HH, Friedman JH. Classification and treatment of tardive syndromes. Neurologist
2003;9(1):16Y27.

Continuum (Minneap Minn) 2016;22(4):1327–1336 www.ContinuumJournal.com 1329

Copyright © American Academy of Neurology. Unauthorized reproduction of this article is prohibited.


PMP—Preferred Responses

The primary care provider also reports having noticed some mild movements
in the patient’s mouth but attributed these to her dentures and felt that her
vocalizations were due to agitation. The primary care provider did not
believe that risperidone caused these movements and therefore requests
further etiologic investigations.

b 4. What is the most cost-effective investigation to rule out other causes of


chorea in this patient?
A. brain MRI
B. CSF analysis
C. genetic panel for Huntington disease (HD)
D. serum glucose
E . serum paraneoplastic panel

The preferred response is A (brain MRI). Brain MRI is the single most useful
test in directing further investigation in a patient with chorea.1 The most
common structural cause of chorea in the elderly is small vessel ischemic disease.
Less common causes of chorea include diabetic nonketotic hyperglycemia,
antiphospholipid antibody syndrome, and polycythemia rubra vera. HD and
HDL disorders (HDL1, HDL2) may have caudate atrophy on MRI. Rare
metabolic and storage disorders would also have abnormalities on MRI
(eg, copper or iron on the basal ganglia). Metabolic disorders including
liver, renal, and thyroid disorders are important to exclude. Liver disease
can cause acquired hepatolenticular degeneration with dyskinesia with
T1 hyperintensity in the pallidum. Increasingly recognized are
paraneoplastic disorders (eg, antiYN-methyl-D-aspartate [NMDA]) and
non-neoplastic autoimmune disorders (eg, antiYleucine-rich, glioma inactivated
1 [LgI1]) that can cause orofacial dyskinesia, usually in the setting of a
patient with an encephalitis. Blood and CSF examination for the presence
of these autoantibodies (eg, anti-NMDA) may be useful, but an MRI remains
the most useful test in this patient prior to further investigation.
1. Hermann A, Walker RH. Diagnosis and treatment of chorea syndromes. Curr Neurol Neurosci
Rep 2015;15(2):514. doi:10.1007/s11910-014-0514-0.

The patient’s blood work for hematologic, renal, liver, and thyroid
functions is normal, and brain MRI shows mild periventricular white matter
changes but no basal ganglia lesions. The patient and family are advised
that the MRI findings are not likely the primary cause of her symptoms but
are related to her history of vascular disease.

1330 www.ContinuumJournal.com August 2016

Copyright © American Academy of Neurology. Unauthorized reproduction of this article is prohibited.


b 5. What is the most appropriate next step in managing this patient?
A. reduce quetiapine
B . refer for neuropsychological testing
C. repeat the brain MRI
D. stop quetiapine
E . switch to a tricyclic antidepressant

The preferred response is A (reduce quetiapine). Reducing the antipsychotic


quetiapine is the next step in treating this patient. Although quetiapine is
less likely than other antipsychotics to cause tardive dyskinesia, it may be
exacerbating the symptoms.1,2 Stopping the medication may be an option as
well, although her mood issues may become more problematic.
1. Correll CU, Leucht S, Kane JM. Lower risk for tardive dyskinesia associated with second-generation
antipsychotics: a systematic review of 1-year studies. Am J Psychiatry 2004;161(3):414Y425.
2. Dodler CR, Jeste DV. Incidence of tardive dyskinesia with typical versus atypical antipsychotics in
very high risk patients. Biol Psychiatry 2003;53(12):1142Y1145.

The quetiapine dose is reduced to 25 mg at bedtime. However, after


1 month the primary care provider calls to say the patient is not sleeping at
night and is agitated with hallucinations. The primary care provider has
increased the dose of quetiapine back to 50 mg/d, with mild improvement
in sleep. However, the orofacial movements are still bothering the patient,
and her vocalizations appear worse.

b 6. What is the best treatment option for this patient at this time?
A. add clonazepam at night for sleep
B . add tetrabenazine
C. add vitamin E
D. stop amlodipine
E . switch from quetiapine to olanzapine

The preferred response is A (add clonazepam at night for sleep). Evidence-based


medicine guidelines suggest Level B evidence for adding clonazepam for
treating tardive dyskinesia.1 Treatment with tetrabenazine or amantadine has
Level C evidence, and treatment with vitamin E has Level U evidence.
Olanzapine is another atypical antipsychotic drug that has a similar predilection
for causing tardive dyskinesia.2

1. Bhidayasiri R, Fahn S, Weiner WJ, et al. Evidence-based guideline: treatment of tardive


syndromes: report of the Guideline Development Subcommittee of the American Academy
of Neurology. Neurology 2013;81(5):463Y469. doi:10.1212/WNL.0b013e31829d86b6.
2. Cloud LJ, Zutshi D, Factor SA. Tardive dyskinesia: therapeutic options for an increasingly
common disorder. Neurotherapeutics 2014;11(1):166Y176. doi:10.1007/s13311-013-0222-5.

The patient returns for neurologic follow-up 2 months later with extreme
daytime drowsiness and no change in her orofacial movements. The
patient would like to try an alternative method of treatment.

Continuum (Minneap Minn) 2016;22(4):1327–1336 www.ContinuumJournal.com 1331

Copyright © American Academy of Neurology. Unauthorized reproduction of this article is prohibited.


PMP—Preferred Responses

b 7. Which of the following medication changes would be most appropriate at


this time?
A. add amantadine
B . add tetrabenazine
C. add vitamin E
D. stop amlodipine
E . switch from quetiapine to olanzapine

The preferred response is B (add tetrabenazine). Adding tetrabenazine is


probably the next best option to reduce the tardive dyskinesia.1 Tetrabenazine
reversibly inhibits the activity of vesicular monoamine transporter type 2,
resulting in depletion of central dopamine. Tetrabenazine undergoes extensive
first-pass metabolism to active metabolites that are metabolized by the
cytochrome P450 2D6 isozymes. This results in variability in dosing requirements
for some individuals.2 The suggested starting dose is one-half a 25 mg tablet,
given 2 times a day and slowly titrated. At the same time, the patient can remain
on a low dose of quetiapine. The patient has ongoing mood issues that are
disruptive to her sleep, and as such she needs to continue with the antipsychotic
that has the lowest risk of causing tardive dyskinesia. Quetiapine has a low risk,
although it is still present. Reducing the dose to the lowest possible is often
tried but with variable efficacy. The patient is counseled about possible side
effects of tetrabenazine.

1. Jankovic J, Clarence-Smith K. Tetrabenazine for the treatment of chorea and other


hyperkinetic movement disorders. Expert Rev Neurother 2011;11(11):1509Y1523. doi:10.
1586/ern.11.149.
2. Mehanna R, Hunter C, Davidson A, et al. Analysis of CYP2D6 genotype and response to
tetrabenazine. Mov Disord 2013;28(2):210Y215. doi:10.1002/mds.25278.

The patient is started on tetrabenazine 12.5 mg 2 times per day. After


1 month, the dose is increased to 25 mg in the morning and 12.5 mg in the
afternoon. She remains on quetiapine 25 mg at night. The orofacial
movements are reduced, and the truncal movements are improved.
However, the patient still has the vocalizations and is still bothered by the
movements. The dose of tetrabenazine is further increased to 25 mg
2 times per day. Two months later she comes back with improved facial
and truncal movements, but her daughter reports that her mother is
moving slowly, including her walking. On examination she arises slowly
from the chair and walks very slowly. She has a slight asymmetric tremor in
her both hands when resting, which is worse on the right side, as well as
intermittent lower jaw tremor and reduced facial expression.

1332 www.ContinuumJournal.com August 2016

Copyright © American Academy of Neurology. Unauthorized reproduction of this article is prohibited.


b 8. What is the most likely explanation for the patient’s new complex of symptoms?
A. akathisia
B. anxiety
C. depression
D. parkinsonism
E . worsening tardive dyskinesia

The preferred response is D (parkinsonism). The patient has developed


subacute parkinsonism that is characterized by slowness of movements
due to bradykinesia. She also has a bilateral asymmetric resting tremor in her
hands and a jaw tremor. Akathisia causes restlessness and hyperkinetic
movements and is a common side effect of antipsychotics and tetrabenazine.1
Tetrabenazine can also cause depression,1 which can cause psychomotor
slowing in the elderly, but would not cause resting tremor as well. Anxiety can
cause tremor but this is usually postural and of high frequency.
1. Chen JJ, Ondo WG, Dashtipour K, Swope DM. Tetrabenazine for the treatment of hyperkinetic
movement disorders: a review of the literature. Clin Ther 2012;34(7):1487Y1504. doi:10.1016/
j.clinthera.2012.06.010.

The patient’s symptoms of slow movements and difficulty walking are


disabling and impacting her daily life. The family requests further management.

b 9. What is the likely cause of her recent-onset parkinsonian syndrome?


A. cerebrovascular disease
B. continued use of quetiapine
C. dementia with Lewy bodies
D. idiopathic Parkinson disease
E. tetrabenazine use

The preferred response is E (tetrabenazine use). The most common cause


of subacute parkinsonism is drug-induced parkinsonism,1Y3 of which the most
likely cause in this patient is tetrabenazine. Quetiapine can cause parkinsonism,
but this patient appeared to tolerate 75 mg/d without this side effect before
so this is less likely. Drug-induced parkinsonism is most commonly due to
dopamine D2 receptor blockers, both typical and atypical antipsychotics, as
well as antiemetics. Clinically, the parkinsonian signs of bradykinesia, tremor,
and rigidity are usually bilateral but may be asymmetrical. Vascular parkinsonism
is a controversial entity but generally describes lower body parkinsonism
with gait difficulties, including freezing. This patient’s symptoms have occurred
over a relatively short time period (2 months), which would be too fast for
a neurodegenerative parkinsonian disorder such as idiopathic Parkinson disease
or dementia with Lewy bodies.
1. Morgan JC, Sethi KD. Drug-induced tremors. Lancet Neurol 2005;4(12):866Y876.
2. Burkhard PR. Acute and subacute drug-induced movement disorders. Parkinsonism Relat
Disord 2014;20(Suppl 1):S108YS112. doi:10.1016/S1353-8020(13)70027-0.
3. López-Sendón J, Mena MA, de Yébenes JG. Drug-induced parkinsonism. Expert Opin Drug Saf
2013;12(4):487Y496. doi:10.1517/14740388.2013.787065.

Continuum (Minneap Minn) 2016;22(4):1327–1336 www.ContinuumJournal.com 1333

Copyright © American Academy of Neurology. Unauthorized reproduction of this article is prohibited.


PMP—Preferred Responses

b 10. What is the best management option for this patient at this time?
A. increase tetrabenazine
B . reduce quetiapine
C. reduce tetrabenazine
D. stop quetiapine
E . stop tetrabenazine

The preferred response is C (reduce tetrabenazine). Parkinsonism due to


tetrabenazine is usually dose-related and thus reducing the dose initially
is the best strategy.1,2 Another option may be to reduce quetiapine, although
this is a less likely cause of drug-induced parkinsonism in this patient.
1. Guay DR. Tetrabenazine, a monoamine-depleting drug used in the treatment of hyperkinetic
movement disorders. Am J Geriatr Pharmacother 2010;8(4):331Y373. doi:10.1016/
j.amjopharm.2010.08.006.
2. Chen JJ, Ondo WG, Dashtipour K, Swope DM. Tetrabenazine for the treatment of hyperkinetic
movement disorders: a review of the literature. Clin Ther 2012;34(7):1487Y1504. doi:10.1016/
j.clinthera.2012.06.010.

The patient’s dose of tetrabenazine is reduced to 12.5 mg 2 times per day,


and after 2 months she reports fewer tremors but her walking is still slow.
She still has some facial movements. However, overall she is managing
and does not want to change anything. The patient returns for follow-up
2 years later, at which time she has more difficulty walking. Her family
reports that she has slowed down more and has become slower when
dressing and washing. The tremor in her right hand is intermittently
worse, and she has a jaw tremor again. She finds writing more difficult.
She no longer has any orofacial dyskinesia or truncal movements, and
tetrabenazine was stopped 6 months ago. The quetiapine was stopped
1 year ago because of daytime sleepiness.

b 11. What is this patient’s most likely diagnosis at this time?


A. idiopathic Parkinson disease
B . quetiapine-induced parkinsonism
C. stroke
D. tardive dyskinesia
E . tetrabenazine-induced parkinsonism

1334 www.ContinuumJournal.com August 2016

Copyright © American Academy of Neurology. Unauthorized reproduction of this article is prohibited.


The preferred response is A (idiopathic Parkinson disease). The patient
now has progressive bilateral parkinsonism that has worsened even with
stopping tetrabenazine. She probably had underlying subclinical nigrostriatal
dopamine terminal loss due to idiopathic Parkinson disease that was
unmasked by the tetrabenazine and has now progressed.1 Quetiapine can
also cause parkinsonism; however, this was discontinued 1 year prior.
Typically, drug-induced parkinsonism will resolve within 3 months of
stopping or reducing the offending drug. About 10% of cases will persist
and progress and are diagnosed as idiopathic Parkinson disease that
was unmasked by the dopamine blocker.
1. Erro R, Bhatia KP, Tinazzi M. Parkinsonism following neuroleptic exposure: a double-hit
hypothesis? Mov Disord 2015;30(6):780Y785. doi:10.1002/mds.26209.

The patient remains clinically unchanged. She has a slow gait but does not
need any help with her activities of daily living. She is started on
mirtazapine 15 mg at night to help her sleep. She presents for follow-up
3 months later and reports some slowness and mild problems turning in
bed at night as well as increased constipation. She is slower while eating and
has slightly excessive salivation. The patient and her family are reluctant for
her to take more medications.

b 12. What management strategy is the best course of action at this time?
A. no changes in medications required
B . start carbidopa/levodopa
C. start pramipexole
D. start rasagiline
E . stop mirtazapine

The preferred response is A (no changes in medications required). The


patient likely has idiopathic Parkinson disease,1,2 but with mild disability;
therefore, no changes in medications are required. At this stage, trying to
manage her symptoms without new drugs is probably the best approach.
Aids to help her get in and out of bed could help, and dietary advice for the
constipation is appropriate. Use of levodopa or other antiparkinsonian agents
(rasagiline and pramipexole) are probably not required at this stage. A full
discussion with the patient regarding benefits versus risks of any new drug
should be made. In early Parkinson disease the time to start antiparkinsonian
medication is determined by the patient’s lifestyle and quality of life, and
the patient’s decision needs to be respected.

1. Lim TT, Ahmed A, Itin I, et al. Is 6 months of neuroleptic withdrawal sufficient to distinguish
drug-induced parkinsonism from Parkinson’s disease? Int J Neurosci 2013;123(3):170Y174.
doi:10.10.3109/00207454.2012.732976.
2. Morley JF, Pawlowski SM, Kesari A, et al. Motor and non-motor features of Parkinson’s disease
that predict persistent drug-induced Parkinsonism. Parkinsonism Relat Disord
2014;20(7):738Y742. doi:10.1016/j.parkreldis.2014.02.024.

Continuum (Minneap Minn) 2016;22(4):1327–1336 www.ContinuumJournal.com 1335

Copyright © American Academy of Neurology. Unauthorized reproduction of this article is prohibited.


PMP—Preferred Responses

The patient agrees not to start any new medications as she is worried
about side effects in view of the prior problems she has experienced. The
mirtazapine did seem to help her at night. One year later she returns for
follow-up, and she has ongoing slowness of all her activities of daily living,
with a slower gait. She uses a walker as she has had a few falls. Her voice
is quieter, and she has some mild dysphagia with liquids. She has intermittent
tremor in her hands that is worse on the right side, especially if she gets
anxious. She also reports stiffness and cramps in her legs overnight that
wake her up. However, she prefers to continue without medications. Her
primary care provider contacts the neurology office a few months later as
the family has expressed concern and anger at the problems their mother
has experienced over the past few years. The primary care provider is keen to
try and learn from the experience.1,2
1. Esper CD, Factor SA. Failure of recognition of drug-induced parkinsonism in the elderly. Mov
Disord 2008;23(3):401Y404.
2. Canadian Clinical Practice Guidelines for the Treatment of Schizophrenia. Can J Psychiatry
2005;50(suppl 1):7S.

1336 www.ContinuumJournal.com August 2016

Copyright © American Academy of Neurology. Unauthorized reproduction of this article is prohibited.

Вам также может понравиться